Vector Calculus (A.r. Vasishtha, A.K. Vasishtha)

Download as pdf or txt
Download as pdf or txt
You are on page 1of 338

https://www.pdfnotes.

co/

Tr:

Jy Educational Publishers
Since 1942

fo\W .‘j'

T S€ri6S

Vector
Calculus

r^-
https://www.pdfnotes.co/

Vector
Calculijs
[A Course on Vector Algebra and Vector Calculus]

{For Degree and Honours Students ofAll Indian Universitites andfor Various
Competitive Examinations like I.A.S. and P.C.S etc.)

Bv

A.R. Vasishtha A.K. Vasishtha


Retd., Head Deptt. of Mathematics M.Se., Ph.D.

Meerut College, Meerut C.C.S. University, Meerut

KRISHNA Prakashan Media(P)Ltd.


KRISHNA HOUSE, 11, Shivaji Road. Mccrui-250 001 (U.P.), India
https://www.pdfnotes.co/
B.Sc. Made Easy Series (Fully Solved)
All Indian Universities

Trigonometry /V. R. Vasishtha & Others


Matrices
A. R. Vasishtha &.Others
Algebra A. R. Vasishtha &.Others
Differential Calculus A. R. Visishtha & Others
Integral Calculus A. R. Vasishtha &.Others
Differential Equations A. R. Vasishtha & Others
Geometry of 2 Dimensions .-Ai R. Visishtha &.Others
Geometry of 3 Dimensions A. R. Vasishtha & Others
Modern Algebra A. R. Vasishtha &.Others
Vector Calculus A, R. Vasishtha & Others
Ordinary &.Partial Differrential Equations A. R. Vasishtha.& Others
Partial Differrential Equations A..R. Visishtha & Others
Statics
A;-R. Vasishtha &.Others
Dynamics A. R.\^sishtha &.Others
Real Analysis A. R, Vasishtha & Others
Numericail Analysis A. R. Vasishtha & Others
hydrostatics A. R. Vasishtha & Others,

Vector Calculus
Book Code: 452-13
ISBN.: 978-93-87705-69-2
Thirteenth Edition :2017

Price :? 210.00 Only


f*<>’^^ffyf<>oranypart<^mbookthereofmaymbereproducedlnanyformorbyat,ynKanswlthoutZwrm6npemtsslon
fnmthepublishersandtheauthon.£veryeffdnhasbeenmadetoavoldermrsor omlsslonslnthlspubilcatlm.tnsplteofth{s,
some errors mighthavecreptIrt. Any mistake,enbrordiscrepancynoted may be broughtto ournotice which shattbetaken care
ofl(>tkenextedltlon.ltlsnotlpedthatneltherthepubllshernortheauthororselterwlllberespenslble/Oranydamageorlossof
action toanyone,ofany kind.In any manner,therefrom.FOrbinding mistakes,misprintsorfor missing pages,etc. the publisher's
1‘obllltylslimitedtoreplacementwlthlnoriemonthofpurchasebyslmllaredltlon.AlleMpenseslnthlsconnectlonaretobebome
by tliepurchaser. ● .

Published by : Satyendra Rastogi 'Mfrrt''


for KRISHNA Prakashan Media(P) Ltd.
11. Shivaji Road, Meerut-250 001 (U.P)India.
Phones :91.121.2644766,2642946, 4026111. 4026112 Fax: 91.121.2645855
Website: www.krishnaprakashan.com
E-mail: [email protected] '
ChirfEditor : Sugam Rastogi
{fitted at : Brahma Offset Printers, Meerut
https://www.pdfnotes.co/

PREFACE

This book on Vector Calculus has been written for the use of the
students of Degree and Honours classes of all Indian Universities.
In each chapter all concepts and definitions have been diseased
in detail and lucid manner,so that the students should feel no difficulty
to understand the subject. The articles have been explained in detail in
a nice manner and all the examples have been completely solved. We
have tried to solve each problem in an elegant and more interesting
way. The students should follow the solutions very carefully and they
should try to reproduce them when they do the problems inde
pendently.
We have tried our best to keep the book free from misprints. The
authors shall be grateful to the readers who point out errors and
omissions which,inspite 6f all care, might have been there.
The authors will feel amply rewarded if the book serves the
purpose for which it is meant.Suggestions for the improvement of the
book are always welcome.
' —The Authors
92/2. THAPAR NAGAR, A.RVasishtha
MEERUT-250001 KiranVasishtha
https://www.pdfnotes.co/

CONTENTS
Chapters
Pages
1. Multiple Products
1—34
Scalar triple product
Vector triple product
Lagrange’s identity for four vectors
Vector product of four vectore
Reciprocal system of vectors

2. Differentiation and Integration of Vectors 35 — 74


Vector function
Scalar fields and vector fields
Limit and continuity of a vector function
Derivative of a vector function with rrespect to a scalar
Differentiation formulae
Curves in space
Integration of vector functions

3. Gradient, Divergence and Curl


75 — 144
Partial derivatives of vectors
The vector differential operator del.(V)
Gradient of a scalar field
Level surfaces

Directional derivative of a scalar point function


Tangent plane and normal to a level surface
Divergence of a vector point function
Curl of a vector point function
Important vector identities

4. Green’s, Gauss’s and Stake’s Theorems 145 — 332


Une integrals
Surface integrals
Volume integrals
Green’s theorem in the plane
The Gauss’s divergence theorem
● Stok^’s theorem
Line integrals independent of path. .
https://www.pdfnotes.co/

1
Multiple Products
§ 1. Triple Products.
We know that the vector product a x b of two vectors a and b
is itself a vector quantity. Therefore we can multiply it by another
vector c both scalarly and vectoridly. The product (axb)*c is
called scalar triple product, which is a pure number. On the other
hand the product(ax b)xc is called vector triple product, which
is again a vector quantity.
Note. Since a* b is a scalar quantity, therefore the products
(a*b)«c and (a* b)xc are meaningless. Moreover in the product
(a*b)*c we can omit the parentheses and we can simply write it
asaxb*c. Obviously the product ax b*c has meaning only if we
regard it as the product(a Xb)»c.
§ 2. Scalar Triple Product.
[Meerut 1982; Kerala 74; Guru Nanak 81]
The scalar product of two vectors one of which is itself the
vector product of two vectors is a scalar quantity called a Scalar
Triple Product*^ Thus if a, b and c be three vectors, then (axb)«c
is called the scalar triple product of these three vectors.
Since the scalar- triple product involves both the signs of
'cross’ and 'dot’ therefore it is sometimes also called the mixed
product.
^Geometrical Interpretation of Scalar Triple Product.
[Meerut 1982; Kerala 74; Allahabad 80]
Let us consider a paral-
lelopiped whose coterminous
edges OA, OB, OC have the
lengths and directions of the
vectors a, b, c respectively*
T,et V be the volume of this
parallelopiped. We shall regard
V, as necessarily positive.
https://www.pdfnotes.co/
2 VECTORS MAPE EASY

i^et axb=n. Then from our definition of ve't*or product, th.e


vector n is perpendicular to the face OADB, and its modulus n is
the measure of the area of the parallelogram OADB. Also^ by
definition, the vectors a, b and n form a right handed triad.
Let ^ denote the angle between the directions of the vectors
OC and n. Then the vectors a, b and c will form a ■ right handed
or a left handed triad according as ^ is acute or obtuse.
Now (axb)»c=|(axb) 1 1 c I cos ^==1 n 11 c 1 cos ^
=(area of the ^rallelogram OADB).(OCcos
[V |c|=oq.
Now OC cos ^ will be positive or negative according as ^ is
acute or obtuse^ Its absolute value will give us the length of the
perpendicular from C to. the plane ofthe parallelogram
Now the volume V of the parallelepiped=(Area of the
parallelogram 0/4Z>if)xlength of the perpendicular from Con
this parallelogram. Therefore(axb)»c=+F,if^ is acute i.e. if
a, b, c form a right handed triad and (a xb)»c=—K if ^ is obtuse
Le. if a, b, c form a left handed triad.
Now we know that if the vectors a, b, c form a right handed
triad, then the vector triads b, c, a and c, a, b are also right
handed. Hence each of the products (bxc).a and (cxa)«b will
have the same value +V or —V according as a, b, c form a right
handed or a left handed triad. Thus we conclude that in all cases
(axb).c=(bxc)«a=(cxa)«b.
Now a*b—b*a and axb=—bxa.
/. (a X b)*c==c»(a x b)=(b x c)*a=a*(b x c)
=»(cxa)*b=b*(cxa)
=—(bxa)»c= —c«(bx a)= —(c xb)»a
' =—a»(cxb)=—(axc)*b=—b«(axc).
From this we conclude that the value of a scalar triple product
depends on the cyclic order of the factors and is independent of the
position of the dot and cross. These may be interchanged at pleasure.
However, an anticyclic permutation of the three factors changes the
value of the product in sign but not in magnitude. [Important]
Notation. In view of the properties discussed above, the
scalar triple product is usually written as (axb)«c=[abc] or
[a, b, cj. This notation takes into consideration only the cyclic
order of the three vectors and disregards the unimportant positions
of dot and cross. Thus [abc]=[bca]=[cab]= —[cba] etc.
The signs of dot and cross can be inserted at pleasure
f.e. [a, b, c]=a*(bxc) or =(axb)»c.
https://www.pdfnotes.co/
MULTIPLE PRODUCTS 3

Note 1. If i, j, k consitute an orthogonal right handed triad


of unit vectors, then [i. j, k]=(ixj).k=k«k=l.
Note 2. The scalar triple product[abc] is positive or negative
according as a, b, c form a right handed or a left handed triad of
vectors.
**§ 3. Distributive Law for Vector Product.
. 7’o/>rove/Au/ax(b+c)=axb+axc. wAerc a, b, c are any
three vectors.. . [Delhi 1978; Allahabad 75]
Let rSax(b+c)-ax'b axe
. Now forming the scalar product of both sides of(1) with an
arbitrary vector d, we get
d.p=d»[ax(b+c)—axb—axe] ...(2)
or
d.r=d.[ax(b+c)]^d«(ax b)-d*(axc)
[Since scalar product is distributive]
Now in a scalar triple product the positions of dot and cross
edn be interchanged without affecting its value. Therefore from
(2), we get
d.r=(d X a),(b+c)-(d x a),b-(d x a)●c
=(d xa)*b-|-(dxa)«c—(dx a)«b—(d xa)*c
[Since scalar product is distributive]
=0.
Therefore either d=sO, or r=0 or d is perpendicular to r.
But the vector d is arbitrary. Therefore we can take if to be non
zero and not perpendicular to r.
Hence r=0 i.e. ax(b-fc)—axb—axc=0
i.e. a X(b+c)=a x b-f-a X c.
§ 4. Properties of Scalar triple product.
(0 The value of a scalar triple product, if two ofits vectors
are equal, is zero. [Agra 1973]
We have[aab]=a'(ax b).
Now a X b is a vector perpendicular to the plane of a and b.
Therefore a*(a x b)=0.
(w) The value ofa scalar triple product, if two ofits vectors are
parallel, is zero.
Let a, b, c be three vectors such that a and b are parallel i.e.
b=ra, where t is some scalar.
Now [abc]=(axb).c=(ax/a)*c=/(axa).c
=/ (0‘C) [V axa=0]
=0.
- ♦*(///) The necessary and sufficient condition that three non
parallel and non-zero vectors a, b, c be coplanar is that [abc]=0.
[Lucknow 1977; Rohilkband 79; Meerut 83]
https://www.pdfnotes.co/
4 VECTORS MADE EASY

Let a, b, c be three coplanar vectors. Now axb is a vector


perpendicular to the plane of a and b. Since a, b, c are coplanar,
therefore axb is also perpendicular to c. Now the dot product
of two perpendicular vectors is equal to zero. Hence(a x b)● c=0
{.e. [abc]<=0. Therefore the condition is necessary.
The condition is also sufficient. Because if [abc]=0 i.e.,
(ax b)»c=0, then c is perpendicular to axb. But axb is a vector
perpendicular to the plane of a and b. Since c is perpendicular
to a X b, therefore c is parallel to the plane of a and b.
Hence a, b, c are coplanar.
(iv) Since the distributive law holds for both scalar and vector
products, it holds also for the scalar triple product.
Thus [a. b+d, c+r]=[abc]+[abr]+[adc]+[adrl. the cyclic
order of the factors being maintained in each term.

*§ 5. To express the value of the scalar triple product [abc] in.


terms of rectangular components of the vectors.
[Allahabad 1980]
Let a=aii+as5j+fl8*s/b=b,i4-bjj+b3k, c-Cji+Caj+Cak.
Now bxc=(bji+bjj-|-b8k)x,(CilH“^2j”i“^3^)
i j k
= b, ba ^8
Cl Cg Cg

= (Vs“V2)»-*CV8-" Vj) j+(V2-^aCi) k.


/, a»(bxc)=(flii+n8J+08k)*[(b2C3—bjCj)!—(bjCg—baC,)j
+(^iC2-Vi)k]
=fliCbgCa- b^^-afJ>iC^-b^c^-\-a^{biC^ -b^c,)
[V i.i=j.j=k>k^-i and i.j=j«k=k-i=0]

[abc]= bi b\ % ...(1)
Cz

Cl Cz C3
Also (axb)*c^=c*(axb)=
?
bi 0.2 bg

Oi fl g a^

bj bz

Cl c. C3

showing that the value of a scalar triple product is independent


of the positions of do: and cross.
https://www.pdfnotes.co/

MULTIPLE PRODUCTS 5

Note. If OA, OB, OC be three- concurrent edges of a para-


llelopiped and if(oi, a^, a^,{bu bg, bg), (Cj/Cj, Cg) be the rectan
gular coordinates of A, B, C referred to O as origin* then the
determinant(1) gives the volume of that parallelopiped.
§ 6. To express the Scalar triple product [a, b, c] in terms of any
three non-coplanar vectors 1/m, n. [Agra 1988]
Let a=<7il;ffl2m+fl3n,
b=hil+b2m+h,n,
and c=Cil-f-C2m+C8n.
Now b X c=(bi\-^.bgm-^bgn)x(cil-f-'c,m-fCaU)
= >< l+biCgl X m+hiCal x n+haCjm x\-\-bgC^ x m
+hgCgm X n-f* Vi“ 1+ m+V&u x n
=(Va—Va)JO X o—(Vs—Vi)“X1
+(Va—Vi)*Xm
[*.● 1X I=sp and 1X m — —m x 1 etc.]
A a«(b Xc)=(ailrfa2m4-n3*>)*[(^aC8-W nixn
—(Vs-Vi)ox l+(Vs-Vi)*xm]
=^a^{bgCg-bgCt) [Imn]—flg (Vs-Vi)
+^8 (Va—Vi) [JoJoJ-
Since [lmn]=[mnl]—[nlm] and all . the scalar triple products
of the type [Iml] in which two vectors are equal vanish.
«8

A [abc]= h bg b^ [Imn].
c.
Note. Since [i, j, k]=1, therefore § 5 is particular case of § 6.
Solved Examples
Ex. 1. Define scalar triple product of three vectors a, b, c
and interpret the same geometrically. [Meerut 1982]
Soi. Scalar triple product. Definition. The scalar products
(axb)«c, a»(bxc) are called scalar triple products of the vectors
a, b, c.
Geometrical interpretation of scalar triple product. Geometri
cally the scalar triple product (axb)*c represents the volume of
a parallelopiped whose three coterminous edges are represented
by the vectors a, b and c.
For complete discussion refer § 2.
https://www.pdfnotes.co/
6 VECTORS MADE EASY

Ex. 2. Define scalar triple product of a, h, c. Prove that the


value of the scalar triple product of a, b,c remains unchanged if
the cyclic order of the vectors is maintained i.e., prove that
a*(bxc)=b*(cxa)=c«Caxb). [Madras 1975]
Sol. For definition of scalar triple product see solved
example r above.
Let a=Uii+Ua j+fls k. i+^ j+^»8 k,
,c=Cii+C2j+Cak.
Proceeding as in § 5, we have
a*(b X c) (Vs-Va)-0%(Vi-Vi)+ Vi)
[Do it here]
^2 Oz

h bz bz

Cl Cg Cg

bi b, bz

ay Ug flg .interchanging Ry and R2


Ci Cz

by b, b.

Cl C? Cs ,interchanging R^ and R^

Oy Oy fla

=-b*(cxa). ...(1)
Again «a ^8 Cy Cg Cg

by bz bz by ' b2 bz

Cz Cz ^2 Oz
4
interchanging 7?i and i?8
Cl . ^2 Cg

Oy Oz Oz /interchanging R^ and R8

by 2 b8
=c«(axb). ...(2)
From (1) arid (2), we have
a*(b X c)=b«(c X a)=c»(a X b).
Ex, 4. Prove that^ a* b x c=a X b»c. [Meerut 1983, 86]
Sol. We have a«bxc=a»(bxc)
https://www.pdfnotes.co/

MULTIPLE PRODUCTS 7

=c»(ax b) [V scalar triple product is unchanged


if the cyclic order of the vectors is
maintained]
=(axb)*c [V dot product of two vectors is.com
mutative]
«=axb»c.
Ex. 5. Show rAa/ i.jX k=1.
Sol. We have i»j xk=i»(jxk)
=i*i [V jxk=i] ,
=1.
Ex. 6. Show that [Xa+(xb, c, d]=X[a. c, d]+^[b, c, d].
Sol. We have [Xa+{ib, c, d]
=s=(Aa+{jib)-(cxd)
=Xa»(cxd)+|ib*(€xd), by distributive law for dot
product
=X[a, c, d]+{i[b. c, d].
Ex. 7. Prove that [i-j,j-k, k-i]=0.
Sol. We have [i-j,j_k, k-i]
=(i-j)-{(j-k)x(k-i)}
=(*—j)*(jxk—jxi—kxk+kxi)
=(i-j)*(i+k+j)
=i*i-j-i*k-f-i*j—j*i—j?k—j*j
=I+0+0-0-0-1=0.
Ex. 8. Find the volume of the parallelopiped whose edges are
represented by
(i) a=2i-3j+4k, b=i+2j-k, c=3i-J+2k
(//) a~i-2j+3k,b=2i+j-k,c=j+k.
Alternative form of the question. Find the volume of the
parallelopiped whose three coterminous edges are the vectors(2, —-3,
4).{U2, and(3.-U 2), (Meerut 1991S)
Sol. (i) The required volume of the parallelopiped is equal
to the absolute value of[a be].
We have [abc]= 2 -3 4
1 2 -1

3 -1 2
https://www.pdfnotes.co/

8 VECTORS MADE EASY

=2(4-1)4-3(2+3)+4(-1-6),
expanding the determinant along
=2.34-3.54-4.(-7)=64-15-28=~7.
Neglecting the negative sign, we get the volume of the
parallelopiped=7 cubic units,
(ii) The required volumes 1 —2 3
2 1 —1

0 1 1
= 1 -2 3 >byR2-2Ri
0 5 -7

O i l
= 1.(54-7)=12 cubic units.
Ex. 9. Show Jhat the vectors I—2j-f3k, —2i4-3j—4k,
I—3j+5k are coplanar.
Sol. Let a=i-2j+3k. b=-2iH-3j-4k,c=i-3j-f-5k.
The vectors'a, b, c are coplanar if their scalar triple product
is zero, otherwise they are non-coplanar.
We h4ve[abc]= 1 —2 t 3
-2 3 -4
1 -3 5
= 1 —2 3 ,by i?2-l-2jR, and R^—R^
6-1 2
0 -2 2 !
=1.(-24-2)= 1.0=0.
Hence the given vectors are coplanar.
*Ex, 10.‘ Find the constant p such that the vectors a=2i—j-fk,
b=i4-2j—3k, c=3i-f-/>j4-5k are coplanar, [Agra 1978]
Sol. If the vectors a, b, c are coplanar, then we should
have[abc]=0.
2 -1 1

Now [abc]= 1 2-3


3 P 5
https://www.pdfnotes.co/

MULTIPLE PRODUCTS 9

=2(10+3p)+l (5+9)+l(/>-6)=7p+28.
[abc] will be zero if 7p+28=0
or p^-4.
Hence for the given vectors to be coplanar, we should have
p=-4.
**Ex. 11. Prove that the four points 4l+5j+k, -(j+k),
(3i+9j+4k)and 4(—i+j+k)are coplanar.
[Meerot 1989;90P; Kanpur 79; Delhi 77]
Sol. Let A, B, C, D be the four given points whose position
vectors referred to some origin O are
4i-l-5j+k. --(j+k),(3i+9j+4k)and 4(~i+J+k).
If the four points A, B;C,D are coplanar, then the vectors
—>●
AB, AC and AD should also be coplanar.
We have position vector of .8—position vector of A
(j+k)—(4i+5j+k)=—4i—6j—2k=a (say).

Similarly ]Jc==(3i+9j+4k)—(4i+5j+k)= —i+4j+3k


=b(say),
and ]iz)=4(-i+j+k)-(4i+5j+k)=8i-j+3k=c (say).
Now the vectors a, b, c will be coplanar if [a b c]=0.
-4 -6 -2

Now [a b c]= —1 4 3
-8-1 3
4(12+3)+6(-3+24)-2(l+32)
= -60+126-66=0.
the points A, B, d,./) are coplanar.
**Ex. 12. Show that the four points —a+4b—3c, 3a+2b—5c,
3a+8b—5c and —3a+2b+c are coplanar. [Meerut 1981]
Sol. Let A, B, C and D be the points whose position vectors
are respectively -a+4b-3c, 3a+2b-5c, -3a+8b-5c and
—3a+2b+c.
We have .<4^=position vector of B—position vector of ^4
=(3a+2b-5c)-(-a+4b-3c)=4a-2b-2c.
https://www.pdfnotes.co/

10 VECTORS MADE EASY

i4C= position vector of C—position vector of A


=(—3a+Sb'-5c)—(—a 4b —3c)=—2a+4b—2c»

and y4i)=(-3a+2b+c)-(-a-i-4b-3c)=-2a-2b+4c;

Now the scalar triple product of the vectors AB, AC and AD


4 -2 -2

=-[AB, AC, AD]=^ -2 4 -2 [a b c] [Refer § 6]


. ● -2 -2 4.
={4(16-4)+2(-8-4)-2(4+8)}[a b c]
=(48-24-24)[a b c]=0[a b c]=0.
■■ y*
Since the scalar triple product of the vectors AB, AC and AD
is zero, therefore these vectors are Coplanar. Hence the points
R, C and Z> are coplanar.
Ex. 13. If a,)), e are the position vectors of A,B, C prove that
axb+bxc+cxa is a vector perpendicular to the plane of ABC,
So!. We have 2S=b-a,jBC~c—b and cJ=a—c.
Let d=axb+bxc+cxa.
Now d* AB=A'(b-a)=(a x b+b x c+c x a)‘(b—a)
=(a X b)* b—(a x b).a+(b x c)« b—(b x c)« a+(c x a)● b
—(cxa)«a
;
i=[abb]-[aba]+[bcb]-[bca]+[cab]-[caa]
= —[bca]+[cab], since[abb]=0 etc.
=i:—[bca]+[bca], since [cab]=[bca]
=0.
Therefore vector d is perpendicular to AB. Similarly, we can
show that d is perpendicular to BC. -
Now since d is perpendicular to twoTines in the plane ABC,
hence it is perpendicular to the plane ABC,
**Ex. 14. Prove that[a+b, b+c,c+a]=2[abcj.
[Meerut 1984, 86P,88P,90; Rohilkhand 76; Agra 80]
Sol. L.H.S.=(a+b)»[(b+c)X(c+a)J
=(a+b)‘[bxc+bxa+cxc+cxal
=(a+b+[b X c+c X a+b X a],
since cxc—0
https://www.pdfnotes.co/
MULTIPLE PRODUCTS
11
=a.
(bxc)+a*(cxa)+a«(bxa)
+b.*(b X c)+b ●(c X a)+b.(b X a)
=[abc]+[.aca]+[abaj+[bbc]+fbca]+[bba]
=[alic]+[bca],
since all the scalar triple products in vwhich two vectors are equal
vanish.
But [abc]=[bca].
Hence the L.H.S.=2[abc].
**Ex. 15. Prove that
l»a l»b I-c
[]mn][abc]= m*a m*b m*c
Q*a n*b n*c
[Lucknow 1981; Meerut 82. 86; Agra 87; Rohilkhand 80J
^I. Let I=/jH-/jj+Zsk. m=OTii-fmjj+OT3k,n=n^i-j-nj+/i3k;
a-=<iii+a,j+flt3k, b=6iiH-6aj-i-6ak, c^cji+cj+cak.
Now L.H.S.=[lmn][abc]
/)? /r
8 02 Oa

= w, Wg m. bt ba
th Oa Ot <7a

^aOa~^Uoa Kbi'^hh^+l^b^
miOi-F/WaCg-f-ma^a ^\bx-\

/iin,+/fa<*2+W808 nA+nzhz+n^bs ^iCj+WaCa H-ngCg


by the rule for the multiplication of determinants of the same
order.

Now '●a=(/.i+y+/ak).(fl.l+<«d+a3k)=/A+/.o.+/^3. etc.


l «a ] *b l*c
Hence the L.H.S.= m*a m*b m*c
n*a D»b n»c
*Ex. 16. Prove that if I, m, hbe three non~coplanar vectors, then
l»a I.b I

[Imn] (axb)= m*a m*b . m


n*a n*b n
[Meerut 1982 . 91P]
https://www.pdfnotes.co/

12 VECTORS MADE EASY

Sol. Let I=/il+/aj+/3k>


D==/lii+/l2j+W8k,
and a=flii+Oaj+a8k» b=i>ji+^aj+*8k.
/j As (s

Now [lmn]= Wii OTa m3

«i ff8 «8

i j k

and (a x b)= a. «8

-^8 ^8

/i /a a i i k
[Inm](axb)= nti ma Wg fl*. 03

Wa W3 ba

mii+maj+mak mifli+maflra+'Ws^s .

Mii+ naj+Wsk WiOi+na^s+ns^'s "A+Waba+ng^a


Now l*a=:(/ii+/2j-f-/8k).(fl,i+flr2j+<*3k)“/i^3fi+/««2-i-/8^8 ®tc.
I l*a l*b I*a l;b 1

.*. [Imn](axb)— m m»a m*b = m»a m*b m

n n*a n*b n*a n*b n

Ex. 17. Show that the vectors 2a—b+3c, a+b—2c and


a+b—3c are non-coplanar where a, b, c are non-coplanar vectors,
Sol. Let A=2a—b+3c,B=a+b—2c,C=a+b—3c.
The vectors A, B and C are non-coplanar if their scalar triple
product is not equal to zero. We have
2 -1 3

[A B C]= 1 1 -2 [a b c] [See § 6]
1 1 3

={2(-3+2)-|-l (-3+2)+3j[l-l)}[abc]
-(-2-1)[a b c]=-3[a b c].
https://www.pdfnotes.co/
MULTIPLE PRODUCTS 13

Since a, b, c are non-coplanar, therefore [a b c]#0.


Hence [A B CJ^^O and so the vectors A,B, C are non-
coplanar.
Ex. 18. Prove /Ant tAc/owrpoto6a-4b-f-10c,—5a-l-3b-I0c,
4a—6b—10c and2b-}-10c are coplanar. [Meerut 1987]
Sol.
Let A,Bt C and D be the points whose position vectors
are respectively 6a-4b-f-10c, -5a-f3b-10c. 4a-6b-10c and
2b-|-10c.

We have ^B=position vectoi; of B-position vector oiA


= —5a-|-3b— 10c—6a-|-4b—10c= — 1 la-}-7b-20c,
/4C=position vector of C—position vector of A
=4a-6b-1Oc-6a-f4b-1Oc==-2a-2b-^20c,
and ^D=2b-j-I0c-6a-{-4b-10c=-6a-|-6b.
Now the scalar triple product of the vectors JS, /5[C and Jo

[iS, if,i]DJ=-ll 7 -20


-2 -2 -20 [a be]
-6 6 0
(“140—40)—6(220—40)}[a b c], expanding the deter
minant along the third tow
=-6 {-180-}-180}[a b c]=0[a b c]=0.
Sinde the scalar triple product of the vectors AB, and 2)
is zero, therefore these vectors are coplanar. Hence the points
A, B, C and D are coplanar.
Ex. 19. Find p in order that the points A (3, 2, 1), B(4, p, 5),
C(4, 2, —2)and D (6, 5, —1)maybe coplanar. [Meerut 1991]
Sol. We have position vector of B—position vector of A
=^(4-3, p-2,5-1)=(1,p-2,4),

^=(4-3,2-2, -2-l)=(l,0, -3),


and iS=(3, 3, -2).
The points A, B, C and D are coplanar if the vectors ~AB, ~AC
and AD are coplanar U.,\t[AB, ic, iS]=0 i.e., if
https://www.pdfnotes.co/
14 VECTORS MADE EASY

1 p-2 4
1 0 -3 =0

3 3 -2
or 1 p-2 7
. 1 0 0 0, applying Cg+aCj
3 3 7
or -1. .p-2 7
»0, expanding the determinant
3 7 along the second row
or -(7p-14~21)=0
or 7p—35=0 or 7p=35 or p=5.
Ex. 20. Show that the vectors a, b, c are coplanar ifh-fc,
c+a, a+b are coplanar. [Kanpur 1983]
Sol. Let A=b+c,B=c+a and C=a+b.
Then proceeding as in solved example 14, we have
[ABC]-2[abc]. [Do it here]
[a b c]«i[A B C]. ...fl)
Now if the vectors A, B and C are coplanar then [A B C]=0.
So from (1), we have[a b c]=0 which means that the.vectors a, b
and c are also coplanar.
Hence the vectors a, b, c are coplanar if b+c,c+a, a+b are
coplanar.
Ex. 21. Prove that thefour points with position vectors a, b,
c, d are coplanar if and only if ■■ ●

[b, c, d]+[c, a, d]+[a, b, d]=[a, b, c].


[Meerut 1992; Rohilkhand 90; Agra 88]
Sol. Let A, B, C and D be the four points whose position
vectors are a, b, c and d respectively.
We have y45=position vector of R—position vector of A
= b~a.
AC—c—a and AD=d—a.
Now the scalar triple product of the vectors AB, AC and AD

=[AB, A^.AD]=^»(A^>< Id)


https://www.pdfnotes.co/
MULTIPLE PRODUCTS
15

=(b-a)-{(c-a)x(d~a)}
=(b—a)'(cxd-cxa-axd4-axa)
(b-a).(cxd-cxa-axd) [V
axa=i]
= b*(cxd)-b.(cxa)-.b.(axd)-a-(cxd)+a.(cxa)
-|-a*(axd)
=[b c d]-[b c a]-[b a dj~[a c d]
[V a»(cxa)=0=a»(axd)]
=[b c dH-[c a d]+[a b d]-[a b c].
Now the points A, B, C and /) are coplanar if and only if

the vectors AB, AC and AO are coplauar


or if and only if [AB, AC,
or
if and only if [b c d]+[c a dj+[a b dj-[a b c]=0
or
if and only if [b c d]+[c a d] j-[a b d]=.[a b c].
§ 7. Vector triple product.
The vector product of two vectors one of which is itself the
vector product of two vectors is a vector quantity called a ‘●Vector
Triple Product'*. Thus if a, b and c be three vectors, the products
-of the form a x (b x c) and (a x b) x c etc. are called "Vector Triole
products’*.

♦♦Theorem.To prove that a x(b x c)=(a»c) b-(a.b) c.


[Meerut 1975, 79, 80, 84, 86P; Luc&now 81;
Gorakhpur 88; Rohilkhand 90; Allahabad 79; Delhi 80]
Letr=ax(bxc) and bxc=d.
Since bxc—d, therefore d is a vector perpendicular to the
plane containing b and c. Also r=a x d. Therefore r is a vector
perpendicular to both a and d. Now the vector r is perpendicular
to the vector d, whereas the vector d is perpendicular to the plane
containing b and c. Therefore the vector r must lie in the plane
containing b and c Hence the vector r can be expressed linearly
in terras of b and c in the form
r=/b+/wc ...(1), where / and m are scalars.
Since r is perpendicular to a. therefore r.a=0.
(/b+/Mc).a=0 or /(b.a)-f-m(c.a)=0.
/ —m
c*a b*a -A (say).
https://www.pdfnotes.co/
16 VECTORS MADE EASY

Putting the values of / and m in (1), we get


r=rX(c.a) b-X (b.a)c
=X[(c.a)b-(b.a)c]. ...(2)
Now we are to find the value of X.
Consider unit vectors j and k, the first parallel to b and the
second perpendicular to it in the plane containing b and c. Then
we may write
b=bijj
and c==C2i-i-Cj,k.
In terms of j.-and k and the other unit vector i of the right
handed systc'ji, the remaining vector a may be written as
a=nii+fl2i+03*^.
Now bxc=bjX(c2j+Csk)=V2J xj+Vai Xk
, [V jxj=0 and jxk=i}
r ==a x(b x c)=(flii+a2j+«sk)x(^2^31)
==«iV3> X i+U2*2C8i XI+fls^aCak X i
jxi=-k and kxi=j] ...(3)

Also r=X[(c.a)b—(b.a) cJ=X[(C2j4-e3k).(flii+<72i+«sk) ^2!

=X [Cafl!a^2j
-[V i.i=l, i.j=0 etc.]
=X [Ua^aCaJ— ...(4)

Now from (3) and (4) we conclude that X=l.


Hence ax(bxc)=(c.a) b—(b.a)c
=(a.c) b-(a.b) c [V c.a=a.c]
Corollary. (axb)xc=-[cx(axb)]
=-[(c.b) a—(c.a) b]
=(c.a) b—(c.b)a.
Rale to remember a x(bxc). It is a vector to be expressed
linearly in terms of b and c which are the vectors within the
brackets. Also
a X(b x c)=[Dot product of a and c] b
—[Dot product of a and b] c.
Similarly we may remember(axb)xc.
https://www.pdfnotes.co/

MULTIPLE PRODUCTS 17

§ 8., Vector triple product is not associati?e.


If.a, b, c be three vectors, then a x(b x c) gives a vector which
lies in th6 plane of b and c and which is perpendicular to a.
Moreover (a x b)x c gives a vector which lies in the plane of a
and b and which is perpendicular to c. Hence, in general,
a X(b X c)vt(a x b)x c. . Thus in the case of vector triple- product
the position of brackets cannot be, in general, changed.without
altering the value of the product.

Solved Examples

*Ex. 1. Prove./Aa/ax(bxc)+bx(cxa)+cx(axb)=0.
(Meerut 1975, 80,82,86S; Kanpm:80; Allahabad 75;
Gorakhpur 88; Delhi 81; Agra 79],
Sol. We have a x(b x c)=(a«c) b-(a^b)c,
bx(cxa)=(b.a)c-(b.c)a.
and cx(a x b)=(c«b) a—(c.a) b.
Adding these three expressions, we’get
a X(b X c)H-b X(c X a)+c x(a x b)
=(a»c) b—(a*b)c+(b*a)c—(b»c)a4-(c«b) a—(c.a) b
=0. (v a*c=c*a, a*b~b*a, b*c=c*b].
Ex. 2. Show that the vectors a x(b x c), b x(c x a), c x(a x b)
are coplanar [Kamatak 1971]
Sol. Let rj=a x(b x c), ra=b x(c xa), rg=c x(ax b).
Now first prove that ri+rj+rg-P as we have done in the
previous exercise.
Since there exists a linear relation between the vectors fg,
Tg therefore any of these ve^ors can be expressed as a linear
combination of the other two. Hence these three vectors are
. coplanar.
Ex. 3. Evaluate i)ixc)xn where a=2i+3j—5k,
and c=4i+2j+6k. [Meerut 1984]
Sol. We have (b x c)xa=(b.’a) c—(c*a) b
=[(-i+j+kH2i+3i-5k)](4i+2j+6k)
-[(4i+2j+6k).(2i+3j-5k)](-i-hj+k)
=(-2+3-5)(4i+2j+6k)^(8+6-30)(-l+j+k)
=-4(4i+2j+6k)+I6(-i+j+k)
https://www.pdfnotes.co/

18 VECTORS MADE EASY

=_16i-8j-24k-16i+16j+16k
=~32i+8j-8k=8(-4i+j-k).
Ex. 4. Verify theformula for vector triple product
ax(bxc)=(a.c) b—(a.b) c
by taking a=i+j, b=—i-+-2k,c=jfk.
Sol. Wehavebxc=(-i+2k)x(j+k)
= i j k
-1 0 2

0 1 1

=(0-2)i-(-l-O)j+(-l-0)k=-2i+j-k.
ax(bxc)=(i+j)x(-2i+j-k)
= i j k =(-l-0)i-(-l-0)j+(l+2)k
1 1 J
-2 1 -1

=-i+j+3k. ...(1)
Again (a»c) b—(a»b)c
=[(i+j)‘(j+k)](-i+2k)-[(i+j).(-i+2k)](j+k)
=(0+1+0)(-i+2k)-(-1+0+0)(j+k)
=1 (-i+2k)+(j+k)=-i+j+3k. ...(2)
From (1) and (2), we see that
ax(bxc)=(a.c) b—(a.b)c.
Ex 5. Prove that(b x c)x(c x a)=[a b c] c.
[Meerut 1983 S]
Sol. Letbxc=d.
Then (b x c)x(c x a)=d x(c x a)
=(d»a)c—(d»c)a=[(bxc)«a]c—[(bxc»c]a
=[b c a] c—[b c c] a
=[a b c] c, since,[b c c]=0 and [b c a]=[a b cj.
Hence(b x<^ X(c X a)=[a b c] c.
Ex. 6. Prove that a x(b x a)=(a x b)x a.
Sol. We have ax(bxa)=(a»a) b—(a*b) a ...(1)
https://www.pdfnotes.co/

MULTIPLE PRODUCTS 19

Again (a x b)x a=-{a x(a x b)} [V ■ uxv=—vxu]


==—{(a«b) a—(a»a)JI»}
==—(a«b)a+(a*a) b
=(a*a) b—(a«b)a» ●●(2)
From (1) and (2), we have '
a X(b X a)=--(a X b)X a.
♦Ex. 7. //a 2j+.k, b—2i+j+k,. c=i+2j—k, find
ax(bxc).
Sol. We have a x (b xc)=(a.c) b—(a.b). c
-[(i:-2j+k).(iH:2j-k)] (2i+j+k)
-[(i-2j+k).(2i+j+k)] (i+2j-k)
=(1-4-1) (2i+j-l-k)-(2-2-f 1) (i+2j-k)
=(-8i-4j-4k)-(i+2j-k)=-9i-6j-3k.
Ex. 8. Show that ix(axi)+jx(axj)+kx(axk)=2a.
[Nagpur 1973; Lucknow 78; Delhi 81;
Meerut 82, 87s, 88 P. 90 P]
Sol. We have ix(axi)=(i»i) a—(i»a) i
=a—(i»a) i [V i.i=l]
jx(axj)=(j.j)a-(j.ayj
=a-(j.a)j [V. j.j-1]
and kx(axk)=(k.k) a-(k.a) k=a-(k.a) k [v k.k=ll
Adding these three expressions, we gk
i X (a X i)+j X (a X j)+k X (a X k)
=3a—(i»a) i—(j»a) j—(k*a) k
=3a—[(a.i) i+-(a»j) j+(a.k) k] [V a*j=j»a etc.]
Now we shall show that
a=(a.i)i+(a.j)j+(a.k)k.
Let a=xi-}-j;j+zk.
Taking dot product of both sides with i, j and k'successively.
we get
x=a.i, j^=a.j, z=a*k.
a=(a.i)i+(a.j)j+(a.k)k. ,
Hence ix(axi)+jx(axj)+kx(axfc)=3a—a=2a.
♦Ex. 9. S'/low t/ra/ i x(jxk)=0. [Rohilkhand 1979]
Sol. We ^ve ix(j>^k)=ixi [V jxk=i]
[V ixi==0]
https://www.pdfnotes.co/

20 VECTORS MADE EASY

Ex. 10. Show that[a x b, b x c, c x a]=[abc]*» and express the


result by means ofdeterminants.
[Meerut 1981, 86S, 87, 88, 89P; Kanpur 79; Lucknow 81;
Gorakhpur 87; Allahabad 80]
Sol. We have [a x b, b x c, c x a]
=(a X b).[(b X c)X(c X a)].
Let us first find the value of(b x c)x(c x a).
Let bxG=d.
Then (bxc)x(cxa)=dx(cxa)
—(d»a)c—(d«c)a -
=[(bxc)«a]c~[(bxc)»c] a
=[bca]c—[bcc]a
. =[abc]c, since [bcc]=0
and [bca]=-[abc].
/. [a X b, b Xc,c X a]=(a x b)«[abc]c
=[abc](a X b)-c=:[abc][abc]
=[abc]2.
Second part. Let a=flii+a2j+a3k, b—^J+b^j+bak;
c=Cii+C2j+rak.
aj flg ^3

We have [abc]=

Cj Cg C3

i j

Again axb= ■;«8

bx 63 bj
={aji>^-bxaii H-fbifla-aiba) j+(flx&2-<»A)
1 j k

Similarly bxc= bx bg bj
Cx Cg ci
“(bgCg—bgCg) i+(Cjba—bgCa) JH-Acg—C]bg) k
https://www.pdfnotes.co/
MULTIPLE PRODUCTS 21

I j k
and 'c:<a=^ Cj ^8

^8-

=(^2^8-«8<^8)i+Kc8-«*Ci)iH^iOa-ajCg) k.
^8^8“”^2^8 ^1^3”“ ®J^8 ^1^2”"^2^1

/. [axb, bxc,cxa]= ^2^8“"^8^2 ^1^8~^X^8 ^1^2*~^2^1

^8^8 ^8^8 ^1^8 —^8^1 . ^l^3~“^2^X

Q . C8 C*8 -^8 ^8

= ^1 >^2 ^8 “ -^1 -^2 -®3

-^2 -^a ^1 ^2 ^8

where the capital letters A^, A^ etc. denote the cofactors of the
coitesponding small letters a^, a^t etc. in the determinant .
. -Oi ^2 «8

bg

Cl c^ Ca

Since [abc]®-[a x b, b x c, c x a],


2
til flj flg Ai A2 Ag

.'. bi ba b3 Bi 'Bg^

Cl . Ci ^8 c, c3
Ex. 11. Prove thatfor any three vectors A, B and C,
(AxBHBxC)x(CxA)=(A.BxC)2. [Meerut 1974, 76]
Sol. This question is the same as is the first pdrt of solved ●
example 10 above.
Ex. 12. Prove thatfor any three vectors a, b and
[axb, bxc, cxa]:s=[a b c]*. [Meerut 1986S, 87]
Hence show that the vectorjs a, b, c are non-coplanar if and
only if the vectors a x b, b x c, c x a ore non-coplanar. [Delhi 1980]
Sol. For .the solution of the first part of this question see
solved example 10 above.
Second part. As proved in the first part of this question. we
have
https://www.pdfnotes.co/
22 VECTORS MADE EASY

[abc]2=[axb, bxc, cxa].


[abc]=0 if and only if[a x b, b x c, c x a]=0
or [abcJ^tO if.and only if[a x b, b x c, c x a]^0.
Now [abc]9^,0 if and only if the vectors, a, b, c are non-cop-
lanar.
Hence the vectors a, b, c are non-coplanar if and only if the
vectors a x b, b x c, c x a are non-coplanar.
Ex. 13. Show that

[axb, bxc, cxa]= a»a * a«b a*c [abc]2.


b«a b«b b*c

c-a c*b c-c


Sol. First prove that [a x b, b x c, c x a]=[abc]2. For com
plete solution see solved example 10. ●
Now to prove the second part of the question, let
a^flTi i+^2 j+fls k. b=*i'H-b2 j-t-ha k. c=c, j+Cg k.
Tb.en [abc]=

hi hg b.

Cl Cg C3

a3 ^3

/. [abc]2= h, b3 ha hg «3

Cl ^ Cg Cg Cl C2 ^3

flihi-fi/ghg-fCghg fliCi-fCgCg-fflaCg

hifli+hgCg-FVs ‘hi^+hg^-fha^ Vi+h^jCg+hgCa

CiOi-fCgCg-fCaCa qhi-fCghg+Cgha Ci^+Cg^+Ca^


by the row-by-row multiplication rule for the product of two
determinants of the same order
a*a . a*b a*c

b*a b*b b«c

c*a c*b c*c


because a ● a=(Oii-F Cg]+^7ak)●(Oii+aJ+a^)
etc.

/
https://www.pdfnotes.co/
MULTIPLE PRODUCTS 23

From the two relations proved above we get the relation


required to be proved in 'the question.
Ex. 14. Prove that (a x b)x(a x c)- d=(a» d)[abc].
Sol. Letaxb—r.
Then (axb)x(axc)=-rx(axc)=^(r«c) a—(r«a) c
=[(axb)*c] a—[(axb)«a] c
=[abc] a-[aba]c
=[abc] a, since [aba]=0.
Therefore (a x b)x(a x c)» d=[abc]a*d
i =(a«d)[abc].
*Ex. 15. If a, h,c be' three unit vectors such that a x(b x c)=]^b
find the angles which a makes with b and c, b and c being non-
parallel. [Rajasthan.1975; Rohilkhand 78; Kanpur 86]
Sol. It is given tha'i a x(b x c)=^b.
(a»c) b—(a«b)c=^b.
(a.c—J) b—(a*b)c=0. ...(1)
Since b and c are non-parallel, therefore for the existence of
relation (1) the coefficients of b and c should vanish separately.
Therefore, we get
a*c— i.e. a*c=^ and a«b=0.
Let 6 and ^ be the Angles which a makes with b and c respec
tively. Since a, b, c are unit vectors, therefore
a*b=cos 6--^^0 e=90“.
^●c=cqs^=i .-. ^==60“.
Ex. 16. Prove that (axb)xc=ax(bxc), if and only if
(cxa)xb=0. [Meerul 1992; Rohilkhand 79; Gorakhpdr 88]
Sol. We have (axb)xc=ax(bxc)
if and only if ● (c*a) b-(c»b) a=(a»c) b—(a»b) c
i.e., if and only if —(c*b) a= —(a»b) c, sincc-c»a=a*c
i.e., if and only if (c»b) a—(a»b) c=0
Ue„ if and only if (b»c)a—(b«a)c=0, since c«b=b»c and a»b=b*a
/.e., if and only if (cx a)xb=0.
Note. (cxa)xb=0 is possible when (i) a and c are cojlinear
, because then c x a=0 or (ii) b is parallel to c x a i.e. b is perpendi
cular to both c and ai or (iii) at least one of the vectors a, b, c is a
null vector. [Meemt 1988P]
PjToducts of four vectors
§ 9. Scalar product of four vectors.
If a, b,. c, d are four vectors, the products (axb)*(cxd).
https://www.pdfnotes.co/
24 VECTORS MADE EASY

(axd)*(bxc) etc. are called scalar products of four vectors.


^Theorem. To prove that
a«c b»c
(axb).(cxd)=
a*d ● b*d
[Mysore 1971; Meerut 83,84,89P,90P; Rohilkhand 92]
Let a X b=r. Then (a x b)»(c x d)=r*(c x d).
Now in a scalar triple product the position of dot and cross
may be interchanged without, altering the value of the product.
Therefore, r*(c x d)=-(r x c)« d.
A (a X b)*(c X d)=[(a x b)x c]* d
=[(c»a) b—(c»b)a]»d
=(c»a)(b«d)—(c«b)(a»d)
=(a»c)(b»d)—(b»c)(a»d)
a*c b*c

a*d b*d
This relation is known as Lagrange*s Identity.
§ 10. Vector product of four vectors.
Let a, b, c, d be four vectors. Consider the vector product
of the vectors ax b and c xd. This product can be written as
(axb)x(cXd) and is called the vector product of four vectors.
It is a vector perpendicular to axb and, therefore coplanar with
a and b. Similarly it is a vector coplanar with c and d. Hence
~ this vector must be parallel to the line of intersection of a plane
parallel to a and b with another plane parallel to c and d.
Theorem. To prove that
fi) (a x‘b)X(c X d)==[abd]c—[abc] d
[Allahabad 1980; Gorakhpur 87]
(ii) (axb)x(cxd)=[acdj b—[bed]a.
[Lucknow 1980; Gorakhpur 88]
(a X b)X(c X d) is a vector which can be either expressed in
terms of c and d or in terms of a and b. To express it in terms
of c and d, let us put a x b=>|. Then
(a x b)x(c X d)=l x(c x d)=(I»d) c—(l*c).d
=[(axb).d]c|-[(axb).c] d=[abd]c-[abc] d.
https://www.pdfnotes.co/
MULTIPLE PRODUCTS 25

Again to express(axb)x(cxd) h terms of a and b, let us


putcxd=m. Then
(a X b)X(c X d)=.(a.x b)X m=~m X(a X b)
= —[(m*b) a—(m*a) b]=(m*a) b—(m*b)a
=[(c X d)» a] b—I(c X d)« b) a
=[cda] b-(cdb] a=[acd] b-[bcd]a.
Linear Relation connecting four vectors. Equating the above
twd expressions for the value of(axb)x(cxd), we get
fabdj c-[abc] d=r-[acdj b-[bcd] a
or
[bed] a-[acd] b+[abd]c-[a,bc] d=0. ...(1)
[Meerot 1984]
which is the required linear relation connecting the four vectors
a, b. c, d.

*To find an expression for any vector r, in space,.as a linear


combination of three non-coplanar vectors a, b, c.
[Kamatak 1971; Allahabad 77; Ranpur 88]
Replacing d by r in the relation (1)just established, we get
[berj a-[acr] b-{-[abr] c-[abcj r=0
or
[abc] r=[bcr] a-[acr] b+[abr] c.
Since a, b, c are non-coplanar, thefefore [abc]7^=0.
Therefore dividing both sides of(2) by [abc], we get
r=M «~[acr] b-f[abr] c
[abef ^
or „_fbcr] a-{-[car] b4[abr]c
[abc] ,since [acr]=—[car]
or r=frbe] a-{-[real b-ffrabl c
[abc] ...(3)
which is the required expression for r. ●
*§11. Reciprocal system of vectors.
If&,h,ebeany three non-coplanar vectors so tAo/[abc]#0
then the three vectors a', b:, c' defined by the equations
b'=£^ nV- axb
[abc]' [abc] * ® [ibc]
are called reciprocal system ofve *nrs to the vectors a. b. c.
(0 To show that =
https://www.pdfnotes.co/
26 VECTORS MADE EASY

bxc a«(bxc) [abc] ,


We have a»a'=a»
[abc]“ [abc] “[abc]’” ’ .

Similarly b»b'—b*cx^ b«(cxa) [bca] [abc] 1.


[abc]~ [abc] “[abc]“[abc]

and c»c'=- ?Ll!* c«(axb) [cab] [ale]=1.


”^ *[abc]" [abc] “[abc]“[abc]
Note. The reason for the name reciprocal lies in the rela
tions a«a'—b«b'=c»c'= 1.
*♦
(«) The scalar product of any other pair of vectors, one
from each system, is zero i.e,
a»b'=a»c'—b*a^=b«c'—c«a'=c»b'=0.
exa a»(cxa) ' [aca]
We have a*b'=a*
[abcJ~ [abc] “[^c]
=^0, since [aca]=0.
Similarly we can prove the other results.
{Hi) The scalar triple product [abc] formed from three
non-coplanar vectors a, h, c is the reciprocal of the scalar triple
product [a' b' c'] formed from the reciprocal system a', b', c' i'.e.
[abc][a'b'c']=4. [Lucknow 1977; Rohilkhand 80; Meerut 88]
We have[a' b; c']=a'.(b'x c')
, bxc exa (aXb)
“ [abc] *|[abc]^ [abc]
(bxc)»[(cxa)x(axb)]
[abej
Now expanding (c x a) x (a x b) by vector triple product trea
ting c>;a as one vector, we get
(cxa)x(axb)=[(cxa)*b] a—[(cxa)*a] b
=[cab] a—[caa] b
“[abc] a, since [caa]=0
and [cab]=[abc].
(b x c).[abc] a f(b x c)*a] [abc]
[a'bV]=^
[abep [abc]3
[bca] [abc]
[abc]®
[abcj® 1
■“[abc]®“[abc] ‘
la'b'c'j[abc]=I.
https://www.pdfnotes.co/
MULTIPLE PRODUCTS 27

Note 1. Since [abcJ^tO, therefore from the relation,


[a'b'c'][abc]= l,.we conclude that [a'b'c']9«t0.
Hence the vectors a', b', c' are also non-coplanar.
Note 2. The symmetry of results proved in properties (i),
(ii) and (iii) suggest that if a\ b', c' is the reciprocal system to
a, b, c then a, b, c is also the reciprocal system to a', b', c^
Note 3. The relation [abc][a' b' c']=I shows that the scalar
triple products [abc] and [a' b' c'] are either both positive or both
negative. Hence the two system of vectors a, b, c and a', b', c' are
either both right handed or both left handed,
(/v) The orthonormal vector triads i, j, )s.form a self recipro-'
cal system.
Let i', j', k' be the system of vectors reciprocal to the system
i, j, k.
jxk i
Then by definition i' T =-i.
[i j
Similarly j and k'=k.
Hence the result.
.§ 13. If a, b, c be three non-coplanar vectors and a', b'; c'
consitute the reciprocal system of vectors, then prove that any vector
. T can be expressed qs r=(r.a') a+(r.b') b+(r.c')c.
[Karanatak 1971]
Let r be expressed as a linear combination of the non-coplanar
vectors a, b, c in the form
r=;ra'-)-j;b-|-^c ...(1)
wnere x, y, z arc some scalars.
Multiplying both sides of(1) scalarly with bxc, we get ●
r.(b X c)=A:a.(b X c)+3;b.(b X c)-1-z.c(b X c)
=x:[abc]+7[bbc]+z[cbcj
=x [abc], since [bbc]=0=[cbc].
r.(bxc) bxc bxc
x== r» r»V, since a'=
[abc] [abc] :[abc]‘
Similarly multiplying both sides of(1) scalarly with cxa and
a X b, we can show that
;^=r»b' and z=r«c'.
’Putting the values of x, y and z in (1), we get
r—(r«aO a-l-(r.b') b+(r*c') c. ..-.(2)
https://www.pdfnotes.co/
28 VECTORS MADE EASY

Note 1. In a similar manner, we can prove that


r=(r«a) a'+(r»b)b'-f-(r.c) c'.
Note 2. Since the system of vectors I, j, k is selfrreciprocal,
therefore from(2) we conclude that
r=(r.|)M-(r.j)j+(r.k)k.

Solved Examples

Ex. 1. find a s^t ofvectors reciprocal to the set


2iH-3j—k, i—j—2k, ~H-2j+2k. [Kanpur 1980; Agra 79]
Sol. Let a=2i+3j-k, b=i-j-2k,c=-i+2j+2k.
Let a', b", c' be the set of vectors reciprocal to the set a, b, c.
Then by definition.
axb
a'-^ r>
~(al>c]’ fib^’® [abc]■
2 3-1

Now [abc]= 1 -1 -2 =2(2)-3(0)-l(l)-3

-1 2 , 2
i j k
and bxc= 1 -1 -2 2i+0j-j-k=2H-k.

-1 2 2
, cxa 2ir}-k
“■[abc]~ 3 =(3

Similarly
[abc]
i j k
^8i+3j-7k
=i -1 2 2
3
2 3-1

and
[abc]
j k !

2 3-1 -7i+3j-5k~
3
I -1 -2
https://www.pdfnotes.co/

MULTIPLE PRODUCTS 29

Ex. 2. Obtain a set of vectors reciprocal to the three vectors


-i+j+k,i-j+k,i+j+k.
Sol. Proceed as in solved example 1.
Ans.
-h i+ik. j+ik.Ji+ij.
Ex. 3. Prove the identity a x[a x(a x b)]=(a.a)(b x a).
Sol. We have ax[ax(axb)]=ax[(a.M a-(a.a) b]
=ax[(a«b) a]—ax[(a.a) b], by dist. law for cross product
=(a«b)(a Xa)—(a.a)(a x b) [V ax(iwb)=/M(ax b)]
=-(a«a)(axb) [V axa~0]
=(a»a)bxa [V axb=-(bx(a]
*Ex. 4. Prove that
aX{bx(cxd)}=(b.d)(axc)-(b.c)(aXd).
Hence expand a x[b x{c x(d x e)}].
[Kanpur 1977; Rohilkhand 76]
Sol. First part. We have
ax{bx(cxd)}=ax{(b.d)c~(b.c) d}
=(b.d)(axc)-(b.c)(axd).
Second part. We have
b X {c X(d xe)}== bX {(c»e) d—(c« d)e)
=(c.e)(fe X d)—(c« d)(b X e).
a X[b X {c X(d X e)}]=a X[(c«e)(b X d)-(c« d)(b X e)j
==(c*e)[aX(bx d)]-(c.d)[a x(bx e)]
=(c«e)[(a*d) b~(a*b) d]
~(c*d)[(a.e)b~(a.b)e].
Ex. 5. Prove /Aar d-fa x{b x(c x d)}]=(b.d)[acd].
Sol.
ax{bx(cxd)}=a x{(b.d) c-(b.c)d)
. =(b.d)(axc)-(b.c)(axd).
/. d.[ax{bx(cxd))]=d.[(b.d)(axc)~(b.c)(axd)]
=(b.d)[d*(axc)]-(b.c)[d.(axd)]
=(b«d)[dac]—(b*c)[dad]
=(b*d)[acd], since[dad]=0 and
[dac]=[acd].
Ex. 6. If the four vectors a, b, c, d are coplanar, show that
(axb)x(cxd)=0. [Andhra 1975]
https://www.pdfnotes.co/

30 VECTORS MADE EASY

Sol. ax b is a vector perpendicular to the plane containing


a and b. Similarly cxd is a vector perpendicular to the plane con
taining c and d
Since a, b, c, d are all coplanar, therefore the vectors a x b and
c X d are perpendicular to the same plane. Therefore a x b and
cxd are parallel.
Now we know that the vector product of two parallel vectors
is equal to a zero vector, therefore(a x b)x(cx d)=0. .
Ex. 7. Prove that
(a X b)X(c X d)+(a x c)x(d x b)-f(a x d)x(b x c)
=-2[bcd]a.
Sol. (a X b)X(c X d)=IX(c X d), where 1=a x b
=(l.d)c~(l-c)d
=[(a x.b).d] c-[(ax b)«c] d
=[abd] c—[abc] d ...(1)
Again (a x c)x(d x b)=(a x c)x m, where m=d x b
=(m*a) c-(m*c)a
=[(dxb)«a] c—[(dxb)*c]a
=[dba] c-[dbc] a
= —[abd]c—[bcdj a. ...(2)
since [dba]=s--[abdl, as we have changed the cyclic order of
the vectors and [dbc]=[bcd], as the cyclic order has been main
tained.
Also(axd)x(bxc)=(axd)xn, where n=bxc
=(n»a)d-(n*d) a=[(bxc).a] d-[(bxc)«d] a
~[bca] d—[bed] a=[abc] d—[bed] a. ...(3)
Adding (1),(2) and (3), we gef
(ax b)x(cxd)-j-(axc)x(dxb)+(axd)x(bxc).
=:-2[bcd]a.
Ex. 8. Prove that
[axp, bxq, cxr]-i-[axq, bxr, exp]
+[axr,.bxp, exq]=0.
[Rohilkhand 1990; Kanpur 86]
Sol. We have[a x p, b x q, c x r]
-(axp)‘[(bxq)x(cxr)]
=(axp)-i{(bxq)‘r}«-{(bxq>c) r]
=(axpH[bqr]c-[bqc]r> ●
=[apc][bqr]r-[apr][bqcj. ...(1)
https://www.pdfnotes.co/

MULTIPLE PRODUCTS 31

Again [a -< q, b x r, c x p]—[b xr, c x p, a x q]


=(b,xr).[(cxp)x(axq)]
=(b X r).{[(c X p).q] a~[(cx p)»a]q}
=[bra][cpq]-[brq][cpa]. ...(2)
And [axr, bxp, cxq]=[cxq, axr, bxp]
=(c>^q)*[(a-xr)x(bxp)]
==(cxqH[(axr).p] b—[(axr)*b] p}
=[cqb][arp]-[cqp][arlS]., ...(3)'
Adding (1),(2) and (3), we get
axp, bxq, cxr]4-[axq, bxr, cxp]+[axr, bxp, cxq]
=fapcl[bqr]~[apr][bqc]+[braj[cpq]—[brq][cpa]
+[cqb][arp]-[cqpj[arb]
=[apc][bqrlr-[apr][bqc]-j-[bra][cpqj—[bqr][ape]
+[bqc][apr]-[cpq][bra]
=0,since [brq]=—[bqr],[cpa]=—[ape]etc.
Ex. 9. Prove that
[a X b, c X d, e X f]=[abd][eef]-[abc][def]
=[abe][fcd]-[abf][ecd]
=[cda][bef]—[cdb][aef].
[Rohilkband 1992]
Sol. We have [axb,cxd, eXf]
=(axb).[(exd)x(exf)]
=(axb)»[I X(exf)], where I=cxd
=(axb).[(l.f)e-(l.e)f]
=(axb).[{(cxd).f} e-{(exd).e} f]
=[cdf][abe]-[cde][abfj
=[abe][fed]—[abf][ecd], since [cdf]==[fcd] etc.
Again[axb,cxd,exf]=[cxd,exf, axb]"
=(cxd).[(exf)x(axb)]
=(cxd).[{(exf).b} a-{(exf).a) b]
=[cda][efb]-fcdb][efa]
=[cda].[bef]-[cdb][aef].
And [axb, cxd,exf]=[exf, axb,cxd]
=(e X f).[(a X b)X(c X d)]
---(eXf).[{(aXb).d} c-((ax b).c) d]
-[efc][abd]-[efd][abc]
=[abd][cef]-[abc][def].
Ex. 10. Prove that
(b X c).(a X d)+(c x a),(b x d)+(a x b).(c x d)=0.
[Meerut 1969; Gorakhpur 87; Rohilkband 77; Delhi 80]
https://www.pdfnotes.co/

32 VECTORS MADE EASY

Sol. We have
b*a b*d
(bxc)*(axd) =(b.a)(c.d)-(c.a)(b.d>
c*a c«d
...(1)
c»b c»d
Similarly(c x a)»(b x d)=
a«b a*d
=(c*b)(a.d)—(a»b)(c«d) ...(2)
a«c a*d
and ■ (axb)«(cxd)=
b*c b*d
=(a*c)(b«d)—(b«c)(a.d).
Adding (1),(2) and (3), we get
(bxc)»(axd)+(cxa)«(bxd)+(axb)»(cxd)=0
since a»b= b»a etc.
Ex. 11. Establish the identity
[a b c] d=:[b c d] a+[c a d] b+[a b dj c
for anyfour vectors a, b, c, d. [Bardwan 1975}
Hence show that any vector t can always be expressed as a
linear combination of three non-coplanar vectors.
Sol. For complete solution of this question refer § 10.
Equating the two expressions for the value of
(a X b)X(c X d), we get
[a b d]c—[a b [c d=[a c d] b—[b c d] a
or [a b c] d=[b c d]a—[a c d] b+[a b d]c
=[b c d] a+[c a d] b+[a b d]c. ...(1)
Now let a, b, c be three given non-coplanar vectors and r. be
any vector. Then [a b c]#0. Replacing d by r in (1), we get
[a b c]r=[b c r]a+[c a r] b-j-[a b r]c
or r-tLlf] b+tlll c
^“[a b c] ■*"[a b c] ^[a b c]
which is the required expression for r as a linear combination of
three non-coplanar vectors a, b, c.
Ex. 12. If Si, h, c be a set of non-coplanar vectors and
axb
[ a b c]' [abc]' [a be]
then prove that
b'xc' ^ c'xa' a'xb'
a
[a' b' c']' * [a' b' c']‘
[Meerut 1987 S]
https://www.pdfnotes.co/
MULTIPLE PRODUCTS 33

Sol. First prove that [a b c][a' b' c'l= 1.


[For its complete solution see § 11, part (iii)]
cxa
Now b'x c"= X axb (cxa)x(axb) ●
[a b c]' [a b c] [a b c]2
{(c X a).b} a—{(c x a). a}b
[aTcp
[c a b] a—[c a a] b
[a b cY
[a b c] a
, since [e a aJ^^O
[a b cY
a
“[a b c]
b'xc'
/. a-[abc](b'xc')=
[a' b' c'l*
[V [abc][a'b'c']=l].
c'xa' , a'xb'
Similarly prove that b=
fa^b' c'] ®~[a' b' c']*
Ex. 13. If a, b, c and a', b', c' are reciprocal system, of
vectors, prove that
(/) axa'-fbxb'+cxc'=0.
[Rohilkhand 1990; Meerot 89; Agra 87]
a+b+c
(1*0 a'xb'+b'xc'+c'xa'=
[a b c]' [Agra 1988]
and {iii) a.a'+b»b'+c.c'=3. [Rohilkhand 1979]
Sol. Since a, b, c and a", b', c' are reciprocal system of
vectors, therefore
bxc
a'
^ [a b c]* ~[a Jb c]' ® “[a b c]'
(i) We Have axa'+bx b'+c xc'
a x(bxc)+bx(cxa)+cx(axb)
[a b c]
(a»c)b—(a«b)c+(b»a)c—(b«c)a+(c«b)a—(c«a)b
[a b c]
1
0[v a.c=c.aetc.]
[a b c]
0.
bxc cxa (b X c)X(c X a)
(ii) Wehavea'Xb'-
[a b c]^[a b c]” [abcp
{(bxc).a)c—{(b c)»c> a [b c a]c—[b c cl a
[a b c]- [a b c]3
https://www.pdfnotes.co/
.34 VECTORS MADE EASY

[a b c] c
2 > since [b c a]=[a b c] and [b c c]=0
[a b c]
I
[abe]
Similarly we can show that
1 1
b'vc a and c's a' b.
[a be] [a be]
1
(a+b+c).
a'xb'+b'.e'H-e'>a-^j
(iii) We have a»a'+b*b'+e»e'
b>;e , ^ exa , _ axb

a.(bxe) b»(exa) e.(axb)


[a b e] [a b e] [a b e]
[a b e]4[b e a]+[e a b]
[a b e]
3[a,fa e]
since [a b e]=[b c a]=[c a b]
[a b e]'
=3.
https://www.pdfnotes.co/

2
Differentiation and Integration
of Vectors
§ 1. Vector Function.
We know that a scalar quantity possesses only magnitude and
has no concern with direction. A single real number gives us a
complete representation of a scalar quantity. Thus a scalar quan>
tity is nothing but a real number.
Let Z> be any subset of the set of all real numbers. If to
each element f of Z), wc associate by some rule a unique real
number/(/), then this rule defines a scalar function of the scalar
variable/. Here/(O is a scalar quantity and thus/is a s.calar
function.
In a similar manner we define a vector function.
Let D be any subset of the set of all real numbers. If to each
element t ofD, we associate by some rule a unique vector t{t), then
this rule defines a vector function of the scalar variable t. Here f(/)
is a vector quantity and.thus f is a vectorfunction.
We know that every vector can be uniquely expressed as a
linear combination of three fixed non-coplanar vectors. Therefore
we may write
f(0=/i(01+/2(0i+Zs(/)k
where i, j, k denote a fixed right handed triad of three mutually
perpendicular non-coplanar unit vectors.
§ 2. Scalar fields and vector fields.
If to each point P (a:, y, 2) of a region R in space there cor
responds a unique scalarf{P), then/is called ascalar pointfunction
and we say that a scalar field /has been defined in R.
Examples: (I) The temperature at any point within or on
the surface of earth at a certain time defines a scalar field.
(2) /(a-, y, r>=.T“ y3_3^8 defines a scalar field.
If to each point P{x,y. z) of a region R in space there
corresponds a unique vector f(/*), then f is called a vector point
function-and we say that a vector field f has been defined in R.
https://www.pdfnotes.co/
36 VECTORS MADE EASY

Examples. (1) If the velocity at any point (x, y,z) of a


particle moving in a curve is known at a certain time, then a
vector field is defined.
(2) '{{x, y, z)=xy^ i+3y^}—2x^zk defines a vector field.
§ 3. Limit and Continuity of a vector function.
Definition 1. A vectorfunction f(0 is said to tend to a limit I,
when t tends to iffor any,given positive number e, however small,
there corresponds a. positive number 8 such that
| f(0-i|<€
whenever 0< I r-/o 1 < S.
If f(0 tends to a limit 1 as t tends to t^, we write
lim

Definition 2. A vector function f{t) is said to be continuous


for a value t^of t if
(i) f{t^ is defined and
(«) for any given positive number e, however small, there
corresponds a positive number S such that
I f(0-f(to)! < «
whenever 1 t—to I < 8.
Further a vectorfunction f(t) is said to be continuous if it is
continuousfor every value of tfor which it has been defined.
We shall give here (without proof) some important results
about the limits and continuity of a vector function.
Theorem 1. The necessary and sufficient condition for a vector
function f(t) to be continuous at t=to is that

, Theorem 2. // f(0=/i(0 H/a(t) j+/a(0 k. t^en f(t) is


continuous if and only iff^ (t),f^ {t).f^(t) are continuous.
Theorems. Ler f(f)=/i(0 i+/a(0 j+ZaCO k
and ● I=/,/+/aj+/3k.
lim
Then the necessary and sufficient conditions that f(0--i
/-►to
lim lim lim
are /a(t) = /a and /3(0=/3.
/-►to
Theorem 4. ... Iff{t), g(t) are vector functions of scalar variable
t and <t>(t) is a scalar function of scalar variable t, then
lim lim lim
(0 [f(0±g(01= f(t)± g(t)-
t->to /“►to /-►to
https://www.pdfnotes.co/

DIFFERENTIATION AND INTEGRATION OF VECTORS 37

lim

m g(0
Urn
(iv) im f(Oj- 'Urn ,, .“I Hm
m
lim lim
(V)
/-►/o
HO = HO .
§ 4. Derivative of a vector Unction with respect to a scalar.
Definition. Let r -^.f{t) be a vector function of the scalar
variable t. We define r+8r= f (/rf-80-
8r=f<t+8/)-f(f).
Consider the vector^J-JJl±Mhl^,
S/ Bt
lim Sr lim
V S/-»0 8?""S/-».0 f (t-\-BO~f(t) ; , , , ^ ,
^exists, then the value of this
I f

limit, which we shall denote is called the derivative of thevec~


tor function r with, respect to the scalar t. Symbolically
dt Hm, (r^Sr)—r lim f(/+80—f(0
dt 8/^0 8/ ~8t->0 Bt , *
dr
If
dt exists, then r is said to be difierentiable. Since ^ is a

vector quantity, therefore, ~ is also a vector quantity.


Successive Derivatives. If r is a vector function of the scalar
dt.
variable t, then ^ is' also in general a vector function of 7’ If dt IS

differentiable, then its derivative is denoted by dt^.


~ and is called tjie
JQ

second derivative of r. Similarly the derivative of ~ is denoted by


dh
and is called the third derivative of r and so on.
dr d^r , . ..
are also represented by r,r,...respectively.
§.5. Differentiation Formulae.
Theorem. If &,h and c are differentiable vector functions of
a scalar t and ^ is a differentiable scalar function of the same vari^
able t, then
https://www.pdfnotes.co/

38 VECTORS MADE EASY

d2i ^
1. ^,(a+b)=
_ d ; .. dh da ,
2. 3-(a.b).a.3^+3j..b [Gorakhpur 1982]
_ d/ dh da .
3. j.(axb)-..ax^+^xb [Agra 1982; Garhwal 81]
. d . ,da d<f> a
4. sWa)-=#s-+rff Garhvral 1983]

a bc
r ^a
u.., dh *1 r , del
5- ir + “ a* + '"’a

J,{ax(b>^4
«● J. x(bx c)+ax xc)+ax(bx0
[Garhwal 1985; Rohiikhand 78]
d litn ((a-]-8a)-^(b-f-8b)]--(a4-b)
Proof, 1. dt
-X (a+b)= 8/
(
lim 8a+Sb ^ lim /Sa , 8b \
8/—►© 8/ 8/“^0
lim ^ lim 8b ^
"'8r^08r “*'8/-^0 8f ""dt ' dt '
Thus the derivative of the sum of two vectors is equal to the
sum of their derivatives, as it is also in Scalar Calculus,
c’ .t t . d . da dh
S imilarly we can prove that ^ '
^ u\ lim (a+8a)*(b+8b)—a-b
'8/-^0 Ft
lim a»b4-a»8b+-8a»b4'8a»8b—a»b
8z—>-0 St
lim a«8b+8a«b+8a»8b I
8r-»0 St
lim 8b ?a , . 8a „
8t-0 “●87+sT''’+87‘®'’
lim 8b lim 8a lim 8a
a* ● 8b
""8r->0 St +8r->0 8/ 8/h^O Si
dh da . , da ^ . ..
=a» dt
^ + dt
^*b f dt since 8b-^zero vector as
8f-»-0
= a* dh da ^

Note. We know that a»b-b»a. Therefore while evaluating

^(a .b), we should not bother about the order of the factors.
dt
https://www.pdfnotes.co/

DIFFERENTIATION AND INTEGRATION.OF VECTORS 39

d (a+Sa)y(b+8b)-avb
3.
$/
lira a y b-i- a > Sb+8a x b+8a x Sb—a 'b
S/— SI
lira a X 8b-f-8a x b+Sa x 8b
Sf
lim Sa
Sl-»-0 f-'
sF^^'Si X b-|-^
a x —4-
lim 8a
SI
X Sb
I
Sl->0 ^ xb+Sl^OSl
Sl “^Sl-*-0 SI
““ ^ySb
*
db da elk ^ ^ ^
~ ^ ^ A 8b-^zero vector as 8l-^0
i.:- a X
db da ^
A<b+0 ^aX
</i+rfi
Note. We know that cross product of two, vectors is not
commutative because a xb=—bx a. Therefore'while evaluating
d ■
dt(a X b), we must maintain the order of the factors a and b.
4 (<^+H)(a+Sa)-<^a

_ lim ^a-f-^Sa-4~S0a-{-S^Sa—<^a lini ^Sa-{-S^a-)-S^Sa


“Sl->0 "Si Sl-*-0 Si
lim - Sa S(j) S^ ^ \
“SI-.-0 ^87+«7*+s7*7
lim , Sa lira S^ lim ^ Sa
~Si->0 ^ Si Si-^0 Si Si-»0 Si

0, since Sa-i-zero*vector as Sl-»-0


dt
, da d<f> , j da dA
='^3r+^»+®“'^3r+ff“-
Note, ^a is the multiplication- of a vector by a scalar. In the
case of such multiplication we usually write the scalar in the first
position and the vector in the second position,
d
5. -r ra Kbc
dt dt a.(bx e)}
d da
=a.^(b c)-f dt .(bxc) [by rule (2)]

7. dc db
[by rule (3)]
xc)+^.(b c)
https://www.pdfnotes.co/
40 VECTORS MADE EASY

=a«

= ab
di
r</a

Note. Here [a b c] is the scalar triple product of three vectors


a, b and c. Therefore while evaluating [a b c] we must maintain
the cyclic order of each factor.
dfX
dt {ax(bxc)}=ax^(bxc)i-dt X(b X c)
6. ~ [byrule(3)J

=ax

=ax
gxc)+ax(bx|)+|x(bxc)
da
=^x(bxc)+ax gxc)+ax(bx^).

§ 6. Derivative of a function of a function.


Suppose r is a differentiable vector function of a scalar variable
s and ^ is a differentiable scalar function of another scalar variable
t. Then r is a function of t.
● An increment 8t in t produces an increment 8r in r and an
increment in s. When 8r->-0, Sr->*0 and 8j-»-0.
We have lim /8s
we nave 8t“8t^O U'r 8sf
_/ lim 8f\ / lim ^
" \8r->0 8t/ \8/->0 8sJ~dt ds'
Note. We can also write $. But it should be clear
dt ds dt
dt. ds
that ^ is a vector quantity and ^ is a scalar quantity. Thus^ ^

is nothing but the multiplication of the vector ^ by the scalar


§ 7. Derivative of a constant vector.
A vector is said to be constant only if both its magnitude and
direction are fixed. If either of these changes then the vector
will change and thus it will not be constant.
https://www.pdfnotes.co/

DIFFEkENTIATION AND INTEGRATION OF VECTORS 41

Let r be a constant vector function of the scalar variable /.


Let r=c, where c is a constant vector. Then
.*. Sr=0(zero vector).

** 8/-S/ "●
. lim Sr lim „ ^
dr
** dt 0 (zero vector).
Thus the derivative of a constant vector is equal to the null
vector.
§ 8. Derivative of a vector function in terms of its components.
Let r be a vector function of the scalar variable /.
Let r=^H-_jJ-|-i:k where the components x, y, z arc scalar
functions of the scalar variable t and i, j, k are fixed unit vectors
We have r+Sr=(x+Sx) i+(y+8y) j+(r+Sz) k.
Sr=(r+Sr)-r=Sx i+S;; j+Sz k.

*● Bt'"8t *'^Bt
lim Sr hm . S;» Sz
S/-^0 S/ St-*-0 St ~^St St

●● dt~dt '^dt^-^dt^'
Thus in order to differentiate a vector we should differentiate -
its components.
Note. If r=xi+>»j-f-2k, then sometimes we also write it as
r=(x, y, z). Id this notation
d^x d^y dh
dr\dt‘ dt‘ dt)! It^' df^' df^}■ and. so on.
Alternative Method.
We have r=xi+^+zk, where i, j, k are constant vectors and
so their derivatives will be zero.

5=5 (*‘+^i+^k)=| (ATi)+ i (yi)+ (rk)


-<* ‘+*5+5J+'>^5+5 '‘+^5
dx . dy . dz di
^di dt vanish.
$ 9>. Some important results.
Theorem 1. The necessary and sufficient condition for the
https://www.pdfnotes.co/

42 VECTORS MADE EASY

</a
vector function a (/) to be constant is that

Proof. The condition is necessary. Let a(/) be a constant


vector function of the scalar variable t. Then a(/+S/)"a(0* We
dsi Urn a (r+8/)-a(0 _ lim 0^
^ St-^O Bt "S/-^0 Bt '
Therefore the condition is necessary.
da
The condition is sufficient. Let 0. Then to prove that
dt
a is a constant vector. Let a(/)~ai(/) i-^-a^it) j+fla(0 Then
i i-L^s k
dt dt ^dt ‘^dt

Therefore ^ 0 gives. ●+p i+p'^=^^-\


Equating to zero the coefficients of i, j and k, wi get
da^ ' dujt
-0 .
==0 , -0,
dt dt dt
Hence Ox, a^, are constant scalars i.e. they are independent
of t. Therefore a(/) is a constant vector function.
Theorem 2. If a is a differentiable vector function of the scalar
variable t and if | a | =a, then
d , ^ da da da
(0
A’"'"' “"a- j
Proof, (i) We have a^—a«a=(a.) (a) cos 0!-a'.

Therefore (a^)=2a
da da da
(ii) We have (a->= (a ● a) Tt-‘+^-m-^-Tt-
Also

^ da ^ da da da

Theorems. If a has constant length {fixed magnitude), then'


da
a and -r #0.
dt are perpendicular provided
Proof. Let 1 a 1 = a—constant. Then a»a T=constant.
da . ^a
j,(a-a)=0. Of -.a+a.j-^O
da
or 2a-^ =0 or a-$=0.
dt dt
https://www.pdfnotes.co/

DIFFERENTIATION AND INTEGRATION OF VECTORS 43

dSL .
Thus the scalar product of two vectors a and dt IS zero.
da da
Therefore a is perpendicular to dt
-j- provided -y
dt is not null vector
da
i e., provided — 9^0.

Thus the derivative ofa vector of constant length is perpendi


cular to the vector provided the vector itself is not constant.
Theorem 4. The necessary and sufficient condition for the

vector a(0 to have constant magnitude is a»^-^^0.


dt
[Rohilkhand 1992; Allahabad 80; Kanpur 78; Agrac82, 85]^
Proof. Let a be a vector function of the scalar variable t.
Let I a I =fl=constant. Then a.a=a®=constant.
d , . ^ da da

da da
or 2a. or
dt'
Therefore the condition is necessary.
» </a
Condition is sufficient. If a*-?- -0, then
dt
da da _

or l(a.a)=0
or a*a—constant
or a2==constant
or a*=constant
or I a I =constant.
Theorem 5. If a is a differentiable vectorfunction of the scalar
variable t, then
d ^d^a
.-●=ax
dt dt^ [Agra 1976]
d i/a\ da da d^a
Proof. We have -r
dt
d-a
= 0“|-a X
dt^ , since the cross product of two equal
da .
vectors -r.is zero
dt
d^a
a ;<
dt^'
https://www.pdfnotes.co/
44 VECTORS MADE EASY

Theorem 6. The necessary and suffiicient condition for the


vector a(r) to have constant direction is
da g.
8Xgj-=0.
[Meerut 199i; Rohilkhand 90; Allahabad 82; Ravishankar 82]
Proof. Let a be vector function of the scalar variable/. Let
A be a unit vector in the direction of a. If a be the magnitude of a»
then a=flA.
da dA , da A.

dA da . , rfA , da AXA
Hence ax-
W+di*-)-
dA
Hi [V AxA=^0]
...0)
The condition is necessary. Suppose a has a constant direc
tion. Then A is a constant vector because it has constant direction
dA
as well as constant magnitude. Therefore -7r-=0.
dt
da
From (1), we get a x ^=u*A X 0^=0.
Therefore the condition is necessary.
The condition is sufiScient.
da
Suppose that a x dt ::0.
dA
Then from (1), we get a*Ax
dA
OI AX-r-=0.
dt ...(2)
Since A is of..constant length, therefore
A.^=0.
dt ...(3)

From (2) and (3), we get dt


Hence A is a constant vector te. the direction of a is constant.
§ 10. Corves in space.
A curve in a three dimensional Euclidean space may be
regarded as the intersection of two surfaces represented by two
equations of the form Fi {X, y, z)=0. Fa (:*. y, r)=0.
It can be easily seen that the parametric equations of the form
x=-fx (/).F=/2(0> 2=/3(0»
https://www.pdfnotes.co/

DIFFERENTIATION AND INTEGRATION OF VECTORS 45

where x, z are scalar functions of the scalar t, also represents a


curve in three dimensional space. Here (x, y, z) are coordinates of
a current point of the curve. The scalar variable t may range
over a set of values a < / -
In vector notation an equation of the form r=f(t), represents
a curve in three dimensional space if r is the position vector of a
current point on the curve. As t changes, r will give position
vectors of different points on the curve. The vector f(t) can be
expressed as /i(0i+/2(0j+/s(0k.
Also if(x, y, z) are the coordinates of a current point on the
curve whose position vector is r, then r-jci+yj+zk.
Therefore the single vector equation r=f(r)
i.e., xi+yi+zk=M)i+A(0 j+/3(0 k
is equivalent to the three parametric equations
x=f,{t).y=^M).z^M).
The vector equation r==a cos t\+b sin Ij+Ok
represents an ellipse, as for different values of t, the end point of
r describes an ellipse.
Similarly r=a/2 i-j-2a/ j+Ok is the vector equation, of a
parabola.

Geometrical significance of

Let r==f(r) be the vector equation of a curve in space, Ut


r and r+5r be the position
vectors of two neighbouring
points P and Q on this curve.
Thus wc have

OP=.t^f{t)
and OQ=r+Sr=^f(r+S0.
A PQ=r.OQ-OP
=(r+Sr)-r
—8r,
8r.
Thus — is a vector parallel to the chord PQ.

As Q-^P i.e. as 8t->-0, chord P0->-tangent at P to the curve.


. lim 8r rfr .
* * 8/->0 ^ to the tangent at P to
the curve r- -f(/).
https://www.pdfnotes.co/
46 VECTORS MADE EASY

Unit tangent vector to a curve. [Allahabad 1979]


Suppose in place of the scalar parameter /, we take the para
meter as j where j denotes the arc length measured along the
curve from any convenient fixed point C on the curve. Thus arc
CP-5 and arc Cfi -J+Sj.
In this casewill be a vector along the tangent at P to the
as
curve and in the direction of ^ increasing. Also we have
dt lim 8r .lim |8r| lim chord PQ _,
ds Ts "^Q-^P arc PQ^Q-^P arc

Thus$ is a unit vector along the tangent at P in the direction


ds
of s increasing. We denote it by t.

§11. Velocity and Acceleration.


If the scalar variable t be the time and r be the position vector
of a moving particle P with respect to the origin O, th6n Sr is the
displacement of the particle in time S/.
The vector is the average velocity of the particle during the
Sr
interval 8/. If v represents the velocity vector of the particle at P,
lim
then v=
S/“^0 8/ dt

Since ^ is a vector along the tangent at P to the curye in


dt
which the particle is moving, therefore the direction of velocity
is along the tangent.
If Sv be the change in the velocity v during the time Sr, then
8v .
3- IS the average acceleration during that interval. If a represents
Sr
the acceleration of the particle at time r. then

^ 'Bt-^OBt~dt~dt\dtJ~dt^'
Solved Examples
Ex. 1. If r =.(r-H) i-f(r» + r+l) i+(r»-f k find
(it , d-T
-r.
(if jp-
Sol. Since i, j, k are constant vectors, therefore
di _rfk
dt d't ~dt ■
https://www.pdfnotes.co/
DIFFEREMTIATION AND INTEGRATION OF VECTORS 47

7,^J,('+ ‘'-^7,('*+'+')i+7,C‘+'‘+‘+1) k
=i+(2/+l)j^t3/H2r+l)k.

Again,

=0+2j+(6/+2)k=2j+(6f+2) k.
Ex. 2. //T=sitt/i+ms./j+/k.fi/ic/
f/2r
(0 T.. (*0 ^/2r[
<//2.(«0 ^.(/V) [Agra 1978]
Sol. Since i, j, k are constant vectors, therefore ^=0 etc.
eft
Therefore
dx d ,. ^. d , d
(i)
j,=y,(sm 0,+j^(cos0j+j,(0 k=COS / i—sin t j+k.
(ii) *r d (dx\ d, . d ,. ,. dk
0 i+g;-
=^- — sin / i—COS t j+0=—sin t i—cos t j.
dr
(ili) ~ -V[(cos r)M-(-sin 0H(1)*]=V2.
dh
(iv) \/[(—sin 0‘‘+{—cos r)2]=l.
Ex. 3.' //r=(co5ittt) l-\-{sin nt)j, where n is a constant and t
dr
variest show that r ;< -nk
dt
Sol. We have
dr d . ., d
dt=7,(cos nt) (sin nt) n sm nt i+n cos nt I
^ dr
*’''^^“-(cos nt i fsin nt j) <(—n sin nt i+n cos nt j)
=—n cos «/ sin nt i x i+/i cos^ nt i x j
—n sin* n/ jX i+n cos nt sin nt jx j
=n cos- nt k-T-n sin- nt k
[V i i-0,jxj=0, i j==k, j i=-k]
-^n (cos* «/+sin* nt) k nk.
Ex. 4. If a,h are constant' vectors, co is a constant, and r is a
vectorfunction of the scalar variable t given by
r . o.v f,)/a+jm o/b.
show that
https://www.pdfnotes.co/
48 VECTORS MADE EASY

... dh dt
(i) dt^-f-to®r=0, and (ii) rX dt=«axb.

[Rohilkhand 1984; Agra 81; Kumayon 82; Madras 83]


Sol. Since a, b are constant vectors, therefore
da. dh ^ '
=0,
dt dt

(cos a+ (sin 6)0 b

=—0)sin 6)/a+6> cos 6>( b.


—6)* cos 6)/ a—o>® sin «< b
●* dt^
6)' (cos at a+sin 6)/ b)= -e>2r.
dh
3j5+«»r=0.

(») sin at a+6) cos at b)


rx^=(cos at a+sin at b)x(—6>
— a cos^ at axb—6) sin^ <ar bx a [V axa=0, bxb=0]
6) cos^ at ax b+o) sin® 6>/ a x b

=a (cos® 6>r4-sin® at) axh—aaxb.


Ex. 5. Ifr=‘{sinh t) a-\-{cosh t) b, where a and b are constant
dh
vectors, then show that ^='-
Sol. Since a. b are constant vectors, therefore
da \ dh'
=0.
di'^’ dt

=(cosh /) a+(sinh t) b.

^=(sinh 0 a+(cosh 0 *>=r.


dr
Ex. 6. 7/r=r® - 2t»- i.) j, show that rXT-=k.
[Utkal 1973]

Sol. We have r
=.rM+(2r»-i5)j.
dt
^ =3t® 14
●● dt ^3)1-
dt
rX r® i+
dt ~
https://www.pdfnotes.co/

DIFFERENTIATION AND INTEGRATION OF VECTORS 49

='● ( **+p) fxl


[V txi^0,jxj=0]
=(&«+?)k+(6l--|)(-k)
IIV lxj=k,jxi=-k]
=(6/»+?-6<»+5)k=k.
Ex. 7. Ijf a+e^» b, where a, b are constant vectors.
d»t
show that -»»r=0.
[Agra 1976]
Sol. Given T=e”‘ a+e"« b, .(I )
where a, b are constant vectors.

s=[s^] ®+*"' ^+[j< (*'"'>]


da dh
a—ne:”* b.
V di~^~di' ^ ^ being
constant vectors
Again differentiating with respect to t, we get
dh
^=5«* a+/i* e~** b=sn* (c«* a+e~« b)=/i* r, from (1).
. dh —«* r=0.
df*
et
Ex. 8. If r=a sin <oI+b cos sin u>/, prove that
d^
dt^ -f-©2r=—a cos at,
where a, h, c are constant vectors and a is a constant scalar,
[Meernt 1991, Marathwada 74]
Sol. Given r;=a sin 01+ h cos sin at. ...(I)
where a, b, c are constent vectors and o is a constant scalar.
● *
●* dt = a « cos 6>r^b CO sin 6)ir+^
a^ sin '—5
a^ ta cos at
dh C
and
3T*= —a»® sin o/—b o>* cos <o/H—5 <0 cos at
C c
a cos t a^ sin at

cos at
=!— ^a sin ot+b cos sin 6>r j+—
https://www.pdfnotes.co/

50 VECTORS MADE EASY

2c
— rH— cos at,from (1).
d*t 2c
● r——,cos at,
at^ a
Ex. 9. Show that r=a c»'+b where a and b are the cons
tant vectors^ is the solution of the differential equation

^-{m+n)^+m«r;=0.
Hence solve the equation
d^r *
—2r=0,where
dt*~~dt
dr
r=i and -^—yfor t=0. [Kanpur 1977]
Sol. We have r=a e«*+b ...(1)
where a and b are constant vectors,
a mc”*+b n^ (2)
*● dt
and w* e«*+b n* c"* ...(3)
From (1), (2) and (3). we get

=3am* b n® e"*—(m+n) [a mc“*+b ne"*]


(m*—m®—iwn+wn) a+e"* (n®— b
=0a-f-0b=P-}“0=0,
Hence r=a c"*+b e^ is the solution of the differential equa-
tion
dh dt
dr* -(m-h«) r=0. ...(4)
Putting m=2 and »i=-l in (1) and (4), we see that the
general solution of the differential equation
^ *-2r=0
dt^~dt ” ...(5)
is r=a e®'+b ...(6)
where a and b are arbitrary constant vectors.
dr
From (6), ^=a 2c®*-b er*. (7)
dr
But it is given that for t=0, r=i and ^=j*
/. from (6) and (7), we have
a+b=i -(8).
https://www.pdfnotes.co/
DIFFERENTIATION AND INTEGRATION OF VECTORS > 51
and .2a-b=j. ...(9)
Adding (8) and (9), we get 3a=i+j or a=f(i+j).
Now from (8), we have b=i—a=i—J (i-fj)t=R

Putting a--=^(i+j)and b=| *“*5 j in (6). the required solu


tion of the differential equation (5) under the given conditions is
er*
r=|(i+j)e»+(? l-ij)
or r=i(«»<+2«->) i+1(««-«-< j.
Ex. 10. Prove thefollowing;
d r dh dsL .T ■rf*b rf®a
(0 “*‘55
●b.
iPh d^a

Sol. (i) Wol^vo|[8"-*.b


d r </b\ d (da .V

=*-5?+“-a5-a5*'’-5.'3F
d^h d^a
=a*
dti
..V d ^b da ■ ”
(■') sl»xy-3j-xb|
d
= dt
da db d ldh\ fd (da\' xb— da dh
W^di'^^^7t\di}-[7t{^)_ 5-X3F
= a X d^h </*a ^
S5-*3Xb-
Ex. Hi If r=r2 i_/ j^(2t+l)X Jindat t=0, the values of
(h dh dr I jd‘r
dt*dt^* 'dty\di^ '
Sol. r=/2i_^ j+(2t+l)k.
V
●●● ...(1)
dh
and =2i ...(2)
dt^
https://www.pdfnotes.co/

52 VECTORS MADE EASY

From (1) and (2), we have


dt
...(3)
dt =V[(20H(-1)*+2«]=5 V(4<*+5)
dh
.and ...(4)
dt» =|2i|=2.
Putting r=0 in (1),(2).(3) and (4), we have at /=0,
dr dh A. dr .. d^r
d/=-j+2k. =21, =V5. =2.
Ex. 12. Ifu=^t\i-tl+i2t+l)k and v={2t-3)i+l-tk,
find ~ (u ● v), when t— 1. [Kanpur 1982]
du d *
Sol. i_,j^_(2r+i)k =2d-j+2k
lt~dt

and [(2r-3) i+j-^ k]-2i+0 j-k.


d, . dy / du

=rrM-r j+(2r+1)k].(2i-k)
+[(2r-3)i+j-r k].(2f i-j+2k)
=2r2-(2r+l)+2r(2/--3)-l-2f
=2r*-2r- H-4r*-6/-1-2t
=6t^-\0t-2
=—6,whenr=l.
Ex. 13. I/A'-=5t^ i+r k and B' =sin t \—cos t hfind

(0 j,(A.B); («) j,(AxB); m


dB
Sol. We have ^=l0t
dt i+j--3r* k and ^=cos t i+sin t j.
d .. . dB dA
Xi) Jr(A-B)=A.-^+ dt
={St^ 1+/ j—k)«(cos t i+sin rj)
+(10f i+j—3/* k).(sin t i—cos f j)
=5t^ cos t-\-t sin r+lOr sin t^cos t
=(5f*—l)cos /+1U sin t.
(ii) We have AxB=(5r2 j—k)X-(sin t i—cos t j)
i j k
'* 5/2 /
sin t ^cos t 0
= —/2 cos / i—(0+/2 sin t)j+(—5/* cos t—t sin /) k
https://www.pdfnotes.co/

DIFFERENTIATION “AND INTEGRATION OF VECTORS 53

= COS / Sin / j-(5/* cos t+t sin /)k.


Ji(A X B)=(t» sin t-3t‘cos t)!-(/» cos t+3t‘sin t)i

-n ..V , » w '+'“S0 k
-1 (t sio./—3 cos f)I-/8(/ cos /-f^3 sin /) J
-(11/ cos /-5/2 sin /+sin /)k.
dA
(iii) |(A.A)=:j7«a+a» dt -3-«3i2A»-s-
dt
=2(S/‘ l+t j—<> k).(10/ i+J—3/> k)
=2[50 <»+/+3<»]=100<»+2<+6<».
Ex. 14. f/r^a cos I i+a sin I j+a/ tan « k,^«<f
dt d^
and r<fr tPt
di^W L*'A*” afsj'
Sol. We have [Meerat 1991 P,92; Agra 82. 88; Kaapor 88]

dt ~ ^ ^ j+tf tan a k
dh
-
rf/a=~o cos t i-a sin / j, ●/ ^
dh
sin / i—a cos t j.
. dr dh i j k
●● dt^dt^ —a sin / a cos t a tan a
-a cos / —a sin t 0
—a^ sin / tan a i—a* ^qs t tan aj + oak.
dt dh // - . 1
●* It^dF^ ^V ^ cos* ^ tan*
«+a<)
sec a.
Also 'A ^ /<*. ^2j.\ d^
dt* dt^‘ dt^]-\dt^dt^)'^
=(a» sin < tan «i_aScos / tan a j+a%).(a sin t I-a cos t f)
=o»sm»ttanai.i+a»cos«t.tan«j.j , [V i«j=0etc. J
=a* tan a (sin* i+cos* t} [ v i.i=l=j.j]
=a*tana.
du
Ex. 15. If ■j=YiXn,^=tyixr, show that

(nxv)=wx(uxv).
Sol. We have [Meerat 1991S, Kanpur 88]
https://www.pdfnotes.co/

VECTORS MADE EASY


54

3(0 X y)=^X v+a X^=<w xn)x y+u x(w x y)


=(y.w) u-(yo)w+(«t.y) v-(a-w) y
=(yw)«—(a*w)V [V Q.vssva]
=(w»v) u—(w*u) v=wx(axv).
Ex..16. IfB. be a unit vector in the direction oj r, prove that
' dB \
Rx r where r= 1| r .
dt ""r*
[Kanpw 1987; Agra 83; Garhwal 86]
1
Sol. We have r=rR; so that R=- r.

dt~r*dt t^dt
(Xdt lir \
HeniceRX^=^ rX \rdt r^dt )
1 dr 1 dr

1 dt
=T-^^dt [V rxr=0]

Ex 17. Show that t x (ft=it xdt)jr^ where t=rr,


[Rohilkhand 1991, Agra 83]
A 1
Sol. We have r=- r.

dr=d(7 =7*+(
Hence rx<rf=(irj x I dt
rxr
' =.pTx*-(^<fr)
txdt
,since rxr=0.

Ex. 18. 1ft is the position vector ofa moving point and r is
the modulus oft,show thi
dt ^ dr
^'3T'It
. Interpret the relations r*$=Oan</jX$=0.
dt
[Rohilkhand 1980]
Sol. We have r«r=l r
https://www.pdfnotes.co/
DIFFERENTIATION AND INTEGRATION OF VECTORS 55

●●●

or r*

dt dr dt dt
or 2r.-r=2r ●r
dt dt ● ^^dt-dt
or , dt dr
^'dtT’' dt

Geometrical interpretation of ^»~=0 and rx$=0.


dt
dt

^=0 is a necessary and sufficient condition for the
dt
vector r (r) to have constant modulus while ;● x 0 is a neces-
dt
saty and sufficient condition for the vector r (/) to have constant
direction.

Ex. 19. If the direction of a differentiable vector function t{t)


is constant, show that r‘x(</r/dr)~0. [Kanpur 1982; Rohillchand 79]
Or
If r (t) is a vector of constant direction, show that its derivative
is coUinear with it.
[Allahabad 1981]
Sol. Let r be a vector function of the scalar variable t having
a constant direction.
If R be.a unit vector in the direction of r, then R is a const-
tant vector because it has constant direction as well as constant
modulus.
If r be the modulus of r, then r=rR.

*● dt~dt^"^^^~dt^-^^ dt
dir r_ dR . . n
~dt- ● ® constant vector

rx
f=(rR)xgR)
● o

=r|(RxR)=0
[V RxR=0]
Now rx(dr/dr)=0 implies that the vector dtidt is> coUinear
with r.

Ex. 20. If rx dr=0, show that jr =constant.


[Kanpnr 1987; Rohilkband 80]
https://www.pdfnotes.co/
56 VECTORS MADE EASY

Sol. Let r be the modulus of the vector r.


Then r=rr.
4f^d(r i)=drr+rdt,
r X dr={r r)x(drr-hrdt)
=(r<fr) rxr4-r*rx<tf
=r*rxrfr [V rxr=0]
rxdr=0 *> r® rxdi-=0>
=>rxrfr=0 ...(1)
Since r is of constant modulus, therefore
^.*=0. ...(2)

From(1)and (2). we get dr=0.


Hence r is a constant vector.
Alternative method.
Let r=xl4-y j+r k, then dr=dx i+dy l+dz k.
rxdir= i j k
X y 2'-
dx dy. dz
={ydz—zdy)\-\-{zdx—xdz) \-\-(xdy~-ydoC) k.
.V rx</r=0 =► (ydz—zdy) i-\-(zdx—xdz) }+(xdy—ydx) k=0
=> ydz—zdy=0, zdx—xdz=0, xdy—ydx=0
dx dy_dz
x^ y~ z
If — then log x=log ;>+log Cj
X y
or x=Ciy. ...(1)
If then log y+log c,=log z
or z=c^y\ ...(2)
Now r=xl+;>j+zk=(Ci y) \+y j+(Ca y) k
=y(ci i+j+cak).
● L.- y(gj i+J+Cafc)
'● ^~\rrV[{Ctyr-\^y^+{c,yf]
Cl i+i+Cfl k which is a constant vector because
it is independent of x, y, z.
https://www.pdfnotes.co/
DIFFERENTIATION AND INTEGRATION OF VECTORS 57

Ex. 21. If T is a unit vector, then prove that


dr dr
^'^dt dt * [Rajasthan 1974]
Sol. Since’r is a unit vector, therefore | r j is constant and so
r is perpendicular to its derivative drfdt.
Now by the definition of the cross product of two vectors.
we have
dr dr dr dr
rx sin 90®= 1.
Jr ='- dt- dt ● dt
Ex. 22. If ^ is the unit vector making an angle 6 with x-axis,
show that defdQ is a unit vector obtained by rotating e through a
rigjht^ angle in the direction of6 increasing. [Allahabad 1979]
Sol. Since e is the unit vector which makes an angle 0 with
x-axis, therefore
e=cos 6 i+sin 0 [Draw figure yourself]
. ^
●● ® Hcos 0 J=cosQ 7c+0|i+sin w+0^ j.
which is a unit vector which makes an angle ^«+0 with x-axis.
Thus die/</0 is a* unit vector perpendicular to the vector e in
the direction of 0 increasing.
Hence dejd^ is a unit vector obtained by rotating e through
a right angle in the direction of 0 increasing.
Ex. 23. If r is a vectorfunction ofa scalar t and a is a cons
tant vector, m a constant, differentiate the following with respect
tot:
dr dr
(/) r.a. (//) rxa. (i7i) rx^r dt‘ «v) r.g;.
1 r+a rxa
(y) (w) w (V/) (v«7)
r*+a2' r*a
Sol. (i) Let i?=r«a. [Note r.a is a scalar]
dR dr da
Then-s-
dt ^di ●a+r«T- dt
dr .. da
~dt ● a+r»0 ● It-= 0, as a is constant
dr dr

(ii) LetR=rxa.
</r , da
https://www.pdfnotes.co/
58 VECTORS MADE EASY

dt dfk
=j^xa-}-rxO V● W
dr dr

(HI) LetR=rxj-
</R dr dr. ^ dh
dt^
A.
=0+rX5^
dh r^.● dt^dt ”
dh

dr
(ivi Let R=r*j^-
dr dr dh (dry+r« dh
™®"*=7f-a+'*55=U) F*'
(v) Let R=r> fL.
dR d
Then -4
dt~dt

^2/1 Jl±
^^dt r^df

(vi) Let R=m


©■
dR d Jdry

dr dh ■ Jr* m dr
~ ^dt’dt» Note^=2r.^^.
-2m* ^

r+a
(vii) Let R
JR 1

[Note that r*+a* is a scalar]


1 1 4
r*+a» (M)-{ (r*+a*)* dt (r»+a»)J.(r+a)
https://www.pdfnotes.co/

DIFFERENTIATION AND.INTEGRATION OF VECTORS 59

dt
It-r.
1 dr dt
(r+a).

r xa
(?iii) Let R r«a
^ dm \ d,
Then^ — :r ("“X xa)
dt r»a dt^
[Note that r.a is a scalar quantity]
1 /A
r«a \dt
dr
Xa+rx
l)-{
-rxa
dt
r*a xa)

dr dr
xa -^●a
It dt da
(rxa) =0
/●a (r.a)« ● dt
Ex. 24. If r is a vector function of a scalar t, r its module^ and
a, b are constant vectors, differentiate the following with tespect to t :
dr
(/) r®.r+aX (//) r® r+(a*r) b, {Hi) r" r, (iv) (ar+rb)®.

dr
Sol. (i) Let R=r® r+aX^-

, „ ^ dr . ^ d~r da
= 3r®-r.r-fr»
dt -x+ax^
dt ● dt
(ii) Let R=r*r+(a.r) b.
dh
Then ^4 r)+|j^ (a-r) . b+(a.r) dt
* , ^de , (da
=2^S'+'"7r+(3r-'+--*r
, dr\ .
r-● ^=oi
dt
dr , 9 dr / dr\ , ●●

(iii) LetR=:r«r.
https://www.pdfnotes.co/
60 VECTORS MADE EASY

£fr
Then r4->-« dr I .dr\ r+r"
7/=("^ ‘3f) w
(iv) Let i{=(flr+rb)2. Then
d dr’
Note. -r. r^=2r»
^=2(<.r+fb).|(or+fb) dt dt
/I / . LX . dr. , db\
=2(ar+rb).|^r+agj+j^ '>+'■*)
da . dh «■]
=2(ar+rb).(a^+^b)
Ex. 25. Find
(0 ^ r </r p jf
<//L * W
d /</r rf%r\“l
m dt 5:
' dr dh-
Sol. (i) Let /?= '' df dt^ ● Then R is the scalar triple

product of three vectors r, and ~. Therefore using the rule


for finding the derivative of a scalar triple product, we have
dR
__ \dr__ dr dh^ dh dh^ dr d»r1
dt -[dF dr dr’dr dr dr,
= r» ^ . sipce* scalar triple products having two equal
vectors vanish.
dr dh~\
(ii) Let R— r, -
dr dr ' Then as in part (i)
dR dr dh'
dt “U' df dfi_ ■
Differentiating again, we get
d^R [dr dr dhl . r dh — _
dhl
dr^ dr dr drJ'*'L * dr’ **J+L'* dt’ dt\
‘ ^ ^rl dr dh
~ dr' dr + *■' dt’ dr

Then R is the vector triple pro-


(iiij LetR=rxgx^).
duct of three vectors. Therefore using the rule for finding the
derivative of a vector triple product, we have
U-rx/—x-Urx/-x-\
dt - dt^\dt^ dt^r^\dt‘^ dt»r^ \dt^ dt’‘)
https://www.pdfnotes.co/

DIFERENTIATION AND INTEGRATION OF VECTORS 61

dx idt d^\ [dt dh\


=axU^3fij+'^U'<3r4
</%■

being vector product of two equal vectors.

Ex. 26. If a=-s/« e \-\-cos 6 j+0k, b=co5 0 i—sin 6 j—3k.

and c=2i-h3j—3k, {ax(bxc)} at 6=-^*


[Kanpur 1987; Rohilkhand 79]
Sol. We have
bxc= i j k =(3 sin e+9) H-(3‘cos 8-6)j .
+(3 cos 6+2 sin 0) k.
cos 0 —sin 6 —3
2 3 -3
.*. ax(bxc)= i J k
sin 0 cos 6 0

3 sin 0+9 3 cos 0-6 3 cos 0+2 sin 0


=^(3 cos* 0+2 sin 0 cos 0-«-30 cos 6+60) i+(30 sin 0+90
—3 sin 0 cos 0—2 sin* 6) j+(—6 sin 6—9 cos 6) k*
***

=(-6 cos 6 sin 0+2 cos* 6—2 sin* 0—3 cos 0+30 sin 0+6) i
+(3 sin 0+30 cos 0+9-3 cos* 0+3 sin* 0—4 sin 0 cos 6) j
+( - 6 cos 0+9 sin 0) k*
Putting 0^7c/2, we get the required derivative
=(4+|Tr) i+15j+9k.
Ex. 27. Show that if a, b, c are constant vectors, then
r—a r*+b r+c is the path of a particle moving with constant acce
leration. ‘
dr
Sol. The velocity of the particle—^=2ra+b.
dh
The acceleration of the particle= <//* =2a.
Thus the point whose path is r=a /*+b r+c is moving with '
constant acceleration.
Ex. 28. A particle moves along the curve x—4 cos t,y=4 sin t,
z=6t. Find the velocity and acceleration at time t=ti and t=^Tz.
Find also the magnitudes of the velocity and acceleration at any
time t.
[Kanpur 1980; Agra 81]
https://www.pdfnotes.co/

62 VECTORS MADE EASY

Sol. Let r be the position vector of the particle at time t.


Then r=x j+z k=^4 cos t i+4 sin t j-f6t k. If v is the
velocity of the particle at time r and a its acceleration at that
dr
time then v~ = —4 sin t i+4 cos t j+6k.
~'dt~
dh
—4 cos t i—4 sin t j.
^~di^~
Magnitude of the velocity at time /=j v 1
=V(16 sin® /+16 cos® r4-'36)=V(52)=2V(13).
Magnitude of the acceleration
= 1 a|=\/(16 cos® r+16 sin® 0=f4*
At /=0, v= 4j+6k, a=-*4i
At/=iTc. v=—4i+6k, a=-4j.
Ex.>29. A particle moves along the curve x=er*, y=2 cos 3t,
z^lsinZt. Determine the velocity and acceleration at any time t
and their magnitudes at i—0. [Gorakhpur 1985]
Sol. Let r be the position vector of the particle at time U
Then r =jc i-\-y j+z k=e~^ i+2 cos 3/ J+2 sin 3/ k.
If v is the velocity of the particle at time t and a its accele
ration at that time, then
dr
—er* i—6 sin 3t j+6 cos 3t k.
"'~dt~

a=^=e-‘i—18 cos 3/ j—18 sin 3t k.


dt^
Putting/=6 in the above relations, the velocity at r=0 is
—i+6k,
and the acceleration at i=0 is i—18J.
Hence at r=0.
the magnitude of velocity=|—i+6k 1 =^/{{—\y●^6^\—^/3^,
and the magnitude of acceleration '
= 1 1-18J 1=V[l^+(-18)®]=V(325).
Ex. 30. A particle moves along the curve j:=/®+l, y—t^,
z=2/^- 5, where t is the time. Find the components of Us velocity
and acceleration at t—I in the direction
[Agra 1979; Rohilkhand 81]
Sol. If r is the position vector of any point (x, y, z) on the
given curve, then
r=.xi+:>^j+zk=(r®+l) i+/® j+(2r+5)k.
Velocity=v=^=3r® i+2/J+2ks=3i+2j+2k at t=l.
https://www.pdfnotes.co/
DIFFERENTIATION AND INTEGRATION OF VECTORS 63
(Pf d idx\
Acceleration=a=

Now the unit vector in the given direction i+j+3k


i+j+3k i+i+3k
=b, say.
| i+j+3k|-V(ll)"
the component of velocity in the given direction
-v.K^(3i+2jif-2k).(i+j4-3k) 11
vm ^70T)-=V(n);
and the component of acceleration in the given direction
=a*b=(6i+2j)«(i+j+3k) 8
V(n) ~V(11)‘
Ex. 31. A particle moves so that its position vector is given by
r=coj <a/ i+j/n <o/ j where vi is a constant; show that(0 the velocity
of the particle is perpendicular to r,(«) the acceleration is directed
towards the origin and has magnitude proportional to the distance
dr
from the origin, (i/7) rXj^is a constant vector^
dr
Sol. (i) Velocity v=^^=—« sin w/ H-<o cos o»/ j.

We have r*^^=(cos u>t a+sin at j)»(—<a sin at i-f-w cos <o/ j)


= —a cos at sin cd/+e>> sin cot cos w^=0.
Therefore the velocity is perpendicular to r.
(il) Acceleration of the particle
dh
=a= «●>“ cos at i— sin j

= —.6)2 (cos at l-hsin at J)= —a^ r.


.*. Acceleration is a vector opposite to the> direction of r i.e.
acceleration is directed towards the origin. Also magnitude of
acceleration I a |=|—6>2r |=6>2r which is proportional to r i.e., the
distance of the particle from the origin,

(ill) rx^=(cos at i-j-sin at j)X(—6> sin at cos at j)


=6) cos2 at ixj—6) sin*©/jxi [V ixi=0, jxj=0]
=6) cos* 6)/k+6> sin2 6>r k [v ixj=k=—jxi]
-a (cos* 6>r+sin* 6>f) k=6)k, a constant vector.
Ex. 32. Find the unit tangent vector to any point on the curve
x=fl cos t,y~a sin t, z=bt.
https://www.pdfnotes.co/

64 VECTORS MADE EASY

Sol. If r is the position vector of any point (:», y* z) on the


given curve, then
t=xl-{-yl+2k=a cos t i+o sin t \-]-bt k.
dt.
The vector -r is also the tangent at the point {x, y, z) to the
dt
given curve.
We have ^=—o
dt sin t i+u cos t \-\-b k.
dt
= sin'^ r-ha* cos* /+h*)=
- dt
Hence the unit tangent vector t
dtjdt —a sin t i+fl cos t j+^k
~\dt/dtr
1
{—a sin t i+fl cos t j-hfrk).

§12. Integration of Vector Functions.


We shall define integration as the reverse process of differentia^
tion. Let f(0 and F(0 be two vector functions of the scalar t such
that4F(<)=f(0-
dt
Then F(0 is called the indefinite integral of f(0 with respect
to t and symbolically we write / f(0 dr=F (/)● ...(1)
The function f (i) to be integrated is called the integrand.
If c is any arbitrary constant vector independent of t, then
f(0.
^,{f(0+c}
This is equivalent to J f {t) dt- 'E (0+*'● ...(2)
From (2) it is obvious that the integral F(t) of f (0 is indefinite
to the extent of an additive arbitrary constant c. Therefore F (0
is called the indefinite integral of f (r). The constant vector c is
called the constant of integration. It can be determined if we are
given some initial conditions,

if £ JF (/)=f (t) for all t in the interval [a, h], then the definite
integral between the limits and can in such case be written
tb
dt

=fF (0+e V . 0
(6)-F (o).
https://www.pdfnotes.co/
DIFFERENTIATION AND INTEGRATION OF VECTORS 65

Theor«m. If t(»)=/.(/) i+f,(,)j+f,(,)k,,hen


S t(0 *=i//,(0 dl+i Sf,(t) dt+k if(0 dt.
Proof. Let F(0=f(/).
...(1)
Then /f(t)*=F(0. ...(2)
Let
™ ^ (')1+ ■'"a (0 k.
Then from (1) , we have
d
J, {Ft (0 i+Fj (0 j+F, (t) k}=f (0
or
{jr ('))■ i+{s F, (t)| j+l^^ F, (,)^ k
~fi (0 ^+fi (0 j+Za (0 k.
Equating the coefficients of i, j, k, we get

s Ft (0=/. (0, 3 Fe (0=/e (0. j, F3 (t)=/3 (t).


- Pi (0=J /a (0 </t. Fg (0=J/a (0 rf/.
F(0=
-{]/i (/) i+jj/a (0 j+| ‘ /a(0^/ k.
So from (2), we get " '

Note.
/J (') *--=1 ift (0 dt+i if it) dt+k.if (/) dt.
From this theorem
r ... . .● that thedefinition of
the integral of a vector fimction/impiies the definition of integrals
of three scalar functions which are the components of that vector
function. Thus in order to integrate r vector fiinction we should
integrate its components.
§ 13. .Some Standard Results.
We have already obtained some standard results for differen-
tiation. With the help of these results we can obtain some standard
results for integration.
dr ds
1. We have ^ (r.s)= -3-»s+r»^
dt dt
Therefore d/a=sr.s-(-c,

where c is the constant of integration. It should be noted that c is


here a scalar quantity since the integr^d is also scalar,
2. We have ^ (r*)=2r.—
Therefore
IK) df=r*-f c.

Here the constant of integration c is a scalar quantity.


https://www.pdfnotes.co/
66 VECTORS MADE EASY

3. We have d (drY_^ M .i
dt\dt) dt*d
Therefore we have
f/^ dt dh\ . (dry+c.
y di'di‘) *“((/»)
Here the constant of integration c is a scalar quantity.
Also
\dt) ~dt*dt
dh
4. We have d( dx\ dt dt . ^dh rX dt^

rf(=rx^J+c.
Here the constant of integration c is a vector quantity since
dh
the integrand tx-^^is also a vector quantity.
5. If a is a constant vector, we have
d da. dt dt
=a x-T*
|(axr)4xr+axgj dt

Therefore
ht) dr=axr4-c.

Here the constant of integration c is a vector quantity.


6. If I r I and r is a unit vector in the direction of r then

dt (T 7“r dt~l» 'dt '●


Therefore

7. If c is a constant scalar and r a vector function of a scalar


/, then obviously ct dt=c t dt.
8. If r and s are two vector functions of the scalar t, then
tdt+ sdt.
obviously J (r+s) d^=J
Solved Examples
Ex. 1. //f(0=(^--^")i+2/»j-3k.^««r

(0 ,f (/) dt and («) j f (/) dt.


Sol. (i) / f (0 dt=! {(t-t^) J-3k} dt
https://www.pdfnotes.co/
DIFFERENTIATION AND INTEGRATION OF VECTORS 67

=i (/-
J-Jdf

where c is an arbitrary constant vector

‘+i
(ii)
‘f(I) </<=£ {(<-(») j_3k} dr
n
i£ d/+] £ 2/’dl-k V.2dt ●
■/8 ^8*12 r /412 2

L^' [^ J1 |'+Ti-».
Ex. 2. Evaluate
Jo
1
Sol. (c'i+e-2»j+/k)<7/
0

=i ‘ r 4/l+k t dt
9 Jo Jo
=i
0 JO Jo

Ex. 3. //f (0=./ l+(J»-2/) J+(3/«+3<») k,/*id ● I


f Jr

Jo
f(0*.
[A gra 1577]
Sol
0
[if i+(/*-20 j+(3/*+3/8) k] dt

i Jo ‘ ((*-2<) <*+k Jo
Jo
('(3<»+3/») *
11
3ini
= 1 i'fJo +j ●I'OT-<*]‘+k
Jo L
»»+^‘
4 Jo
=i i+rt-l) j+(l+ij k=i i-f j+ik.
Ex.4. Ift=ti-t^i+{t-\)kanda=2tH+6tKfmIuaU .

f.
●2
(0 r.s dt. (ii) rxs ffr
0
[Meerut 1992]
Sol. (i) We have r.s=[/ i-i* j+(l-I) k].(2/a i+6l k)
=2/»+6/(/-l)«2f«+6l»-6/.
https://www.pdfnotes.co/

68 VECTORS MADE EASY

● r«s |-0(2<* 16/^—6/)dt

^+2/8-3raT=8+16-12= 12.
.2 . Jo
(ii) Wehaverx.s= . i J k
. t -/a t-\

2t^ 0 6/

=_6r8 i_[6/a-2fa(/_!)]j+2?« k
=-6/* i-(8fa-2/»)j+2/4 k.

rxsrfr=f [-6/* i-(8<a-2/*) J+2r^ k] dt


0 0
n Y2 2
=i -6t* dt-}^ i%t^-2t«) dt+k 2t*dt
0 JO 0
la r8r* t*V ,, r2/»ia
-I" -j
JO iT-ji+HTj.,
24 i-(y-8)j+yk

=_24i-fi+fk.
Ex. 5. Eva/tutfe j (a»hxc)dt, where
a=/ i-3j+2/ k. b=i-2 J+2k, c=3 i+/ J-k.
[Garbwal 1^771
Sol. We have a»bxc=[a b c]
t -3 2t
1 -2 2

3 / -1
t -3-r2r 0 , by C,+2Ci
and Cj—2Ci
1 0 0

3 t+6 -7
-l.{-7(2f-3)-0}.
expanding the detenninant along
=7(2t-3);
https://www.pdfnotes.co/
DIFFERENTIATION AND INTEGRATION OF VECTORS 69
i
● Y(a.bxc) f 7(2/-3) dt=l 1.
(2/-3)dt
12
7 /2-3r „7[(4_6)-(1-3)]=7.(-2+2)=0.
Ji

Ex. 6. Evaluate
i: rx^'dt. wherer=2t^ i+^ j-^3/* k.
[Kanpur 1975]
Sol. Given r~=2t^ i+r j-3/» k.

j^=^t i+j-9/2 k and ^=4 i+0 j-18 t k.

rX^=(2/2 i+/ j-3f» k)x(4 i+0 j-18 Ik)


i j k

2/2 / -3i8

4 0 -18f
=-1812 i_(_36 /«+12 /») j-4/ k
=-18/2 i+24/» j-4/ k.
" d"2r . fa
(-18 /* i+24/2 j-4/k)
,1

=-18 i J'‘l>*+24j
/» dt-A k
i: tdt

=-l8 i -4 k
2 1

=-6(8-1)i+6(16-l)j-2(4-1)k
=-42i+90j-6k.

Ex, 7, Find the value of t satisfying the equation


where a is a constant vector. Also it is given that when t- O, r=0
. dt
and -r=u.
dt [Agra.1981]
dh
Sol. Integrating the equation ^2=®» we get'
di
dt
=/a+b, where b is an arbitrarv constant vector.
dt
But it is given that when /=0,
https://www.pdfnotes.co/
70 VECTORS MADE EASY

o=0a4-b or b - a.
dt
=^a+n.'
dt"
Integrating again with respect to //we get
r=i/* a+/a+c, where c is constant.
But when /=6,r=0.'
' 0=04-0+c or c=0.
r=J/2 a+/u.
. dh
Ex. 8. Solve the equation ^=a where a is a constant vector;
dr
pven that and dt=0 when t=0.

Sol. Proceed as in Ex. 7. Ads. r =^/%.

Ex. 9. Find the value of r satisfying the equation^ =/a+ b,


where n and b are constant vectors. [Agra 1979; Rohilkhand 83]

Sol. Integrating the equation /a+b, we get


dr *
— =i/* a-|-/b'+c, where c is constant.
dt
Again integrating,:we get
r =|/* a-|-^/* b+/ c+d, where d is constant.'

Ex. 10. Integrate ^=-nh.

Sol. We have ~= —hh.


dt^ : ●● 0)
Forming the scalar product of each side of(I) with the vector

Now integrating we get

-nV+c, where c is constant.


/
Ex. 11, If r»dr~0,show that
| |
r —constant.
[Agra 1975; Rohilkhand 79]
Sol. Wehaver.dr=^0s> 2r.dr=0 ^V(r*r)=0
https://www.pdfnotes.co/

DIFFERENTIATION AND INTEGRATION OF VECTORS 71

=> </(r®)=0 => r*=constant => I r p=con8tant


=» I r |=constant.
Ex. 12. Find the value of r satisfying the equation
dh ●- ■
24l*j-l-4 sin /k.

given that r=2i+j and —i—3k at t—0.


dh
Sol. Integrating the equation 24t*j-l-4sin Ik, we get
dt
dt =3/2|—8t*j--4 cos tk+b, where b is an arbitrary constant
vector.
But it is given that when t=0, drldt=—l—3k.
/. _i_3k=_4ic^.b or b--i+k.
dr
/. ^=3t^i—8t*j—4 cos /k—i+k
=(3r2_l) i_8/8j+(i-4 cos t) k.
Integrating again w.r.t. t, we get
r=(t8—r) i_2r*j-f-(/—4 sin /) k+c,
where c is ah arbitrary constant'vector.
But it is given that when /=0, r=2H-j.
/. 2i+j=0+c=c. ^
j=(l8-0 i-2r*j+(/-4 sin t) k-{-2i4-j
or r=(t»-t-^2) i+(l-2<^) j+(t-4 sin t) k
is the required solution of the given differential equation;
dr
Ex. 13. Show that
arbitrary constant vector.
K'-S) dt—rx +c, where c
dt is an

d t dr\ d^ dr dr .
Sol. We ha:ve
dty^d^
^d^ . dr dr ●
=rXjj5, since
Integrating both sides with respect to //we get

'('xg <i'=(rx|) +c, where c is an arbitrary constant


vector.
https://www.pdfnotes.co/
72 VECTORS MADE EASY

Ex. 14. Integrate ^'<^=zh, where a and h are constant


vectors.
d^r
Sol. We have i
dt
dh
Therefore integrating a x <//2=»b, we get
dr
'*^^=^h+c, whece c is constant.
Again integrating, we get
aX fc+d, where d is constant.
Ex. 15. 7/r(0=5/2 i+/ j_,s prove that

<//=_14i+75j_15k.

[Meemt 1991, Kanpur 87; Agra 82, 86; Rohilkband 85]


Siol. We have
('«£)
<//=r X-5--j-c.
dt
dtr

dr
Let us now find r x -r We have $=10/i+j—S/^k.
dt
'
dr
*^x i+O‘-^®.k)x(10/ i+j-3/2k)
i J k
5/2 / -/3
=-2/3 i+5/4j-5/3 k.
10/ 1 —3/2
. r/ dh\ 12
●● Ji y^dt^) «^^=[-2<«i+5/«j-5/2kJ
2/3
J*
i+fs/^l^’-fs/^r
L ji L Ji
k=-14i+75j- 15k.
Ex. 16. Given that
r(/)=2i— j+2k, when t=2
=4i—2j+3k, when /=3,
show that W/= 10.

[Kanpur 1986; Rohilkhand 84; Agra 83, 87J


Sol. We have
/H) <//=^r2+c.
https://www.pdfnotes.co/
.#

DIFFERENTIATION AND INTEGRATION OF VECTORS 73


n3

a(4) dt — -Jr*
- J3
When 1=3,r=4i-2j+3k.
when /=3, r2=(4i-2j+3k).f4I-2j+3k)= 16+4+9=29.
When /=2, r=2i—j+2k.'
When/~-2, r2=4+l+4=9.

a('4) [29-91=10.
Ex. 17. The acceleration ofa particle- at any time / ^ 0 w
given by
ds
a= =12 coj 2M-8 2/ j+16f k.
dt
If the velocity v and displacement lare zero qt t=0,find ? andt
at any time. [Meernt 1991 P; Kerala 74]
d\
Sol. We have ^=12 cos 2M-8 sin 2t j+16/ k.
Integrating, we get
12 cos 2t di+i 8sin2/^/+k 16/^/

or v=6 sin 2/ i+4 cos 2t j+8/2 k+c.


When /=0, v=0.
0=- 0i+4j+0k+c
or c=—4j.
dr
6 sin 2t i+(4 cos 2/—4)j+8/*k.

Integrating, we get

r=i J 6 sin 2/ f//+j J(4 cos 2/-4)dt+k


8/2 dt
= —3cos2/ i+(2sin2/—4/)j+f/*k+d, whered is constant.
When /=0,r=0.
0=-3i+0j+0k+d. d=3i.
r= —3 cos 2t i+(2 sin 2/-4/)j+f/^ k+3i
=(3-3 cos 20 i+(2 sin 2t~4t)j+f/sk.
Ex. 18. The acceleration ofa particle at any time t is
et- i+e2f pfufi giyg„ y =i-|_j at t—0.
[Meerut 1991S; Kanpur 88]
https://www.pdfnotes.co/

74 VECTORS MADE EASY

dy
■^=e* i+e*'J+k,
Sol. Given dt where v is the velocity
vector of the particle at any time t. Integrating with respect to
we get
V=gr j+fk+C,
where c is an arbitrary constant vector-
But at /=0, it is given that T=i+j-
i+i=i+ij+c or c = y
j+rk+ij
= c*i+i (e2/+l)j+tk.
https://www.pdfnotes.co/

Gradient, Divergence and Curl

§ 1. Partial Derivati?es of Vectors.


^ Suppose r is a vector depending on more than one scalar
yariable. Let r=t(Xyy, z) i.e. let r be a function of.three scalar
variables x,y and z. The partial derivative of r with respect to x
IS defined as
lim . y, z)—f(x. y. z)
dx Bx-*0 8jc
if this limit exists. Thus 3r/9ji: is nothing but the ordinary deri
vative of r with respect to x provided the other variables y and z
are regarded as constants. Similarly we may define the partial
derivatives and ~.
'By dz
Higher partial derivatives can. also be defined as in Scalar
Calculus. Thus, for example.

Bx^ dx\Bxy dy^ 0y/' Zz^~dz\dzy


Bh g /8r\ Bh . a /0r \
Bxdy~dx\dy)' ByBx~Zy\zx)
If r has continuous partial derivatives of the second order at
least, then. i*L__ J!l
dx dy ~ dy Bx i.e. the order of differentiation, is imma-
terial. If r=f(x, y,z)the totol differential dr of r is given by

§ 2. The Vector.Differential Operator Del.(v)»


The vector differential operator y (read as del or nabla) h
defined as '

and operates distribulively.


https://www.pdfnotes.co/
76 VECTORS MADE EASY

The vector operator V can generally be treated to behave as


an ordinary vector. It possesses properties like ordinary vectors,
be treated as its components along
The symbols |;.|;. scan
i, j, k.
§ 3. Gradient of a scalar Field. Definition.
Let f{x, y, z) be defined and dijferentiable at each point
(x, z)in a certain region of space (i.e., defines a differentiable
scalarfield). Then the gradient of f, written as v/ or grad /. is
defined as

[Kerala 1975; Allahabad 791


It should be noted that v/»s a vector whose three successive

components are |^,^.and|^. 0z Thus the gradient of a scalar


field defines a vector field. !ff o scalar point function, then v/
is a vector pointfunction.
§ 4. Formulas involving gradient.
Theorem 1. Gradient of the sum of two scalar point functions.
Iffand g are two scalar point functions, then
grad if-\-g)=gradf+grad g
or V (f+g)^ Vf-\rVg-
Proof. We have v
(/+g)--{i|+l^+>^4)(/+ir)
=i I;(/+s)+J (/+s)+k 0-^(/+?)
- i Vj-iM-l-i V . i ®l4-k^+k^
” dx'^ dx^^ dy^^ 9z^“ dz

=V/+V /+grad g.
Similarly, we can prove that v (/—g)=V/-^Vg*
Theorem 2. Gradient of a constant. The necessary and suffi
cient condition for a scalar pointfunction to be constant is that
v/=o.
https://www.pdfnotes.co/

GRADIENT, DIVERGENCE AND CURL 77

Proof. If/(jc, y, z) is constant, then

dx o.|=o.
9/
Therefore grad/=i^+j|^+k|^=0i+0j+0k=0.
Hence the condition is necessary.

Conversely,Jet grad /=0. Then i|^+j ~+k|^= n.


ox oy oz

Therefore. ^=0,|^=0,
dx 'dy ?^=0.
dz
f must be independent of x, y and z. ●
/must be a constant. Hence the condition is sufficient.
Theorem 3. Gradient of the product of two scalar point func
tions. Iff andg are scalar point functions^ then
grad ifg)=fgrad g+g gradf
or V(/g)=/Vfi^+^ V/-
[Meerut 1972; Bombay 69]

Proof. We have v (/?)=('

=/V g+g v/=/grad gfg grad/.


In particular if c is a constant, then
V (c/)=c V/+/V v/+0=c v/.
Theorem 4. . Gradient of the Quotient of two scalar functions.
Iff and g are two scalar pointfunctions, then

2
g [Jiwaji 1982]

Proof. We have v

=i

r|-AI i(L\ ’‘ZyJjy


But
’(ih g ’dy\g) g‘
9'
s●
https://www.pdfnotes.co/
78 VECTORS MADE EASY

^ dz dz
and 1/Z)=
dAs!
■■■
§£_/●?£
dx ■'dx

=p{? v/-/ V s|-


Solved Examples
Ex. 1. IfF=e^y i-\-{x—2y) j+x sin y k, calculate
dt 02F 02F a^F
(0 1^. («) 9/
(»70 (/V)
0x3;;'
(V)
3/'
3F
Sol. (i) ^

+ {x$m y) k
—(ye^^) i+n) j+(sin y) k=ye*y H-j+sin yk.

(ii)
S=lli; ] ‘+[1/^-2;') ]i+[|; k
=xe*->' \—l\+x cos ;; k^
02F 8 /3F\ a r ...y . . * . ● ● n
(iii)
ra
= I (;’«*'■)] *+l5j+ 85 (““●>’> I'
=^2e*y i+0+0 k=y*e»'i.
02F
(iv)

=(fi^+x;;e*J') i+O+cos ;; k
=e*^ (x;;+1) i-j-cos ;; k.

(V)
P=Mi)=1-
=x*e^ i-rO—X sin ;; k=x*e*' i—x sin y k.
https://www.pdfnotes.co/
-GKADIENT, DIVERGENCE AND CURL 79

Ex. 7. If if,(X, y, 2)=A^2 and t=xz i-xy fc, show


that f)at(2,-1, I)/14 i+2j.
[G^bwal 1985]
Sol. We have ^ f==jy;* z(jf2 i—jc;;j+;;22 fc)
i— z z® k.

●'● lx W=2x/ ,●» i-2xy> r j+;4 2« k,

& ^ i-2xyazi+y»^ k]
=>2y^ z® 1-2/ z i+6k=2/ z® 1-2/ z j
and
d:^dz “Fz ] =1 (27^ ‘-2/ ^ i)
=4/. 1-2/j.
a*
.*. at the point (1—1, 1),
dx^dz (^f)=4.(- 1)2.1 1
~2.(-l)8j=4 1+2 j.
Ex. 3. If f = (2xy—x*) 1+(e*y—y sin x) j+x^.coj ;; k, verify
that
. a^f - a®f
dyd~dxdy ^Agra 1981]
Sol. We have
●»’-**)] i+[^ ,(^-7’sin^)J j
+[|j(x»cosj-)]k
={^xy-A:ii+iye=^y-~y cos x) j+2x cos y k.
. 0*f 8 /*
“ dxdy~dy |)=4 l+iye’r-y COS ;c) j
+2x cos j?k]
ra
— ^xy
[dyy —4/) j 1+ (ye*y -y cos x) j .
●0 '
+l^<2xcos^) k
=(4j+(e«»+xv e^y-cos x) j+(-2x sin y) k
=4>f (ew+x;* e*y—cos x) j—2x sin y k. ...(1)
0f
Again r-
dy px‘y—xt) j+ ^(eyy—ysiBx) j
ra
+ [^(^<=os.V) k
=2j+(xzw-sin X) j—x'.sin y k.
https://www.pdfnotes.co/
VECTORS MADE EASY
80

dH
. -1(^)=fI- (2;c»)l i+f^(.ve*>’-sm x) j
●● dybx~dx\cy) Idx^ 'll Idx^ 'j*
- i(A:2sin;^)']k

=4x cos x)\—2x siny k. ...(2)


^ m 3f
From (1) and (2), we have 53^-33^-
Ex. 4. Jfu=xy2 i-\-xz^ j— fe and|=a3 i—xyz j+x^rk.cfl/-

c«toepx^Io"A«P<»'“(1- 1-0)
du
Sol. We have^
ar[J “
=.vz i+0 j—3/ k=fz i—Sy"^ k
82u- 9 /0u\ _Q .— 6yk=i—6yk,

0V
Again
=3x2 i-yzjJf-2xzk

?!? ?!! (—6>’k)x(6xi+2zk)


a/^ 0x2
=-36x;;kxi—12j)z kx
=-36x;;j [Vk xi=j andkxk=0]
02| a*v ●
at the point(1, 1, 0), we have gy g;5^2=*“26j.
Ex. 5. If A-=X*:))Z i-2xz® \-\-xz^ k, l=2z i+>)j—x® k, find
0*
the value of (AxB)nr(l,0, -2). [Kanpur 1985, 81]
0xa;F
Sol. We have A xB i j >
x^yz — 2xz« x^2
2z y -X®
-.(2x»z»—x;iz®) i+(2xz®+x*j'^) j+(x®>’®4x2*) k.
/. ^(AxB)=-xz2 j+2x®^2k.

Again
0x8>>
https://www.pdfnotes.co/
GRADIENT, DIVBROBNCB AND CURL 81

= —z® i+4^z j+4xyz k. ...(1)


Putting 1,3^=0 and z=—2in (1), we get the required
derivative at the point(1,0, —2)= —4i-8j.
Ex.6. If /(x, y, z)=3jc®j^—;;*z®, find gradf at the point
(I,-2,-1). [Agra 1978^RohilkIiand 83]
Sol. We have

grad /=v/=(i|i+j|j+kI)
=1|j(3*»>'-j^z«)+j (3i^-y>!^)+ky3x»y-^
=i(6xy)+i(3*“-3/z»)+k (-23>*2)
=6xy i+(3x®-3>^®z®) j-2j;»zfe.
Putting x=l, y=—2,z= —1, we get
V/=6(l)(-2)!+{3(l)®-3(-2)® (-1)®}j
-2(~2)»(-l)k
12i-9j-I6k.
Ex. 7. If r— \ r \ where r=xi+yl-{-zk,prove that
(0 Vf(f^) --f'(r)^r, (i0vr=ir,
[RobUkhand 1984]
(*7) v/(r)xr=0, (/v) V
[Meerut 1991]

(V) V hg\r\ =p. [Meerut 1991;JCanpor 88]


(vi) y r«=^=nr"-® r. [Kanpur 1984; Agra 86;Roh'ilkhand90;
Garhwal 84]
Sol. If r=--xi+j/j+zk, then r=
| ri —
/. r*=x®4-/+^*.

(i) V/(.0=(i|f+j.|;+kfJ/rr)
=i
|^yi:r)+j|;A^)+k|/(r)

=i/'W|+j/'(r)|+k/'(r)|

==/'(r)(i|+j|+k|)=/'(Dvr.

(ii) 'Wehaveyr=l|+j|+k|-
0r
Now r®=x®4-;>®+z*; 2r ^=2x U,,|^=--
ox r
https://www.pdfnotes.co/

82 VECTORS MADE EASY

Similarly and gj--=-

V'’
r= r.
7‘+?1+r ‘‘^"7(*‘+^+'k)“r
(iii) We have as in part (i), v/(0-/'(^)

But as in part (ii) vr=^ r.


1
/. V/(r)-r(r)^r.
\\v/(r)xr= xr)
|/'(r) ir|xr=ji/'(r)f(f
—0,since rxr=0.

(W) Wehavev(i)=i |;(7)+J |;(r)+^|?(r)

I®®®

1 1
=-78{^*+3l+^k)= -;s r.
(v) We have V log 1|
^ =V log r
=i
Lldgr+J^logr+kilogr-
It. i+it i+ii'’k=iV5 i+y j+£.k\
"=r dx dy dz^ r\r ● /

=p(*i+M+*)=^'-
(vi) Wehavevr"=i~ r«

=1 nr«-> |+l„r»-|;+k nr-|=»r-*(i|+J|+k|)

=nr«-* 1 r ●/ vr=p as in part (ii)


r .
=wr»-8r.

Ex. 8. Prove that f{u) yM«*v J f{py^u\


https://www.pdfnotes.co/
GRADIENT, DIVERGENCE AND CURL 83

Sol. We have v //(») du

=2 /(«)</»} {by def. of gradi^t]'

Ex. 9. Show that


(0 grarf(r.a)=a, («) gra</[r, a. b]=axb,
where a and b are constant vectors. [Rohilkband 1981; Kanpar 87]
Sol. (i) Let a=ai l+Oi j+Os k. Then aj, Oj, <73 are constants.
Also r=A:i+yj+zk.
r.a=fliX+j8y+«8^.
grad (r.a)=v(r.a)=v (aiX+a2:)'+«8^)
=si
9 0 ' 3
^(OiX+hay+asz)+l^(f^iX+^2y+asz)+k^{aiX^a^-{’af^z)
=^7i i+fla j+<7a k=a.-
(ii) grad [r, a, b]==grad {r«(a x b)}, where a X b is a constant
vector
=a xb M in part (i)..
Ex. 10. Tf 4r(x, y, z)=?fiy-{-'fx-i-z^, find at the point
(1, 1. 1). (Agra 1979]
Sol. We have-
9a: ty*^ dz -
■0
0Af (xay-fyajf+z2), i+ ^^o^yj^jfixJrz’^) j

^{2xy+y^) i+(;c*-f 2xj;) j+2z k.


Putting x== 1, ;;=i= I, z= I, we get
at the point (1,1, l)=3l+3j+2k.
Ex. 11. Find grad f, where f is given by
f=x^-)fl-^xz\ at the point (1, —], 2). [Agral977J

Sol. We have grad f=-^=^ i+ ^ j+ ^ k

dy

+[|(^-y+»*“)] k
=(3*»+z«) i+(-3y>) l+2xz k.
Putting 1, f «2g we get
https://www.pdfnotes.co/
84 VECTORS MADE EASY

7/at the point(1, ~1,2)=7 l-3j+4k.


Ex. 12 if w~yz’\-zx-^xy» prove
that
{grad u)»[{grad v)x(grad >v)]~0.. [Kohla^nr 1978]
k
Sol: We have grad ^ ^J+
=11+1 j+lk-i+j+k,

-“I‘+ i l+2z k
a , Sw., dw i , dw.
andgtad w=^l+^j+^ k
=0'+r)i+(2+Jc> j+(JC+J') k.
/. grad u*[(grad v)x(grad w)J=scalar triple product of the
vectors grad u, grad v and grad tv
1 1 1 =2 1 1 1
2x 2y 2z X y z

y+z z-\-x Xr]-y. y-{-z r+x x+y


=2 1 1 I

.'x+y’i-z x+y-\-z x-\-y-]-z * -^2+^8

y+z z+x x+y


--=2(x+y+z) 1 1 1
1 1 1
y+z z+x x+y
=2{x+y+z).0.
the first two rows of the determinant being identical
szi0.

prove that
(0 r^rxv/. «0 F.r=0,(/«)F.F/=0-

SM. We bave viT-^i+l^l+^kand r-xi+Fj+ek


https://www.pdfnotes.co/
GRADIENT, DIVERGENCE AND CURL 8$
(i) rXv/= i j li;
X y z.

dfidx ^fi^y mz
k=F

(ii) F.r=(rxv/)«t [V F=rxv/]


=0, b^use the value of a scalar triple product
having two vectors equal is zero,
(iii) F.v/=(rxv/)«V/=[r,v/, V/]
=0, because the value of a scalai- triple product
having two vectors equUl is zero.
,.A»r
Ex; 14. Prpve that A»|v 7’)= ~

Sol. First prove that v


^r r*
[For its complete solution see Ex. 7, part (iv)]
A»v
Ex^ IS. Prove that.vr-®= —3r"* r. [Agra 1974]
Sol. We have
r=xi+;;jH-2rk and r=| r| =y(x>+/+za)
so that ra=;c3+;;8+23.

Now vr~* ^
dr dr
.= _3r-4^|._3^ dz

-HI'+I'+M- ...(1)
Differentiating both sides of r^=x^+y^-i-z^ partially w.r.t.
X, we have
dr
?L_f.
2r^=2x or dx~r‘

Similarly 0=^and dz r
from (1), we have
https://www.pdfnotes.co/
86 VECTORS MADE EASY

yr-®=—3r-*

— —3r“® (jfi+;)j+zk)=—3r-® r.
Ex. 16. Prove that ^<ffdt==d4>,

Sol. We have v^= ?^i+|f


bx 8;^ j+^k.
dz ...(1)
Also r=xi+yj+rk =► dr=dx \-\-dy \-\-dz k. ...(2)
From (1) and. (2), ^^●dr

=(i ‘+ i *+‘'>’1+* «
=i ‘'●‘+1 ‘»+| <fe=</f
Ex. 17. Show that
wAere r=xi+;>j+zk

and ^ iJ a function of x, y and z.

Sol. We have V^=|^«+ ^ J+ ^ ...(1)


</r dx, dy. dz
Also r=xi+>>j+2k W"d^ ds^^ ds'^' ...(2)
From (1) and (2), we have

*-<l'+l'+t‘HS‘+ti+S'‘)
ds

™ 8xds by ds"^bz ds~'ds


Ex. 18. p and p are two scalar point functions such that
p is a function of p; show that
dp
VP=^VP.

Sol. Wehave w=^i+^j+|k. ●..(1)


Since p is a function of ^/therefore
f ^ ?F f[p£P
8x“^8x’ ^“^0/ 0z^ <//) bz
from (1), we have
If
Vp=
0JC fljp Sj' ● dp bz

\ ●
https://www.pdfnotes.co/
GRADIENT, DIVERGENCE AND CURL 87

az y^dp VP*
\i
~-dp\dx^^
Ex. 19. rf <f> -=:(3r2-4rV8 f 6r-*/3), show that
V<i>=2 (3-r-*/«-r-’/®) r.
Sol. We have r=xi+>'j+2:k and i 1- 1 - Vix^+f+z^)
so that r®
Now tf> is a function of r.

[See Ex. 7 part (i)]


-[6r-4.|r-»/H6.(-i) r-*l^]
1
=(6r-2r-i/2-2r-‘/«)^ r *.* “r
-(6-2r-3/2-2/-’/») r-2(3-r-3/2
Ex. 20. (0 Interpret the symbol a»v.
(ii) Show that(a*^)
(Hi) Show that(a*^) r—a.
Sol. (i) Let a=aii -f Then^
a*v
=(«.i+ad+^).(i|j+j.|;+Jc|)

Thus the symbol a«v stands for the operator


a , a , 8
dx^^^dy'^^^dz

(ii) (a.V) 4- - («1|;+'>a|;+‘'. *■


Also 00
a. V#=(a.i+aJ+aak).(^ i+ 63'

Hence (a*v) a»V4>.


/ 0 , 0 , 0\
(iii) (a.v) r-
0-zj'
8r , 0r , 8r
0J? oz
0r dr
But rr~.Yi+>'j4-2k. .* ?r.k
dz -
■ dx dy
(a.v)r=«ii-fa,jH-<73k=a. .
https://www.pdfnotes.co/
88 VECTORS MADE EXSY

§ 5. Level Surfaces.
Let f{x» y, z) be a scalar field over a region R. TThe points
satisfying an equation of the type
f{x, y;z)~c,(arbitrary constant)
constitute a faniily of surfaces in three dimensional space. The
surfaces of this family are called level surfaces. Any surface of
this family is such that the value of the function/at any point of
it is the same. Therefore these surfaces are also called iso-f-sur~
faces.
Theorem 1. Letf{x, y,z) Jbe a scalar field over a region R.
Then through any point of A there passes one and only one level
surface.
Proof. Let z^ be any point of the region R. Then the
leveLsurface f{x, y,z)=f{x^, y^, Zi) passes through this point.
Now suppose the level surfaces/(.v, y,z)~Ci&ndf{x, y, z)=--Ca
pass through the point yx,z^. Then
/fe and yv 2i)=C2.
Since/(x, y, z) has a unique value at(Xj, yi, Zj) therefore we
have c,.
Hence only one level surface passes through the point
(.Xx,yi,z;).
Theorem 2. Yfis a vector normal to the surface f(x, y» s^^c
where c is a constant. [Agra 1968; Kerala 75]
Proof. Let r=,xi+yj+zk be the^position vector of any point
P {x, y,z) on the level surface f(x,y,z)~c. Let
Q (a:+8a:, z+8z)
be a neighbouring point on this surface. Then the position vector
of e=r+8r=-(jf+8-^)i+Cv+Sy)j+(^+8z)k.
Pe=i.(r+8r)-r=-8r=8A:i-i-8yj+8alc.
As Q-^P, the.line PQ tends to tangent at P to the level sur
face. Therefore dr~dxi-{-dyi-^dzk lies in the tangent plane to the
surface at P.
From the differential calculus, we have
https://www.pdfnotes.co/
GRADIENT, DIVERGENCE AND CURL 89

&3

Since/{x, y,^z)=constant, therefore df=0.


V/*<fr=0 so that v/is a vector perpendicular to </r and
therefore to the tangent plane at P to the surface
f{x, V, z)=c.
Hence v/is a vectdr normal to the surface/(x, z)=c.
Thus if f{x, y;z) is a* scalar field defined over a region R,
then V/at any point {x, y, z) is a vector in the direction of nor-
mal at that point to the level surface f{x,y, z)=c passing through
that point.

§ 6. Directional Derivative of a scalar point function.


[Agra 1972; Kolhapur 73; Bombay 70j
Definition. iW /{x, y, z) define a scalarfield in a region R
and let P be any point in this region. Suppose Q is a point in this
region in the neighbourhood of P in the direction of a given unit
tk
vector 2k.

Then lim fi.Q)-f{P), if it exists,is called the directional


Q-^P PQ
derivative offat P in the direction of 9.
Interpretation of directional derivative. Let P be the point
(x, y,z) and let Q be the point (x+^x, y-\-By, z+Bz). Suppose
PQ^Bs. Then is a small element at P in the direction of
a^ lf Bf=f(x-[-Bx,y-jrBy,z-\-Bz)-f{x,y,z)=^f(Q)-f(P), then
8/
represents the average rate of change of / per unit distance

in the direction of a. Now the directional derivative of/at P im

l-l
the rate ofchange of/with respect to distance at point P in the
direction of unit vector a.
Theorem 1. The directional derivative ofa scalar fieldfata
point P (x, y, z)in the direction ofa unit vector a is given by
df ^ ■
^=V/-a.
[Allahabad 1982;Poona 70]
https://www.pdfnotes.co/

90 VECTORS MADE EASY

Proof. Let fix, y,z) define a scalar field' in the region R.


Let r= YH-jj+2^k denote the position vector of any point
P{x, z)in this region. If s denotes the distance of P from some
fixed point A in(the direction of a, thqn 8s denotes small element
IS a unit vector at P in this
at P in the direction of a. Therefore ^‘
dr
direction i.e. =a.

dr dx. , dy.. dz,


But r— -f j - ds
ds * -y k=a.
Js^^ds
dy
Now V/«a= dx 'f cj’ ^ dz) + r¥'+.s‘‘)

^dxds'^dyds'^dzds
—^^directional derivative of/at P in the direction of a.
ds
Alternative Proof. Let.g be a point in the neighbourhood
of P in the direction of the given unit vector a. If I, m, n arc
the direction cosines of the line P0, then/i+mj+nk=the unit
vector in the direction of PQ=a. Further if PQ- 8s, then the.
co-ordinates of(> are {x+l8s, y+ni8s, z-\ n8s). Now the directional
derivative of/at P in the direction of a is
lim f(Q)-f(P)
=^Q-^P PQ
lim fix ^I8s, y-\-m8s, z+n8s)-f(x, y, z)
~8s-^0 85

lim f(x.y,z)x (.■It*!.* 8


85->-0 85
on expanding by Taylor’s theorem

='05+'"a?+'’ dr

Theorem 2. If n be a unit vector normal to the level surface


https://www.pdfnotes.co/

GRADIENT, DIVERGENCE AND CURL 91

f{Xy y, z)~c at a point P(x, y, z) and n be the distance ofP from


somefixed point A in the direction ofn so that ^n represents element
of normal at P in the direction of n, then
df -
gradf^ dn n.
[Agra 1971]
Proof. We have grsidf- '^f—
dx^ dy^^dz*^'
Also grad/is a vector normal to the surface f{x, y, z)
A
Since
n IS a unit vector normal to the surface/(;c, z)=c, therefore let
grad f~ A n, where .4 is some scalar to be determined.

Now ^ - directional derivative of/in the direction of n

~ V/.n

=>4n* n [V v/=^grad f=An]

/. grad/=v/=^n.

Note. If the vector n is in the direction of /increasing, then


df . ..
^ IS positive. Therefore y/ is a vector normal to the surface
f(x‘> y, z)=c in the direction of/increasing.
Theorem 3. Gradf is a vector in the direction of which the
df
maximum value of the directional derivative of fi.e. -t;
ds occurs,
[Agra 1971]

Proof. The directional derivative of/in the direction of a


is given by /*»

V v/=-T-n
dn
df ,A A

dn(n*a)
df
dn cos 6, where 6 is the angle between a and n.
https://www.pdfnotes.co/

92 VECTORS MADE EASY

df
Now df is fixed. Therefore Tn cos 0 is maximum when
cin
cos 0 is maximum i,e., when cos 0=1. But cos 0 will be 1 wheu
the angle between a and n is 0 /.e. when a'is along the unit normal
vector n.
Therefore the directional derivative is maximum along the
normal to the surface. Its maximum value is
df
dn=jgrad/l.

§ 7. Tangent plane and Nonnal to a level surface.


Tofind the equations of the tangent plane and normal to the
surfacef{x,y,z)=c.
Let f{x, y,z)=c be the equation of a level surface. Let
be the position vector of any point P{x, y, z) on this
surface.

Then v/= vector along the normal to


the surface at P i:e. v/is perpendicular to the tangent plane at i*.
Tangent plane at P. Let R=A'i-f-T j+Z k be the position
vector of any current point Q {X, T, Z)on the tangent plane at P
to the surface. The vector

i>e=R-r=(2r-x:) j+(Z-r)k
lies in the tangent plane at P. Therefore it is perpendicular to the
vector v/.
(R—r)«v/='0
or [(X-*) H-(r-j-)j+(Z-z)k]. i+ 0
)“

»y ...(1)
is the equation of the tangent plane at P.
Normal at P. Let R=2Ti+Tj4-Zk be the position vector
of any current point Q {X, Y,Z) on the normal at P to the
surface. The vector ^=R—r=(A'-Jc)i+(T—>^)j+(Z—r) k lies
along the normal at P to the surface. Therefore it is parallel to
the vector y/.
https://www.pdfnotes.co/
GRADIENT, DIVERGENCE AND CURL 93

(R-r)xv/=0 ...(2)
is the vector equation of the normal at P to the given surface.
Cartesian form. The'vectors

(AT-.X) i+(r-y)j+(Z-z)k and v/=^ i+


will be parallel if

(Z-.)i+(y^,)j+(Z-z)k=p(Ii+|i+|k ).
where p is some scalar.
Equating the coefficients of i, j, k, we get

or X-x Y-y Z-z


¥ “£ " y
dx By dz
are the equations of the normal at P.

Solved Examples
Ex. 1. Find a unit normal vector to the level surface
x^y-ir2xz=A at the point(2, —2, 3).
Sol. The equation of the level surface is
fix, y, 2)=x2v-1-2xz=4.
The vector grad/is along the normal to the surface at the
point(x,
We have grad/-V (^*;^+2xz)=(2x;/-|-2z) i+x* j+2x k.
at the point(2, -2. 3). grad/= -2i^4j+4k.
~2i+4j+4k is a vector along the normal to the given,
surface at the point(2, —2, 3).
Hence a unit normal vector to the surface at this point
-2i4.4i+4k -2i+4j-|-4k IS .
●“ I ~2i+4j+4k I “7(4+id+16)=“»*+«+**'-
The vcctDr -(.-^i+fj-pik) Le., ii-fj-fk is. also a unit
normal vector to the given surface at the point (2, —2, 3).
Ex. 2. Find the unit normal to the surface z=x^+y^ at the
point i-U-2, 5), [Kanpur 1986]
Sol. The equation of the given surface is
/ y,
https://www.pdfnotes.co/
94 VECTORS MADE EASY

The vector grad /is along the normal to the surface at the
point {x, y, z).

We have grad /=^ ^ ^k--2.v- i+2;^ j—k.


/. at the point(—1, —2,5), grad/~ —2i—4 j—k.
_(2 i+4 j+k)is a vector along the normal to the given
surface at the point (—1, —2, 5).
Hence the required unit normal vector to the surface at this
point
grad/ -(2 i+4 j+k) -(2i+4j+k)
--TiiidTr""T^(2 1+4 j+k) I - V(2H4*+1*)
-(2 i+4 j+k)
V(21)
The vector
2 i+4 j+k is also a unit normal vector to the
V(21)
given surface at the point(—1, 2, 5).
Ex. 3. Find the unit normal to the surface
x*-3xyz-\-.z^-\-1 =--0
at the point(1, 1, 1). [Gorakhpur 1988; Allahabad 79],
Sol. The given surface is
/(x, y, z)^x*-3xyz-^z^-\-1 -O. ...(1)

We have grad f- Vi+Vi+afk


.(4.^-3y2) l+(-3;tz)j+(-3jt;-+2z) k.
Now a vector normal to the surface (1) at the point (1, 1* 1)
grad/at the point (1, 1, l)=(4--3)i+(—3) j+(—3+2)k
^i-3j-k.
grad/
the required unit normal vector=^ 1 grad/I

i-3j-k i-3j-k i-3j-k


- 7lZ3PkT^7P+F3F+(-i)^r v(H) ‘
Ex. 4. Find the unit vector normal to the surface x®~/+a=2 ●
at the point (1, -- 1, 2). [Ravi Shankar 1981}
Sol. The given surface is
/(x, 2')=xa-/+z-2=-0. ●●●(I).

W'e have grad /^ dx

=2x i-2;»j+fc.
https://www.pdfnotes.co/
GRADIENT, DIVERGENCE AND CURL 95

Now a vector normal to the surface (1) at the point(I, -1,2)


=grad/at the point (1, — 1, 2)=2 i-J-2 j-fk.
Hence the required unit vector normal to the surface (I) at
the point (1, - -1,2)
2i+2j+k 2i+2Rk
I grad/I “ I 2i+2j+k I
Ex. 5. Find the gradient and the unit normal to the level
surface x'^+y—z=A at the point(2, 0, 0).
Sol. The given surface is
fix. y. z)=x^+y-z~4=^0. ●..(1)

We have grad/=|^ i+^j+|^k=2A:i+j-k.


/. at the point (2, 0, 0)> grad/=-4 i+j—k.
Now a vector along the normal to the surface (1) at the point
(2,0,0)
==gfad/at the point (2,0, 0)=4 i+j—k.
Hence the required unit normal to the surface (1) at the point
(2, 0, 0)
4i+j-k 4 i+j-k 1 .
- I 4i+j-k 1 ~V06+1-1-1)== 3:72
Ex. 6. Find the directional derivatives of a scalar point func^
tion f in the direction of coordinate axes.
Sol. The grad / at any point (x, z) is the vector

The directional derivative of/in the direction of i

=- grad f»\^.
\dx'^Ty^^Wz^y^-Tx.
Similarly the directional derivatives of /in the directions of i
are k are
By
Ex. 7. Find the directional derivative offix. y. z)=x^yz-\-iixz^
arthe point (I, —2, — 1) /« the direction of the vector 2i—j—2k.
[Allahabad 1978]
Sol. We have f(x. y, z)=x*y2-j-4xz2.
A grad f - (2^j-z-f4z«) j+(:r*j;+8xr) k
81—j*" 10k at the point (1, -2,
https://www.pdfnotes.co/
96 VECTORS MADE EASY

If a be the unit vector in the direction of the vector 2i—j—2k,

then a~ 2i-i-2k 2 1,_2


■ V(4+l+4)“"3 3^ 3 *
Therefore the required directional derivative is

grad/● a=(81—j— 10k)«(fi--Jj—fk)=1^+1+


Since this is positive,/is increasing in this direction.
Ex. 8. Find the directional derivative of

at the point(1, 1, —1)in the direction o/2i+j—k. [Agra 1979]


Ans. 8/V6-
Ex. 9. Find the directional derivative of thefunction
f=,x^—y^+2z^ at the point P(1, 2, 3)in the direction of the line PQ
where Q is the point(5,0, 4). [Agra 1987]

Sol. Here grad/=|^ ^


2x i-2;» j+4z k=2i-4j+12k at the point(1. 2, 3).
Also P0=position vector of Q—position vector of P
=(5i+0J+4k)-(i+2j+3k)=4i-2j+k.
If a be the unit vector in the direction of the vector PQ,
^ . 4i-2j+k 4i-2j+k
then a- ^(21)’
the required directional derivative
4i-2j+k
=(grad /).a=(2i—
4j+12k).-| V(21)
28 28
“7(T[)“21 V(21)“3 V'(2I)-
Ex. 10. Find the directional derivatives of the function
f=:xy-\-yz+zx
in the direction of the vector 2i+3j+6k at the point(3, 1, 2).
[Rohilkhand 1980, 81; Agra 75]
Sol. We have /(x, y, z)=x>»4-
grad /=(a//0x)i+(0//0p)j+(0//3^) k
=Cf+z)i+(2T+:«)i+(^+P).k
=(1+2)i+(2+3)j+(3+l)k at the point(3,1, 2)
«3l+5i+4Is.
https://www.pdfnotes.co/
GRADIENT, DIVERGENCE AND CURL 97

If a be tne unit vector in the direction of the vector


2l-}-3j-j-6k, then
a 2i+3j+6k 2l+3j+6k I
I 2l+3j+6k I “v'(4+9+36)~7
,(2l+3J+6k).
/. the required directional derivative
=(gTad/).a =(3i+5j+4k).i(2H-3J+«t)

=i(3i+5j+4k).{2i+3j+6k)
1 45
(64-15+24)=:y.
Ex. 11. Find the directional derivative of ^=xy-\-yz-\-zx in
the direction of the vector i-f-2j+2k at(1,2,0); (Agra 1982]
Sol. We have ^(x, y, z)=xy-\-yz-\-zx,
grad -(a^/ax) i H-(a^/a>^) j+(a^/az)k
={y-{-z) j+(x+3')k
=(2+0)i+(0+l)j+(l+2)k
at the point(1, 2,0)
=2i+j+3k
If a be the unit vector in the direction of the given vector
i-f-2j-|“2k, then
a= i+2j+2k i+2j+2k 1
1 i+2j+2kT“V(l+4+4)=3 ^*+2J+2k).
the required directional derivative
=(grad^).a=(2i+j-f-3k).i(i+2j+2k)
10
=H2+2+6)=~
Ex. 12. Find the directional derivative of
(Xyy.z)=x^yz-{-4x:fi
at the point(1, —2, 1) in the direction of 2\—\~-2k,
[Indore 1983]
Sol. Do yourself. Ana —13/3.
Ex. 13. Obtain the directional derivative of tf>=x^-\-y^ at
the point(2, —1, 1) in the direction of the vector i+2|+2k.
[Allahabad 1975]
Sol. Do yourself. Ans. —3.
Ex. 14. Find the directional derivatives of ^=xyz at the
point(2, 2, 2), in the directions
<0 I (ii) I (iii) l+J+k. [Agra 1981]
https://www.pdfnotes.co/
98 VECTORS MADE EASY

Sol. We have.^ {x, y,z)—xyz.


\\ grad ^={Z4>ldx) i-\-{d^!dy) \+{Z^ldz)\a=yz \+zx}+xy k
.=s4i+4j+4k at the point(2, 2, 2).
(i) Directional derivative of^at the point (2, 2, 2)in the
direction of the unit vector i
=grad ^.i=(4i+4j+4k).i=4.
(ii) Directional derivative of ^ at the point (2, 2, 2) in the
direction of the unit vector j
=grad ^.j=(4i+4j+4k).j=4.
(iii) Unit vector a in the direction of the vector i+j+k
i+j+k i+j+k
I i+j+k I V3
directional derivative of 4> at the point (2, 2, 2) in the
direction of the vector i+j+k

=grad ^/a=(4l+4j+4k).

Ex. 15. F^d the directional derivative of^=xyz at(1, 2, 3)


in the direction of the vector i.
Sol. Do yourself; Ans.6.
Ex. 16. Find the directional derivative of the' function
xy^-\-yz^-\-zx^ along the tangent to the curve x=t,y=t^,z=t^ at
the point(1, 1, 1).
Sol. Let ^(X, y, z)=xy^-^ysfl-\-zx\
Then grad ^=(8^/0x) i+(0^/0;»)j+(0^/ar)k
=(f+2zx\ i+(z«+2xy)j+(x»+2;;z) k
=3i+3j+3k^at the point(1, 1, 1)
-3(i+j+k).
Also for the curve x~t,'y=t^, z=t^, we-have
dxfdt=l, dy{dt=2t, dzfdt=3t\
At the point (1, 1, 1) on the curve x=t^ y=t^, z=‘t^,we
haver=l.
Now a vector along the tangent to the above curve at the
point(X, y,z)
=(dxfdt) i-^(dyldt)^+{dz/dt)k
=i+'2t j+3t»k.
Putting L a vector along the tangent to the curve at the
point(1, 1. l)=l+?|+3k.
https://www.pdfnotes.co/

GRADIENT^ DIVERGENCE AND CURL 99

If a be the unit vector in the direction of this tangent, then


i+2j+3k i+2j+3k
ji+2j+3kr=^“7
the required directional derivative
=a«grad ^ at(1, 1, I)
i-f2j-h3k q /| I {● |.\

Ex. 17, In what direction the directional derivative of


4>=x^yHfrom (1, 1, 2) will be maximuiH.and what Is its m' agnitude!
Also find a unit normal vector to the surface x}y^z=2 at the point
(1,1,2).
Sol. We know that the directional derivative of ^ at the
point (x, y, z) is maximum in the direction of the normal to the
surface^=3constant /.e., in the direction of the vector grad 4>.
Now grad <f,={mdx) \-\-{d^ldy) j+(9^/0r) k
=Ixf^z i -\-2x^yz j+jcV k
=4i+4j-f k, at the point (1, 1, 2).
Hence the directional derivative of ^ at the point (1, U2) will
be maximum in the direction of the vector 4i+4j+k.
Also the magnitude of this maximum directional derivative
=modulus of grad ^ at<l, 1, 2)
= I 4i+4j+k 1 =V(16+16+1)=V(33).
The unit vector along the normal to the surface x^y^z=2 at
the point (1, 1, 2) .
grad ^
,at(l, 1,2>

4i+4j+k 4i+4j+k '


■’T^4j+k 1 - V(33) ‘
Ex. 18. Find the greatest value of the directional derivative of
the function 2x^—y—z^ at the point (2,-1, 1).
Sol. Let ^ (x, y, z)=2x^—y—:^.
Then grad <S=(0^/a;c) i+(0^/0;F) j+(0^/0z) k
=4a: i—j—42® k
=81—j—4k, at the point (2, —1, 1).
Now the greatest value of the directional derivative of 6 at the
point (2.-1,1)
https://www.pdfnotes.co/
100 VECTORS 2ilADB EASY

=>modulu8 of grad ^ at the point {2,— 1,1)


= I 8i-j-4k I =V[8*+(~1)*+(-4)*]=V(64H-1+16)=9.
Ex.19. Find the maximum value of the directional derivative
of yz at the point(1,4,1). [Bombay 1970]
Sol. We have grad <^=(d<f>ldx) j+C k
=2xyz i+jfiz li-jfiyk
=8l+j+4k, atthe point(1, 4,1).
Now the maximum valqe of the. directional derivative of
<fim^3fiyzat the point(1,4, 1)
» I grad ^ at the point<l, 4, I) j
= I 8l+j+4k I =V(8*‘+lH4«)=V(8i>-=9.
Ex. 20. Calculate the maximum rate ofchange and the corres-
porting direction for thefunction ^—x* y^ ^at the point 2i+3j—k.
[Allahabad 1982]
Sol. The coordinates of the point whose position vector is
2l+3J-kare(2,3;-l).
We have grad 4»—iHl9x)i+(34/8!y) j+(8^/3z)k
=2xy»a« l+3xVz« j+4xy28 k
=1081+ 108j~432 k, at the point(2. 3. -1)
=.108(i+j-4 k):
Now the rate of change of^ (tci, the directional derivative
of at the point(2, 3, — 1) is maximum in the direction of the
vector grad ^ at this po*nt i.e.,. in the directioa of the vector
108(i+j-4k).
Also the magnitude of the maximum rate of change=modu-
lus of grad ^ at tho point(2, 3, — 1)
= I 108(l+jT4k) I =108 I i+j-4k I
=108V[1*+1*+(~4)*]= 108V(18)=324V2.
Ex. 21. Find the values of the constants a, b,c so that the
directional derivative of ^^ax^-{-by*-\-cz^ at{\, 1,2) has a maxi
mum magnitude 4 in the direction parallel to y-axis.
Sol We have grad ^ =ib^ldx)i^id4>:dy)j+(8^/0z)k
=2&;c i+2b>'j+2crk
=2o i+2bj+4c k, at the point(1, 1, 2).
Now the directional derivative of^ at the point (1, 1, 2) is
maximum in the direction of the v^tor grad ^ at this point.
According to the question this directional derivative is maximum
in the direction parallel to y-axis i.e., in the direction parallel
to the vector J. So If the direction of the vector 2ai+2bJ+4c k
https://www.pdfnotes.co/

GRADIENT, DIVERGENCE AND CURL 101

is parallel to the vector j, we must have 2a=0, 4c=0 i.e., a=0


and c=0.
Then grad ^ at (1, 1, 2)=26j.
Also the maximum value of directional derivative
= Jgrad^|.
.V 4= I 2bl |, since according to the.question the maximum
value of directional de.**ivative is 4.
/. 2^=4 or b=2.
Hence a=0, b~2, c=0.
Ex. 22. In what direction from the point (1,1, ~1) is the
directional derivative of a maximum 2 Also find
the value of this maximum directional derivative,
Sol. We have grad/=2jci—4;;j+8zk
=2i-4j—8k at the point (1, 1,-1).
The directional derivative of/ is maximum in the direction
ofgrad/=2i-4j-8k.
The maximum value of this directional derivative
=1 grad/HI 2i-4j-8kl-V(4+16+64)-V(84)=2V(21).
Ex. 23. For thefunctionf=yl{x^-\-y^),find the value of the
directional derivative making an angle 30® with the positive x^axis
at the point(0, 1). [Agra 1981]

Sol. We have grad /=|^ i+^j


—2xy
at the point (0. l).

If a is a unit vector along the line which makes an angle 30®


with the positive jr-axis, then
a=-cos 30® i-fsin 30® j>^i+lj,
2
the required directional derivative is

grad/.a=(-j).|^^
Ex. 24. What is the.greatest rate of increase of u=xyz^ at thf
point {i.Q, 3)2 [Agra 1968]
Sol. We have v« =^yz^ i-j-xr® j+2xj;2r k.
at the point(1,0, 3), we have
V«=0 i+9 j+0 k=9 j.
The greatest rate of increase of u at the point(1, 0, 3)
=the maximum value of ^at the point(1,0, 3)
https://www.pdfnotes.co/
102 VECTORS MADE EASY

=1 v« I* at the point(1, 0, 3)
=19jl=9.
Ex. 25. Show that the directional derivative of a scalar point
function at any point along any tangent line to the level surface at
the point is zero.
Sol. Letf(x, y,z)be a scalar point function and let a be a
unit vector along a tangent line to the level surface/(x, y, z)—c.
We know that v/ is a normal vector at any point of the
surface/(x, ^)=c. Therefore the vectors v/ and a are per¬
pendicular.
Now the directional derivative of/in the direction of a
=a*v/=0.
Ex. 26. Find the equations of the tangent plane and normal to
the surface 2xz^—‘ixy—Ax=l at the point (1, — 1, 2).
Sol. The equation of the surface is
f(x, y, z)=2xz^-3xy-^4x=7.
We have grad f—{2z^—3y-\-4) i-3x}-\-4xz k
=7i—3j-|-8k, at the point(1,-1, 2).
/. 7i—3jH-8k is a vector along the normal to the- surface at
the point(1,—1,2).
The position vector of the point(1, --1, 2) is =r=i—j-l-2k.
If R=A!'i+yj+Zk is the position vector of any current point
{X, Y,Z)on the tangent plane at(1, ~1,2), then the vector R—r
is perpendicular to the vector grad/,
the equation of the tangent plane is
(R-^r).grad/=0,
i.e. {(2rH-H+Zk)-(i-j+2k)}.(7i-3j-|-8k)=0,
i.e. {{X-\)i+(r+l)j+(Z-2)k}.(7i-3j-f8k)=0,
i.e. 7(2r-l>-3(T+1):F8(Z-2)=-0.
The equations of the normal to the surface at the point
(I, —1,2) are
X-X r+1 Z-2 X-l r-H-1 Z—2
.u.— -3 “ 8 *
^ ~ d£
dx dy dz
Ex. 27. Find the equations of the tangent plane and normal to
the surface xyz=4 at the point 2, 2). [Meemt 1991 P, Agra 82]
Sol. The equation of the surface is
/(X, y,z)=x;;2—4=0.
We have grad f=yzi-\-xz}-\-xy'k
https://www.pdfnotes.co/

GRADIBNT, DIVERGENCE AND CURL 103

=4i+2j+2k; at the point(1, 2, 2).


4i-f2j+2k is a vector along the normal to the surface at
the point(1, 2, 2).
The position vector of the point(1, 2, 2)is =r -- i+2j+2k.
If R =>fi+Tj+Zk is the position-vector of any current point
(X, Y,Z)on the tangent plane at(1, 2, 2), the equation of the
tangent plane is
fR—r).grad/=0,
i.e. {(Z!+rj+Zk)-(H-2j+2k)}.(4i+2j+2kW0,
i.e. {(X-\)i+(F-2)j+(Z-2)k}.(4i+2j+2k)=0,
i.e. 4(Z-1)4-2(r-2)+2(Z-2)=0,
i.e. 4Z+2r-|-2Z= 12, i.e. 2X-\-Y+Z=6.
The equations of the normal to the surface at the point
(1, 2, 2) are
X-l Y-2 Z-2

d'x dy dz
i.e» X-l r—2 z-2 x-l Y-2 Z-2
4 - 2 1 *
Ex.- 28. Find the equation of the tangent plane to the surface
yz-zx-\-xy-\-5=0, at the point (I, -1,2).
Sol. The equation of the given surface is
f{x, y, z)=yz—zx+xy+5—0
We have grad/=(0//0x) i+(df/dy) j+(0//02)k
={-z+y)i4-(^4-A*) k
= —3l4-3j—2k, at the point(I, — 1, 2).
— 3i+3j —2k is a vector along the normal -to the surface
fix, y, r)=: 0 at the point(1, — 1, 2).
The position’vector of the point(1, — 1, 2)
=i—j4-2k=r, say.
If R==^yi4-Tj4-Zk is the position vector of any current point
iX, r,Z)6a the tangent plane at(1, -1,2), the equation of the
tangent plane is
(R—r).grad/=0
i.e., {(Zl4-rj-fZk)-(i-j4-2k)}4-3i+3j-2kj=0
or iiX-l)i+(r4-I)j+(Z-2)k}.(-3H-3j-2k)=0
or . -3 iX-1)+3(F-f l)-2(Z-?)=0
or -3Z+3F-2Z4-34-34-4=.0
or 3F-3F4“2Z= 10.
https://www.pdfnotes.co/

m VECTORS MADE EASY

Ex. 29. Find the equations of the tangent plane and normal to
the surface at the point(2, — 1, 5).
Sol. The equation of the given surface is
f{x, y, z)=xa+/-2T=0. ...(1)
We have grtad f=^{Zfj^x) i+(d//0;/) j+(0//0z) k
=2^+2j;j-k
=4i—2j—k,at the point (2, —I,5).
/. 4i~2j—k is a vector along the normal to the surface(1)
at the point (2, — 1, 5)/.c., perpendicular to the tangent plane
to the surface (1) at the point(2, — 1, 5).
Hence the equation of the tangent plane to the surface(1) at
the point(2, — 1, 5)is
{(^i+W+^k)-(2i-j+5k)}.(4i-2j-k)=.0
or {(x-2) 1)j+(r-5)k}«(4i-2j-k)=0
or 4(;c-2)-2 0;+1)-(2-5)=0
or 4x—2y—z=5,
The equations of the normal to the surface (1)at the point
(2, 1, 5) are
x—2 z—5
dffdx~"^fldy'~"dffdz
I.C.,
x-2 y+l z-^5
4 T"=-=T
Ex. 30. Find the equations of the tangent plane and normal to
the surface x^-\-y^ -f2® —25 at the point (4, 0, 3).
Sol. The given surface is /{x, y,
...(1)
We have grad /=2xi+2>i+2zk
=8i+0j+6k, at the point(4,0, 3).
the direction cosines of the normal to the surface(I) at
the point (4, 0, 3)are proportional to 8, 0, 6.
Hence the equation of the tangent plane to the surface (1)
at the point(4, 0, 3)is
8(x-4)+0(y-0)+6(z-3)=0 or 4x+3z=25,
The equations of the normal to the surface (1) at the'^oint
(4,0, 3) are
x—4 y—O z—3^ x—4 y z—3
8 ■■ 0 ~ 6 4 ~0~ 3
Ex. 31. Given the curve x^+y*+z*= 1, x+y-\-z—\ {intersec
tion oftwo surfaces), find the equations of the tangent line at the
point(1,0, 0). [Agra 1983J
https://www.pdfnotes.co/

GRADIENT, DIVERGENCE AND CURL 105

Sol. A normal to 1 at(1,0, 0)is


grad/i-grad(^a+;;a+z^)=2:d+2j^j+,2rk=2i.
A normal to x-\-y+z= \ at(1,0,0) is
grad/a=grad (AJ+:F+^)=^^lH-lj+lk=ri+j+k..
The tangent line at the point (1,0,0) is perpendicular to both
these normals. Therefore it is parallel to the vector
(grad/,)x(grad/a).
Now(grad/i)x(grad/a)-2i x(i l-i+k) .
=2l>ej+2ixk=2k-2j=0i-2j+2k.
/I equations of the line through the point
(1,0,0)and parallel to the vector 0i—2j+2k.
The required equations are
X-l y-0 Z-0
0 “ ~2
i.e.. y__z
-1 ~ 1 *
Ex. 32. Find the angle between the surfaces
and 2=x5»H-:»;a-3 at the point (2, -1,2).
.[Meerut 1991S; Kanpur 78,80]
Sol. Angle between two surfaces at a point is the angle
between the normals to the surfaces at the point.
Let ^2 and/a=jc«+;;a-2.
Then grad /,=2xi+2yj+22k and grad/a=2xi+2yj-k.
pointy-7®2f
ni=4i-2j+4k and na=4i-2j-k.
The vectors Oj and Ua are along normals to the two surfaces at
● the point(2, —I,2). If6 is the angle between these vectors then
01-08=^^I Ui I I ng J cos 0
or
I6-f4-4=-\/(164-4+16)v'(16+44-l) cos 6.
16 8
cos 0 or 0=cos“*
6V(21) 3V(21)
Ex. 33. Find the angle ofintersection at {4, —3,2)ofspheres
^+>'^+^=29 and j;*+2^+4x—6jv—82—47=0.
Sol. Let/i~ ^2_ 29 and f^=x^ 4x
-6y~82-47.
Then grad>;=2x i-|-2y j+2z k
and grad/a=(2x+4)i+(2y-6)j+(22-8)k.
Let n,=grad/i at the point(4, —3,2)
and n2-=grad/a at the point(4, t-3, 2). Then
ni=8i-6j+4k=2(4i-3j-|-2k)
and
no==12i-12j-4k=:4(3i-3J-k).
https://www.pdfnotes.co/

106 VECTORS MADE EASY

The vectors Oj and are along normals to the two spheres at


the point(4, —3,2)and the angle 0 between these two vectors is
the angle of intersection of the two spheres at the point(4, —3,2).
We have cos 0 nj.Da 8(12+9-2)
I B, 1 1 Bj|-2V(16+9+4).4V(9+9+1).
19
~V(29).V(19)
or 0=cos-V(19/29).
Ex. 34. Find the constants a and b so that the surface
ax^‘—byz={a-\-7) x will be orthogonal to the surface 4x^y+:^=4 at
the point(1,-1. 2).
Sol. The given surfaces are
fx^ax^—byz—{a-\-2)x=0 ...(1)
and /3=4jc®j^+z’*—4=0 ...(2)
The point(1, — 1, 2) obviously lies on the surface (2). It will
also lie on the surface (1) if
a+2h-(a+2)=0 or 2h-2=0 or 6=1.
Now grad /i=[2afx—(u+2)] i -bz\—by k
and grad ft -Sxy i+4jc® J+3z2 k.
Theil Dji=grad fi at the point(1, —1,2)=(d—2)i—26j+6k
and D3=grad /a at the point(1,-1,2)=—8i+4j+12k.
The vectors Dj and are along the normals to the surfaces
(1) and (2) at the point (1, —1, 2). These surfaces will intersect
orthogonally at the point (1, —1,2) if the vectors Oj and are
perpendicular i.e,, if Dj●0^- 0
or -8(u-2)-86+126=0 or 6-2a+4=0 ...(3)
But b— 1, as already found.
Putting 6= 1 in (3), we get u= 5/2. Ans. 0=5/2,6=1.
Ex. 35, If F and f are point functions, show that the coni’
ponents of the former tangential and normal to the level surface
V/x(Fxy/) and (F»V/)V/
f=0 are
(v/y {VfY *
Sol. The unit normal vector to the surface/=0 is
V/
“I V/f
/. The magnitude of the component of F along the normal
=F. V/
IV/I
https://www.pdfnotes.co/
GRADIENT, DIVERGENCE AND CURL 107

the component of F along the normal


Jf. ^ (F»V/) (F»V/)
V/
r i^riv/i I v/l“ (v/)>
Consequently the tangential component of F is
F (F»V/) F-(F.v/) V/

v/x(Fxy/)
(V/)
2 [V ax(bxc)=(a.c)b-(a.b)c].
§ 8. Divergence of a vector point function.
Definition. Let V ))e any given differentiable vector point
function. Then the divergence ofV, written as.
V*V or divV,

is defined asdiv\=S;’y=^i^+i|;+k Ij.V


_. 8V 8V «V 8V

[Sagar 1983; Kerala 74; Bombay 70]


It should be noted that div V is a scalar quantity. Thus the
divergence ofa vector pointfunction is a scalar pointfunction.
Theorem. IfV=V^-\-V^4-V^ is a differentiable vector point
function, then div
ox dy oz
Proof. We have by definition
. av
divV=V*V=i» d;c +j-
dy dz

Now V=f'.I+Kd+Kak; .●
● 3x-dx
ij. i+25 k
dx ix
av
.V i*~
0JC
i+ dx j+8Z.
\dx
dV dV.2
Similarly ]● and
dy ^ dy dz dz

Hence div V=?:*+^+^»-


dx^ dy ^ dz
Solenoidal Vector. Definition. A vector V is said to be sole-
noidal if div V=0. [Meerut 1991S; Calcutta 75]
§ 9. Cnrl of a vector point function. Definition.
Let f be any given differentiable vector point function. Then the
curl or rotation of f, written as V X f, curl f or rot f is defined as
https://www.pdfnotes.co/

108 VECTORS MADE EASY

=ix
g3^+jx^+kXg^-SlXg^.
[Sagar 1983; Bombay 86; Paii|ab 88]
It should be noted that curl f is a vector quantity. Thus the
curl ofa vector pointfunction is a vector pointfunction.
Theorem. //f=/i i+/a j+/a k is a differentiable vector point
function, then
k.

Proof; We have by definition

curl f=vxf=ix^+jx^+kx^^
=ix
4(/i *+/» i+/> «+i (/« j+/»
+kx|:(/,i+/,j+/3k)
iy/%44.%i+%k
3/b 3/s
‘■►rfi+e‘)

(dA 9A\ ,./9/i ^8\ ., 1% e/i\ k.

Note. It Should be noted that the expression for curl f can be


written immediawly if we treat the operator v as a vector quantity!
Thus

Curlf=vxf=(i l^+j |;+k g)x(/xi+/sj+/sk)


i j k

0 a a .
dx 0y 0Z

ft /a fz
0 a i 0 a j+ 0 a k
dz 0JC dz dx By

fz fz fi fz fl fz
https://www.pdfnotes.co/
GRADIENT, DIVBROBNCB AND CURL 109

But we must take care that in the expansion oi the determi-

nant.^e operators ^ must precede the functions fx,


Inrotational vector. Definition. A vector f is said to be irrota^
tionalif^xf==0. [Meerut 1991 S]
§ 10. The Laplacian operator
The LMplacian operator y* is defined as

If/is a scalar point function, then

It should be noted that y yis also a scalar quantity.


If f is a vector point function, then

^ 3;ra+0y»+Oz«*
It should be noted that y*f is also a vector quantity.
Laplace’s equation. The equation is called Laplace's
equation. A function whirh satisfies Laplace’s equation is called
a harmonic function.

Solved Examples
Ex. l. Prove that div.T=^Z.
[Agra 1S178; Rohilkhand 81; Eanpnr 75; Gorakhpur 88]
Sol. We have r=xi+yj+zk.

By definition, div r=y.r ●r

=i»+,.|E+k3i
3jc ' 82

=i.|+j.j+k.k r ●.- ?£-» *_i ?f_k


I 3x '’ey->’Zz~^
=1+1 + 1 =3.
Ex. 2. Prove that curl r=0,
[Agra 1968; Kanpur 75. 79; Rohilkhand 76; Gorakhpur 88]
Sol. We have by definition
https://www.pdfnotes.co/

110 VECTORS MADE EASY

Curl r=vx<^
=(‘fe+i|?+4z)xr
=’x|+i><l+'‘X3-;-
Nowr^xi+jKl+A. k.

Curlr=ixi+jxj+kxk=0+0+0=0.
Ex. 3. Jf f=J^y 1—2x2l+2y2 k> find
(i) div t (ii) curl f, (Hi) curl curl f. [Agra 1986]
Sol. (i) We have

diy f=v*f
=(* 1;+^ !)●<**>’ ‘-2*^
=fe h (~2xz)+ I (2yz)=2xy+0+2y=2y (a:+1).
(ii) We have curl f=v x f= i j k
a d 6
ax dy dz

x^y —2xz 2yz

- }y k (-H '-[h k <*“^>] 1


=(2z+2x) I -0 j+(-2z-x2) k=(2A: f 2z) i-(x2+2z) k.
(lii) We have curl curl f=vx (v X f)
=VX[(2x+2z)i-(x*+2z).k]
i j k

i. L I
ax dy dz

2x+2z 0 z?-2z

= Ty [h (-^-2^)- k (2*+2t)] j
+[o-|;(2*+2z)]k
=0 i-(-2x-i) J+(0-0) k=(2*+2) i.
https://www.pdfnotes.co/
GRADIENT. DIVERGENCE AND CURL 111

Ex. 4. Find the divergence and curl of the vector


i+2xy S+(y^-xy)k. [Agra 1982]
Sol. We have div f=7*f

=h^^-J^+l^(2xy)+yy‘-xy)
=2x-i-2x+0=4x.
Also curl f=vxf

=(* b+J i+2xyj+(;y^^xy)k}


i j k
0 0 0
dx dy dz
x^—y» 2xy y^—xy

= Xy(i^-=^y)-li<.^y)] i+[4 J
+[|(2*.V>-|;(*«->»)] k
=:[(2y^x)~0] i+(0+;>») j+(2;;+2;;) k
=(2;'-^)i+;J+4yk.
Ex. 5. Find div F and curl F where
¥=grad {}^-\-}^:\-z^-2xyz),
Sol. We have F=grad (x^-^y^-\-z^~3xyz)

=1* ^i^-Vf+^-^xyz)-^} |;(x8+y»+z3-3.v;;z)


+k^(x^+y^+z^—3xyz)
={3x^-3yz)i+{3f--3xz)l4.(3z^-3xy) k.
NowdivF=V*F

r
=-6x+6y-\-6^=6.(x+y+z).
Also curl F=v xF
i j k
0 0 d
dx dy dz
d7^--3yz 3y<^^3xz 3^-~3xy
https://www.pdfnotes.co/
112 VECTORS MADE EASY

\
fa
~w
+[?-(3x>-3;)z)- |^(3z*-3ai3-)J j

+[|j(3y‘-ixz)-^(3**-3;-z)] k
=(-3*+3x)i+(-3;>+3;-)J+(-3r+3z)k
=0i+0j+0k=0.
Ex. 6. Given ^=2jc® z*,find div {grad ^).
Sol. We have grad ^=grad
=1
(2z»j.V)+j L(2x»/z4)+k I(2x»/z*)
=6x^y^z* i+4jfiyz* j+8x®;^®2* k.
Now div (grad ^)=v*(grad ^)=v»(6xV^ i+4^^yz* j
+8x:/2®k)

=|j(6*‘J«Z‘)+|;(4ifiyz>)+i(g^yV)
^\2xy^z^-\-4x^z*+24x*yh^.
Ex.7. Ift=xy^ i-|-2xV^ j—3yz^ k,find div f and curl f.
What are their values at the point(1, 1, 1)?
[Rohilkband 1982]
Sol. We have div f=v ●f

—y»-{-2x^z—6yz,
div fat(I, -1. l)=(-l)*H«;2.1M-6.(-l).l=l+2+6=9.
Also curl f=vxf= i j k
8 8 8
8x dy d2T

xy^ 2x^yz -Syzf

=[|; ]‘-[I;(-33’^’)-^(*3^]i
+[|(2*“3'z)-|(*;-»)] k
^{-3z^^2x^y)i-(0-0)iH^xyz-2xy)k
=—{3:^+2x^y)i+(4x;;2r—2xj|?) k.
curl fat Cl. -1. l)=^[3.ia+2.P.(~l)]i
+[4.1.(-l).l~2.1.(~l)Jk
https://www.pdfnotes.co/
GRADIENT, DIVERGENCE AND CURL 113

=-i-2k.
Ex. 8. // F=x^ i-j-xy^zk, find div F, curl F at
(h-Ulh [Garhwal 1979; Madras 78]
Sol. We have div F=v»F

=2xz-~6y'^:^-\-X}^,
divFat(l, -1, l)=2.1.1-6.(-l)Ma+l.(~l)«
=2-6+l=-3.
Also curlF=v xF= i j k
8 8 8
dx dy dz
x^z -2t^ xy^z

= Jy(*A)-4(-2A“)]i+[|,(X^)- I
+
=x{2xyz-\-4y^z) i^{x\y^z)j-f-(O-O) k
=2(xyz4-2fz) i-{-'{x^'-fz)I
curl.Fat(l, -1, l)=2[l.(~l).H-2.(-l)M]i

, =2(-l~2)i+(l-l)j=-6i+0j
\ =—6i.
Ex. 9. 7/ F={y^-{-z^-x«) i+(za+x«-;j;8)j+(;c«+>;a~^) k.
find div f cttr/ f. '
Sol. We have div f=v.f

=‘^2x--2y—2z=x—2(x+y-{-z).
Also liurl f=vxf ■ /

i j ■
k /

d d \\
dx dz

y^+z^—x^
https://www.pdfnotes.co/
114 VECTORS MAOE EASY

+[4(2“+*“-^)-^(v>+2:“-x») k
H2y-2z)i+(2z-2*)j+(2j:-2;-) k
^2(y-z)i+2(z-x)i+2(x-^y) k:
Ex. 10. lft=^x-\-y+ J) i+j+(—JC—k,prove that
f*cw/f=0. [Kanpnr 1988; Agra 86]
Sol. We have curl f=v x f
i j k
0 d a
ax dy dz

x+y+l 1 -x-y

= Fy <--^-y)-Fz(*)]*+[fz(*+^+ i

=(-1-0)i+(0+1)j+(0-1)k=-i+j-k,
f.curl f=[(z+;)»+1) i+j+(-*-;i-) k].(-i+j-k)
=(*+^+1).(-1)+1.1+(-x-y)-(-1)
=-:-x—y—l+l+x+y—0.
Ex. 11. and v~xi?—2y, then find
grad[(grad u).(grad v)].

Sol. We have grad u=i ^ Qx^y)+^fPx^y)


=6xy i+3A-2 j+O k—6xy i+3x*j.

Also grad v=i^(;vz2_?;;)+j i(xz2-2>»)+k ^(xz^-2y)


=z^ i—2j+2xzk.
/. ● (grad «)»(grad v)=--(6xy i+3x® D*(z^ i—2j+2xz k)
=6xyz^—6j^.
Hence grad [(grad M)»(grad v)]=grad (6xy^—6x^)

=i^(^xyz^—6x*) {6xyz^~6x^)+k^{6xyz^—6x2)
=(6;^z2-12x) i+(oxz2) j+(12x>»z) k.
Ex. 12. If u=x^-)^-\-4z,show that m=0.
u
So!. Wehavev“»=g+,5+g
02« 82« 02m
https://www.pdfnotes.co/

GRADIENT, DIVERGENCE AND CURL 115

Now {x^-f+4z)=2x.

■* dx^~dx\^x)~dx^^^~^‘

-2y.
. d^u d /du\ d,
*● dy^-dyidyl'-'^^'^^y^ ^*
Finally |^=^(^-1'H4^)=4.
. d^u_d(du\ a .

. d^u dht 0®« ,, ^ , .


●● w»+ap+^~^~^+®=®-
Hence y2«-o.
Ex. 13. Iff=fi i+/g j4/a K show that
V*f=V/i*i+V/a*j+V/8‘fe-

Sol. Wehavev/i-l‘l+1 j+|k.

●●● '^>i-‘=C4‘+|i+i^)-‘=i-
Similarly V/a*j= ~ and v/s‘k= Zz’

.*. ^ V/,«i+v/2*j+V/a‘k

-g+f+S-«-'-»■>■
Ex. 14. Prove that v(r®r)=6r®.
Sol. We have r=xi+jjH-zk.
.% r»r=r8(xH-yj+^k)='^-rf+r®yj+r8zk.

.-. v.(F«r)=div(r»r)=|j (r» x)+i (/^ j-)+^ (,’ 2),


dr dr dr
=r8+3ra xgj+r^+Sr* ys^+r^+Sr*
0y 2 02
^

=3'*+'^ (4+^|+4) ...(1)


Now r2=jc24.y8-j.jj2.
dr X
.?. 2r~=2x or
d^ dx~F'
116 https://www.pdfnotes.co/
VECTORS MADE J^SY

Similarly
. ?p=-
Zy r and Zz r

\ from(l), v(r^)=3r«+3r*(;c|+;^/+r.£ ■I
=3r«H-3ra j=3r«+3f».p
=3r8+3r«=:6r».
Ex. 15. Find the constants a. b, c so that the vector
F~(x-]r2y-\^az) i-\-{bx—2y—z)\-\-{Ax+cy-\-2z) k is irrotational.
Sol. The vector F is irrotational if curl F=0.
We have curl F=v x F
i j k
a 0 a
a^: zy 0i

x+2y-\-az bx—Zy—z Ax-\-cy-^2z /

=(^?+l)i+(«-4)j+(h-2)k.
/. curl F=p (c+1) i+(fl_4) j+(6-2) k=0
c+l=0, a-4=0, a-2=0
=» c=—1, a=4, 6=2.
Hence the vector F is irrotational if o=4, 6=2, c=—1.
Ex. 16. Determine the constant a so that the vector
V=(x-\-2y) i+(y^2z) j+(x+02) k is solenoidaL [Kanpur 1978]
Sol. A vector V is said to be solenoidal if div V=0.

We have div V=y.V=L (x+3y)+^^ 0-2*)+gJ (x+az)


=l+l+a=2+a.
Now div V=0 if 2+a=0 i.e. if o=-2.
Ex. 17. Show that th^ vector
V~{sin yArz) i+(* cos y-z)\-\-{x—y) k is irrotational.
Sol. A Vector V is said to be irrotational if curl V=0.
We have curl V=vxV
https://www.pdfnotes.co/

GRADIENT, DIVERGENCE AND CURL 117


i j k
d a d
dx dy dz

sinj>+z x&ysy—z X'-y

●0 3 *

-(—1+1)1—(I-l) j+(cos cos y) k=0.


V is irrotational.

Ex. 18. If\ Is u constant vector, show that


(/) </iVV=0, («) curl V=9.

Sol. (i) We havedivV=:i* 8v, , av,. av


ax+l'83r+‘“8F
=i.0+j.0+k.0=0.

(ii) We have curl V=ix~+jxt—+kx~


ax By Bz
-=ix0+jX0+kx0=0.
Ex. 19. If VL is a constant vectorsfind
(/) rfiv(rxa), fRohilkhand 1981, 84]
(I'O curl{tXsi). fRohilkhand 1984; Indore 83]
Sol. We haver ==xi+>j+2k.
Let a=Ui i+Oa J+agk. Then the scalars ^2* are all
constants.
We have rxa= i j k
X y z

«1 «2 Oa

=(oy;—tfgt) i+(ajz—agx) i+(a^—a^) k.

(i) div (r X a)=— (aay-aaz)-h- ~ (a^z— a^xX-h ^ (d^x-a^y)


=0+0+0=0.
(ii) curl (r>«0=V‘X(Txa)
https://www.pdfnotes.co/
118 VECTORS MADE EASY

j k
a a a
dx dy dz

a^y—OiZ a^z-r-a^x a^x—a^


^9 0 1 P0
“15' 95 J (o^-oiy)

-'^(«»>'-<V) i+[|^ (<>sJ'-a»2) k


= —2aji—2flaj—l«8k= —2(flii+fl8j+flak)= —2a.
Ex. 20. {\+\-\-]L\find curl
[Meerat 1991P; Kanpur 87; Agra 83]
Sol. We have curl V= i i k
a a a
dx az
exyz e*y‘ gxyz

=h ‘+[1 j
+ |^(c*^0-|7.(e*^0Tk
dy
=e*y‘(xz-xy)i+e*y*(xy-yz)j+e*J'*(yz-^xz)k.
Ex. 21. Evaluate div f where
t=2j^zi^xj^zl+3y^xk. [Kanpur 1988]
Sol. We have

dhr f= Vf=(i|j+j|;+k|)-(2*>rf--VJl+3/A:k)

—4xz—2xyz+0=2xz(2—y),
Ex. 22. Show that v*(JC/r®)=Q.
[Meerut 1991 P; Rohilkhand 92]
sol.
(p)={l?+|ra+|?)(?)●
Now
aJi:* \r*/“ax\aJCV'’®/J r* dxf
Lp 2f^ir- r*=?jc*+j>®+2* gives
ar x'
ax\f» r* r|L dx r\
https://www.pdfnotes.co/

GRADIENT^ DIVeRGENCB AND CURL 119

. I5Jc«ar
dx\f» r<if~ r*dx r« + ^
3x 6x ISx^x 9x I5x^
*= r*r r» ^ r" r« r»
Again ?L/£ULJL/£U_i_J 3Ar0rl
dy^\r^} 3;'\a:v\r«/f dy\ r* dyf
",, dr y"
dy\ r* Ff - * ^y~r _
L( ^j^y\ 3^ ISxydr 3x . ISjcj;*
dy\ r^J r® r® dy r®
^ 3jc 15jcz2
Similarly
3z2 \r®j r®.“ r’

Therefore adding we get

_ 9x^l5x^ ^ 15xy^ 3x . ISxz^


r’ ■* r5”~;s"H—p-

=- (-«H3^+2r2)= - 15a: . 15x , ^


7^+—'-0-
§ 11. Important Vector Identities.
1. Prove that <//v.(A+B)=</iv A+<//v B
or V ●(A+B)= V ● A+ V *B. [Meerut 1992]
Proof. We have

div(A+B)-vKA+B)=[i L+j ^+k|).(A+B)


= !●
■ li<'^+®)+i-|;(A+B)+k.|(A+B)
-i. , 9B\ , , /3A 3B\
-* (3^+^j+rw+5r)+'‘'(ar+te)
3A
/: 3A. .
=(.-g-+J- dy *iW"S
= V*A+V*B=div A+div B.
^B . > 9B\
+j-l?
dy

2. Prove that curl (A+B)=o«r/ A+cwr/ B


or
V X (AH-B)=V X A+7 X B.
Proof. We have curl (A+B)=v x (A+B)
g idA 3B\
= (‘|j+i |.+'‘3y x(A+B)= 2ix^(A +B)=2ix 13a:
https://www.pdfnotes.co/

120 VECTORS MADE EASY

BA 0B
=2ixx—1-Six dx =7curl A-{-curl B.
dx
3. If A is a differentiable vectorfunction and <f> is a differentia
able scalarfunction, then
div A^-^ div A
or V(M)=(V^)*A+^(V‘A).
[Meerut B. Sc. Physics 1983; Gorakhpur 85; Garhwal 84;
Rohilkhand 82; Agra 81; Bombay 86]
Proof. We have

div(M)=V«(M)

M(i')●*}+"{♦
[Note a«(mb)=(wa)*b=m (a»b)]

=.|2^ i^.A+^S (i-^)={V«-A++ (y.A).


4. Prove that curl (4>A)=(grad 4>) x A-i-0 curl A
or V X (^A)=(v^) X A-1-^ (v X A).
[Agra P;68; Meerut 67, 68,72; Bombay 68;
Kanpur 76; Punjab 63]
Proof. We have

curl WA)=VX(M)=(i |j+l |;+k|-),x(4.A)

=2{lx|j(M)}=s{ix(|A+^|^)}

=2

[Note that a x (mb)=(»ia)x b=/w (a x b)]

{^ (ix^)=(v«xA+^(VXA).
https://www.pdfnotes.co/

GAADIBNT, DtVBRGENCB AND CURL 121

5. Prove that div(AxB)=B.cwr/ X—k^curt B


or
V «(A X B)=B(v X A)—A*(v x B).
[Agra 1984, 85;JKanpnr 87; Calicat 74;
Allababa4'79; Gvrakbpor 88J
Proof. We have

XB+AX
div(AxB)=2|i.L(AxB)|=2.i.|^ #)(

-l|(£ »)J
[Note a.(bxc)=(axb).cand a.(bxc)=—a.(cxb)]

=(curl A)*B—(curl B)*A=B*curl A -A»curl B.


6. Prove that
curl(AxB)=(B.V) A-B div A-(A.V)B+A div B.
[Meerut 1991; Agra 74; Allahabad 77; Ravisbankar 82]
Proof. We have curl(A x B)=v X(A X B)
xB
dx

=2 A~(i.A)

B.2i l-U
=(div B)A-(A.V)B+(B.V)A-(div A)B.
dx -W-i)}*
7. Prove that
grad(A.B)=(B.V)A+(A- V)B+B xc«r/ A+A x curl B.
[Allahabad 1980,82; Rohilkhand 78; Jiwaji 81]
Proof. We have
0B aA \
grad (A.B)=v(A.B)=2i~(A.B)=Si|a-

...(1)
https://www.pdfnotes.co/

122 VECTORS MADE EASV

Now we know that ax(bxc)=(a«c) b—(a.b)c.


(a.b)c=(a*c) b—ax(bxc).
dB
■■■ (*-g)i=(A.i)^-Ax (fx')
0B
=(A*i)^+AX (■xg)
Thus 2

/. 8B\
=|A.Sil}B+Axr
=(A-V)B+Ax(vxB). ...(2)
Similarly S (B-V)A+Bx(vxA). ...(3)
Putting the values from (2) and (3) in (1), we get
grad(A.B)=(A.V)B+Ax(vxB)+(B.V) A+Bx(vxA).
Note. If we put A in place of B, then
grad (A»A)=2 (A* 7) A+2Ax(vxA)
or i grad A*=(A« V) A+Axcurl A.
8. Prove that div grad
i.e. v*(v^)=yV- [Rohilkhand 1981; Garhwal 85]
Proof. We have

-dx\dxl’^dy\dyrdz\dz)
, av . /9" ^9® 9M ^
~W'^dy^'^dz^-\dx^^^~^Wy
9. Prove that curl of the gradient of ^ is zero
i.e. VX (V^)=0, i.e. curl grad if>=Q.
[Meerut 1991S; Rohilkhand 81: Agra 74:
Garhwal 82; Kerala 74; Jiwaji 83]
Proof. We have grad i+|^ k.
ox oy dz
.●. curl grad y X grad ^
https://www.pdfnotes.co/
. GRADIENT* DIVERGENCE AND CURL 123
i j k
d d d
dx dy dz

H H H
dx dy dz
av. 3®^
\dydz dzdyj'^Xdz dx dx dzl^^\dxdy dydx)
=0i+0j+0k=0,
provided we suppose that ^ has continuous second partial deriva
tives so that the order of differentiation is immaterial.
10 Prove that div curl A=0, i.e., v(v X A)=0.
[Meerut 1992; Kanpur 89; Agra 82; Rohilkhand 90]
Proof. Let A=^,i+^2j*+^3k.
<.Then curl A=vxA= i i k
d d d
dx dy dz

At A2 Aa
ZdA, d£A /d_At d^)i ,/dAa 3^i^
-\dy ~dz )^^\dz ~dx ~W)
Now div curl A=v(V X A)
„3_/3^3 ?AA±fdAi 2£3\ 1/dAs dA
~dx\dy ~dz rdy[dz. dx }^dz\dx dy ■)
3% d^A, I d^A, 0^3 3% d»A,
dx dy dx dz~^dy dz dy dx'^dz dx dz dy
=0, assuming that A has continuous second partial derivatives.
11. Prove that
VX(vxA)=v (V*A)—v^A. [Meerut B.Sc. Physics 1983;
Kanpur 86; Allahabad 81; Rohilkhand 90]
Proof. Let A—■'f
Then vxA= i j k
3 3 3
dx dy dz

At Aa ^8
https://www.pdfnotes.co/

124 VECTORS MADE EASY

(dAs ^4 3^3
~\dy ~~dz *■)

vx(vxA)= i J k
a ■ 3 3
dy dz
dA 3
^A^_dA^ dA^_dAi
dy dz ‘ dz dx dx ~ dy
\ dA 8
dx
3M 3
)}-]
dy I dz \dz ~dx
dU.
dy dx^dz dx y&>S‘M
=iU d A dA,
■f
d^Ai. 32^1
dy ^dz -(dy^ +3^2)}']
dx +0J1 +0Z ) ^0A:a .
=2 i
{e (V-A)-(vMoj.
=2
■{|;(V-A|i]-7’“S^ii=V (V.‘A)-v>A.
Solved Examples

Ex. 1. Prove that gradf(u) ~f' («) grad u.


Sol. We have

grad/(u)=i |j/(«)+j |j^/(«)+k


du
dz
=i/'(«)|+i/'(«)|+k/'(a)^;
'. du , ,du , . du
/'(«) 3r -f{u) grad «.

Ex. 2. TaW/ig F==;c2j; i+xz j+2yz k, verify that


Jiv c«r/ F=0. [Agra.1986; Rohilkhaod 85]
Sol. We have Curl F= i j k
3 a 3
a.'c 9.V a^
x^y xz 2yz
https://www.pdfnotes.co/
GRADIENT, DIVERGENCE AND CURL 125

-(22-Jf)1-0 j+(z-;«2^ k^(2z-x)i+(2-;c») k.


Now div curl F=div i(2z^x) i+(z-x^)k]
--1+1=0.
Ex. 2. Verify that curl gradfr^O, where
f=xy+2xy+z^. [Agra 1973]
Sol. We have &adf=(df/dx) i-\-(df/dy) j+(0//92)k
=(2^+2y)i+(x2+2x)jH-22k.
.*. curl grad f=^vx[(2xy-h2y) i+(:fi+2x)j+22k]
i i k
a
dx
- i
dy dz

2xy+2y x^+2x 2z

^[fy (^+2x) i+j^i (2xy+2y)~l^ (2z)]J


+[k(^+2^)-L(2*y+2y)l k
-(0—0)i+(0-0)}+(2x+2-2x-2)k
=0i+0j+0k=0. ■
Ex. 4. Prove that
curl(<[»V^)=v<^Xv^=-Cttr/(^V<P). [Bombay 1986]
Sol. We know that curl(M)=(v^)x A+^ curl A.
In the above formula replacing <f>hy^ and A by we have
curl (4'V^)=(v<|»)X7^+(|» curl
=V4»Xv^+0
[*.* curl =curl grad ^=0]
=V'I'Xv^.
.,.(1)
Similarly curl(M)=(v^)x v4»+^ curl
=V<{>Xv<p+0
=v^xv4'
=-v4»xvf (2)
From (1) and (2). we have
curl v^=-curl
https://www.pdfnotes.co/
126 VECTORS MADE EASY

Ex. 5. Show that curl(a«r) a=0, where a is a constant vector,


Sol. We know that curl (^A)=(v0)x curl A.
Replacing ^ by a*r and A by a in the above formula, we have
curl (a»r) a=[v (a«r)]xa+(a»r) curl a. ...(1)
But if a is a constant vector, then curl a=0 and v(a«r)=a.
from (1), we have .
curl(a«r) a=axa+0=0+0=0.
Ex. 6. Find and
| i *^hen

Sol. Let Then we can write <h=r^e’^'.

Now i+|j+|k.
We have
But
dr
Therefore 2r r-=2x or
dx jdx~~r

So d4>
dx=re-'(2—r) (2—r)e"' x.

Similarly
^=(2~r)e-' y and ^=-(2-^r)e"'z.
Therefore v^=(2—r)er'(xi+yj+2k)=(2—r)e”' r.
Also I 1=1(2—r)e-' r|=(2—r)er' j r 1=(2—r)er' r.
Ex. 7. Prove f/rot div (r* r)=(«4-.3)
[Gorakhpur 1985; Rohilkhand 78; Kanpur 87]
Sol. We have
div(M)=^(div A)+A»grad
Putting A=r and j>=r’* in this identity, we get
div (r" r)=r" div r+r«grad r"
=3r"+r«(nr"“^ grad r)
[V div r=3 and grad/(«)=/'(«) grad u]
1
V grad fi=r= r r
=3r"+r* /ir"-r^r
=3r"+«r""'*(r«r)=3r"+nr"“2 r*=(/i+3) r«.
Ex.8. Prove that v* (r" r)=n(n+3)r"”^ r. [Kanpur 1988]
Sol. We have v*.(r" r)=v [v(r" r)]=grad [div (r" r)]
BBgrad [(grad r")*r+r" div r]
https://www.pdfnotes.co/
GRADIENT, DIVERGENCE AND CURL 127

=grad r).r+3r«]=grad [nr^^ r^+Sr"]


=grad [«r«-2 /●2-h3r"]=grad [(n+3) /●"]
“(«+3) grad r«=(«.-|-3) nr”-^ r=n (/i+3) r.
Ex. 9. Prove that div
[Banaras 1978]
Sol. We have div
(73-r)=div (/~»r)
=r-3 div r+r.grad r-8=3/-8H-r.C-3r"« grad /●)

=3r-a+r.^-3r-«ir^
=3r-a-3r-» (r.r)=r3r-8-3r^. r2=3r-a-3r- 8.-=0.
the vector/-a r is solenoidal.

Ex. 10. Prove that div r=2/r. [Kanpur 1979]


Sol. div (r)=div Now proceed as in Ex. 7.
Alternative Method,
div r=div div
n

=div

Ir a*/ + (r ~r‘ 0?]


Now r2=A:2+;;a+^2. 2r —=2x i.e. ~ ^
dx dx r

3 3 r® 3 1 2.
r r3 r r r r
Ex 11. Prove that vec/or/(r) r is irrotational.
[Agra 1974j Kanpur 75]
Sol. The vector/(r) r will be irrotational if
curl [/(r) r]=0.
We know that Curl (M)=(grad ,t)x A+^ curl A.
Putting <^=f(r) and A=r in this identity, we get
Curl [/(r) rj=[grad/(r)]xr+/(r) curl r V
*» [/'('’) grad r] X t+f(r) 0 [V curl raO]
https://www.pdfnotes.co/
128 VECTORS MADE EASY

1 1
= /V)“f xr=/'(r) -(rxr)=0, since rxr=0.
The vector/(r) r is irrotationaL

Ex. 12. Prove that /(r)=/"(r)+^f\r), [Agra 1917]


Sol. We know that if ^ is a scalar function then
v¥=vW).
V*/(r)«=v{v/(0}=div{grad/(r)}

=div {/'(f) grad r}=div /'(r)rl


1 1
=-^/'(r)div r-hr.grad /'(r)^

/'«+■●{J, m } grad
/'('●)+."[{-7./'('-)+^/"(r)}' |r r
=7-/W+[i|-i./'(r)+i /'<(f)}] (r.r)
=-f/'('●)+[^-{-p/'(<-)+7/"w}]
=7-/'(r)-7/('-)+/"(r)=/"('-)+7/'('-).
Ex. 13. If V* /(r)=0, show that

Ar)=j-\-Czf
where and Ca are arbitrary constants. / ●'
[Bombay 1989]
Sol. As shown in the preceding example, if

then f'(.<■)■

if vV('')=0, then

/''(r)+^/'(r)=0 or
Integrating with respect to r, we get
log/'(r)= —2 log r+log c, where c is a constant
, c
=log -pj-
https://www.pdfnotes.co/

gradient, divergence and curl 129


Again integrating,
Q
/(r)= — -+C2 where is a constant

=7+^^2. replacing —c by c^.

Ex. 14. Prove that y* div


(«™<7)=0.
[Meerut 1991S; Agra 84; Sohilfchand 81; Kanpur 791
Sol. We have

V’(i)= v(v )-)=div|grad


=div
(4
' ‘gradr)=diy(_^l,)=diy(_^r)

(_L)divr+r.grad|_l)

-^+^(>-->')=-p+^r*=0.
1/r is a solution of Laplace’s equdtion.
Ex. 15. Prove that div grad r*=n (/i+1) r"-«,
i.e.
V*/●»=«(«+
[Kanpur 1978, 80; Rohilkfaand 81; Agra 84; Jiwaji 83]
Sol. We have r"=y«(yr«)=dlv (grad r«»)
=div grad r)=div
n/fl-i 1 fJ —div r)
=(ifr"“2) div r+r.(grad nr»-8)
=3»/^a+r.[n(n-2) r»>2 ^ad r]
=3w«-2+r. n («~2) r»-«i r

=3«r"-2+r.[/i(n_2) r]=3/ir«-2+n (/i~2) r«-* (r.r)


=3nr»-*+n («-2) r»-^ r^=m»-^ (3+„_2)=„ („+1)
Note. If «= — I, then y2 (r-r)=(_ j) (—14. i) ,^8=0.
Ex. 16. Prove that curl grad r^=0.
[Kohilkhand 1992; Garhwal 81]
»oI. .Let r"=^. Now proceed as in identity 9 of § 11.
Ex. 17. Jf r is the position vector of the point (x, y, z) show
that curl (r« r)=0, where r is the module oft. [Kanpur 1978]
Sol. We know that curl (M)=(y^)xA+^ curl A. Putting.
9—and A=r in this.identity, we get
curl (r« r)=(yr»)xr+r« curl r
=(nr«-i yr) Xr-fr« 0
https://www.pdfnotes.co/

130 VECTORS MADE EASY

[V V /(^)=/V) vr and curl jr=curl(xi+;^+rk)=0


1 ■
V vr=-r
=nr*»”*(rXr)=nr"-2 0 [V rxr^=d
=*0.
Ex. 18. Prove that r" r is an irrotational vectorfor any value
of n but is solenoidal only if w+3=0. [AgrR 1976; Rohilkhand78]
Sol. Let F=r« r.
The vector F is irrotational if curl F=0. Proceeding as in
Ex. 17 show that curi(r" r)=0 for any value of n.
.*. r" r is an irrotational vector for any value of «.
The vector F is solenoidal if divF=0. Proceeding as in
Ex..7,show that div (r« r)=(n+3)r"
the vector r"'r is solenoidal only if(«+3) r”=0 /.c., only
if «+3=t=0i.c.y only if n=—3.
Ex. 19. If n—(1/r) r, show that v x a=0. [Kanpur 1979]
Sol. We have vx®=c“rl ir=curl [(1/r) Ir].
We know that curl(^A)=(v^)xA+ii curl A.
Replacing 4» by 1/r and A by r in this identity, we have
curl [(1/r) r]=[V (l/r)]Xr-l-(l/r) curl r

=[(-^)v<-]xf+(i/'-)0
[V curl r=0 and v fif‘)=f'{r) \jr
1
~?r V Vr=jT
1 1
=-j5(rxr)=--3 0=0.
Hence vxu=0 if u=(l/r) r.
Ex. 20. If VL—{\lr)tfind grad {div o). [Kanpur 1976]
Sol. Proceeding as in Ex.'10, first show that
div o=div [(1/r) r]==2/r.
grad (div n)=grad (2/r)=(—2/r*) grad r
[V grad/(r)=/'(r) grad r]

Ex. 21. If V*/(r)=0 show that /(r)«Ci log r+Cj where


and Cj, are arbitrary constants. [Poona 1970]
Sou We have
https://www.pdfnotes.co/

GRADIENT, DIVERGENCE AND CURL 131

Now|/(,)=/'(r).|.
But from we have 2r =2;c or ^
oX dx r *

02
●● 0.V2

=;^'«-p|/'w+r/"w|
1
/'«+p2/"(r)

Simifarly, by symmetry,

^/('●)=;/' (r)-i/' (r)+^/" (r).


X^+)^
●●● V^/l:'■)=|/'(r)- r (/●)-+ x^-\-f
r» r2 /" (r)

'1
=7/' w+r' (/●).
/. ifvV(/')=0,then
/" W+i/' (/●)=0 or 0£)_ i
r ir) r-
Integrating with respect.to r, we get
log/' (r)= —log r+log Cj, where is a constant
=log (Cj/r).
f ' {r)=cjr.
Again integrating,
/(r)=Ci log r+Cg, where Cj is a constant.
Hence/(/●)=Cj log r+Cj, where c^, c, are arbitrary constants.
Ex. 22. Prove that iv«*=(a‘V) a+axcwr/ a.
[Kanpor 198$]
Sol. Pcoceed as in identity 7 by taking A=a and B=a
1 1
We have
5 Va2=jv (a.a)
0a . da
=jSi|j(a.a)=i2i(a. )
https://www.pdfnotes.co/
132 VECTORS MADE EASY

=is2l(..|)=2(a.|)i.
We know that Ax(BxC)=(A»C)B—(A«B)C.
/. (A.B)C=(A.C)B-Ax(BxC).
da
■■■ (-1) i+“H)-
Thus Z

=(a« V)a+axcurl a.
(-1)
a+ax2 (a.V)a+ax(V xa)

Hence a*=(a« V)a+ax curl a.

Ex. 23. Show that curl a ^(r)=p ^'(r) r x a,, where & is a cons
tant.vector. [Kanpnr 1982]
Sol. We know that curl(^A)=(v^)xA+^ curl A. Repla
cing ^ by tft(r) and A by a in this identity, we have
curl [a ^(r)]=[v ^(r)]x a+^(r)curl a
=[^V)V/']xa-|-^(r)0
[V. a is a constant vector=>● curl a=0
1 1
= <k\r)-T xa V vr=-r r
4>V)rxa.
Ex. 24. Prove that
[Meerut 1972; Bombav 86]
Sol. We have vW)=V'[vW)I
=V -W (V'I')+^ (V^)]=» V ‘[^ (V4')l+ V -[+(V^)]
=4>V'(V«|/)+(v«P).(V^)+^|»V(v4»)+<V'l')'W)
=^V®++2v^ ● V'['+'f'VV.
Ex. 25. Prove that div X v<I;)=0.
Sol. We know that*
div (AXB)=B.curl A—A*Curl B.
/. div (v^X V'[')=(V«I'HCurl v^)-(V^)»(Curl v<J;)
=(V«I;).0-(v^).0 [V curl grad <|>=0J
=0.
Ex. 26. If A and B ar£ irrotational, prove that AxB is sole-
noidal. [Bombay 1M8; Kanpur 77. 79]
https://www.pdfnotes.co/
GRADIENT, DIVERGENCE AND CURL 133

Sol. If A and B are irrotational, then


curl A=0, curl B=0.
Now div (AxB)=B.curl A-A.curl B=B.0-A»0=0.
Since div(AxB)is zero, therefore AxB is solenoidaf.
Ex. 27: Prove that curl grad (|>)=0.
Sol. We know that
curl(^A)=grad ^ X A+^ curl A.
Putting grad <f> in place of A, we get
curl(0 grad ^)=grad ^xgrad curl grad A
=0+^0.
Here grad ^xgrad ^=0, since it is the cross product of two
equal vectors. Also curl grad ^=0.
/. curl grad. =0+0=0.
Ex. 28. If f and g are two scalar pointfunctions, prove that
div {fsjg)=f 72^4.^/.vg. [Meerut 1972]

Sol. Wehavevj=|i+|j+|k.

Therefore/vg=/|i+/|j+/|k.

^ \dx^^dy^+dz^r[dx 35;.+^ Y^)


?!_. ?!● 9" \
dx^'^dy^'^dzy

^\dx '^dy
i+®£ i+?f k
)
=/V'^+V/-Vg.
Ex. 29. A vector function t is the product of a scalar function
and the gradient of a scalar function. Show that
i*curlt=Q. ,
[Kerala 1975}
Sol. Let f=c|/grad^, where tp and are scalar functions.
We have curl f=curl (it-grad <f>).
We know that curl (M)=(grad A) x A+^ curl A.
.*. curl (i}» grad (grad «|>) x (grad (curl grad <f,)
-=(grad «[;) x (grad <f>) [ v curl grad (|>=0]
Now f.curl f=(i[i grad ^)*{(grad <I<) x(grad 0)}
-[4» grad grad 4,, grad <f,]=r\, [grad <f>, grad grad <f>]
134 https://www.pdfnotes.co/
VECTORS MADE EASY

=0,since the value of a scalar triple product is zero if


two vectors are equal.
Ex. 30. Given that p F=VjP» where p, p, F are pjoint functions,
prove that F*c«r/ F=0. [Kerala 1975]
Sol. We have F=(l/p) vp, where 1/p and p are scalar func
tions. Now proceed as in Ex.29,

Ex. 31. Prove that V


or. div [r grad r~*]=3r-*. [Meerut 1991P]
Sol. We have y
(^)=6rad r-»

But

dr dr X
Therefore 2r or —
dx dx r

=~3r~8 X.

-3r-8 z.
Similarly ^(r-«)= —3r"8 y and ~ (r-*)=

Therefore y
(p)=-3r-® (xiH-pj+zk).

.-. ry
(i)=-3r-V^i+;^+zk).

^ y (-3r-‘ y)+li(-3r- z).


dr
x-3r-*
Now ^ X)= 12 r-5 0a:-

=12 r-8 — x-3r-4=12r-8


r

Similarly ^(—3r~* p)=12r"« 3r“*


and (-3r-8 z)= 12r-« z2-3r-^

Hence V ●|ry ^j= I2r"® (x*+/+z2)-9r-®


=12r« r*-9r-®=12r-«-9r-*=3r-®
/ 1\ a«r
Ex. 32. Prove that Sk
●(v -)=-7r-
https://www.pdfnotes.co/
GRADIENT, DIVERGENCE AND CURL 135

Sol. We have
j1 1 j 1 1 I
grad -=-;3grad r=-^ - r=
●● a*
Ex. 33. Prove that
3(a»r)(b«r) a*b
b*v r&
(a.V i)
where a and b are constant vectors.
Sol. As shown in the last example, we have
a*r
a»V - r=

b-V
(a-V ^)= b«V
=b.2i

=b.Si

[V a is a constant vector
dr jcl
=b.2i j a»i 3x ^ A
●j-—+pr (a-r)[ r- ?I=iand|j=-j
=b.2 -i(a.i)l+i,(a.r)*i

=b..|-ia+|(a.r)r|.
[V 2(a«i) i=a, and 2xi=r]
a«b 3 (a»r) (b»r)
, . r®
Ex. 34. Prove that div (Axr)=r»curl A. [Robilkhand 1979]
Sol. We know that
div (AxB)=Bacurl A—A*curl B.
div (Axr)=--racurl A—A»curl r
=r»curlA—A«0 [ .● curlr=0]
= r*curl A.
Ex. 35. If Si is a constant vector, prove that
div {r" (a x r)}=0. [Allahabad 1980; Robilkhand 77]
Sol. We have
div (^A)=^ div A+A*grad
div (r» (a x r)}=r” div (a x r) -{- (a x r) ● grad
=/* div (a X r) -^(a X r)«(nr"“^ grad r)
136
https://www.pdfnotes.co/
VECTORS MADE EASY

—r"(r*curl a—a*curl

=r« (r.0-a.0)+nr»-a
f)+(axr).^
●r
li
h)
[V curl of constant vector is zero and curl r=0]
=nr”-^ [a, r, r]
'=0, since a scalar triple product having two equal vectors is
zero,
Ex. 36. Prove that
V ●(U^V-V^U)=U^^V-VvU.
[Meerut 1969; Bombay 89; Agra 70]
Sol. We have V. (U^V— VyU)
= V*(£7vF)-V*(KvC/).
Now V(C^V)^=C^{V*(V^')}+(VC0*(V^

Interchanging t/ and V, we get


V *(^VC0= ^V*c^+(v^0«(v£0-
V*(C^V^^-KvC0
=[CTv2F+(vC0-(v^0I-[)^v"£^+(vn-(v£0]
= c/v^l^—Fy2j7.
Ex. 37. If & and b are constant vectors, prove that
(0 div [(r X a) X b]= -2b.a. [Rohilkhand 1979]
(«) curl [(r X a) X h]=b X a. [Rohilkhand 1979; Gorakhpur 87]
Sol. (i) We have (r x a) x b=(b.r) a-(b.a) r.
div [(rxa)xb]=div [(b-r) a-(b.a) r]
=diV [(b ● r) a]—div [(b ● a), r] ...(1)
But div (<f>A)=<f> div A+A»grad
Taking ^=b»r and A=a, we get
div [(b.r) a]=(b.r) div a+a.grad (b-r).
Since a is a constant vector, therefore div a==0.
Also let b = hi*+h2j+h3k.
Then b.r=(h,i+h2j+h3k).(xi+j^j+zk)
~\x-\-b^-\-b^z where b^, b^ are constants,
grad (b.r)=hii-{-h2j+h3k=b.
div [(b.r) a]=a.b. ...(2)
Again div [(bi*a)] r=(b«a) div r+r.grad (b.a).
But div r=3. Also grad (b.a)==0 because b.a is constant,
div [(b.a) r]=3 (b.a). ... (3)
Substituting the values from (2) and (3) in (1), we get
div [trxa)xb]=(a.b)-3 (b.a)=-2b.a.
https://www.pdfnotes.co/
GRADIENT, DIVERGENCE AND CURL 137

(ii) Curl[(rx a)xb]=curl [(b-r) a-(b.a)r]


=curl [(b.r) a]-curl [(b.a) r].
But curl(M)=grad 0xA+^ curl A.
.*. curl [(b.r) a]=[grad (bT)J x a+(b.r)curl a
-bxa[v curl a=0 and grad (b.r)=b]
Also
curl [(b.a)r]=[grad (b.a)] xr+(b.a)curl r
-0[V grad (b*a)=0 and curl r=0]
curl[(rxa)xb]=bxa—0=bxa.
Ex. 38. Ifn is a constant vector, prove that
axr a 3r
curl
fA -~;:3
[Rajasthan 1981]
Sol. We have
, aXr
curl-^=VX

...(1)
da
Now ^=0 because a is a constant vector.
6r .
Also r=xi-|r;^j4-zk.

Further
ax r
.*. (1) becomes
a /axr\ 3X 1
dx\ ; “(axr)-f-;3(axi)
r^r
3x .. 1
=-7T (axr)+j5(axi).
ix a /axr\ 3x . 1
to(-F-)=-7Tix(axr)+i i X(a X i)
3x
= a-(i-a)r l+i [(i.i) a-(i.a) i]
3x 1 1
=—■pr ^a+ r& ajT-\- r»
[V i«r=x and i.a=ai if a==<7ji-f Oai+flak]
1
https://www.pdfnotes.co/
138 VECTORS MADE EASY

1
■{“1 a+|p r+^ a p2».i
3 2 , 3 , . ,3 1
a
fS

[V 2x*=r*, SfliA:=r»a, 2flii=al

= -^+p(aT)f-
Ex. 39. Prove that div
[Agra 1981]
Sol. We have
div ^' (xi+yi+zk)

r ...(1)
_f(r) .
r dr {4
J/(r)\0r
dx
f\r) 1
.*>+,
{‘i 4^«}f f' (*■>-?
Similarly

and
0zl r
Putting these values in (0, we get
div

=^/t^)+/' (r)=U2i/(r)+r^ f'(r<\J^^\r^f{r)


Ex. 40. Evaluate div (a x (r x a), where a is a constant
vector. [Kanpur 1976]
Sol. We have div {a x (r x a)}
=div (a X b), where b=r x a
=b»(V X a)—a«(v x b) [See identity 5 of § 11]
=—a»(vxb) [V a is a constant vector VXa=0]
=—a*[vx(rxa)]. ...(1)
Now r=Jci-}-j>j+zk.
Let a=fliiH-U2j+fl3k. Then the scalars a^, a^, a^ are all
constants.
Proceeding as in Ex. 19 part (ii) after § 10, we have
curl (r X a)=V X (r X a) =—2a/ [Do it here]
https://www.pdfnotes.co/
GRADIENT, DIVERGENCE AND CURL 139

Hence from (1), we have


div (ax(rxa)}=-a.(-2a)=2a.a=2a*.
Ex. 41. If a and b are constant vectors, then show that
V»(a.br)=3a.b.
Sol. We know that V*(M)=(V^)*A+^(vA)
[See identity 3 of§ 11J
Replacing ^ by a.b and A by r in the above identity.
we get
V(a’*»r)=[V(a*b)]*r+(a*b)(v«r). ...(1)
Since a and b are constant vectors, therefore a»b is a cons
tant scalar.
V (a.b)=0'.
Also vr=v(Jfi+>'j+2k)

Substituting these values in (1), we get


V(a*br)=0.r-f-3(a.b)=3a.b.
Ex. 42. Prove that 2r-«.
Sol.
We know that v(M)=^(vA)+A.(v^). .,.(1)
Putting A=r and 1/r^ in this identity, we get

V-(y=j5(vr)+r.(vi)
3 r 2 "1
=p+r- -psV^
[V V*r=3 and v/(0=/'(0 V^’I
1
V V'‘=-r ,
3 1
2 , , 3 2 r^=~-
r^~r^ ^2

=V.(-|vr)=v(4ir)

v.(4.)
https://www.pdfnotes.co/
140 VECTORS*MADE EASY

=(—^)(v*r)+r. V »,“sing the identity(1)

=4-3+^-.[^Vr
6, /8 1 \ 6 8
—li+is r«r

6 .8 , 6 8 2 ^ ,
- ;4+^ -^-2r ●
£x: 43. Prove that curl [rx(aXr)]=3rxa, where a is a
constant vector. [Gorakhpur 1983]
Sol. Curl [rx(axr)]
=VX[(r-r) a—(r.a)r] [V ax(bxc)=(a«c) b—(a«b) c]
=VX[r2a—(r.a)r] [V r.r=r2=r2]
=VX(r8a)-vx[(r.a)r]
[V vx(A+B)=vxA+vxBJ
/'2)xa+r2 (yxa)—[v (r»a)]xr—(r-a)(vxr)
[V vx(«|)A)=(V^)XA+^(vXA)]
(2r vr)xa+r20-[v(r.a)]Xr-(r.a)0
[V Vf{r)=--f{r) vr; V X a=0, a
being a constant vector; and vxr=0j
1
2r -rjxa—[v (r.a)]xr
=--=2rxa—axr [V V (r»a)=ia, if a is a constant vector.
See Ex. 9 after § 4. Do it here]
=^-2rx a-frxa=3rxa.
Ex. 44. Prove /Afl/ vx(Fxr)--2F—(v-F)r+(r.v) F.
[Allahabad 1980]
Sol. We know that
VX(AxB)=A(vB)-B(v-A)+(B.v)A-(A.v)B.
[See identity 6 after § 11]
Putting A=F and B=r in this identity, we get
V x(Fxr)=F (v*r)-r(v.F)+(r.v)F-(F.v)r. ...(I)
Now V«r=v(-’f*+J'j+2k)

=6^«+3^W+8lW='+
' +‘=3- ...(2)
If F=Fii+FJ-[-F8k, then
https://www.pdfnotes.co/

GRADIENT, DIVERGENCE AND CURL 141

F-V=(WJ+^,k).(i|.+j I-+.I) ,

/. (F.V)r
|i+^® y(^i+M+zk)
=F,|^(A:i+j1+zk)+...+
=F,i+/y+Fak=F. ...(3)
.*. from (I),(2) and (3), we get
VX(Fxr)=3F-(V»F)r+(r* V)F-F
=2F-(VF)r+(r.v)F.
Ex. 43. If^ and h are constant vectors,prove that
grad[(r x a).(r x b)J=(b x r)x a+(a x r)x b.
[Kanpur 1977]
Sol. We have (rxa).(rx b)
r*r r-b
a»r a*b ,by Lagrange’s identity
=(a.b)(r-r)—(r.b)(a.r).
grad[(rxa).(rxb;]
=grad [(a.b)(r.r)-(r.b)(a-r)]
“grad [(a.b)(r.r)]-grad [(r.b)(a.r)]
=(a.b) grad (r.r)+(r.r) grad (a.b)
“(r*b) grad (a.r)-(a.r) grad (r.b) ...(1)
[*.* grad (^tp)=^ grad and '4>j
Now grad (r.r)=grad r^=grad (jc^+j^Hz*)
=2xi+2yj4-22k=(xi+;;j-|-2k)=2r.
Also if a and b are constant vectors, then a.b is a constant
scalar and so grad‘(a.b)=0.
_ Further if a is a constant vector, then grad (r.a)=a. Similarly
b IS a constant vector implies grad (r.b)=b.
Putting the above values in (^)ar,we have
grad [(rxa).(rxb)]=(a.b)^+(r.r)O-(r.b)a-(a.r) b
=[(a.b)r-(r.a) b]+[(a.b) r-(r.b)a]
=(b X r)X a+(a x r)x b.
Ex. 46. Prove that curl[r«(a x r)]=(n+2)r«a—nr"-*(r.a) r,
vhele a is a constant vector. [Rohilkhand 1977]
https://www.pdfnotes.co/

142 VECTORS MADE EASY

Sol. We know that curl (^A)=(grad ^)x A+^ curl A.


Putting ^=r" and A=axr in this identity, we have
curl [r"(a x r)]=(v/'”)x(a x r)+r" curl(a x r). ...(1)
Now Yr=nr”"^ (1/r) r=nr”-^r.
V (v^'*)x(axjr)=(«/'"“*r)x(axr)
=nr"-^ rx(axr)=nr«-2[(r»r) a—i,r-a) r]
=nr”-^[r^a—(r«a) r]
=nr"a—«r""2(j-.a) r. ...(2)
Aiso r=jci+yj+zk.
Let a—a^i+a^+a^k, where the scalars a^, a^, are all
constants.
Then axr= i j

ax a2 as
X y z
=(a^—a3y)i+{a^x—a-iZ)l+ia^y-a^x) k.
curl(axr)= i j k
d a d
dx dy dz

a^z—a^ a^—QyZ a^y-oix \


*9 9 1 r9

9 H T9 9 n
j+ ^^{a^x-a^z)—^^{a^-a^)^k
=(fli+fli) i4-(fl2+«2) j+(«8+«s) fe=2(aii+fl2j+r7ak)=2a
...(3)
Substituting from (2) and (3)in (1), we get
curl [r«(axr)]=/ir« a—«r"“2(r»a) r+r"(2a)
=(n+2)r” a—nr”~^(r«a) r.
Ex. 47. Prove that a*{v(v*a)—v x(? x a)“)=</iV v,where a is a
constant unit vector.
Sol. We know that
V(A.B)-(A.V)B+(B.V)A-KA;<(vxB)+Bx(vx A).
/. V (v*a)=(v. V)a+(a* Vl v+vx(vxa)+ax(vxv)
...(1)
https://www.pdfnotes.co/

■GRADIENT, DIVERGENCE AND CURL 143


Let v=Vi i+Vg jfva k.

Then v. V=(Vj i+Vg j+Vj

=”>lc+'’»|+'»rz-
a
V (v.V)a=(v.i+v,|+V3y
_ 9a 9a 9a

=0, because a is a constant vector.


Also V xa=0, a being a constant vector,
from (1), we have
V(v*a)=(a.V) v+ax(vxv) ...(2)
Also we know that

V X(A X B)=(V -B) A-(V - A) B+(B. V) A-(A. V) B.


VX(?Xa)=(V.a) V—(V *v) a4-(a« V) v—(v» V) a
= -(V*v) a+(a»V) V ...(3)
[V V *a=0 and (v.y) a=0]
Subtracting (3) from (2), we get
V(v.a)-vx(vxa)=aX(vxv)+(V.Y)a ...(4)
Multiplying both sides of (4) scalarly by a, we get
a.[v (v*a)~vx(vxa)]=a.[ax(vxv)H-a.[(V.v) a]
=[a, a, vx v]+(V .y) (a*a)
=0-f (V«v) a®, since the scalar triple product
[a, a, vx v]=0
= y.y [V a2=j a |2 = L a being a unit vector]
=div y.
Ex. 48. If a is a constant vector, then prove that
(i) V (a«u)=(a«y) u+a x car/u,
(ii) V *(a X n)= ~si»curl u;
(iii) VX(axa)=a<//vn~(a.V)n.
Sol. (i) Proceed exactly as in Ex. 47.
We have V (a.u)=(a.y) n+(u.y) a+ax(vxu)
+ox(vXa).
https://www.pdfnotes.co/

144 VECTORS MADE EASY

Since a is a constant vector, therefore


(u»V)a=0 and vxa=0.
y (a»n)=(a» V)o+axcurl u.
(ii) We know that V ‘(A x B)=B«(V x A)—A.(vx B).
V(® X “)=“*(V X a)—a ●(y x a)
\ =u*0—a*(v X u), since y x a=0.
a being a constant vector
\ =0—a«curl u=—a»c}irl n.
(iii) Proceed exactly as in Exx47, Using the identity for
yx(AxB). \
\
\
https://www.pdfnotes.co/

4
Green’s, Gauss’s and
Stoke’s Theorems
§1. Some preliminary concepts.
Oriented curve.Suppose Cis a curve in space.Let us orientC by
taking one of the two directions along C as the positive direction; the
opposite direction along Cis then called thenegnhVe direction.Suppose
is the initial point and B the terminal point of C under the chosen
orientation. In case these two points coincide, the curve C is called a
closed curve.

Oriented closed curve

Smooth curve.Let r(r) =x{t)i+y(r)j+z(r)k,where r(r)is the


position vector of(x,y,z), be the parametric representation ofa curve
C joining the points A and By where r=/j and r=/2 respectivety. We
dr.
know that IS a tangent vector to this curve at the point r. Suppose
dt
the function r(r)is continuous and has a continuous first derivative not
equal to zero vector for all values of t under consideration. Then th^
curve C possesses a unique tangent at each of its points. A curve
satisfying these assumptions is called a smooth curve.
https://www.pdfnotes.co/

146 VECTOR CALCULUS

A curve C is said to be piecewise smooth if it is composed of a


finite number ofsmooth curves.The curve C in the adjoining figure is
piecewise smooth as it is composed of three smooth curves C2and
C3. The circle is a smooth closed curve while the curve consisiting of
the four sides of a rectangle is a piecewise smooth closed curve.

Smooth surface. Suppose S is a surface which has a unique


normal at each of its points and the direction of this normal depends
continuously on the points ofS.Then S is called a smooth surface.
If a surfece S is not smooth but can besubdivided into a finite
number ofsmooth srufeces,then it is called a piecewise smooth surfara.
Thesurface ofasphere issmooth while the surface ofa cubeis piecewise
smooth.
Classification ofregions. A region R in which every clos^ curve
can be contracted to a point without passing out ofthe region is called
a simply connected region. Otherwise the region R is multiply-con
nected. The region interior to a circle is a simply-connected plane
region. The region interior to a sphere is a simply-connected region in
space. The region between two concentric circles lying in the same
plane is a multiply connected
plane region.
If we take a closed curve in
this region surrounding the inner
circle,then itcannotbe contracted
to a point without passing out of
the region.Iherefore the region is
not simply-connected. However
the region between two concentric
shperes is a simply-connected re
gion in space.The region between
two infinitely long coaxial cylin
ders is a multiply-connected re
gion in space.
§2. Line Integrals. Ai^ integral which is to be evaluated along a
curve is called a line integral
https://www.pdfnotes.co/
GIVEN’S,GAUSS’S AND STORE’S THEOREMS 147

.V Suppose r(/)= X(t)i+)> {t)\’¥z(t)k,where r(r)is the position


. of(x,y,z) i.e., r(r)= x i+>^j+z k,defines a piecewise smooth
curve joining two points^ and B.Letf=ti atA and t=t2 at B.Suppose
F(x,y,z)= Fi 1+^2j+-p3 k is a vector point function defined and con-
tinuous along C. If 5 denotes the arc length of the curve C, then
dr
ds t is a unit vector along the tangent to the curve C at the point r.

TTie component of the vector F along this tangent is F ● The


dr
integral ofF ● — along C firom^ to B written as

r dr
ds
ds
-r F*dr ¥*dr

is an example of a line integral. It is called the tangent line integral of


F along C.
Since r = x l+yj+z k,therefore,dr = dr i+tfyj+dz k.
/. F *dr =(Fii+F2j+F3k)»(dr i+dyj+dz k)
= dr+F2dy+Fs dz.
Therefore in components form the above line integral is written
as

JT F *dr = (Fi dr+F2dy+F3dz).


The parametric equations of the curve C arex=x(t),y^y(t) and
2=2(0.
Therefore we may write

Circulation.IfC is a simple closed curve (Le. a curve which does


notintersect Usey"anywhere), then the tangent line integral of¥ around C
is called the circulation of¥ about Oltis often denoted by
F-dr =
1^ (Fjdr+F2dy+F3dz).

Work done by a Force.Supposea force Facts upon a particle.Let


the particle be displaced along a given path Cin space. Ifr denotes the

position vector ofa pointon C,then^is a unit vector along the tangent
to C at the point r in the direction ofs increasing. The component of
https://www.pdfnotes.co/
148 VECTOR CALCULUS.

dr
force F along tangent to C is F ●-j-* Therefore the work done by F
dr
during a small displacement ds of the particle along C is F« ds ds

t.c., F»dr. The total work Wdonc by F in this displacement along C, is


given by the line integral

-X F«dr,
the integration being taken in the sense of the displacement.
§3. Surface Integrals.
Any integral which is to be evaluated over a surface is called a
surface integroL
Suppose 5 is a surface of
finite area. Suppose/(x,y,z) is a
single valued function of position ss*
defined over 5. Subdivide the area
●Pk
5 into n elements of areas s
dSi,SS2^ .,.ydS„. In each part
dSfc we choose an arbitraiy point
Pf^ whose coordinates are

We define
fiPtd *yk*^ld- Porm the sum

k=l
Now take the limit of this sum as n-*co in such a way that the
largest of the areas dS^ approaches zero. This limit if it exists, is called
the Jiiifflce integrfl/of/(x,y, z) over 5 and* is denoted by
f(x.y,z)dS.
It can be shown that if the surface S is piecewise smooth and the
function/ z) is continuous over 5, then the above limit exists i.c ●»
is independent of the choice of sub-division and points
Flux. Suppose 5 is a piecewise smooth surface and
F(z,y,z)
is a vector function of position defined and continuous over 5. LetPbe
any point on the surfoce S and let n be the unit vector at P in. the
https://www.pdfnotes.co/

GREEN’S,GAUSS’S AND STORE’S THEOREM? 149

direction of outward drawn nor-


mat to the surface S at P. Then ?. JO
F -n is the normal component of
F at P. The integral of F *n over
Sis

£ F*ndS.
Itiscalledthe/Zaxof F
over 5.
Let us associate with the dif
ferential ofsurface area dS a vec
tor dS (called vector area) whose
magnitude is dS and whose direction is that of n. Then
Therefore we can write

jtt F*dS,
Suppose the outward drawn normal to the suifece SziP makes
angles a, y with the positive directions of andz|Kes respectively.
If/, m,n are the direction cosines of the outward drawn normal,then
/= cos a,/w = cos/S,n = cos y.
Also n = cosai-l-cos/5j+cosyk = /l-f-wj+nk.,
Let F(r,y,2)= Fi i+F2j+Fg k. Then
F-n = Fj cos a+F2cos^+Fg cos y =Fi/+/?2 /I.
Therefore we can write

£ ¥*ndS
-£ (Fi cos a+P2cos/8+F3cos y)dS

^J]s if we define

£ Fi cos adS

£ p2 cos dS = p2dzdx,

£ FscosydS -
Note 1.
£ F^dxdy.
Other examples ofsurface integrals ar^

£i'‘^£”''^
where/(x,y,2)is a scalar function of position.
https://www.pdfnotes.co/

VECTOR CALCULUS
150

Note 2. Important In order to evaluate surface integrals it is


convenient to express them as do^le integrals taken over the orthog-
"" onal projection of the surface S on one of the coordinate planes. But
this is possible only if any line perpendicular to the ra-ordinate plane
chosen meets the surface S in no more than one point. If the surface
Sdoes not satisfy this condition,then it can be sub-divided into surfaces
which do satisfy this condition.
Suppose the surface S is such that any line perpendicular to the
jy-plane meets5in no more than one point. Then the equation of the
surface 5 can be written in
the form ZA
z=h (x,y).
Let be the orthogo-. S
nal projection of S on the
jcy-plane. If y is the acute
angle which the undirected
normal n at/*(r,j)>,z) to the
surfaceS makte ^thz-axis,
then it can be shown that
cosy dS =idxifyt X
where dS is the small ele R
ment ofarea ofsurface5
atthepointP.
Therefore = kis the unitvector along ■
cosy lii*k|
z-axis.
Hencea =ii F ●ndS F *11
dxdy
l“*kl
Thus the surface integral on S can be evaluated with the help of

a double integral integrated over P.
§4. Volume Integrals.
Suppose K is a volume bounded by a surface S. Suppose
f (x,y, z) is a single valued function of position defined over V. Subdivide
the volume Kinton elements of volumes^Fi, <51^2,...» In each part
dVk we choose an arbitrary point whose co-ordinates are
(^fc >yk»^ik)- tlefine/ (P/^ = / ^ik)-
Form the sum

lf(Pk)^Vk‘
ik=l
https://www.pdfnotes.co/

GREEN’S,GAUSS’S AND STORE’S THEOREMS 151

Now take the limit of this sum as n-*oo in such a way that the
largest of the volumes 6Vi^ approaches zero. This limit, if it exists, is
called the volume integral off(x,y,z)over Kand is denoted by

It can be shown that if the surface is piecewise smooth and the


function f(r,y,z) is continuous over K, then the above limit exists
i.e., is independent of the choice ofsub-divisions and points P^.

parallel to the threeco-ordinates axes,thenrfF = dxifydsand theabove


volume integral becomes

JJJv f(x,yyZ)dxdydz.
If F(x,y,z)is a vector function,then

JJJv
is also an example ofa volume integral.
Solved Examples
Ex.1.Evaluate F *dr,where F = l+y^jand curve Cis the arc
ofthe parabola y =3?in the x-yplanefrom (0,0)to(1,1).
Sol. We shall illustrate two methods for the solution of such a
problem.
Method 1.The curve Cis the parabolay=jc^ from(0,0)to(1,1).
Let jc=r; then y = r^. If r is the position vector of any point
(r,y)on C,then
dr
●●● t=
Also in terms off,F = 1+r^j.
At the point(0,0),r = x = 0. At the point(1,1),r = 1.

● ● X =X(*'*f)<*=
>,2/1*
3"^ 8. 3+4“ 12*
o"
Method 2.In thexy-plane we have r = x l+yj.
dr = dxi+dyl.
Therefore F ●dr = (x^ l+y^ j) ● (dr l+dK j) = ^ dc+y^ dy.
https://www.pdfnotes.co/

152 \'T'CTOR CALCULUS

=^ 0^dx+/(fy).
Now along the curve C, y = Therefore dy-lxdx.

● ● X*'-* 1
7
8 12

Ex. 2. Evaluate F ●dr, where F = i+jcyjfl«dc«/veC

is flrc of the curvey = jc^ from (0,0) to (2,8).


Sol. The curve C is the curve y=^3? from (0,0) to (2,8). Let
x=t, theny=A If r is the position vector of any point {x,y) on C, then
r(r)=xl+yj = M+r^j.

.. f =
Also in terms of r, F = l+r^ j.
At the point (0,0) r=jc=0. At the point (2,8), r=2.

JT F ●* = JT = X^vf'-^ l+t"*j]*(i+3«^j) dt
= X^ = X^ [<^+2<*ld'
r8.256
+ 824
3' 7 " 21
“b ’ Jo L
F ●dr, where C is the curve
Ex. 3. F = 3*y I-y^ J, evaluate f,
in thexy-pUme, y^2jr,from (0,0) to (1,2).
[CaUcut 1983; Kanpur 78; Agra 76; Garhwal 85]
Sol. The parametric equations of the parabola y=2ic^ can be
taken as
x^t,y-2t^.
At the point (0,0>, x=0 and so r=0. Again at the point
(1,2), Jc=landsor=l.

Now X F »dr = (3xyi-y^f)^{dxi+(fyf)


[V r = jc l+y j, so that dr-dx j]
https://www.pdfnotes.co/

GREEN’S,GAUSS’S AND STORE’S THEOREMS 153

1
dt)
= I (3x2f^.l-4A4r)d/

[●/ x-ty y=2p' so that dx/dt=l diaA dy/dt-4t]


1
= I {6^-\6^)dt=^ -16-
^J o
6^16^3 8 7
4 6 2 3 6

Ex. 4. Evaluate JT F ●dr where F isx^ »+y j and C isy^=4x in


thexy-plane from (0,0) to (4,4). (^ra J986» Kanpur77)
Sol. In thex_y-plane, we have r = x i+y j so that dr = dr i+dy j.
/. l^*dv={3?y^\+yiY{dxi+dy\)
= j^y^dx+y(fy. [●.' i ● i = 1, i ● j = 0 etc.]
(A'^dx+ydy), where C is the given curve

y^-4x from (0,0) to (4,4)

ydy

[●●● y=4jr]
t4
=4 -X
in = 256+8 = 264.

Ex. 5. Integrate the junction ¥-3^ i-xyi from the point


(0,0) to (1,1) alongparabola ^=x.
[Rohilkhand 1978]
Sol. Here the parabola_y^=x lies in thex>'-plane. If r is the position
vector of any point (r,};) on this plane, then
r = X i+y j so that d r = dr i+ify j.
Let C be the curve y^=x from (0, 0) to (1, 1). The parametric
equaions of /=x can be taken asx=/^, y=r. At the point (0,0) we have
r=0 and at the point (1,1) we have r= 1.

X F.dr =
Ic (^i-xyiy(dxi+dj>i)
https://www.pdfnotes.co/

154 VECTOR CALCULUS

= (x^dx-xyify),wherex=P, y=t

dt
“JIo 1
= Jo
Ti 1.4
=2 -t
4 Jo 3 4 12
I
Ex.6.Evaluate F*dr where F =(x^+y^)i+Jcyjand the curve

C is the arc ofthe parabola y=x^from (0,0)to(3,9)in thexy-plane.


(Kanpur 1981)
Sol. We have
i ¥*dr

r=xi+yj]
=f l<^+y^i+xyiV(d^i+dyi) tv

=f. (^+y^dx+fxytfy
1/2
.yify [v y^=xand

for the curve C, x varies form 0 to 3 and3; varies from 0 to 9]

- 3jr+^x-j^+^ |y jQ
774
=9+^+1 ● 243 =j[45+243+486]]=^*
Ex. 7. Evaluate / F ●rfr along the curve j^+y^=h z=l wi the
positive direction from (0,1,1) to (1,0,1) where
F = (2x+yz) i+xzj+(xy+2z) k.
Sol. Let the given curve be denoted by C and let.4 and B be points
(0,1,1) and (1,0,1) respectively.
Along the given curve C, we have r = x i+y j+z k.
dr-dxi+dyj+dzk.

“ Jc ^ i+xzj+(;y+2z) k]»(dr i+^fy j+d!zk)

...(1)
= Jf^ [(2x+yz) dx+xz efy+(xy+2z) dz].
https://www.pdfnotes.co/

GREEN’S,GAUSS’S AND STORE’S IHEOREMS 155

In moving firom>4 to ByX varies from 0 to 1 ,y varies from 1 to 0


andz remains constant. We havez=1 and so dz=0.
Hence from(1)
●0

X'^**=r {7x-¥y)dx^- xdy+0

= £ px+V(l-^dx-£
= [4 = 1,
the last two integrals cancel by a property of definite integrals.
'Ex. %. Find the work done when a force
F= i-(2x7-fy) j
moves a particle in xy-plane from (0, 0) to (1, 1) aU>ng the parabola
^ (Kanpur 1980)
Sol. Let C denote the arc of the parabola y^=x from the point
(0,0) to the point (1,1). The parametric equations of the parabola
y^=x can be taken as x=r^, y=r. At the point (0,0), r=0 and at the
point (1,1), r= 1. The required work done

= JC
rF*dr = J^{(x^-/+x)i-(2xy+y)j}«(dri+dvj)

dt
dt

= "-6 2 0
- = 1 — 1 1 = —2
3 2 2 3
Ex. 9. Find the work done in moving a particle in a force field
F = 3x^l+(2xz-y)j+zk
along the line joining (0,0,0) to (2,1,3).
Sol. Let C be the straight line joining (0,0,0) to (2,1,3). The
parametric equations of this straight line are
https://www.pdfnotes.co/

156 VECTOR CALCULUS

JC-0 _ y~0 Z-0


2-0 ■" 1-0 “ 3-d = t say
or Jif=2r, y=^ty z=3t.
At the point (0,0,0), we have /=0 and at the point (2,1,3), we
have/=l.
The required work done

F*rfr = JT [3jc^i+(2jcz-y) j+3 k]*(<dr l+ifyj+dzk)

= [3x^dx+(2xz-y)(ty+zdz]

= [3 (2r)^ dt+ {2(2/) (3/)-/} dt+{3t) 3 dt]

= £^^i2At^+12^-t+9t)dt = J^(36^+St)dt

= 36^/3 +g = 12+4 = 16.


. Jo Jo
Ex. 10. Evaluate
X Y ●dr where

Y — c [—3a sin^t cos t i+fl (2 sin t—3 sinh)^+b sin 2t k]


and C is given byr — a cos t i-¥a sin / j+hr kfrom t = jr/4 to jt/2.
dr
Sol We have
dt = -a sin t i+a cos tj+b k.

● ● X -X=./4ra^'
jt/2 f
dr\

Jfjr/2 c [(-3fl sinT cos /) {-a sin t)+a(2 sin t-3 sin^/)
(a cos t)+(b sin 2t) (b)]dt
*31/2
c [3a^ sinVcos t+a^ (2 sin t cos t-3 sin^t cos t)
’jt/4

+b^ sin 2t]dt

="Xr
-\3t/2

sin 2idt-c (n^+h^) cos 2/


Jjr/4

=c {(-1)-0}| = ic (a^+*^
Ex. 11. Find
X t«</r where t is the unit tangent vector and C is the
unit circle, inxy-plane, with centre at the origin.
https://www.pdfnotes.co/
GREEN’S,GAUSS’S AND STORE’S THEOREMS 157

dr
Sol. For any curve, —.= unit tangent vector = t.
●●● ds
Jc L \^/ =JT (ft)*
ds
[*.' t«t = 1, t being a unit vector]
i2n

“J, «=0 dSy since along the unit circle C,s goes
from 0to 2jt
■]2jt
S = 2jt.
JO

Ex. 12. Evaluate


J dy-ydx) around the circle = 1.
Sol. Let C denote the circle jp" = l. The parametric equa
tions of this circle arex^cos r,y=sin t.
To integrate around the circle C we should vary t from 0 to 2k.
dx] dt

ut

“ Jo (^s^^+sin^r) dt^2K.

where
Ex. IZ. Evaluate
X F-dr,
F = icosy—\xsiny
and C is the curve in thexy-plane from (1,0) to (0,1).
Sol. We have
X F*rfr

= Jc siny)*(iifr+j tfy)

= JT (cosy A - jcsinyrfv)
1(0.1) = 0-1= 1.
^f^d(xcosy)^ fJtcosy
p -l(i’O)
Ex. 14. Evaluate ^*dvywhere¥ =xyiA-{j?-^y^landcurveC

is the, arc ofy = j?-A from (2,0) to (4,12). (Garhwal 1981)


https://www.pdfnotes.co/

158 VECTOR CALCULUS

Sol. We have
X F*dr

=X = X^^*-^X Q^+fydy.
Along C,y=;c^-4 anda:^ = y+4.

12
\jA ol^ [y^ y = 732.
= [^3Jo
Ex. 15. Evaluate
X Jcy®<fa. wl>ere C is the seffttent of the She
y^lx in thexy-planefrom (-1,-2)to (1,2).
Sol. The parametric form of the curve C can be taken as
r(0 = M+2rj (-1<^<1).
dr
We have = i+2j.
dt
-. dr dr ds
dF = &*
dr dr ds ds
^»because^is unit vector.
dt ds dt dt ds

^=|i+2j|=>^5.
dt

.●●X^/*=X ds\
^
dt) dt = f^^t(2tfV5dt
16
= 8VsJ^^^ t"^dt^^
>^5

Ex. 16. Evaluate F*dr,

where ¥=xyi+yz^+zxk curve Cisr^t 1+/^ k, r varying


from -I/O +1.
(Tirupati 1989, Rohilkhand 92)
Sol. Along the curve C,
r = X i+>^ j+z k = t i+t^i+^ k.
https://www.pdfnotes.co/

GREEN’S,GAUSS’S AND STORE’S THEOREMS 159

/. x=t, y=P', and ~= i+2rj+3/^ k.


.*. Along the curve C,we have
F =(rxr^)i+(r^xr^)j+(r^xr)k = j+r"* k.
Hence F.^U
= Jl,(<^i+<’j+f‘'k)-(I+2<j+3<^k)= (*®+a‘+3f*)<*

= (l^+5t*)dt = f^^Pdt+sf^^t^dt
7l' 10
=0+5(2)J^Vd(= 10 ^ 0 7

Ex. 17. If F =(3x^+6y)i—14yzj+20xz^kf then evaluate

^F*drfrom (0,0,0)to(1,1,1)along the curve


x=ty y^?, 2=/^.
Sol. Along the given curve C,we have
r = X1+;;j+2 k = r i+r^j+^ k.

●●● f=i+aj+3/^k. (1)


Also from the equations ofthe given curve we find that the points
(0,0,0)and (1,1,1)correspond to r=0 and r=l respectively.

K) dt

J[[(3r2+6y)i-14yzj+20x22k]*(l+2rj+3/2k)</r,
from(1)

= X=o[(3*^+^)-28l'a+60a:2^<^]dt
1
= i [(3t^+6t^)-28t^+60t^]dt,

puttingx=r, y-t^,

=^ (9/2-28r^+60r^)i/r= 3r^-4r^+6r^® ^
= 3-4+6 = 5.
https://www.pdfnotes.co/

160 VECTOR CALCULUS

Ex. 18. IfF =(2x+y)i+(3y-x)j, evaluate F*dr where C is

the curve in the xy-plane consisting ofthe strai0it linesfrom (0,0)to


(2,0)and then to (3,2).
Sol. The path of integration C has
been shown in the figure. It consists of the
straight lines OAmdAB. B
We have
X F*dv
/
13,2)

O
0,0) i(2,0)
(drl+<fyj)

Now along the straight line


OAy y=0, and X varies from 0 to 2. The equation of the straight
line is
2-0
y-o = 3-2(jc-2) i.e., y=^2x-A.
alongylB, y=2jc—4, dy=2dx andx varies from 2to 3.

.●■XF-dr = P I(2i+0)<fc+0]+ f H2x+2x-4)dx


+(6x-12-x)2dx]

= (14Jt-28)<fc = 4+14X (x-2)dx


r 3
= 4+14 = 4+7 = 11.
2 2
F*dr
Ex.l9. If F = (3x^+fy) i-14yzj+20xz^k, evaluate JT
where C is the curve consisting of the straight tines from (0,0,0) to
(1,0,0) then to (1,1,0) and then to (1,1,1).
Sol. We have F*dr
= [(3r2+^)i-14yzj+20xz^k]«(dri+<^j+dzk)
= {^’¥6y)dx-\Ayzdy'¥2Qx^dz.
Let Cj denote the straight line joining (0,0,0) to (1,0,0). Then
along Ci^ y-0, z=0 and x goes from 0 to 1. Obviously along
Cj, ^fy=0anddz=0.
https://www.pdfnotes.co/

GREEN’S,GAUSS’S AND STOKE’S THEOREMS 161

Let C2 denote the straight line joining(1,0,0)to(1,1,0). Then


along C2, x=l, z=0 and y varies from 0 to 1. Obviously along
C2, dx=0anddz=0.
Again let C3 denote the straight line joining (1,1,0)to(1, Ul).
Along C3,we have
x=1, y=1 so that dx=0, dy=0.Obviously along C^,z varies from
0 tol.
Now F»dr.

We have
X,*'**= ^J(3>^+(!y)<i>c-14yztfy+20xz^dz]
= f'
Jx=0
[*.' along Cj, y=0, z=0, <fy=0, dz-0,andx varies from 0to 1]

=3M‘= 1.
. Jo

Again X/-*= j[‘o —14yzify


[V along C2, dx=0, dz-0 andy varies from 0to 1]

=rJy=0 -14y .O^fy


=0.
[: along C2, z=0]

Finally =

[●/ along C3, dx-0, rfy=:0 and z varies from 0 to 1 ]

= 20[iz3j^ = f.
Hence
X |^F*dr=l+0+^ = ^-
Ex. 20. Iff = (^+3) i+xirj+^z—x) k, evaluate F*<ir where
C is the path consisting of the straight lines from (0,0,0) to (0,0,1) then
. to (0,1,1) and then to (2,1,1).
Sol. We have F»rfr
https://www.pdfnotes.co/

162 VECTOR CALCULUS

=[(^+3)i+xzj+(yz-x)k]*(dx l+dy^+dz k)
=(^+3)dx+xz dy+(yz-x)dz.
Let Cl denote the straight line joining (0,0,0) to (0,0,1), C2
denote the straight line joining (0,0,1) to (0,1,1) and C3 denote the
straight line joining(0,1,1)to (2,1,1).
Along Cl, jc=0, y=0sothatdr=0, dy=0.
Also along Ci, zvaries from 0to 1.
Along C2, x=0,z=1 so that dr=0,
Also along C2, y varies from 0to 1.
Along C3, >>=1,2=1 so that <fy=0,dz^O.
Also along C3, x varies from 0 to 2.

£ =
F*dr = f F»dr+Jcz
Jci
f F*dr+JC3
f F-dr
=r (O.z
Jz=0\
= 0+0+5 rxl^= 10.
(2.1+3)dr

L Jo
Ex.21.Evaluate Y*dr where F =(x^+>^)i-2x>>j,curve C is
the rectangle m thexy-pldne bounded byy=0, x=<i, y=by x=Q.
(Andhra 1992; Meerut81;Kanpur 79)
Sol. In the:9>-planez=0. Therefore
r v=xi+y|and dr = dri+^fyj.
The path ofintegration Chas been shown in the figure.It consists
of the straight lines OA.ABy BD and DO.
We have
jrF.*= £l(i^+/)l-'2xyi]>(dxl+dyi)
y^i

B
m ia.b)
4 i

O
(0,0) A(a,0)
https://www.pdfnotes.co/

GREEN’S,GAUSS’S AND STORE’S THEOREMS 163

Now on OA^ }^=:0,


=X dx-Txydy].
and jc varies from 0to a,
onABy x-Oy dx=0 andy varies from 0 to by
on BDy y-by dy^d and jc varies from a to 0,
on DOy x=0, dr=0 and>> varies from b to 0.

X F-*= XVdr-X‘2«y<fy+X“ Oify


r^r r..2i
-2a ^ + j+b^ +0= -2ab‘^.
nO

3 0 . .® °
Ex.22.Find the total work done in movingaparticle in aforcefield
given byY = i“ xy i—5zj+lOx k along the curve x=i^+l, v=2i^, z=P
from r=1 to t—2. [Tinipati 1984, Madras 83,Kanpur 78]
Sol.LetCdenote the arcofthegiven curve fromr=1 to/=Z Then
the total work done

= X F’*= X (3j;>’l-5zj+10xk)*(<irl+<<Hj+<irk)
= X (3-zy <ir-fe<fy+10j:<fe)

rhf dt dt)
dt

^ Si (^+1)(^fm-(5p)(4t)+10(^+1)(3^)]dt

= J^ (12f^+12/^-20r'*+30r'^+30/^)</r

= J^ (12r^+10r'*+l2r^+30/^ <ir = 303.


Ex.23.Find the work donein movingaparticle once around a circle
Cin the xy-plane, ifthe circle has centre at the origin and radius 2and if
theforcefield F is given by
F = {2x-y+2z)\+{x+y-z)l+{3x-2y-5z)k.
(Kanpur 1979)
Sol. In thex}>-plane,we havez=:0. Therefore
F = C2x-y)i+(x+>^)j+(3x-^)k.
The circle Cis given byx^+3;^ =:4 orx=2 cos r, y-2sin t.
r = X 1+yj= 2cos r 1+2sin r j.
https://www.pdfnotes.co/

164 VECTOR CALCULUS

dr
dt - -2sin/i+2cosfj.
Also F =(4 cos t-1 sin t)i+(2 cos t+2sin0j+(6 cos t-A sin t) k.
In moving round the circle once t will vary from 0 to 2tz.
2ji

The required work done is = JT F*dr =

■r [-2sin t(4 cos t-2sin r)+2 cos t(2cos r+2sin t)]dt

^ [4(sin^r+cos^r)-4sin t cos t]dt


i2jt
(4-4 sin t cos t)dt = 4r-2sin 2tJO = &r.

Ex. 24. // F = (3cc^+6y)i-14yzj+20xz^ k, evaluate F*dr

where Cisa straight line joining (0,0,0) to (1,1,1).


(Meerut 1983; Bundelkhand 79)
Sol. The equations of the straight line joining (0,0,0) and
(1.1,1) are
x-0 y-0 _ z-0 = t (say).
1-0 “ 1-0 1-0
Then along C, x=r, y=r, z-t.
J Also r = X i+y j+z k. dr = (i+j+k) dt.
AlsoalongC, F = (3r^+6r)i-14r^j+20r^k.
At (0,0,0), r=0 and at (1,1,1), r=l.

X = X=0 [(3^+a)-i4<^+20'^l * = y

EK.2S.IfF =^yi-xj,evaluate ¥*drjrom (0,0) to (1,1) along


the followingpaths C:
(a) the parabola y =x^, (Agra 1973)
(b) the straight lines from (0,0) to (1,0) and then to (1,1).
(c) the straight line joining (0,0) and (1,1).
Sol. The three paths of integration have been shown in the figure.
We have

X F-dr
https://www.pdfnotes.co/
GREEN’S,GAUSS’S AND STORE’S THEOREMS 165

(a) C is the arc of parabola


from (0,0)to(l, 1).
Here dy^lxdx and x varies
from 0 to 1.

Jo 3
(b)C is the curve consisting of
straight lines OB and BA.
Along Qfi, y=0, dy=Q and jr varies from 0to 1.
Along BA, x=1, dx=0 andjc varies from 0 to 1.
F*rfr =
£odx^£ -14,= -1.
(c)C is the straight line OA.The equation of04 is
1-0
3^-0 =
1-0(x-0) i.e., y^x.
dy-dx andx varies from 0 to 1.
1
(xdx-xdx)= 0.

Ex. 26. Calculate ●dr where C is the


curve :
(0 y^=x joining (0,0) to (1,1).
(«) x'^^y joining (0,0) to (1,1).
{Hi) consisting of two straight lines joining (0,0) to (1,0) and
(1,0) to (1,1). V / V ;
(tV) consisting of three straight lines joining (0,0) to (2, -2)
(2, -2) to (0, -1) and (0, -1) to (1,1).
Sol. Here r = x i+>; j so that dr-dx i+dyy

■■■ £ F-dr = £ [(x^+/)d>:+(x^-/)ify] ,.(1)

(i) Let C be the curve y^=x from (0,0) to (1,1). Then along
C, y^=x, X varies from 0 to 1 andy varies from 0 to 1.
from (1),
|["F.dr
https://www.pdfnotes.co/

166 VECTOR CALCULUS

= ¥^J^\
^ Jo Aiy'-lA'
Jo
_Ci4.lV4.fi il -5_A=21
■“Vs 2j^5"3j 6 15 30 10
(ii) Let C be the curve from (0,0) to (1,1). Then along
C, y=x^y x-y^^^yxy2ihes from 0 to 1 andy varies from 0 to 1.
.*. from (1), F»rfr

_ 'k^i^ 4. fi_i'i - 24.1 _ M


- (3'^15j 1^9 3) ” 5*^9 "45
(iii) Let be the straight ine joining (0,0) to (1,0) and C2 be
the straight line joining (1,0) to (1,1).
Along Cl, y=0 so that <fy=0 andx varies from 0 to 1.
Along C2, x= 1, dr=0 andx varies from 0 to 1.

.●.from(l).XF-* = X/***X,F«dr
= 1^+/) dx+(?-/) ❖}+X2{ dx+Qc^-/)Ify}

irsi* r 1 3I' iJ, 1^ 1.


= 3^0+>'-F
L Jo =3"^ \1-3/
(iv)Let Cl be the straight linejoining(0,0)to(2, -2), C2be the
straight line joninig (2, —2) to (0, —1) and C3 be the straight line
joining(0,-1)to (1,1).
The equation of the straight liiie joining(0,0)and (2,-2)is
https://www.pdfnotes.co/

GREEN’S,GAUSS’S AND STOKE’S THEOREMS 167

2 . ■
—^xory=-x.
^i» y=-Xy dy=-dx and jc varies from 0to 2.

from (1),
2
16
2 4jc^
3 3
0
The equation of the straight line joinihg(2,-2)and(0,-1)is
--■gr(-i) 1
(jc-0) or 7+1 2^ or >>= -1-4
■X
2-0 2^
1
or
>’=-^(r+2).
1 1
along C2, f(^+2). ^ 2 «fr and jc varies from 2 to 0.
from (1),

X/-*=£^ j:^+|(a:+2)^ <&+ i-^-|(j:+2)* (~2*

=rf( dr

0
7 3
+rjc
8 3 4 2
2

= _Z.3-3 = -^.
3 3
The equation of the straight line joining (0, ^1) and (1,1) is
1+1
y+l:=
1-0 (x-0) or 7+1 = 2ror>> = 2x-l.
along C3, y—2x—1, dy=2 dx andjr varies from 0 to 1.
.vfrom(l),

Sc, “ X=o [{^+(2j;- 1)^} dr+{^-(2i- if} (2 dr)‘

=X*
= -^x^.+2x^-x = -1+2-1 = I:
L ^ Jo ^ ^
https://www.pdfnotes.co/

168 VECTOR CALCULUS

Hence F*dr

_ 16 25
""3 3 ■*’3" 3

Ek, 27. Evaluate Sc F*dr, where F = yz i+zxj+xy kandCis the


portion of the curve r = a cost\-¥b sin tj+ct k ,from f=0 to t^n/1.
Sol. Along the carve C,
r = xl+y j+z k = a cos r i+h sin r j+cr k.
X = a cos r, & sin r, z = cr.
Now Y*dt = 0>z i+zx j+xy k)*(dr i+dyj+dz k)

= r (yzd[r+zxdy+xydz) =
-it=n/2
xyz ],=« = (a cos t). (b sm t). (cf)j ^
r . lJr/2
= ahc rcosrsinf ^ =s abc (0-0) = 0.

Ex.28.£va/aare
jr ¥*dr where ¥ =^zi+x j+ykand C is the arc
of tile curve r = cos t i+sin r j+r k from t=0 to t=2jt.
(Agra 1!>77; Garhwal 86)
Sol. The vector equation of the given curve is
r = (cos t) l+(sin r) j+r k. ...(1)
the parametric equations of (1) are
x=cos r, >=sin f, z=r. ,.(2)

From (1), ~ = (-sin 0 i+(cos t) j+k.


dr\
dt
●● X “X ; F*
dt)
= (2i+xj+yk)»[(-sinr)i+(cos0j+k]d^

= (-zsinr+xcosr+y)dr

l/=0 (-r sin r+cos^r+sin r) du


putting for X,)', z from (2)

'0 r (l+cos2r)dr + c sin r dr


https://www.pdfnotes.co/
GREEN’S,GAUSS’S AND STORE’S THEOREMS 169

2n 2n

r(-cosO}jj>^+ X COS tdt +i

23tl
Z
r+lsin2/
Z
12jt

JO
+ -cosr
T27r
JO

=— . +. sint ^ +1[2;r]-f[-cos 2;r+cos0]


= 2jt+7i 7m. , .

^29Evaluate F»rfr, where F = xy i+yzj+zjc kand Cis the


● JG
arc ofthe curvet =\a cos 6)i+(a sin d)}+a 6 kfrom 0=0to 6-\n.
Sol. The parametric equations of the given curve are
x-a cos 0, y=a sin 0, z=ad. ...(1)
dr
Also
dd ={-a sin 0)i+(fl cos 0)j+^i k.

^ 2i+zxky[(-a sin 0)i+(a cos d)j+a k]dd

= {-axy sin 0+ayz cos 0+ozz)dd


●ir/2

'0=0 {-a^ cos0sin^0+iz^0sin 0cos0+a^0cos 0)dd,


putting forz,y,zfrom (1)
n/2
cosddd + \a^ 0sin 20^0
'0
n/2
+a
’X d cosddd

o r, ^ T _ w/2 a/2
l sin^0 ^cos20'\
cos20rf0
^ c ^ r®— /o 2j0

a/2 ^a/2
sin 0 dd
^0sin0^ -J^
a/2 1 a/2
+a 3 1
\ )0 l jt+ (cosd^j 0

Ex. 30. IfF—yz i+zxj—xy k,'frnd F*dr where C is given by

x=t, y=t^, z=i^from P(0,0,0) to 2(2,4,8). [Madurai 1985]


https://www.pdfnotes.co/

170 VECTOR CALCULUS

Sol. Along the given curve C,we have


r = jc i+yj+2 k = r j+/^ k.
^
at = i+2rj+3r^k-
Also along the given curve C,we have
F = .P)i+(r^.0j-(^ ● ^ = k.
At the point P(0,0,0), we have r=0 and at the point Q(2,4,8),
wehavef=2.

● X*'-*=X F. f*V dt
W.
= [(r^ i+r'*j-r^k)*(i+2rj+3r^k)]dr

=£ (r5+2t^-3r5)<* = 0*= 0.
Ex. 31. Evaluate x~^(y+z)dSy where C is the arc ofthe circle

= 4j/i the xy-planefrom A (2,0,0)to B(V2,V2,0).


Sol. Let z=2cos r, y=2sin r, z=0 be the parametric equations
of the circle = 4, z=0.
For the point/l,z=2,3;=0,z=0 and so r=0 and for the point5,z=V2,
y=yf2, z=0 and so t=jt/4.
Ifr is the position vector ofany point(z,y,z)on the circle C,then
r =z i+yj+z k = 2cos r i+2sin /j+0 k.
dr
-2sinr i+2cosrj.
dt

— =4sin^r+4 COST = 4.
●● [dtj
\2
Id^'^ o dr
But d]^ (dr ds ^ t ,where t = — is unit tangent
[dtj ds dt^ [dtj ds vector
(dsf [.. |2_ t being unit vector]
[dtj
ds
= 4o<^^ = 2.
c y+zds dt
X dt
https://www.pdfnotes.co/

GREEN’S,GAUSS’S AND STORE’S THEOREMS 171

=j: jt/4
2sin r+0 .
2cosr
●2<* = 2

= 2 log see r 1 ot/4 = 21ogV2


X
W/4
xantdt

Jo
= 2-
Ilog 2= log 2.
Ex. 32. Evaluate
dSy where C is the arc of the
circular.helix
r(t) = cos t l+sin ti+3t k
●*«
from A (1,0,0) toB (1,0, 6jz).
Sol. The equation of the curve C is I
r(f) = cos t i+sin r j+3r k.
dr
dt =-sinM+cosrj+3k.
\2
* -T = sin^r+cos^r+9 = 10.

. (drds'f {^'f 2 dt
^ I t = 10, vsrhere t = ^ is unit tangent
vector to C at the point V

or (dsf
~ =10
[V t =t»t=l, t being unit vector ]
ds
or ^ = ’/(10).
Parametric equations of C are
x = cosr, y = sin/, z = 3r.
At the point z=l,3;=0,z=:0 and so r=0 and at the point
S, z=l, _y=0, z=6;r and so t=2n.

●● X =X 2ds
^ (cos^r+sin^r+9/^) ^dt
dt

= X^ (i+ia^+su") dt
-l2ji

= ^(10) 2®+6(27cf+^(2r)5 .

Ex. 33. Find the circulation ofV round the curve C, where
F — e^ sin yi+e^ cosy j
https://www.pdfnotes.co/
172 VECTOR CALCULUS

and C is the rectangle whose vertices are{Q,0),(1,0),(1,\n),(0,\jz).


Sol.The rectangleC lies in the;cy-plane.Ifr is the position vector
of any point (x,>>)on this plane,then
r =jc i+yjso that rfr = dr i+ify j.
By definition, the circulation of F round the curve C

■i F*dr
= (^’^sinydr+^cosydy). ...(1)
. \
Draw figure as in solved example 21.
Let O be the point (0,0), A be the point (1,0), B be the point
(l,ljr)andD be the point (0,l;r).
Now on OAy y=0, and x varies fi-om 0 to 1,
onABy x= 1, dr=0 andy varies from 0 to i jt,
on BDy y=| JT, (fy^O andx varies from 1 to 0,
on DOy jr=0, dr=0 andy varies from ^ jt to 0.
1 'n/2
e^.0dx+ eco sydy
.-. from (1), ^ F*dr = ^JC=0 fy=0
●0
+ h-1 e^sin^Ttdx + cosydy
fy=7t/2

-\n/2 r nO nO
= e siny Jo + e j+ siny Jjr/2

= e+l-e+(0-l) = 0.

Ex.34.//F = y evaluate around any dosedpath


0^+/)
C in the xy-plane.
Sol. In the xy-plane, r = x i+y j so that dr-dx i+^ j.
"
'C
F®dr = f'c ?+/
, (dxi-^dyj)
\ /
—ydx+xdy
●J'C x^+y'^
We change to polar coordinates by putting x=r cos 6, y=r sin 6.
https://www.pdfnotes.co/

GREEN’S,GAUSS‘8 AND STORE’S THEOREMS 173

dx= -r sin 6dd+cos6dr


anddy-r cos6dO+sin 6dr.
. -ydx+xdy if .
●● ~? ”'’sm0(-rsin0rf0+cos0rfr)

+r cos 6(r cos0de+sin B dr)


_ r^(cos^d+sm^6)
dd^dd.

●X»^’*=X de (1)

5-
X

■>x

Fig (i)

Case I. If the origin O lies inside the closed curve C as in fig. (i),
then for the curve C at the point we have 9—0 and when after a
complete round we come back to>4, then at^, 6=2ji. So from (1)

X = Xv<' = 2»-
Case II. If the origin O lies outside the closed curve C as in fig.
(ii), then for the curve C at the points, we have 0=0q and when after
a complete round along C we come back \.oA, then also aty4, B^Bq. So
from (1)
^0

X"'“*=X = 00
dB= B
-1^0

J0Q
= 0.

Ex.35. IfF — (2j^+y^) i+(3y-4x) j,evaluate ¥● dr around the


triangle ABC whose vertices are A (0,0), B (2,0) and C (2,1).
https://www.pdfnotes.co/

174 VECTOR CALCULUS

Sol. Let Cl denote the curve consisting of the straight lines


AB,BC and CA.Then

J* ¥*dr around the triangle


ABC
●(dxi+dyji) C(2,l)

= l(2i^+/) *+(3>’-4x) tfy 1 A(0.0) B(2.0)

+ JBC
f [(2j^+/)dx+(3y-4x)dy]

f [(2j^+/)dx+(^y-4x)dy] ...(1)
+ JCA
Now along the straight line AB, }> = 0, = 0 and.*: varies from
0to2;
along the straight line BC, jc = 2, dr = 0 and y varies from 0 to
1;
1-0
and along the straight line CA, }’-0 = ^Zq ^
1
i.e., y=\x or x 2y so that dx-2dy andy varies from 1 to 0.
.●.from(l), ●fC\
T F*dr
1

Fjc=0 jPj{2.(2y)V}(2<fy)
+(3y-^)<fy]
2 1

= 2i? + 2!^.
. Jo 2

V*dT where F = jcy i+(r^+y^) \andCis the


Ex. 36. Evaluate JF
x-axisfromx^2 toxMand the straight linex-4fromy=^Q toy=l2.
https://www.pdfnotes.co/

GREEN’S,GAUSS’S AND STORE’S THEOREMS 175

Sol. Here the cun-e C consists of the straight lines A5 and BD


where/I,B and D are the points (2,0),(4,0)and (4,12)respectively.
We have
X F*dr

D(4,12)
=X V)j]*(<& i+<fyi)
== £pcydx+(x^+/)dy]
x=4
j<

0
...(1) >1(2,0) B(4.0) ^
Along the straight line
AB, y-0,<fy=0 and x varies from
2 to 4.
Along the straight line BD, x=4,<fr=0 andy varies from 0 to 12.

from (l)i jr F*rfr


=x:.<^ *’>‘*+X=o
il2

= 0+ 16y +^/ = 192+576 = 768.


0

Ex.37.Evaluate
£ r'dtyWhere ¥ =xyi+Qc^+^iandCislhe
rectangle in thexy-plane bounded by the linesy-2,x-4,ys:iQ andx=l.
(Kanpur 1982)
Sol.Here the curve C
consists of the four straight
lines AB,BD,DE and EA.
Along the line (U0)B >>^10 D(4,10)
AB, y=2,ify=0 and jc varies
from 1 to 4.
Along the line A
BD, x=4,dx=0andy varies .\'=4
from 2 to 10.
Along the line .v-2
DE, y=10,dy=Q and x var
ies from 4 to 1. li'a. 2)
O ■>
https://www.pdfnotes.co/

176 VECTOR CALCULUS

Along the line EA,jc= 1,dr=0 andy varies from 10 to 2.


We have F^dr ^ ^y\+(^+y^)iV(dxi+dyi)
-xydx+(j^+fydy.
Now
‘4 1
F*rfr+
L F*rfr

2xdx+ I0xdx+
!x=l —4
. -iio 1 r 1
=P ,+
LJl 3 J2
„ L J -^ Jio
8 8 1000
= 15+ 160+-^-32-^ -75+ 2+|-10-^
= 15+160-32-75+2-10 = 60.

Ek.3S.Evaluate f ^^^^:~^*dT,whereC is the boundary ofthe


Jc (^+yy
square x=±a, y-±a in the counter clockwise sense.
Sol. Let F =
(X^+/)^ D
We have {a, a)
(-C,a)
F*^/r= d■ S±il.ir
^ Vi+£j.●{dxi+(fy}) 0 i \

_ -y^dx+x^dy
^ (xWf ‘ A {-a,-a) (fl-a)5
The curve C consists of the
four straight lines >45,5A and
EA.
Along the line>45, y= -a, dy=0 andjc varies from -a to a.
Along the line 5A x-a, dx^O and)> varies from -a to a.
-a.
Along the line D5, y-a, dy=^0 and jc varies from a to
Along the line EA, x- -a, dr=0 andy varies from a to -a.
F»dr
We have^ F-dr = F-dr+ '■●‘'●●+X4
https://www.pdfnotes.co/

^tnlUj-IAOMOrOriV 8^1
GREEN’S,GAUSS’S AND STORE’S THEOREMS 177

[ It; A
=r Jx=—a
a^dx

Xx^+a^^
(a)-I'vfe (“V'-.- .
+ f
Jy=-a
'I
^ v'J Vf
nii='i

c
dx
2

i'{ii(J>-'(£)+7ii C'H-xjpfy.]
i Jy=a

-£ .2
●7^^
!
4;^
:r£)+yL(*-x4-xS)]^^

€ X fXz)dx^
5 J ff(y\<fy =
f jis (
- In f(x)dx
y .

-*5 . i-.
r 1
i-
c £ I
S.
X

= &l
3 as&?dde
J® a^sec% x=flt^n0sothat
^ ^ dx^aief^de

^ cos^ed$
J^ji/4 V' = 4 'i4ji/4»4-()ii--0f!
(f:+cos ^ 4P“‘
c ?,1 ~ <●.
1 Jr/4 . ^
I '3U^w\Xiv''i (b) .Op .>\^!
= 4 e+^sm2B 0 f 4 T+X >S*: JT+2.
,^}?j(V●^Ti●)='^^
Ex. 39. Find the cirqilatMnxf^rpi^^l^^
to

(1,1) and the cufvh^=ixfwm (1,1) to (0,0).


Sol. Here the closed curve C consists of arcs OAP and ^
' "’'^^'^^Bgt^Gjf^dnote the hrc' OAP^'axuA. ,> ■s

o c
^K,
^j (o)
:VA
J5 )>=jf^sothat.
4i;n. ,i ,il)
dx and i varies frpmjO to 1●;r -rnuD n.'i' ig i)ilx Si%«A .fog
Along C2, we havex=y^ y-.so'‘■■^that
-;- s i-
< »●
dx=:2y dy andy varies from i td'O.
Also .V
'-i bf>G
\
F*dr = [(2x V) *+(3)'-4jc) j]« <\ '
(<fcl4^j),i ●51

\f.
P ~^x
Now circulation of Fround C 0"-^V»
https://www.pdfnotes.co/

178 VECTOR CALCULUS

Wdr

= [(2jt+/)<iit+(3y-4x)<fyl+J^J,(2x+y^dic+(^-4x)dy]

= \(2x+x*)dr+(3t^-4j)2xdx\

+JPi K2/+/)■fy dv+(3y-V) M


= ^ (lx-Sx^+6]^+x*)dx+j^ (3y-^+(j^tfy

Ji
-1 1x1 1
3‘*‘2‘*’5 2'^3 2
4 1_3_^ 30-40+6-45 49
3"^5 2"" 30 “ 30

Ex. 40. (a) Evaluate ¥● dr where

F = (2x+y) l+(3y-Jc) j+3® k


and C is the curve x^2t^yy=U from t=0 to t-1.
(b) IfA=(7y+3) i+Jtzj+0?z-Jc) k, evaluate A«dr a/o«g the
curve C:
x=2^^, y=ty z=Pfrom ^=0 to r=1. (Kakatfya 1992)
(c) Evaluate A*dr where C is the line joining (0,0, 0) and
(2,1,1) told A = (^+3) i+xz j+(yz-x) k. (Nagaijuna 1991)
Sol. (a) Along the given curve C, we have

r = a* l+r J+/^ k, ^ = 4( i+j+3^ k


and F = (4/^+0 l+(3t-7p)i+i* k.

●●●X^’‘'"=X u Wj
= X=0 ('b^+0+(3<-2/^+3<‘] dr=J^^ (3t^+l6p+2t^+3t)dt
https://www.pdfnotes.co/

GREEN’S,GAUSS’S AND STORE’S THEOREMS 179

s- 2^7+4^4^2^ 3^ _ 3+4+2 3 OT.

(b) Along the given curve C,we have


r =jc l+yj+z k = 2r^ 1+/j+/^ k,
on putting the values ofx,y,zin terms of/.

f= 4rl+j+3<2k.
Also A =(2/+3)1+2/^ I+(/^-2/2^ k.
dt

= (8r^+12/+2/^+3/®~6/^ rf/

-I+#;4.i4.3_6 280+630+35+45-126 864 288


3‘*’'’'^3‘^7 5= 105 "*105=ir’
(c) The equations of the straight line joining (0,0,0) and
(2,1,l)are
x-0 _ y-O z-0
2-0 1-0"1-0“/ (say).
Then along C, x=2/,)^=/,z=/.
At the point(0,0,0), /=0and at the point(2,1,1), /=!.
Along the curve C,we have
r=xi+3^j+zk = 2/l+/j+/k and so dr = 2l+j+k.
Also along C, A ’=(2/+3)i+2/^j+(/^-2/)k.

X “X=0P(2'+3)+1^+1.(«^-2<)1*
“X* (^+®+2<^+'^-2<l*= X* l3^+2»+61*
= r/3+/2+6/i!=
JO
1+1+6 = 8.
EX.4L Evaluate

{(2jy^-y^cojx)<&+(l-^5OTX+3ry)<fy}
https://www.pdfnotes.co/

,5i03Hr E-axoTZ awA e*?;--..


180 VECTOR CALCULUS

whi^^^tstH^i^dSfiheparabhrd^^^^fhhT^byV) (|jr, 1).


ev
Sol. We know that x+Nm IS. an exact differential if
dM dN ,>5 + -■ A x+t v-^i -t. ■■●■■ ●■;

ni r-.?0lt»7 edJ gmtmq no

Here M = 2x^-^cosx; v^cosx.


*’ > dv ~
: A O'.iA
Also N=l- ^SinX+3iVr.,^^s ●^COSX%,6]^
"T
dM dN/ }b)
Thus ● Therefore Mflr+'jVIcfy'is an exact differential.
■^r n;:~N)-S£+^s-n-(£-^^s;) “(●●i j =●-
JC
Let 4>{Xyy) be ^dcifthat
d4> =
Then ^dx+^(fy = (2x^-~y^ cosx) dx+(l-^ sin!j:+3r^^) rfy.

...(1)
.*. ^ = (2x^-y^ cosx) whicl;^ »iv^ <f> '^jry-^p' siax+fi^j
88S
?fAIs(R
y# :^inif.m ar^^hich giv^ <ji» <=y-y^. sin x
rf'*:

bniB (0*0,0; ifigiBU? 3(is lo imob^ ?

+jrV+jr#)-.l@
The values of^.giy^ji^ty (l](kB3 £2;Psgfeeiltwe:take/i()’)=>’ and
/2(»:)=0.Then^ ,A-.-. .
●■■.Pegiyen.integ!3^,,rinE&ncO=i ,(0,0.0) ; n
n 'r
SiCOl
.>l+t+3 ;; ^ t* J_ '/_-T

y-
A
Dlv ■ ■
1-lxsin—+—xl -0

Ex. 42. Find me circulation of Yround the c^e C where


?r=5o?5Mf4-txJt[r
OL
id+-s+si -
and C is the circlex^+^ = 1, z=0. Jt*
Sol. By definition, the circulation of Fcalong the citrve Cis
{h Cf 'A-fxsu?.\^~l)+:d»(x'AO^V^'O) ,4
= 6 F»dr, where r = x i+yj+z k
https://www.pdfnotes.co/

GREEN’S,GAUSS’S AND STORE’S THEOREMS 181


■j J. ● ; iOdV m
JC
Of 0 moil ?.ohtiv i 5! . ii i gnofe /)nR0=s
ow O /I'.iih yilj ytioiA
j. ■-. <1? \ riW pn C, j2[=|f.pd d?=0]

2ji

X \
.{, J i ttit;

sin0(-sin0)J0 fi5 rol ^


2x >2ji r S.^
fo 1 > sin%:dflib ?|>]^lh4.t0s 20)!^^
2ji
— 1 sin 2d
2 = T5^i“"?o;H-rnrcj
0
Ex. 43. Find the circulation^M E, round iHf curve C, where
F = (x-y) i+(x+7) j and Cis /fecircle's? = 4, 254(T
; oC\ TTie|),aranie^fic eqiiatiotts^frtljk(^clfrjc?+jj2 ^4j,;g_Q are
jc=2 cos ty y=2 sin /, z=0.
{i-r lUiV
‘ By dPfiiiition, the circulation of F dfeSg lbe curve
X™ 1 i>V/ ,Io8
fc
U s'H-i'(ti+i ^I woW
J
'C [(r^) i+(r+j^)J] 0
-{X lS:+7!b'‘s'{;jL i£ -

. . r i i'”-
n> i. '1 ■' X.
' ■ ■●'■ ' Ll-+-M>^£Xl) (li) C'l) 0--V

S^^'\P f '/=0 dt
^ dt
2n
j
X [(? cos t-2 sin t) ● (-2 sin r)+(2 cos r+2 sin r) *2 cos t}it

JO 4 (sin^i-k<igi^t)dt^^X^r^t i^4.f ^ 1 ^ = 8nr.


0t“‘ '/l^v L Jo
Ex. 44. Show that

c
.'< -A I V.™S^.+. s-i
C w r^e
circle j? = 1 ih. ^,xX^Fl(^ ,dpqi^^
dldd^if^^sentfe, ^
https://www.pdfnotes.co/

182 VECTOR CALCULUS

, Sol.The parametric equations of the circle arex=cos r, j;=sin r,


z=0 and along the circle t varies from 0to 2jc.
Along the circle C,we have
r =jc l+yj= cos ^ i+sin rj.
dr
●● dt = -sin t i+cos rj.
Hence the given integral
2si
=1=0 [(-sint I+cos fj) ●(-sin t I+cosr j)]<fr

= (sin^r+cos^)<fr

=J. <“=L r Jo = 2jt.

Ex, 4S. If <p = 2xyi^j and C is die curve jc=r^,)>=2r,2=r^ from


r=0 to r=l, evaluate
X (Kanpur 1974)
Sol. We have r =xl+y j+z kso that dr = dx i+dy^+dz k.

Now ^ dr = 2xyZ^{dx l+dy j+dz k)

dz
= 21 ^df+7kf x>^
2dt
= ^X=0 ('*)* 2* *+?| (?) (2») (?f
+2k
f^(?)Qt)(?f'i?dt

= 81
I

Ex. 46. Tjf F = i-z j+x^ k and C is the curve x-P',y-2t,z=/^


from.t=0 to r = 1, evaluate X (Kanpur 1974)
https://www.pdfnotes.co/

GREEN’S,GAUSS’S AND STORE’S THEOREMS 183

Sol. We have r = x l+yj+z kso that


rfr = <& i+<fyj+<fe k.
i j k
Fxdr — xy —z
dx dy dz

= -i(2dz:+x^<fy)-j{jcydz-3?dx)-\-\i{xydy’\‘Zdx).
Changing in terms of t with the help of parametric equations
x=r^,)>=s2r, we have
Fxdr = -I{^'3t^dt+t^-2dt)-^(2^-‘^dt^t'^-2tdt)
+k(2^^2dt+^-2tdt)
= “i(3^^+2 dt-j(4 dt+k(4^+2^dt.

£ (3/5+2<^*-jJ[‘ 4»5dr+kJ[‘ (,^+2&idt


= -i 6^5
M
Ex. 47. Let r denote the position vector of any point (x,>>,z)
measuredfrom an origin O and letr^\r\.

Evaluate jffi
centre atthe origin,
^*dS where S denotes the sphare ofradius a with

Spl. The equation to the sphere5isx^+y^+z^ =


A vector normal to the sphere S at the point(x,y,z)is given by
V(x^ = 2»r 1+^j+2z k.
If n denotes the unit vector along the outward drawn normal to
the sphere S at the point(x,y,z),then

n 2ri+2yj+2zk _xi+yj+zk
>/(4r2+4/+4z^) a

since x^+y^+z^ =(?on the sphere S.


Again r=xi+yj+zk
and r= |r| = V(x^+y^+z^).
Let F =4
https://www.pdfnotes.co/
184 VECTOR CALCULUS
ZWHUIOilHT ?/3.'^OT2 Ci'A/-. ?;J>EUAO ,2'i<a:l>IO

Then F*nJS
}{>)■» I ‘'> 4 -f f i X ~ '1 owfi oW

■ '■ :

i I

-JK (x i+y j+2 k)*(x i f^jj+z k) dS \ I V- i -sliX ●? .●.

.(* ^ "rp D
^vKri fr '

-SS) l-(fcS-<N^'9*»fiS*W>«*

s:4^r^«s48tt^i ^) [v !5u^ce 6ftl^4*phere5 = 4jra^]


cr f*\ {-\ ●'V
}* r
= "tV.X'I

J^r^i^)^yaluai F* yzA+zx^+xy kand


/k5 in the
octant . I i #Ieeiiitl$84^|;;Agrav74;riS^pur.iZ9>
i\V;'-:‘< o'

jc^+)^+2^ = a\0^ Xyy,z <a^ .^b’n'v'j 'i^\ V.^

evaluate ly^ndStvvhheh^mUi^'elc^i^dldHgm^

iissi-roir'; Sol. (a). Avect6^dWl^tbtWsul3iabe'.^ is^enby


Oi :vj -ji -■ojon'ji) n M
mZ oiofiq? :x!j
f
Therefore ^.=£ay]>n4 normaUtp . t5

aio- point ..
2xi+^j+2zk
Ti4ib>4^!tn «(>=●', -R T ii.
V(43c^+4/+4z^ i4! / .»● T nisg.A
t ~ briii
since jc^+y^+z^ =1 on the surfacCtS?- H
T
.:r ?‘JJ
https://www.pdfnotes.co/
GREEN’S,GAUSS’S AND STORE’S THEOREMS 185
l.ii rjJA',.' yio l' d'6i

We have
E F»nrf5 F»n
■E
“ Jn»k|,
where R is the pro-
^
jection of S on thoA^'-plane. The region bounde<l'1)y'ji:-Jxis, y-axis

andthedr^e^t#%^=:Or.«v.':--«.o,r£i , , ...
We have F*n = (yz i+zx j + xy k)*(x i +yj 4 ^z k) 4.3xyz.
(jjf I +^j + z k)»k = z. _n*k| =z.
vVj> B im Hentei F^ndS
fS J
1
^3 r f (^^sih^0|rrf$,if^, 0^^chan|irtg tb polars

J^al2{/\^ r .,/
■j cos6sinW^=|^'^
0 ' ^ a ?

(b) A vectcif ttoriiiail to'tKeyufhce ils g^nbf \


M.

V (x^4^^ ?>b « / 5 : V) i
/. n = a unit normal to any poiijt (x,y,z) of 5 r
(t*T bimibi f '1.

.icH
° ^(4:^ I 4>^ + 4|?) ° ;
■3f-: #ce on the surface 5.
Since 0 s xjy^ z :S a, th^refqre the surface 5 is that part of the
spherex^-l^^+z^=fl^ which liesrin the positwe octant.
i’{) "T “ ('rx — j _ j ~ ( vjS - 1.) 'j 'i “ -
We fihve F*n = ‘ y i+(jt- 2xz) j k] 4 (x f+!y j + z k)
J
J a

j'tjf - ^):-xyz] = - (2xy - 3xyz)


? ●

J
yfl a; Z ■5v:jfjqr: -jrf} i>i Ifiraioa
r {●

.A -i" 'ppi^ '■ ’■ ~.i + “t* ./ i 7


T9
where <^;is the projec-
fit tj'7 ‘>;|t
tion ofS on thezz-plane. The re|ionif ^Xraxis,z-abas
S^
and
1.
the circle x^ + z^ ,^'^2 ^ q --a _ i:_
r
..irla, LifU no -=
'K i '< + i Z) '■■ (' 4 :- ^ .... , i ..p , , J
"We haven == -^ a (X i -i-)rj^-/k)^j* ^ f

Hence
E F*ndS
■J
S y "7 -h dx dz
y/a
https://www.pdfnotes.co/

186 '/ECTOR CALCULUS

R {2x -3xz)dxdz

Jpji/2 ra [2r cos0“3(r cos 8)(r sin 0)]rdddr,


$asQ Jr=iO
on changing to polars
r To o 1 r^/2[fa .
L « I
e=0 Jr=0 cos8de-3Jd=0 I ydr cosdsind^d
L . Jr=0
ji/2 'jt/2
2.3 f cosdsinddd
3 Jo cos0d0-^a*£
2% 34 1_2 3 34
~3 4 2“3 8
Ex.49. /jfF =yi +(x - 2ez)j-xyli,evaluate
(V xF)*ndS where S is the surface ofthe sphere

jp" +y^ +2^^ a^ above thexy-plane.(Kanpur 1980;Bundelkhand 79)


Sol. let f= V X F = i J k
d d d
dx dz
y x-2xz -xy
=1
^(-xy)-^(x-2xz)^ -j

+ k[^(x-2xz)-|;(^)
= xi +yj-2zk.
A vector normal to the sphere5at the point {x,y,z)is given by
V(x^+y^+ z^)= 2x1 + 2yj+ 2zfc
If n denotes the unit vector along the outward drawn normal to
the sphere S at the point(x,y,z),then
n = 2ri + 2yj+ 2zk _xi+yj+zk^
a
/(4x^ + 4y^ + 4z^)”
since x^ + y^ +z^ = on the sphere 5.
(xi+yj-2zk)*(xi + yj+zk)
Noyjr(V X F)«n = f*n = a

a
https://www.pdfnotes.co/

GREEN’S,GAUSS’S AND STORE’S THEOREMS 187

f»n dxdfy »where R is the


We know that
ffs '●“<« = Jl n »k
projection of the surface S on the;i^-plane. Obviously the region R is
the area bounded by the drclejc^ + z = 0 in the jy-plane.
We have n»k = - (x i + y I + z k)«k = -●

Ss = f*n^5

rr
E dxdy
(f.n) z/a
JJR a z/a
wherez^-a^ -jp"

-JJ. '/((?'-3?-^)
dxdy

3(x^+/)-2a^
■E dxdy

C‘
W=0 Jr=0 r dO dty changing to polar coordinates

= 2n: (*“ r(3/-2a^ dfy first in tegrating with respect to 9


Jr=o

= 2n:
r a sin ((3fl^ sin^ t - 2a^
a cost a cos tdty
putting r = a sin r so that dr -a cost dt

H
Jt/2 Ji/2
= 2® a* sin*«ft-2
X sinr^r

= 2;rn^ 3-^-2-1 = 27Tfl^(2 - 2) = 0.


Ex. 50, fjf F =y i + (x - 2tz) j evaluate
(y xF)*ndS where S is the surface of the sphere
jp + z^ = a^ which lies in the first octant (Kanpur 1974)
Sol. Proceed as in solved example 49. Here R i.e., the projection
of the surface S on thejiy-plane is the area in the jQ^-plane bounded by
https://www.pdfnotes.co/
188 VECTOR CALCULUS
.'>T
f-.r/ av?A %'■ J /\i

x-sods,y-axis and the circle ji:? + = a^,z = Q^<This area is in the form
9j^a.qi9adt^t-pfabii^e/> H j| ujjij v/on>i ^>W ■
; -s- j^Oh^teii
> ■:
(Vx F)r,iijdS,jii, ●ic.y..) naa adi lo noimjoiq
. .^id:rJc/2
T" 3r^- 2a^' 't',X ' .vi’jVK) 'id* V.d BO'B i'dJ
-^-=—:rrdddr ):
'0=0 ----- :^«o jvcd sW
r=ov^2^^). .
= 0, proceeding as in solv^ example 49. 11
Ex. 5i. Evaluate <!>n di^here <p = a/w? sLlhe surface

ofthe cylinder = l&includedm^-the


S
first octant
VI)-'ij , between z = Oand
^ ” 5* V (il*)) li
■ *

, , SoL A vector normal to tW'surface 5 i.e., tlie*cylinder

Therefore n = a unit npteilloran^^poinL^^^^


K || ~
= 16, on ilie surface S.
V(ix^ \
We have ● "v>j V

.●iblXfii.
of on thexzrplane.
S>)V 0--d>.
We have n*j = | (x i +y j)*j = fy*<● r
V*
:k£ ■
Jjs \ -4 s )Vj y/4 '^
3 ●5
r..^'
i:.« iU'n sincey^= \^(16 -x^)on5
'2] 5 i. T-n t*. ●
■?A 3-A
0\.>
u^Jo
r ^.<fiKt;integraU*g!iw^
1 25
8 ' 2 rk‘+(-
Jo - A |)j®.l-f ) 4
: .
: 75 i 1 2 6V i IVjSVh’iV) u»(''? X 7)
3 2 3^^^
C# -r + " f.
ibnV«“si “ “

44^i 1 r.Bxti b .d ioani as boD'jo i^I Jo?,


b'jbfijxld' ft T ?! f'o Z o'JBiiu?. ori! 10
https://www.pdfnotes.co/

LAO
GREEN’S,GAUSS’S AND STORE’S THEOREMS 189

■-oiq ofii -75’. j(54S v/ ■


'li:: -fi5
£v . 11
, ■ <! ii n
'.£
4 ● ^ Vv'i ■> ^

O.'i

u ; 0 }0 XjiI .'I
6V/ o?x-y nnniij b)R’iy;H bUjorf;. .
i wh^eV is tfie dd^ed^aon
' Rb ^ ■vAb.tiibn;^"- {

6oM/irfe</ the cylinder:^ ^V the planes x = 0,y == 0,^ = land


1981)
Sol. TTie cylinders = 4^'^{^meets thejc-^ (i.e., >« = 0, z = 0)
at x^ = 4ot x = 2onthe positiVesidei.4atthepoint(2,0,0). It meets
z-axis(/.e., z = 0,y = 0^dtz?ii‘4^:^dtthe)^M^; 0/4)f^tfiere^
limits of integration fbrz ^ from 6 to 4 4^, forz from 0 to 2 and for
i.
y from 0 to 2. <■
'i

Z no
4^n' ■●● ■● V-

.●.X(2.+y).K=£jjPJ, z^Q ^x: 4-y) dxdydz


ji
i

P f2 nW
/ ?. . V.
ft ‘^4’
V vb
C ?; .
V-'

)
(8T .dv MV i : "I'f
*x=0 2ic/+^ ' ^ V

= X(4-rf*?H4, t^ %]dx ^ c
(4 y- (2x^+4)
1
!>i’42 ?, -r

> 2 b .
(li—1, - ,:4
JC . 8 80
= 2 4z + 4z^" ^ 2 8 + 16 01
8
3 2 3 3
0

i shyh^-:^^l^vduate is ' F?n id5, F-=;2rJ + z j ^by^zk and


● . \-
Sis the surface of tKedjdirtder^ =il6. included in the first octant
between z"ii^ ^ = 5. o ^ i, .; I = * (Tirupati 1993)
sfii 8cS9kA<yjBetor;iipfm^ to :theiSurfa(C»;5jis giYeiil)y
●<s- a> -R ji fi :0: biv ■I/;] H A'
?A
■/£ - V”' ^M^^e6 ) == 2a:i’ 4i 2)) j;
i’ 01
i ^nnlJ|iefefi^«;“ atfnit iiormal^to aiiyipolnt-bfS'
2zi + 2yj
= »sincez^ + / == on the surface S.
V(4jc^ + 4y^)
https://www.pdfnotes.co/

190 VECTOR CALCULUS

We have
£ F*ndS =
£ F*n ’WhereR is the pro

jection of5 on the x-z plane. It should be noted that in this case we
cannot take the projection of 5 on the x-y plane as the surface S is
perpendicular to thejr-y plane.
Now F*n =(zi+xj =}(xz+xy).

Therefore the required surface integral is


xz+xydxdz
■£ 4 yM
-r f 4 xz
Jz-0 Jx = 0 V(16-x^)
*5 V
+ x dxdz^smcey = V(16-j^)onS
/

= J^ (4z + 8)rfr = 90.


Ex. 54. = (2x^ - 3z) i - 2xyj - 4x k, dien evaluate

s ^●FdV where Vis the closed re^nbounded by the planes


jc = 0, = 0, z = Oand2c + ^ + z = 4.
(Osmania 1989,90; Kanpur 7d, 78)
Also Evaluate
J£ VxFdV.

Sol. WehaveF = (2z2-3z)i-2xyj-4zk.


.-.V»F =
‘ to i te) * ‘
= ±(2at^-3z) + |(-2x^) + |(-4x)
= 4z - 2x = 2z.
V*FrfF= Txdxdydz [V dV ^^dxdydz]
A-Tx-Ty
:=0 X dxdydz.
[Note that we have taken a thin column parallel to^-axis.as the
elementary volume. It cuts the boundary atz = 0 andz = 4 — 2z —
Also the projection of the plane 2z + 2y + z = 4 on the jy-plane is
https://www.pdfnotes.co/

GREEN’S,GAUSS’S AND STORE’S THEOREMS 191

bounded by the axes y = 0, jc =0 and the line x + y = 2. Hence the


limits fory are from 0 to 2 — x and those forx are from 0to 2]

JE V*FdK=2
2-x
rr
%fx=0 »fy-0
X z
T4-2c-2y’
2=0 dx^fy

=2f f
jx=0jy=0
x(4- 2r“^)dx{fy
● 2-X
dx
.y“0

=2£'iP-4x^+ 4x]dr,on simplifying


= 2 4x^ 8
4 -^x^ + 2x^ =2 4-^+8 3
^ Jo L ^
Second part We have
VxF= i j k
d d d
dx dz
2j^-3z - 2xy -4x

“[I -i(- 2*^)]‘-[I (2^-3^)]j


+[|(-2*3’)"i(2^-3^)lk

=0i-(-4+ 3)j+(-2y)k=j-2yk.
●. Jf]0[, V X Fdy= fjfyd- 2yk)dx^dz
= r r~^
Jx=oJy=0 Jz=0 (J - 2yk)dr^d?

=r r 2-jc

●wr=0 *fy=0 (j-2yk)(4-2x-2y)drdy


t2-jc
“ X=0 j (‘‘3’ “ 2*3' ->'2) - 2k(2;^-x/ -1/) dr
J y=0

“X=ofj<2--t)(‘»-2Jf-2+jr)
https://www.pdfnotes.co/

192 VECTOR CALCULUS


>5lil ( lun 0 -■ -f J" ●“ 'oxfi c^ilJ hubcifo-t
I ■:■ ; ; . ■■ '
: ;■ V'JI bil.y 7. ■ & ^ '{l.Oy Itir'i!
dx

= j^ (2-xf^l-^l(2-xfkfdx >. _ \ k'' (■ *.

(^-’r Lo--a,
dx S'
r
L ! “'i ? ■ ● -Ct.;*
‘*-'U a12 ! n-r.
r 3i^ S tf',
_ (^-2f 2 (X- 2/ n'
3 i 0
i- * ; rjr
^
*1 “ '
■■■<
0
"‘‘■1

if no ,75-) 17 i - 4
= |i-|K = f
h »
(■j y^.^lh&closed
y *Xtt
region bounded by the planes 4r ^ 2^ ®
Sol. We have
/»2i fl4-2x: rSf4x-‘2y ^ | -,''4 X V
<pdv = XiJLo J.4
?
r
i ? ^ .
= 45
JJ ^2 ^'V4-2c
xisl-
0 ,/>'=0
4-2*1I
-|8-4c-:^ I
z y
0
'^dy
t
6
1.
xG y(8-4r- Vv)j
{

4-2c
>5 i(^^: ■ .. i

L = 45
jCf^i 2 3 «
dx

i: + := -j
'1 !!
"W-i'l’”
= 45
7ii j’b. ;
128.
ii.’-i X V
i4if4c
Ex. 56. Evaluate
JJs » vS- vS

plane
where F = (Jt + /) l' k *a«ii S is (/fc -f-S-
2*: +)' + 2z = 6 in the iiburl970)
Sol. Avector^ ^al to&'suiia^iS is g^enby« o-xi.
;; : y (2r t >» + ^) = 21 + 2 k.
n = a unit nbhttai'vi^tbr at aiij^ point
_ 2i+j + 2k 1 s
V(4 + 1 + 4) S)iJ
J
https://www.pdfnotes.co/
GREEN’S,GAUSS'S AND STORE’S THEOREMS 193

We have
£ F*nd5
-E F«n dxdy
In-kl *
where R is the

projection of5on thejQ^-plane.The region/?is bounded byx-axis,)'-axis


and the straight line 2r + y = 6,z = 0.
We have F*n =[(i + iV 2r J-.+^kj»(|I +1j+|k)
= Ux+/)-ix + iyz =i/+ iyz.
Also
n.k =(|l+|j+|k)*k =|.
Hence
iE F»ndS
=JK lIl^ +iy^Hdxdy
“JQIr +
=/X + 2y
(6-2x-y\
2 /j
dx dyy using the fact that

2= ^ ^ from the equation of5

+6y-2xy- y^)dx(fy =
(6-y)/2
^E y(3-x)dxify

‘0 y(3 —x)dxdy.
[Note that R is bounded byx-axis,y-axis and the straight line
2r + y = 6,2 = 0. To evaluate the double integralover/?,keepy fixed
6~ V
and integrate with respect tox^fromx =0tox = ^ ^ ;then integrate
with respect to y from y =0 to y = 6. In this way R is completely
covered].
l(6-yy2
r* x2
=2j,=o>’ tfy
x-0

3(6-y) {6-yf
2 8 tfy

_ 3y 36. 12)- /

36 >-^1 .
8 8 =r ^
https://www.pdfnotes.co/
194 VECTOR CALCULUS

[g ^_36x36 =[162-81]= 81.


2 16 ^ 2 16
JO

Ex. 57. Evaluate F*ndS, where F=yl + 2rj-zkand


S is the surface ofthe plane 2x +y = 6inthefirst octant cut off by the
planez=4.
Sol. A vector normal to the surfoce5is given by
V(2r+y)= 2l+j.
Therefore n = a unit normal vector at any point(x,y,z)of5
21+j 1

We have
jK F*n »wherein is the pro
jection of S on the zz-plane. It should be noted that in this case we
cannot take the projection on the jiy-plane because the surface 5 is
perpendicular tojy-plane.
(2
Now F«n =(yi + 2cj -zk)●
1
Also
n*j=:^(21+j)*J = vs'
The required surface integral is
rc (2 2 ^
= JJ« + 2(y +x)dxdz

‘ff. [6- 2r +x]drdz,sincey =6- 2con5

-it (6-x)drdz = 2

f3 r l4 [■ jZ]®
= 8 18 -1 =108.
JO
Ex. 58. Evaluate
it ¥*ndSy where F = 18zi - 12j + 3y k
and S is the surface of the plane 2x + 3y -I- 6z = 12 in the first octant,
Sol. A vector normal to the surface S i.c., the plane 2x + 3y +
6z = 12 is given by
V(2x + 3y + 6z) = 21 + 3j + 6 k.
.*. n = a unit normal vector at any point (x,y, z) of 5
https://www.pdfnotes.co/

GRIiEN’S,GAUSS’S AND STORE’S THEOREMS 195


\
_2i + 3j+6k_2l + 3j +6k
V^(4 + 9 + 36)
We have
If. F^ndS
=ir. F»n »where R is the pro¬
jection ofS on the j^-plane. The region R is bounded byjc-axis,}'-axis
and the straight line 2jc + 3y = 12,z = 0.
We have F«n =(18z I - 12j+ 3y k).f2i + 3j 7
+ 6k
)
1 18
= j(36z - 36 + 18y)= y(2z +y - 2).
Alson*k =
i(2i + 3j+ 6k)*k =|-

;x
Hence

= rr
dxdy
6/7

JjR (6z + 3y -6)dx^

= rr
jj/? [(12- 2z -3y)+ 3y -6]
t since 6z = 12- 2z- 3y from the equation of5

jj^ (6-2x)dxdy = 2jJ^ (3-x)dxdy


(12-2x)/3
=2 p f
jx=0jy=0 (3 -x)dxdy
r -I(12-2c)/3'
=2f
J*=o(3-x) Ly J3'=0
I
dx

(3 -jr)(6-x)dx

18z-
2 3 Jo
=I(108- 162 + 72]=|-18 = 24.
Ex.59. ^ F s= 2y i — 3j+ k and S is the surface ofthe para~
bolic cylinder y^ = 8xin thefirst octant bounded by the planesy = 4and
2 = 6,then evaluate F»n dS.

Sol. A vector normal to the surfaced U.,the qrlinder


8jc = 0is given by V(8x-^)=8i - 2yj.
https://www.pdfnotes.co/

196 VECTOR CALCULUS

n = a unit nonnal vector at any point(x,y,z)ofS


8i-2yj _ 4i-yj
>^(64 + 4y^) V(16+/)

We have
■it
F*n dS = F«n
n*i
where R is the projection ofS on the yz-plane.

We have F*n = (2y I - 3 j + k)*


V(16+/)
8y + 3y _ lly I

V(16+/) V(16+^)
4i-yj 4
Also n*i =
V(16+/) ‘ V(16+/)

Hence
E ¥*ndS = rr lly dydz
JjR ^(16+/) 4/y/(l6+/)

■'iE ydydz - n4 Jy=0«fz=0


r f ydydz
r n<» 11
=114 Jy=0
r
11
“ 4 *
.. [d‘
2 0
11
= ^.6-8
4 =132.

Ex. 60. Evaluate F*n dS over the surface S of the cylinder

j^+y^ = 9 included in the first octant between z = 0 and z = 4 where


F =zi+jcj-yzk.
Sol. A vector normal to the surface S Ley the cylinder
jp’ +y2_9 jggivenby V(jc^ +y^) = 2rI + 2yj.
n = a unit normal vector at any point of S
2xi + 2yj zi+yj
» sincez^+y^ = 9onthesurface5.
” V(4j^ + 4y^) 3
F»n = (zi+zj-yzk)* i(jti+yj)

= |(zz+xy) = |z(z+y). ‘
https://www.pdfnotes.co/

GREEN’S,GAUSS’S AND STORE’S THEOREMS 197

Also dS = elementary area on the surface of the cylinder


= 3dddz, using cylindrical coordinates r,0,z.

ffs
wherez = 3cos 9, y = 3sin0

Jp4 rji/2
*n/2 t4
'0=0 3cos0 ^z^ +z.3sin0
dd
z=0
'ji/2

■4 COS 9 [8+ 12 Sind]

= 12

= 12
^So
n/2
cos0d9 + 3

1
= 42
r Sind cos

Zl + 3-1
Ex. 61. Evaluate
E F»ndS, where F = 4xi -r 2y^j +z^k
and S is the closed surface consisting of the cylinder j? + = 4 and the
circular discs z = 0 and z= 3.
Sol. Here the surface S consists of three surfaces: (i) the surface
Sj of the base /.e., the plane face z = 0 of the cylinder (ii) the surface
^2 of the top i.c., the plane facez = 3 of the cylinder and (iii) the surfece
^3 of the convex portion of the cylinder.
For the surface i.e., z = 0, F = 4x i - 2y^ j, putting z = 0 in
F.
A unit vector n along the outward drawn normal to is
obviously — k.

/I F*ndS = E (4zi-2y2j)*(-k)d5

■E 0^5 = 0.

For the surface ^2 z = 3, k = 4z i - 2y^j + 9 k, putting ^


z = 3 in F.
A unit vector-n alonjg the outward drawn normal to 52'is
obviously k.
https://www.pdfnotes.co/

198 VECTOR CALCULUS

a. F*ndS = (4jci-2/j + 9k)«krf5

-£ 9rf5 = 9
;x. = 9-2JT.2 = 367T

[●.● area of the plane face S2 of the cylinder = 7jtr = Tjt.l]


For the convex portion = 4, a vector normal to
53 is given by V {3? + y^) = 2jc i + 2y j.
n = a unit vector along outward drawn normal at any point of ^3
2xi + 2yj
= »since = 4on^3.
V(4jc^ + 4y^)
/. on53, F«n = (4jc 1 - 2y^j + k)*|| (x i +yj)
= 2x^-y^.'
Also dS - elementary area on the surface
= 2dddZy using cylindrical coordinates r, d, z.
F*ndS = (2x^-/) 2dddz,
//.
wherex = 2 cos 0,y = 2 sin 6

= X=o£ (8oos2e-8sto»0)2d9*
i3
=2 dd
1^=0 8(cos^0 - sin^0) z
Jz=0
2n
= 48
. COS^ Jo ^
/2
= 48 4
f cos^0rf0 - 0 *.* sin^ (27T - 0) = - sin^ 6

= 1924- ? = 4&t.
2 2
Hence the required surface integral

■iK F*nrf5 =
1 ¥*ndS +
;x. ¥*ndS

= 0 + 36;r + 48;r = 84jt.


^ffs, F*n dS
https://www.pdfnotes.co/

GREEN’S,GAUSS’S AND STORE’S THEOREMS 199

Ex.62. Evaluate o^i+z^j+xYk)●ndS where Sis


the surface of the sphere jp' + ^ = 1 above thexy-plane,
Sol. Let F=/z^l+zVj+jcVk-
A vector normal to the surface S U.yj? + = 1 is given by
V (jc^ = 2x1 + 2yj + 2zk.
Therefore n = a unit normal vector at any point (x,y, z) of 5
2xi + 2yj + 2zk
= xi +yj +zk,
V(4x^ + 4/ + 4z^)
since^ + + z^ = 1 on the surface 5.
We have
ffs F»n
n*k where j? is the pro
jection of 5 on theji)>-plane. Obviously the region jR is the area of the
drclex^ +y^ = 1, z = 0 in thejQ’-plane.
We have F«n = (yV i + zV j + x^^ k)«(x I +y j + z k)
= xyV + yzV + zx^.
Also n*k = (x i-f-y j + z W «k = z.
Hence
J5C F»n dxdy

■& xyz iyz + zx + xy)

■£ xy[xy + z(x+y)]dxify

=JP[ [^ + (^+x)d'^{i-(^ +/)>] <&4'.


sincez = V(1 -x^ -y^ on 5

Jr2ji
0=0 Jr=o ^ sin^ 6 + (;^ cos^ 6 Sin 0+/^ cos 0 sin^ 6).
V(l-r^)]rd$dr,
on changing to polars
= J6=0
Cl Jr=0
f\r^cos^0sin^6d0dr

ft^o 1=0'“ ^ «»s*9stee<»*


https://www.pdfnotes.co/

200 VECTOR CALCULUS

=4 f f cos^6 Odd dr
J$—0 Jr=0
cos^0sin 0 “0and cos0sin^0d0= O
'0=0 V=o
r.6V
=4 cos^0sin^0 dd
6 r=0

=- r
6 Jo
»/2
2/» ● 2/,j/i 2 1-1 JT
cos^0sin 0d0 = -r*
3 4-2 2
jr
24
Ex.63. Evaluate

theeWpsoid
JOE r● dS where Sis the part ofthe surface of

j? ^ ?
^+ —+^=1
^
fying above the plane z = 0, the normal at any point being directed
outwards.

Sol. A nonnal vector to the surface


?
+ “o - 1 = 0 is
a^ b^ ^
i_ n (■** j. j. _ 2£i j. 1 X lit
givenbyV ^2 + j,2 + ^2 ~
Hence the unit vector n along the outward drawn nonnal at any
point (jc,y, z) of the surface S is given by
jlx/a^) 1 + (2y/h^) j -i- (2z/c^) k
" ~ '✓[(4jc^/a^) +(4y^V)+(4z^/c^)l
{x/a^ \ + j+(z/c^)k

Now
£ r»dS i*n

whereR is the projection of5on thej^-plane. Obviously the region R


n*k

is the area ofthe ellipsex^/zi^ + yVb^ = 1, z = 0lying in the^y-plane.


Nowr=o:i +yj+zk.
1
r*n = on the surfaced.
https://www.pdfnotes.co/

GREEN’S,GAUSS’S AND StOKE’S THEOREMS 201

z/c^ z
Also n*k =

£ vndS =
£ 1
V[2(x2V)] z/[,?yl{1(:?/a*)}\
dxdy

dxdy
■‘‘£ ^-^£ - ^[1 - (zVfl|2) _ {//b^)] ’
since on 5, z = c >^[1 - (^/a^ -
=cf r
Jx=-aJy=
bV(i- jcVo

-Ma {1 _ (j(2/a2)}
= 46c rr
Jx=0jy=0
W(l-xVfl^

V[6^{1 -
dxdy
-y2]
n6V(l-xVfl^
-1
= 46c dx
bV(l-j?/a^ J)>=0
Tfl
= 46c p££- dr = 46c- —●
7T
X = 2n:a6c.
Jo 22 2 L JO
Ex. 64. Evaluate
Jfjj^ A»ni/S’, m^6ctc
A=xy i-x ^+(x + z)k,S is the portion of the plane 2x + 2y+ z=6
included in the first octant and nisa unit normal to S (Meerut 1974)
Sol. A vector normal to the surface S i.c., the plane 2x + 2}? +z
= 6 is given by
V(2x + 2y+z) = 2i + 2j + k.
n = a unit normal vector at any point (x,y,z) of 5
_ 2i + 2j + k _ 2i + 2j + k
\^(4 + 4 + 1)

WehaveJJ^ A*0^/5 '£ A*n


n«k »where R is the pro
jection of S on the jy-plane. The region R is the area of the triangle in
jy-plane bounded by x-axis, y-axis and the straight line x + y = 3,
z = 0.

We have A»n = [xy i -x^ j + (x + z) k]»[j (21 + 2 j + k)]


= \xy-\i? + \(x + z).
AJson»k = i(2i + 2j+ k)»k = i- ■
https://www.pdfnotes.co/

202 VECTOR CALCULUS

Hence
If. A»n dS
= JI 5[2*y-2^+^+zlff
“f£ (2xy-2x^+x+z)dxdy

"Jit (2xy-2x^+x +6-2x-2y)dxtfy, .


since z = 6- 2x - 2y from the equation of5

-f\r'\2xy-2x^
jx=0jy=0 ^
-X - 2y + 6)dxdy
n3-x
dx
=X!o -2x^-xy-/+ 6yJy=0
= (3-xf-2x^(3-x)-x(3-x)-(3-xf +6(3-x)](tx

^ (3x^-12z^ + 6z + 9)dr= - 4jt^ + + 9zJ o


243 243 27
^4 - 108 + 27 + 27 = 4 -54 =^
Ex.65. Evaluate
S ¥dV where F=xl+}>j+zk and V is
the re^on bounded by the surfaces z = 0,z = 2, y = 0, >> = 6, z = 4
andz = x\ (Andhra 1992)
Sol. Here the limits ofintegration for the region Karez = to
z = 4, y = 0toy =6 andx = 0toz = 2.
We have
m s FdV = (zl +}'j +zk)(^dy<fe
=/j I +/2j+ /s k,say.
/●2 i*6 r4 f2 re t4
Now /i = I « I « I 2 xdxdydz= I « I « xz
- ^ dxdy
Jx=0 Jy=ti Jz=x jx=0jy=0 |_ jr'=x

=X!oX!o<'^ =X!o dx

3 4]^ = 48 - 24 = 24,
(24z-6jc^)dr= 12z^-| J o
ri re r4 ri re t4
yz ^dxify
'2 = X=oX=oJw = J.=oJ,=o L JZ=JC

dr
= X^=o('‘>’ = £0 [^ -
https://www.pdfnotes.co/

GREEN’S,GAUSS’S AND STORE’S THEOREMS 203

=^(72 - ISx^)dx= 12x-6x^Y = 96

and =
X=oX!oXl’ ^‘^‘^‘^=X!oX=o[t Z<=X
l6
^dx^

dx
y=0
t2
48x- 96 384
£‘(48-3x‘>)dx==^ =96-^
5 J« 5 5

j, F(/P'=/,i + /jj + /3k = 24I + 96j+^k.


Ex.66. Evaluate
X Fdyfor F =X i +yj+z k where V is the
region bounded by the surfaces x = 0, = 0, y = 6, z = 4andz = x^.
Sol. If we putz = 4 in z = x\ we getx^ = 4 orz = ± 2.
tne limits of integration for the region V are z =x^ to
z = 4,3/ = 0 to)^ =6 andz =0toz = 2.
Now proceed as in Ex.65.
§5. Green’s theorem in the plane. Let R be a closed bounded
re^on in thex-yplane whose boundary Cconsists offinitely many smooth
curves. Let M and N be continuousjunctions ofx andy having continuous
dM .dN
partial derivatives and inR. Then
dy dx
'dN dM
E ^dx dy ^ {Mdx-¥ Ndy)y

the line integral being taken along the entire boundary CofR such that R
IS
on the left as one advances in the direction ofintegration.
(Meerut 1984,Rohilkhand 90,Osmania 89,Andhra 90,Tirupati 89,
Nagarjuna 91, Kakatiya 91)
Proof. We shall first prove the theorem for a special region R
bounded by a closed curve Cand having the property that any straight
line parallel to any one of the coordinate axes and intersecting R has
only one segment(or a single point) in common with R. This means
that R can be represented in both of the forms
a ^x<b,f(x)^y<g(x)
and c<y<d,p(y)<x<q(y).
https://www.pdfnotes.co/

204 VECTOR CALCULUS

In the adjoining figure, the equations of the curves AEB and


BFA zxQy =/(x) and>> = g(x) respectively. Similarly the equations of
the curves FAE andEBF arex = p(y)andx = ^(y)respectively.

y
y=d F c
A FzF.
I
>-=-c E
x=b

x-a
O ^x

We have
> m dy
Jtt *b
jC m^ dx

M(x,y) dx
fx-a jy^Kx)

*a
=- M[x,/(r)]dr- M{x,g{x)\dx

=- JP Af(x,y)dx,sincey =f(x) represents the curve


AEB andy = g(x)represents the curve BFA.
If portions of C are segments parallel toy-axis such as GH and
PQ in the figure on ^he next page,then the above result is not affected.
The line Mdx over GH is zero because on GH, we have
integral J*
X = constantimplies^ = 0. Similarly the line integraloverPQ iszero.
The equations of and HP are y =f(x) and y = g(x) respectively.
Hence we have
dM
-E R dy dx{fy =^M(x,y)dr.
...(1)
https://www.pdfnotes.co/

GREEN’S,GAUSS’S AND STORE’S THEOREMS 205

'f

Similarly,

j£f--x:r" -p(y) dx ^ ^
●\x=q(y)
dy
Jx=/»(y)

=fJy=c L^ [9 0').}'] -N\p (y),y]J dy


^l9(y)*y]4' + J^Lp(y)>y]}(y

= %N{x,y)dy. ...(2)

From (1) and (2), we get on addin^

dxdy = (Afdr + Ndy).


jJr (ar 9y)
The proof of the theorem can now be (extended to a region R
which can be subdivided into finitely many special re^ons of the above
type by drawing lines {TS in the figure on the next page). In this case
we apply the theorem to each subregion and /?2 in the figure) and
then add the results. The sum of the left hand members will be equal to
the integral over R. The sum of the right hand members will be equal
to the integral over C plus the line integrals over the curves introduced
for subdividing R. Each of the latter integrals comes twice, taken once
in each direction (as ST and TS in the figure). Therefore these two
integrals cancel each other and thus the sum of the right hand members
will be equal to the line integral over C.,
https://www.pdfnotes.co/

2()6 VECrOR CA1.CULUS

R-2

o
Note. Extension of Green’s theorem ,in plane to multiply con
nected regions.
Green’s theorem in the plane is also valid for a multiply con
nected region R such as shown in the figure below. Here the boundary
C of R consists of two parts; the exterior boundary Cj is traversed in
the anticlockwise sense so that J? is on the left, while the interior
boundary C2is traversed in the clockwise sense so that Rison the left.

In order to establish the theorem, we construct a line such as


AD(called a cross cut)connecting the exterior and interior boundaries.
The region bounded byADEFGDAPQJLHA issimply-connected and so
Green’s theorem is valid for it. Therefore

Mdx + Ndy ■K ^ar dy,


dxdy.
ADEFGDAPQLHA
https://www.pdfnotes.co/

GREEN’S,GAUSS’S AND STORE’S THEOREMS 207

The integral on the left hand side leaving out the integrand is
equal to

JL *fc,
=X,+X,’ since f =-f
JAD JDA

= £(Mdx + Ndy).
Hence the theorem.
§6. Green’s theorem in the plane in vector notation.
We have r =jci +yjso that^/r = dr i + dyj.
Let F = Mi + A^j.
Then
Mdr + Nefy =(Mi + ATj)*(dr i + dyj)= F*dr.
Also curl F = V X F
i j k
A ± ± dN, . dM
i+ 'dN_m\
3r dy dz dz dz J + lar
M N 0

.*.(V X F)»k = m __ m
dr dy *
Hence Green’s theorem in plane can be written as
(V X F)-kdR = F«dr

where d/? = dr dy and k is unit vector perpendicular to thex-y plane.


Ifjdenotes the arc length ofC and t is the unit tangent vector to
C,then
dr
dr = ~dy = tdy.
Therefore the above result can also be written as

(VxF).kdR = F*tdj.
K|
Solved Examples
Ex. 1. Verijy Green’s theorem in the planefor

^(^y +y^)dx +x^ify where Cis the closed curve ofthe region
bounded byy = x andy -x^. (Kakatiya 1991)
https://www.pdfnotes.co/

208 VECTOR CALCULUS

Sol. By Green’s theorem in plane,we have

dN dM\
dxdy = (Mdx + Ndy),
& chc dy)

HereM = Jcy+/, N =j^.


The curvesy-x and intersect at (0,0) and (1,1). The
positive direction in traversing C is as shown in the figure.
'''* 'dN aM'
We have

K (2x-X -2y)dxdy =ii (x-‘2y)dxefy

%dx
= Xlo (x-2y)<fy<lx =

4l' 1 1_ 1
-
" 5 4" 20

Now let us evaluate the line integral along C. Along y =


dy^Txdx.Therefore alongy = the line integral equals

jl[{{x){iP‘)+x*)dx-\-:?(7x)dx\

■x<-* + /)dr = f.
Along y = jc, rfy = dr. Therefore along y-x, the line integral
equals
https://www.pdfnotes.co/

GREEN’S,GAUSS’S AND STORE’S THEOREMS 209

[{(x)(x)+x^}dx+x^dx]=
Therefore the required line integral =5-1=- Hence the
theorem is verified.
Ex.2. Verijy Green's theorem in a planefor k

^ [(X^ -■2xy)dx + +
where C is the boundary ofthe region defined byy^ = & andx = 2.
(Osmania 1991)
Sol. By Green’s theorem in plane, we have

HereAf = - 2ry, AT = jc^ + 3.


The parabola = 8j: and the straight linex = 2 intersect at the
points P (2,4) and Q (2, -4). The positive direction in traversing C is
as shown in the figure and R is the region bounded by the curve C.

We have ff
JJti dx dy j
dxtfy

°J[|r
■E (2xy + 2x)dxify

=r r V(8x)
Jx=0jy=‘-V(8x)
(2xy + 2x)dxdy
https://www.pdfnotes.co/

2JO VECTOR CALCULUS

[*.* for the region R, x varies from 0to 2and y


varies from - V(Sx)to \^(8jc)]
■V(&c)

=i 2x
2+y .y=-V(&) dc, integrating with respect
toy regarding^ as constant

2x.[0 + 2.V(8x)]vfc
= 4V8 r^^dx = 8 V2-15 M
Jo L Jo 5 = ^-
5
...(1)
Now let us evaluate the line integral along C. Along = 8jr, we

have jc = y^/S, dx-^yify andy varies from 4 to -4. Ther^ore along


y^ - 8x, the line integral equals

r' \i^ -^4 4^4'+


Jy=A ^64
1 fy^
\ + 3 dc

= f ^ — + ^-—/ + 3 dy
J4 [256 ^64 16^ ^

=_ rJ-4 256 64 -^/ + 3 dc

=-rf- i/ + 3 dy

V fy(t)dx = 0or2£mdx
according as f(x) =-f(x) or/(- x) = f(x)

1
= -2 ●4^ + 3.4
80
Jo
128
5
Along the st. linejc = 2, we have dx = 0 andy varies from - 4 to
4. Therefore along ;r = 2, the line integral equals
https://www.pdfnotes.co/

GREEN’S,GAUSS’S AND STORE'S THEOREMS 211

XI-4[«+(2V +3)4-]=X4(4v + 3)<fy


/:Ay dy.^Q

JT4
0 = r t4 = 6.4 = 24.
Therefore the total line integral along the curve C ii.e.,

^(Mdr + Af<fy)= -24 + 24 =^ ...(2)


From (1)and (2),we see that
rr
jJr {ar 9y)
= {Mdx + Ndy\

which verifies Green*s theorem in plane.


Ex.3. Verify Green*s theorem in a planefor

£[(3x^-^)dc +(4y -6xy)tfy]


where C is the boundary of the region defined x = 0, 3; =0 and
X +}> = 1. (Osmania 1992)
Sol. By Green*s theorem in plane,we have
ff
Jm (a>; dxify = (Mdx + N^).
dy)
Here Af = Af = 4y -6xy.
The closed curve Cconsists ofthe st line OAy the st.linevlB and
the straight line BO.The positive direction in traversing Cis as shown
in the figure and B is the region bounded 1^ C.
YA

B(0,1)

A X
(or® c (1.0)
https://www.pdfnotes.co/

212 VECTOR CALCULUS

We have
;x 'dN dM'
dx(ty

= JI [i -6xy)--^(3>^-8y^)jdx<fy
~JJil [-^+^6y]Octfy = loJJ^ ydxdy
1-x
= 10 rfjx=0jy=0
ydxdy [*.' for the region R, x varies from
0 to 1 andy varies from 0to 1 -x]
l-;e
= 10 n^i dx, integrating with respect toy regarding^
Jo [2Jy=o
as constant

=5r(1 =5jj'(X - ifdx=![(*- if]i


=|[0-(-l)3]=|.
Now let us evaluate the line integral along the curve C.
Along the st. line OAy we havey = 0, £fy = 0 andx varies from 0
to 1.

line integral along OA = r


3x‘'dx=
= 1.

Along the st. line.<4B, we havex = l-y,^=-^andy varies


from 0to 1.
line integral ^VmgAB

= X[{3(1 -yf- (- dy)+ i4y -6y(1 -y)}<fy]

= £l-3{l-2y+)i^)+^+ 4y-6y +6/]^

= £(n/+4y-3)<fy=\^/4-^^3yJO
11 8
“3 ^“3‘

Along the st. line 50,we havejr = 0, dr =0andy varies from 1


toO.

.*. line integral along BO = -2.

.’. total line integral along the closed curve C


https://www.pdfnotes.co/

GREEN’S,GAUSS’S AND STORE’S THEOREMS 213

= l+i-2
3
= ^.
3 ...(2)
From (1)and (2),we see that Green’s theorem is verified.
Ex.4. Verify Green *s theorem in the planefor

£[(3*2-^)<fc +(4y - 6xy)tfy],


where C is the boundary ofthe region defined byy ^Vx^y-
(Kakatiya 1990)
Sol. By Green’s theorem in plane,we have

HereM = 3r^ - AT = 4y -6xy.


The parabola y = Vx i.e.,/=jr and the parabola y = x^ inter
sect at the points(0,0)and (1,1). The closed curve Cconsists ofthe arc
Cl of the parabolay = x^ and the arc Cjof the parabolay = Vx, Also
R is the region bounded by the closed curve C.

7A

Q P(ll)
7?
0 X
(0.0)

We have
E fdN
dx
dM
)
dxdy

“JOr <*<?»’
~ JJr 4y =ff lOydxtfy
https://www.pdfnotes.co/
214 VECTOR CALCULUS

ri pVx
lOydxity [’/ for the region R, x varies from 0
Jx=0 Jy’=x^
to 1 and 3^ varies fromjc^ to Vx]

0 ^ r]y=xi dx^S
=5 15 ^3 ...(1)
2 5 0^2 5 10 2
Now the line integral along the closed curve C

=^(Mdx + Ndy)=£,{Mdx + Ndy)+ {Mdx + Ndy).


Along Cj, jc^ =y, dy - 2jcdr andx varies from 0to 1.

/. line integral along Cj= [(3x^-8x^)dx +(4x^-6x^)2xdr]

^ (3jc^ + 8x^-20x^)dr = r + 2x^-4jc^Jq


J*1 T1 = 1 + 2-4 =-l.
Along C2,>^ = X. dx = 2ydy and limits fory are 1 to 0.
line integral along C2

= X K3/-8/)2y4-+(4y -6/),fy]
-22y^ + 4y)<fy =
Ji
=-1 + 11-2
^^ 2 ^
=^
2
total line integral along the closed curve C
=-1 + 42 3.
2 ...(2)
From (l)'and (2),we see that Green’s theorem is verified.
Ex.5. Verify Green's theorem in theplanefor

f(x^-x/)dx +(/-2xy)^,
where Cis the square with vertices(0,0), (2,0), (2,2),,(0,2).
(Meerut 1974)
Sol. By Green’s theorem in plane,we have
rr Yw
JJR dx drdy =^{Mdx + Ndy).
«■

Here M =^-xy^, N =j'^ ^ 2xy.


https://www.pdfnotes.co/

GREEN’S,GAUSS’S AND STOKE’S THEOREMS 215

The closed curve C consists of


the straight lines (24, ABy BD and
C
DO. The positive direction in tra D <■
B
versing C is as shown in the figure (o,2) (2.2)
and R is the region bounded by C.
We have
c \f
£ (dN dM\
dxdy R

SSr 0
- — (jc^ “jcy^)l dxdy (0,0) C M2,0) ^

-£ (- 2y + 3x/)dxify = j[^ (- 2y + 3x^)dxdy

’2
dx== (-4 + 8x)dx

= \-4x+4j^]1
Jo =
- 8 + 16 = 8 ...( 1)
Now let us evaluate the line integral along the closed curve C.
Along <24, y-Qy €fy = 0 and jc varies from 0 to 2;
alongvid, jr = 2, rfr = 0 andy varies from 0 to 2;
alongBDy y = 2, = 0 andjc varies from 2 to 0;
and along DO, jr = 0, dr = 0 andy varies from 2 to 0.
We have

£ (Mdx + N^) = £^(Afd>:+N,fy) + £^ (Mdx + N(ty)

(Mdx + Nify)

+ S=0 ^ - 8jc)dr +
^ + ^-2y2 + ^_4jc2 + ^
3 0 ^ 0 2 ^2

-3 + 3-8-3 + I6-3-8 ...(2)


From (1) and (2), we see that
https://www.pdfnotes.co/
216 VECTOR CALCULUS

’^_m
\dX ^ ^(Mdr + JVdfy).
This verifies Green’s theorem.
Ex.6. Verify Green’s theorem in the planefor

f[(2xy-^)dx +(i?+/)dy\,
where C is the boundary of the re^on enclosed by y—r? and ^-x
described in the positive sense. (Meerut 1973)
Sol. By Green’s theorem in plane, we have

JK ,dx ^ J dxdy^ ^^(Mdx + Ndy).


HereM = 2jcy-jc2, N =
The parabolasy^ =x andx^ = y intersect at the points(0,0)and
(1,1),[See figure ofsolved example 4]. The closed curve C consists of
the arc of the parabola y=j^ and the arc C2 of the parabola
y^ = X. Also R is the region bounded by the closed curve C

= JJ(r [i(^V)-|;(2x>-ac^)]dr^

“Jjlfi (2x-2x)drdy = j[|^ 0dr<fy = 0. ...(1)

Now the line integral along the closed curve C


(Mdx + Nify)= (Mdx + N(fy)+ (Mdx +Ndy).

AlongCi^ j? =yy dy = 2xdxandxvariesfirom0to 1.


the line integr^ along

->c^)dr +(x^ + x"^)2xdx]


1
4 X^
= l\4^^2x^)dx = 3
^
3
^ 0

_1 1
3^3
https://www.pdfnotes.co/
GREEN’S,GAUSS’S AND STORE’S THEOREMS 217

Along C2, =Xy dx=ilydy and;;varies from 1 to 0.

the line integral along Cj ^[(2/-/)2ydy +(/+/)dy\

= J (Sy*-2y^+/)dy = /-l+l
^ 3 3
/V
1
1 1
=-l+i- =- 1.
3 3
total line integral along the closed curve C
= 1-1 = 0.
...(2)
From (1) and (2), we see that the two integrals are equal and
hence Green’s theorem is verified.
Ex.7. Evaluate by Green theorem

^(x^-coshy)dx +(y + sinx)tfy,


where C is the rectangle with vertices(0,0), (;r,0), (jt, 1), (0,1).
Sol. By Green’s theorem in plane, we have

£ (dN dM\
\ar dy j dxdy = ^^(Mdx + Ndy).
Here M-j^~ coshy, N^y + sinx.
. dN 6M
dx = COSJC, dy =- sinhy.
Hence the given line integral is equal to

(0.1)
(n,1)
yf
R
4\

o »
(0,0)

1
(cosx + smh.y)dxdy =
. Jx=Q Jy=0(cosj: + sinhy)rfvdtc
t1
ycosx + coshy dr =
'oc=0 L Jy=o h=0[cosjc + cosh \-l]dx
https://www.pdfnotes.co/

218 VECTOR CALCULUS

-\3t
= sinjc +jccosh \-x
L Jo -(cosh 1 - 1).
Ex.8, Evaluate by Green’s theorem in plane
(e sinydx + e ~^cosydy), where C is the rectangle with vertices

(0,0),(JT,0), , ^0, .
Sol. Draw figure as in solved example 7. By Green’s theorem in
plane
bN dM\
dx dxdy.

HereM-e ^siny, N = e ^cosy.


dN -X dM
dx =-c ""cosy, — = « ~^cosy.
Hence the given line integral

= JpT cosy - cosy)drdv,


where R is the region enclosed by the rectangle C
Pn pn/2 _
LoJy^o -2e-"cosy*<fy

=r Jx=0

r
sinv
-iJr/2

L 'Jy=o
—n
dr
=r- 2e"^dr

= 2 e"* -!)●
L Jo =2(e
Ex. 9. TjfF =(x^ -y^)i + 2xyj and r = x I +yhfind the value
¥»dr around the rectangular boundary x = 0, x = a,y = 0,y = b.
(Gauhati 1973)
Sol. Here the four vertices of the rectangle taken in order are
(0,0),(a,0), (a,b)and (0,b). Draw figure as in solved example 7.
Let C be the closed curve traversed in positive direction by the
boundary of the rectangle and R be the region bounded by this curve
C.
We have

= fcl(^-/)^ + 2xydx]=J^ Mdx + Ndy,


https://www.pdfnotes.co/
GREEN’S,GAUSS’S AND STORE’S THEOREMS 219

whereM N-2xy
(dN dM\
lar dxdy. by Green’s theorem
dy )

-f£ (2y +2y)dxdy =4X=oX=o>’‘*'‘^


4Jr=0
r li]'
2
L J>’“0
dx^Tb^ dx^2ab\

Ex.10. Apply Green *s theorem in the plane to evaluate


{(y -sinx)dx + cosxtfy},
where Cis the triangle enclosed by the lines y -O^x-lji, jty = 2;c.
(Agra 1973)
Sol. Here Cis the closed curve traversed in positive direction by
AOAB andR is the region bounded by this curve C.

We have {(y ~smjc)dr + cosxd);}

~Jc where Af=y - sin jc, N = cosjc

=1 fdN
(ar
dM'
dy ^
dx dy, by Green’s theorem

"fit
rjr/2 r(2/nyc

Jx=0 Jy=0 (- sinjc - l)dxdy [*.● for the region


R, y varies from 0 to (2/jr) x and jc varies from 0 to jr/2)
https://www.pdfnotes.co/

220 VECTOR CALCULUS

rn/2 - - (2/ji)x
—)^sinjc —y dx
“J;c=0 y=0
jr/2 r 2 .
2 1 ax
- —jcsinx - —X j
'0 It It

=-^r Jo
n/2
n/2
{x-¥xs\nx)dx
*7C/2
2^ 9 r 1 9
cosxdx
jr
2 0 -|[at(-cosx)]^ -^ Jo
2 £^__2 ^
;r ’ 8 “4 ~ 5F’
Ex.11. Evaluate by Green's theorem

^(cosX siny—xy)dx-^sin x cosy(tyjWhere C is die circle:?'+y^ = 1.

Sol. By Green’s theorem in plane,we have

JJR (dN
\dx ^
9yj dxdy — y^(Mdx+Ndy).
Here M = cosx siny-jcy, N = sinx cosy.
dM dN
^ = cosxcosy-jc, dx = cosxcosy.
dy
Hence the given line integral is equal to

£ xdxdy = r COS6rdd dry changing to polars

=^
3 rsin0l^
L Jo = i(O)
^ = O.
Ex. 12.Apply Green's theorem in the plane to evaluate

jr &+(/+/)<»].
where.C is the boundary of the surface enclosed by the x-axis and the
semi-circle y -(1—:?)^^?
Sol. Here C is the closed curve traversed in the positive direction
by the straight line>40B and the semi-circle BDA.Also R is the region
bounded by this curve C.
We have [(2j?’-)?)dx+{:?+y^)dy]
https://www.pdfnotes.co/

GREEN’S,GAUSS’S AND STORE’S THEOREMS 221

. (0,1)Z) C „
C 3'=V(1 -AT^)
7?

^(-i,o)^c 0 T m,of ^
= JT Mdx+Ndyy where Af = 2r^-;^,
rr dx dy^ by Green’s theorem

=JI [i(^V)-|(2^-^^)]<&4-
= Jfg (2x+^)dt4>
=r r
Jx=-\ Jy-Q
V(l-x^
2(jc+y)dx cfy,since for the region R^y varies

from C to V(1 and x varies from -1 to 1

■‘£._,^+2J)'=0„ *

2^\ <*

and
/«<& = 2£f(x)dc m-x)=f(x)

=2L-^1 =2(i-|)=|-
Ex. 13. If C is the simple closed curve in the xy-plane not enclosing
the origin, show that
https://www.pdfnotes.co/

222 VECTOR CALCULUS

X ¥*dr = 0,where ¥ = zizlJiL.

Sol. Let R be the region enclosed by the closed curve C.


We have ¥*dr
JC

X (dxt+^i)

=r Jc ^4.

= (M<it+W<fy).whereAf=

Since the closed curve C does not enclose origin,therefore both >
thefunctionsMandJVare defined at the origin.So by Green’s theorem,
we have

X <“-»«■ XT. (f-») dxdy

^ rr 10i^+/)-x(2x) y+y^-y(^) dxdy


JjR 0^+^)^ (j^+y^)^

Qdxdy = 0.

Hence
X ¥»dr = 0.

Ex. 14. Show that the area bounded by a simple closed curve C is

g^ven by\j^ (x dy-y dx). Hence find die area of the ellipse
x-a cos 6j y=b sin 6. (Tinipati 1993; Agra 74)
Sol. ^ Green’s theorem in plane,if is a plane region bounded
by a simple closed curve C, then
rr idN dM^
JjR ^ar ey) dx{fy = ^ Mdx+N(fy,

Putting M= -y, N-Xy we get


https://www.pdfnotes.co/

GREEN'S,GAUSS’S AND STOKE’S THEOREMS 223

dxify = 2A,where>4 is the area bounded by C.


Hence
^=l£.^4’-ydc).

The area of the ellipse — (xify-ydx)


.
_ 1
2 Jtf=o
acos0 -bsinO dx\dd
dd dB)
2si
(ab cos^a+ab sin^ff)dd = \ab
Ex.15.Find the area bounded by one arch ofthe cycloid
I. dd = jtab.

x^a {e-sin 0), y = a(1-co50), a>0


cmdthex-aas.
Sol. Let C be the closed
curve traversed in positive di
rection by the straight line OA 0=JC
and the arch ABO of the given B(an,2a)
cycloid. C C
At the point 0,0 =0and
at^, d—2jt.
(0,0) 0=27C
>x
The area bounded by one
arch ofthe given cycloid and the 0=0 ^ C A
x-axis (2fl7C,0)
the area enclosed by the sim
ple closed curve C

“2 Jc Green’s theorem

“h SoA^^^-y^^^iLohABo
Now along the straight line OAy we havey=0, dy^Q andx varies
from 0to 2ajt.

Hence the required area =


| 4
2 JfochABO {xdy-ydx)
https://www.pdfnotes.co/

224 VECTOR CALCULUS

if f de
2je=2jt\^de
=if [a(0-sin 0).a sin 6-a(1-cos0).a(1-cos0)]d0
, 2j2ji

^ r [0 sin 0-sin^0-1+2 cos 0-cos^0]d0


2 J2n
-f! r (0sin 0-2+2cos 9)d6
2 J2jt
2jc

2 Jo (2-2cos0-0sin 6)d6
2 2jt
= u6-2sin 0+0cos 0-sin 0
2 L JO
2
= y[4jr+2;r]= 3;r«^
Ex. 16. Use Green’s theorem tofind the area bounded by the hypo-
2/3
cycloid = a- fl>0.
Sol. The parametric equations of the given curve
^3^.^2/3 _ ^2/3 Ijg
jc = a cos^0, y s= a sin^0.

9 =TC A0=O

6 = 27c

Here C is the simple closed curve traversed in positive direction


by the whole arc of the given hypocycloid.
At the point A, 0=0 and when after one complete round in
anti-clockwise sense along the curve C we come back to then at
Ay 0=2jt.
https://www.pdfnotes.co/

GREEN’S/GAUSS’S and STOKE’S theorems 2251

The area bounded by the given hypocyt^Hpid , , F

= the area enclosed hjr the simple Closed curve C


= i(f
2 Jc (x dy-y dx)y by Green’s therein '; ;

2jt
1-^a; y eos?0, sin?0
2je=o ( da
1
2 Jo [a cos^d.3a siii^d cos O^a sin^0r.:(-:3« cos^0,^in e)\dO
3a^ dd
:f’ ,-

2 Jo sin^0+sin^0 CQS %j
;0'
3a2 /●:[ ■ yiMJ^V \ t|0(H tiKil ■
= 2* -I (cos%sin?0+sin^0eos%)dai^i ii'

= 4-
3a
2 -j:Ji/2

2 3^ 3T 3.1.1 ;r
(cos'*© sin^^+sin'*© cBs^0)d$

6a 2
3;rd^2
rj i\[.:
16 ,8
[6:4jl'2- 6A:i 2^
Ex. .17. Find the area of the loop of the folium j^+y^ = 3flA:y,
a>Q.
Sol. Let C be the simple closed curve formed by the loop of ihc
given curve.
y A
!
I

r=i

i "’i ●● i

V- > ^U i.. a 1
V,:.. : :V'^: I '
->x 1
O f-9
(0.0) ● -J ?

v/' '.V. : o ''


' By Green’s Ih^rem the arw bf the lobp
= the area enclosed by the simple closed curve C

= ^ ^ (jc dy-y dx), by Green’s theorem ● ● ●. ■;;h ■

Tir V. -- A -iij
*(■ A/., ■
https://www.pdfnotes.co/

226 VECTOR CALCULUS

1 f ^rx^-ydxl 1 f
2jc^ Jf2 2jc^^[xj
(1)
puttingys^jc.
Puttingy=rjir in the given equation of the folium,we have
jc^+rV = 3axtx
or j^(l+^ = 3atj^
3at
or

Also for half the loop t varies from 0to 1.


/. from (1),the required area of the loop

kSc l+("
dt

r a.¥
=2.i2 J/=0 "● dt, since the loop is symmetrical about the

liney=x
= 3a dt

= 3a" iiidiiir
-1 , by power formula
0
t1
1 3a^
= 3fl2 - ‘
= 2

Ex. 18. Introducing A = N i-M j, ^Aoiv r/iar theformula in Green *s


theorem may he written as

E divAdxdy = ^ A*tkdSy

where n is the outward unit normal vector to C and sis the arc length
ofC.
Sol.WehaveA==iSri-MJ.
divA =
dx ^
'dN dM'
.●●j[XdivA<ir4' = JXR (ar
dxdy
https://www.pdfnotes.co/

GREEN’S,GAUSS’S AND STORE’S THEOREMS 227

=^(M dx-\-Ndy% by Green’s tbeorem.

=(Mi+Ni)*(dxi+dyj)=(Mi+Nj)*dr

dr
Now if t is a unit tangent vector to C,then t = ds^ ● Also if k is a
unit vector perpendicular toxy-plane,then t = kxn.
Mdx+Ndy =[(Mi+Af =[(A/i+Afj)*(kxn)]d:s
= [(Mi+Arj)xk]*nd;y =(Mixk+7Vjxk)*ndly
= i-Afj)»n/fa = A»n dly.
Hence the result.

Note. Puttin^ = in the


div(V0) above
dxify =^ result,
(V0)*n weds
get

or
JJ* V^^dxify =^^ds,
dn since ^ dn n.

§7.The Divergence theorem of Gauss.


Suppose V is the volume bounded by a closed piecewise smooth
surface S. Suppose F(r,y,z) is a vector friction ofposition which is
continuous and has con^uousfirstpartial derivatives in V. Then

m. F*n dS,
where n is the outwards drawn unit normal vector to S.
(Meerut 1983;RohUkhand 80;Allahabad 80 $ Kanpur 79;
Agra72; Tirupati89,93;Madras83)
https://www.pdfnotes.co/

228 VECTOR CALCULUS

Since F*n is the normal component ofvector Fitherefore diver


gence theorem may also be stated aS;Xollows:
The surface ifitegral ofthe normhljdihponent of a vector F taken
over^ a closed surface is equal to the integral ofthe dive^ence of¥ taken
over the volume enclosed by the surface.
Cartesian equivalent of Divergehce Theorein* !(Osmania 1990)
V " dFi dp2 dp2
UtF = i+F2j+Fa k.Then = div F p—+—
Ifa, Y are the anglessvhichoutward drawn unit normaln makes
with positive directions ofjc,y,z-axes,tten cos a,cos cos y are direc¬
tion cosines of n and we have
n = cos a i+cos +COS y.k.
F«n =(Fi i+F2j+Fs k)«(cos a i+cos/8j+cos y k)
A cos
Therefore the divergence theorem can be written as

dxdydz

(F{cos cos'/8+F3 cbs y)dS


; JJS - ● ,v '● I ; ● VJm j ' ●

(Fi dy(ki^^2^xJx+)fydx rfy).

The significance of diveigencp th^^r^ fact that a


surface integral niay be expressed as a voliiime iiite^ and vice versa.
Proof of the divergency theorem.(V' '
We shall first prove the theorem for a special region V which is
bounded by a piecewise smoQth Closed surface S and has the property
that any ^rai^t line pardilei to' i^^ dhe^ of the coordinate axes and
intersecting Khas only one segment (or a single point) in common with
V. IfR is the orthogonal projection of 5.oi}ijthexy^plane, then Kean be
represented in the form ^ /
●s -V
where (^ y) varies in F, 1

Obviously z=g(x,y) represents the';hpper portion of 5,


z=/(z,y) represents the lower portion 'SiMjS. and thqre may be a
:v'b KVa', '
remaining vertical pOrtibrt iS^ 'df S. ' ' '
hmiil.
V
t:: \
https://www.pdfnotes.co/
GREEN’S,GAUSS’S AND STORE’S THEOREMS 229

% Hi ■< ■

‘Yil
r S

I
/ I ^ S2
1
:l ^ I
I
I V. , ,!■ V
I I
0 — T >
I':.- ' I
■ , I'.M
i
I
\
R
}'.i

M. dm
dz JJR dxdy
Jz^f(x,y) dz \
:o

f.ip
m
:
Ig /"a !<.>'./(*.y)l<irrfv, (1)
Now for the vertical portion of S, the normal H3 to 1S3 makes
J
a right angle y with k. Therefore
' J

= 0,.,Sjwe§nK'V%n=X)‘ : !. ● ,;

For the upper portion 5i 9f 5, the rtofirial hi to il inakes an acute


angle yj with k. Therefore
r
k*nirf5i = cos yid5i =-.dxjcfyi Henco

£ F3 k<*i9i.^iS'i.
https://www.pdfnotes.co/

230 VECTOR CALCULUS

For the lower portion ^2 of 5, the normal n2 to S2 makes an


obtuse angle 5/2 with k. Therefore
k*n2dS2 = cos/2d$2= -dxdy. Hence

£ F3k*n2</5'2 = -

/X jp3k*n3C^53 + F^k^n2dS2

E
^0+ P3
or with the help of(1),we get
hi‘.yJi^<y)¥>‘^

I F3k*nrf5 = JE V dz
dV. ...(2)
Similarly,by projecting5on the Other co-ordinate planes,we get

rr
Js F2j*nd5 = V dy
dV ...(3)

and dV (4)
Adding (2),(3)and (4),we get

E (Fii+F2j+i^3k)»nd5 =
(at dy dz^
dV

or
V.FdF=JJ ¥●11 dS.
The proof of the theorem can now be extended to a region V
which can be subdivided into finitely many special regions of the above
type by drawing auxiliary surfaces. In this case we apply the theorem to
each sub-region and then add the results. The sum of the volume
integrals over parts of V will be equal to the volume integral overK
The surface integrals over auxil
iary surfaces cancel in pairs,
while the sum of the remaining
surface integrals is equal to the
surface integral over the whole /
boundary S of V, Ui
Note. The divergence the- W=
orem is applicable for a region y:
V if it is bounded by two closed >
surfaces and $2 one of which
lies within the other. Here out-
https://www.pdfnotes.co/

GREEN’S,GAUSS’S AND STORE’S THEOREMS 231

ward drawn normals will have the directions as shown in the figure.
§8.Some deductions from divergence theorem.
1. Green’s theorem. Let 4> and tp be scalar pointfunctions which
together with theirderivativesin ar^direction are uniform and continuous
within the re^n V bounded by a closed surface S,then

jj[]r^(^vv-¥-vV)dK=jf][ {(pyip—tpVip)● n dS,


(Meerut 1979;Agra 71;Gauhati 92;Indore 79)
Proof. By divergence theorem,we have
F»n dS.

Putting F = we get
V*F = V*(0VV')
=ip(V*V^)+(V^)»(V^)= ^vV+(v^)*(vv>).
Also F»n =(0Vv>)*n.
divergence theorem gives

f£ [^vV+(V^)«(Vv»)]dK
If. (1)
(Osmania 1989, Meerut 70)
This is called Green'sfirst identity or theorem.
Interchanging <p and ip in(1),we get

Jj[|,.(vvV+(vv-)*W)l</K
°JJs (2
)
Subtracting(2)from (1), we get
Vv>—ipV<p)*ndS (3)
This is called Green’s second identity or Green’s theorem in sym
metricalform.
Since Vip = dn n and V<p = d± n,therefore
dip
dn
(<pVip-ipV<p)● n = ^ dn n — ip
^ dtp
dn nj »n
Hence(3)can also be written as
https://www.pdfnotes.co/

232 VECTOR CALCULUS

■ ’
'dS. i<, ■:<
V JJS \<P dn
-t drt^ /
1 (Mwnitl980)
Note. Harmonic function.'// a jcj/flr /?owr^ndtO)h ^ satisfies
Laplace’s equatian V^4> ^ '(^fiim^ called harmonic fiirfiiiipA} If ^ and
xp are both harmonic fthctioM, then vV - P» a● i

d:
Hence from Green’s second identity, we get
t; .O':.-;
d5' = 0.
dn

<l>ndSi ‘
2. Prove that Jjjy '5 M >

(Agra 1972iAllaliabad 77)


Proof. By divergencejtheorem, we have

'V
V*FdF
1 FfndS.

Taking F = 0C where C is an arbitrary cdhstanrnp^i^ro vec^^


'j-it

* i
we get
...(1)

Now^ V«(0C) = (V0)*C+0(V»C)


●iZJ-
(V^)*C, since V*C = 0
Also(0C)»n = i' ‘ ■fb i ;;
.'. (1) becomes f ® <i

JHc.W)dK= JJ; C(^rt)dS


or C»
:G‘Jj^(0n)dS
c*
or
SSvn‘^ ifS
(pads 0.
'i

Since C is an arbitrary vector, therefore we miKt have


V(pdV =
JJ5
3. Prove that VXBdK nxBdS
V s
(Gauliati 1974)
Proof. In divergence theorem taking F BxC, where C is an
arbitrary constant vector, we get
...(1)
iSC V-(BXC) JJ (BXC).ndS:
https://www.pdfnotes.co/

GREEN5S,GAUSS’S AND STORE’S THEOREMS 233

Now y ●(B X C)= C ● cui;^ C;


= C«curl B,since curl C = 0.
Also(BxC)«n =[B; C‘li] hi Bj= C*(nxB)
,.●.(1) brines ? 'sruid/’ii

(C*curlB)rfF t'N: C*(hXB)dS


t’

or €●
M (VxB)dV=C* f
^ -.c
(nxB)rfS

or C*
m (VxB)tiKr^ |J^nxB)d5 =0.
Since € is an arbitrary yectifr therefore we can take C as a
non-zero vector which is not per^enjlt^lar to the vector
dS.
JS
Hen<;e we must have . ^ .
(nxB)rfS = p
or

Solved Exahiples

Ex. 1. For any,closed surface^f^Yove'didt


\ :)
Jj^ c«/f F*ha.?y or ■
Sol. By divergence theorem^^e have
Ifs curlF.ddS ■= curl »\ where Kis the
f'
■ Vblume enclosed by 5
0, siiicies div curl F=0
Ex. 2. Evaluate
^ r ● hi/s, where Sis a closed surface.
(Andhra 1992, Madi^ 83, Rphiliyah^ 75)
Sol. By the divergence theorem, we havb
ff r*ndS= rfr VTdF V

-M 3 dVy since V*r « diy r=3

-3V, whet^ V is thewoliime ehelosed by S.


/

https://www.pdfnotes.co/

234 VECTOR CALCULUS

Ex. 3. // F = axl+by^+czk, a^b^c are constants show that


4
F»n dS = -Jt(a+b+c),where S is the surface ofa unit sphere,
(Rohilkhand 1992,Allahabad 82,Agra 1980)
Sol. By the divergence theorem,we have

ffs F-ndS= j[H'^(V*F)dK


where V is the volume enclosed by S /
E [V*(ax i+byj+cz k)]dV

^(ax)+^m+i(cz)]dy
4
= (a+b+c)dV^(a+b+c) (a+b+c)jTt,
since the volume V enclosed by a sphere of unit radius is equal to
t.e., |;r.
Ex.4.Ifnis the unit outward drawn normal to any closed surface
S,show that
E divndV=S.
Sol. We have by the divergence theorem,
(Andhra 1989)

rfrV divndV
Ex.5. Prove that
=1 -SL n*ndS dS = S.

j[j()r^V^.AdK= JJ JJf^^V.AdK
Sol. By divergence theorem,we have

(^A)«nrf5 (1)

Now V*(^A)=(V^)*A+0(V«A).
Also (0A)*n = ^(A*n).
Hence(1)gives

((V0)«A+0(V*A)]dF =/x <f>K*ndS

or (v<py\dv <pW*AdV.
https://www.pdfnotes.co/

GREEN’S,GAUSS’S AND STORE’S THEOREMS 235

Ex.6. Prove that ●dS = 0.

Sol. We have
I =X (^<p^VtpyndS.

■X V» (V^ x Vv») dV, by divergence throrem


=0.
[●.' V»(V^xVv») = 0]
Ex. 7. Prove that

X'» ●c«r/FdF
-X (F X VtpydS,
Sol. We have
X (F X V^).ds = X (F X
^<P) by divergence theorem applied to
the vector function F x

“ Jv F -curl V<p) dV

[V V*(A X B) = B»curl A - A ●curl B]


■X V^«curlFdK [*.* curl = 0]
r »n
jOltf=JX
Ex. 8. Prove that dS.

Sol
E, ■EM
r*n
dS

^r\ ^
*ndS

dVy by divergence theorem.

NowV* (A 1 /i \
1
p- = ^(VT) + r»V

-1,^=1.
r2 ,3(
Hence
E r *11
dS ^
dV

^ that for a closed surface S

C JX (pP »n dS. (Rohilkhand 1978,79)


https://www.pdfnotes.co/

236 VEC^rOR CALCULUS

Sol. By divergence theorem,we have.

'y [V*(Wl<iK
Now V(^ F)-(V0»F)+0{V«F)= F»F<*
= = F^,sinceV^^ = 0.

Hence ●ndS= I IV F^dK

Ex. 10. IfF = V^,VV =- Ajipy show that

JT. JK F*nrf5=-4jr
Sol. By divergence theorem, we have
pdV.

it -s 47rp.
Now y <*F = v*(v^) = yV

-I ■s F»ndS (- 4;rp) dV=~ 47t \yP‘»'-

Ex. 11. IfC =|V X B/B = V X ;


i|\:
● ■ .:1 ■
A X B*ni/S.+ 1^ A.CdK
2 '5

Sol. We have by divergence theorem


^ ..
iJI
1
(A x ^)indS 2JL V*(A^X‘B)dK?
. 'V'’
Now V«(A X B) = B»curl Ar A»curl B
= B«(V X A) - A*(V X p)= B*B - A»(2(G) 2i[A-C).
m.
Hence \ 2{A*C)]dV
S (A x B)-ndS = l Jj(r (B'
_ 1 \*CdV ,\

or
1
2

Ex. 12. If4> is harmonic in V, then


JJ AxB*n<iS + s A*CdV.

iK »
d<p dS = 0

(Meerut 1972)
where S is the surface enclosing V. ■ ■' \\

Sol. We have n
S dn
https://www.pdfnotes.co/

GREEN’S,GAUSS’S iWD STORE’S THEOREMS 237

■JJ,
} ' f

' »■

: ■>

y ● (V^) dy, by divergence theorem

'^{pdV

- 0, since = 0 in ^because <p is hhrmonic in V.


Ex. 13. If <p is harmonic mV,‘theri

E <P dS = \^<p\^dV

(Meerut 1979; Agra 70)


Sol. We have
I
/
I £ <P dn dS s
I
dn (^V^yndS

Jy dV,hydivergentthwrem

[<y4>f +Vy^^]dK
IV^I^dF.since = Oand (V^j^=
' i . '

Ex. 14. If d> is harmonic in Vand C on S, then <f> is constant


0
inV
Sol. Since^ is harmonic in V, therefo/e as in exercise 13, we have

E <P dn
dS \V^\^dV.

But
dh = 0 on 5. Therefore JJ^ ^dS = 0.

|V0|^ = OinK
V0 = OinK.
= constant in V.
Ex. 15. If 4> and ip are harmonic in{Vand d<p _ dip on S, then
dn 7; dn
(psitp ■{‘CinV, where c is a com
https://www.pdfnotes.co/

238 VECTOR CALCULUS

Sol. We have, = 0,vV =0in K


V') = = 0in K
Therefore^ is harmonic in V.
Again on 5, a« = 0.
dn dn
Thus <p — yf is harmonic in V and on S we have
A(0_^)
dn = O.
Hence as in exercise 14,we have
<p-yf^Cy where c is a constant
or ^ + c.
Ex.16. Ifdiv F denotes the divergence of^vectorfield Yota point
P,show that

div F =
lim
dV^O
JL dV
where dVisthe volume enclosed by the surface dSand the limitis obtained
by shrinking dVto the pointP.
Sol. We have by the divergence theorem.
¥*ndS. ...(1)

By the mean value theorem ofintegral calculus,the left hand side


can be written as
divF ^dV=dw¥dV,
where div F is some value intermediate between the maximum and
minimum of div F throughout6V.Therefore(1)gives

divF<5K= £ F»nd5

or divF £ F»ndS
dV
Taking the limit as 5K-> 0such thatP is always interior to dV,
div F approaches the value div F at point P.Hence,we get

divF =
lim
dV-*0
£ ¥»ndS
dV
Ex.17. Show that
JK ndS^ Ofor any closed surface
https://www.pdfnotes.co/

GREEN’S,GAUSS’S AND STORE’S THEOREMS 239

Solution. Let C be any arbitraiy constant vector. Then


c*

(V»C)rfK,by divergence theorem


= 0,since div C = 0.
ThusC»
jtt n = 0,where C is an arbitraiy vector.

ndS = 0.
Therefore we must have JJ^
Ex.18. /Vove that
xndS =s0for any closed surface S,
SoLJ^t C be any arbitraiy constant vector. Then
c*
E TXndS=JJ^ C*[(rxn))<iS

■E (C X r)»nrf5

[V*(C X r)] dVy by divergence theorem

-SE [r«curl C - C*curl r] dV


= 0, since curl C = 0 and curl r = 0.
ThusC
■E r X n ^5 = 0, where C is an arbitraiy vector.

Therefore, we must have rXndS^O.

Ex. 19. Prove that

Sol.
E (y<p) X ndS = Qfor a closed surface S,
Let C be an arbitraiy constant vector. Then
c*
SI C*[(V^) X n]d5

[C X Vtp]»ndS

y [V*(C X V0)] dVy by div. theorem

= fjfy [V^-curl C - C*curl V^] dV


= 0, smce curl C = 0 and curl = 0.
ThusC*
E (V^) xn dS" = 0, where C is an arbitraiy vector.
https://www.pdfnotes.co/

[i ●
] VECTOR CAECULOS

Hence we must have (V<^)Xnrf.S = O.V

>
Ex.?0. Prove that fL n x (a x r)dS = 2Ka
where a is a constant vector and Vis the volume ericlokd by the closed
surface S.
Sol. - We know Aat n.> J

VxBdK n X BrfS.
V K &
'.i i -.
[See §8, part 3 page 232]
Putting a x.i^w?get , ;

I
. €J
n X;(^ jJJv
y X (a X

ciiri(a x

1^ 2a dVy since (a x r)= 2a:

Ex. 21. A vector% is alivayi normal to a ^yen closed surface S.


Show that
s curlBdV
Sbl^ We know ttat
i
O^i^e Vis the region bounded by S,

Since B is normal to 5, therefwe


r:
B is parallel to n. Therefore
n X B =a
n'X 0.
'S
V

curl B rfK=0
V

Ex.22. Express Jy {(grad pi)● V + p div v}(fVias a surface inte-


(Gauhati 1977)
gral.
Sol. From a vector identity we loiow that
diV i^'(gfatl/j>*vVpdiv v.

‘ div(py)^^
{(gradp)*v +i£)diyvfdK=
https://www.pdfnotes.co/
GREEN’S,GAUSS’S AND STORE’S THEOREMS 241

-X V*(pv)dV
= (pv)● n dS,by Gauss divergence theorem

Ex.23. Usmgthe divergence theorem,show that the volume Vofa


region Tboundedfyji surface Sis

‘'■fl xdydz^ JQC

~^JJs ’^^dxdy).
Sol. By divergence theorem,we have

ffs j’**=i!X (i(^))’^=X0C


Adding these results, we get
3K
=JI (xdydz +ydzdx +zdxify)
or
■iM (xdydz +ydzdx’¥zdxify).
Ex. 24. (a) Ver^ divergence theorem for
F = (x^-yz) I + (y^ -zx) j + (z^-xy) k
taken over the rectangular parallelopiped
0 X ^ a, 0 ^y < h, 0 ^ z :S c.
(Andhra 1990, Meerut 76)
Sol. We have div F = V »F

= j^{^-yz) + -^(/-zx) + -xy) = 2x-h2y + 2z.


.*. volume integral = f£ JJ)[,2(x+y + z)rfK

=2 P P P
«/zs0 ●/y=0 Airs
(x +y + z)dxdydz
TO
x2
2p P
Jz=0 Jy=0 —+yx
2 + zx
x=o
https://www.pdfnotes.co/

242 VECTOR CALCULUS

2 rr
^2=0 Jy=0 2
a2
●\- ay-¥ az dydz

lb
\a^ / dz
2 ‘ ^a- -\-azy
=0 —y
. y=o
*c ' 2 c
a^b ab^ a^b ab^ z2
2;
12=0 2 2
+ abz dz = 2 z + z + ab —
2 2
2 0
= [(P'bc + ab\ + abc^] = dbc {a + b + c).
Surface Integral. We shall now calculate

ff. F*^ndS

over the six faces of the rectangular parallelopiped.

Z t t

C
B

D E

F
X

dver the face i)£FCx,


n-i, x = n.
Therefore,

jj
DEFG
F*ndS

= X=oX=o + 0^ -“)J + (2^ -


=XIoX!o =X=o k - Jy=0
dz
https://www.pdfnotes.co/

GREEN’S,GAUSS’S AND STORE’S THEOREMS 243

=rJz=0
a^b — =
4 0

= a^bc -
cV
4
Over the face^CO, li =-i; jc = athei^re

= Jff'ndS = Jj\(0-yi)i + ..

=r r
●fz=0«/y=0
yzdydz 'z=0
/r dz

= Jz=o
r ^2
y=0

Z<fe =
4
Chrer the face^£F, n = j, y = h.
F*nd5
" £o ^ -“)i
: + (2^ - te ) k] »j dr d?

=r r
«/Z-0jzs
Over the fe(« OGZ)C; b = - j,
- zx)dxdz =
== 6: ther^^
-2.2
fl C
^

F -ndS -r f a
zxdxdz ,«v
«/z=0«/x=0 4
OGDC
Over the fece FCDE, n = k, z = c. Therefore

ir
BCDE
F*ndS= *r^=o*/x=d
P P (c^- xy)dxdy = c^ab - 5^.
4

Over the face^GO, n =- k, z =i OMherefore

ir F «iid5= Jy=0
j4EC?0
a
r Jx=0
f xydx^ 4

Adding the six surface integrals, we get

g F.n^=fa%c_£^+ 4
c^V
4 4 *j
2i.2
+ c^ab-^j-^
4 4_
https://www.pdfnotes.co/
244 VECTOR CALCULUS

= abc(a+ b + c).
Hence the theorem is verified.

Ex.24.(b). = 4jcz i - and Sis the surface bounded


X =0,)> = 0, z = 0, jc = 1, )> = 1, z = 1,evaluate

fjs (Osmania 1990)


Sol. By Gauss divergence th^rem,

jj,F»ndS V*FdK
where Kis the volume enclosed by the surface S

^.(4x2)+|;(-/)+|0-z)]dK
(4z —y)dx<fydz

=r r r
Jx=0 Jy=0 ●lz=>0
(4z-y)dxdydz

= /‘o/‘
Jx—Ojy^O [22*
L
Tl
-yzJz=0 dxdy =r r
«fx>=0 Jy—0 (2-y)dxdy

=x: =0
/1‘
Jy = 0
dx
=rhii‘^=§r dr =
3
2

Ex. 24. (c). Evafuare [Axzdydz-^dzdx+yzdxdy]


- where S is the surface of the cube bounded by the planes x = 0, y = 0,
z = 0*x=l, y=l arulz = 1. (Osmania 1992)
Sol. By Gauss divergence theorem, the given surface integral is
equal to the volume integral

=Iffy [i i
where K is the volume enclosed by the surface S.

Ex. 24. (d). Evaluate J^T F»n<iS^F=xyi+z^j + 2yzk


over the tetrahedron bounded by die planes x = 0, y = 0, z = 0 and
x+y + z=l.
Sol. By Gauss divergence theorem,
V«F dK, where Fis the volume enclosed by
https://www.pdfnotes.co/

GREEN’S,GAUSS’S AND STORE’S THEOREMS 245

the surface S of the tetrahedron

fJjy +
M (y + 0+ 2y)dV
1-X
3 P P ^P ^
Jx=‘0jy=0 Jz=0
’11-x-y
z=0
dxdy
1-x
=3P f >' [1 -X —y\dxdy
●/x=0jy=0

°^X=oX=o *>’-^1*'^ l-x

= 3 f‘ dx
[2 2 3J^„,
= ^X* - 5^(1 - j(l
aV
4
-1 = 1
24 8
0

Ex. 24. (e). Find the vali^ of JJ* (F X V^)«nrf5


where F = x^ I +/j + ^ -xy+yz-{: zx.
Si— X = ± 1,)^ = ± 1, z = ± 1. (Nagarjuna 1991)
So. By Gauss divergeii(» theorem,

jjs (FxV^)-n^5 = ^ V. (FxV^)dK,


where V is the volume enclosed by the surface S.
Now from a vector identity, we know that
V*(A X B) = B«(V X A) - A«(V X B).
V*(F X V<p) = V^«(V X F) - F»{V X (V0)}
= V^»(V X F). [[*.● V X (V^) = curl grad ^ = 0]
Now V X F = V X (x^i -i-/j -I- z^k)
i j k
d d d
dx ^ dz
^
https://www.pdfnotes.co/
246 VECTOR CALCULUS

= 0i + 0j + 0k = 0.
.*. V»(F X V0)=(V^)*0 = 0.
Hence the given integ^

=M
Ex. 25. Verify divergence theorem for ¥ ={2x-z)l+J^yi
— xz\ taken over the regfon bounded hy x = 0, Jc = 1,)'= 0,>? = 1,
z = 0,z = 1.
(Rohilkhand 1999;Agra 85)
Sol. By divergence theorem we have

JJ F.n^= \y (yF)dV,
where 5 is the surface of the cube bounded by the planes x = 0,
X = 1, y = 0, y = 1, 2 = 0, z = 1 and V is the volume enclosed by the
surfaces.
We have V«F = V« [(2x -z)I +x^yj-xz\]
=±(2x-z)+±oPy)+±(-x^)
= 2+x^-2xz.

jJI,(V.F)rfK= (2 -2xz)dV
= X’oXloiloP
=r r
Jz=0 Jy=0 2x + y-X^Z
1

Jt=0
(fydz

= r r 24-zU<fc= rj'ji-
jz=0jy
y=o
1

3
ii.
j jz=0jy=0 \3
z dydz

-T fl U
dz
L Jy=0 -Jo [3 "J*
- Z,_i,2V_2_i_u .,.(1)
~P 2^Jj“3 2“ 6
https://www.pdfnotes.co/
GREEN’S,GAUSS’S AND STORE’S THEOREMS 247

We shall now calculate


/X F*n dS over the six faces of the cube.
Draw figure as in solved example 24 (a). Over the face OABC
which lies in theyz:-plane, n =-i, X = 0.

s ■r r
Therefore
OABC
%/z—O (-ziy(-i)ifydz

rr =r
a/z-O Jy=0
dz ZdZ=:
11

2
0
1
2
Over the opposite face Z)£FG, x = 1, n = I.
Therefore

If = X=oX=oK2-^)‘+)'i-^]
DEFG

= r r (2-z)4,dz=£^
JzssQ Uy—0 (2-2) y
1

z^V = 2-1-1
-X' (2-z)&= 22-^
2 0 2 2
Over the face OGDC which lies in thezr-plane,y = 0, n =—j.

IfF*ndS= r r
OGDC
Jz=0Jx=0 (2x-z)i-xz^k ●(-j)rfzdr

=r r
●/z=0 ●/jc=30
0dfedr = 0.

Over the opposite face y = 1, n = j.

XX rf
F»n</5 =
1

●/2=0 ●/X!= 0 [(2x —z)i + x^j -xz^ k]»jdziir

= Pc
Jz=0 Jj(=0 r
Jz=0 3
x=o
dz = JoP-3 dz

3LJo 3
Over the face OGF4 which lies in thejiy-plane,z = 0, n =- k.

OGFA
=X!Xo(^‘ (- k)dr^
https://www.pdfnotes.co/

248 VECTOR CALCULUS

=r r
jx’=0jy=‘0
Odxity^^O.
Over the opposite face BCDE,z = 1, n = k.

JJ
BCDE
¥*ndS =

=r r
Jjc=0 Jy=0
— xdxify -f- ni
Jx=0 X \'jy=0
y
■X‘-
1
2 0 2
Adding the six surface integrals, we get
11
...(2)
ffs F*ndS = i + | + 0 + i + 0-i = 6
From (1) and (2), w^s^ that

fS =1 F*ndS.
This verifies Gauss divergence theorem.
Ex. 26. yieri^ divergence theorem for F = 4xzl +yzk
taken over the cube bounded byx = 0,.r = l,y = 0,>» = 1, z = 0,z = 1.
Sol. Proceed as in solved example 25. Here we shall have

M SSv (V*F)dV=

-LXX (4z - 2y +y)dx(fydz

The six surface integrals will come out to be 2,0, -1,0, | and 0.
Their sum = I*
Hence the theorem is verified.
Ex. 27. Evaluate

Jjjl^ (fy dz + y^ dz dx + 2z {xy-X -y)dxdy


where S is the surface of the cube
0^z< 1, < 1, 0sz< 1. (Meerut 1986)
Sol. By divergence theorem, the given surface integral is equal
to the volume integral
https://www.pdfnotes.co/

GREEN’S,GAUSS’S AND STORE’S THEOREMS 249

= X=0 X!o X!o 12* + 2y + 2a:)-- 2a:- 2>>]dr &


=2 f r r r r \iyV
Jz=0 Jy=0 Jx=0 xydxdydz = 2 Jz=0 Jy=0 2^ tfydz
x=o.

dz
=0
1
L2jy=o
1,
= X=oi*= H^Jo
Ex. 28. Evaluate, by Green’s theorem in space (i^., Gauss diver
gence theorem), the integral
Axz dy dz —^dz dx + yzdx dy,
where S is the surface of the cube bounded by the planes jc - 0, = 0,
2= 0, X = 1, }> = 1, z = 1. (Me6rut 1974,Kanpur 77)
Sol. Let Kbe the volume enclosed by the surface S. Then by
Gauss divergence theorem,we have

JJ^ Axzdydz-^dzdx+yzdxdy

=JK [i(4**)+i(-^)+1O'*)]
= I.as in solved example 26.
Ex.29. Use Gauss divergence theorem to show that

Jf {(x^-yz)^-2x^yj■t:2k}^adS = ja^,
where S denotes the surface of the cube bounded by the planes x = 0,
x-a,y = Q,y = a, z = 0, z = a. (Rohilkhand 1979, Agra 77)
Sol. Let Kbe the volume enclosed by the surface S of the given
cube. Then by Gauss divergence theorem, we have

Jfj|[ {(x^-yz)i-2x2yj + 2k}*nd5

^ i(^'3'") + |;(-2x2y) + A(2)jdK


+ 0)dK
https://www.pdfnotes.co/
250 VECTOR CALCULUS

= XioXlo J = X=oX=o
na a5
= tJ.=oH,=o‘^ =tJo ‘’'^ = Th„"■ 3

Ex. 30. Evaluate I


g (x i + ;/j + 2 k) *11 dS wh ere S denotes
the surface of the cube bounded by the planes x = 0, y = 0, z = 0,
x=^a,y-a,z - a by the application ofGauss divergence theorerru Verify
your answer by evaluating the inteff'al directly. (Agra 1979)
Sol. Let Kbe the volume bounded by the surface S of the given
cube. Then by Gauss divergence theorem, we have
rC (xi + yi + zkyndS = f(T [div(xi + yj + zk)]dV

3 dV = 3K = Za^, asV=a^ = the volume of the cube


whose each edge is of length a.
To verify our answer we shall evaluate
over the six faces of the cube.
' Draw figure as in solved example 24 (a).
Over the face (24BC which lies in theyz-plane, n = - i, x = 0.

ff
04RC
(xi +yj +zk)»iwi5 =
OABC
(yj+zk)*(-i)d5 = 0.

Over the opposite face DEFG, x = a, n = i.

' fj (>:i+yj+zk)*nrf5 =
DEFG
J
DEFG
(ai + yj +zk)»i<iS

~JJ adS-a dS - a. area of the face DEFG


DEFG DEFG
= art — a .
Similarly calculate the other four surface integrals.
Adding the six surface integrals, we get
https://www.pdfnotes.co/
GREEN’S,GAUSS’S AND STORE’S THEOREMS 251

j[j^ (xi +>>j +zk)»nrf5 =0+ + 0+ <2^ +0+ = 3a^.


Since
J[j[ (;cI+yj+zk).n<iS =
therefore our answer is verified.
II [div(jfi+3;j+zk)]dK,

Ex.31.(a). By transforming to a triple integral evaluate

J0[ (x^dydz +x^ydtdx+x^zdxdy)


where Sis the closed surface bounded by the planesz = 0, z = bandthe
cylinderjp" +y^ = a\ (Meerut 1980)
Solution. By divergence theorem, the required surface integral
Iis equal to the volume integral

My [i +i i
=ioJl. +x^+i^d,dydz
=4x5 rb pa pV(a^-y^
Jz^O Jy=0 Jx‘=0 x^dxdydz

= 20 fri
Jz^O Jy=0 3 jc=0
dydz

-f [(●■ - f jC.‘
Puty = a sin t so that dy = a cos tdt.
n/2

3 a^cos^t(acost)dt
Ji/2

-f-H f Af 3 JT 5 4t
COS tdt^-ab-^—^-jiab.

Ex. 31. (b). Evaluate


Jf (zx^dttfy+Ptfydz+yx^dzdx)
»’hereSistheclosedswfaceconsistmgofthe<ylmdcrii^ +/ = iandthe
dmdar^z = 0andz=:i. (Osmanla 1990)
Sol. Proceed as in Ex. 31<a). (Ans.60te)
Ex. 32. Apply Gauss’s divergence theorem to evaluate
https://www.pdfnotes.co/

252 VECTOR CALCULUS

[{j^-yz)dydz-Ix^ydzdx+zdxdy]

overthe surface ofa cube bounded by the coordinateplanes and theplanes


x=y =z = a.
Sol. By divergence theorem,we have

JJ^ (Fitfydz+ F2dzdx+ F2dxdy)


IdF,
1
■M ay dz / dxdydz.
HereFj -]? -yz^ F2-- 2jc^y, F^= z.
dF-t dFn dFg
*—+dx -^ dy
+ dz = 3jc^-2x^ + 1=x2 + 1.
the given surface integral is equal to the volume integral

■^ +x ifydz
Jf*fl pa
z~0 Jy=0 3 JC=0

fa C"* (a^ \ 2(0^


= Jz=0
I I
●/y=0 3 + dydz = a^ ^ +n ●
\ / \3 /
Ex. 33. 7/F = Jd - yj f (z^ - l)k,^ the value of

HE F*ndS where S is the closed surface bounded by the planes

2 = 0, z = 1 and the cylinder y^ = 4.


(Kanpur 1980)

-s
Sol. By divergence theorem, we have

jo: F»ndS = divFdK

HeredivF = A(^) + |(_^) + A(^_l)


= 1 - 1 + 2z = 2z.

Jtt divFdF = rr r V(4-y^


^z-O tJy- —2 Jx =-V(4-y^
Izdxdydz

=r r [
●/z=0*/y=-2L
2zz
https://www.pdfnotes.co/
GREEN’S,GAUSS’S AND STORE’S THEOREMS .253

dy
2=0

i2
=4te.
= 4 |\'(4-/)+ 2sin *1JO =4[2sin-‘ 1J =4(2)|

where
Ex.34. Find
iJ A»n dSt

A =(2x + 3z)i -(jcz+y)j+(3;^+2z)k


and S is the surface ofthe sphere having centre at f3, -1,2)and radius
3. (Kakatiya 195K), Meerut 74)
Sol. Let F be the volume enclosed by the surface S. Then by
Gauss divergence theorem,we^have

Sis A*»‘« = divAdK

NowdivA =^(2z+ 3z)+ (xz+y)} + 2z)


= 2- 1 + 2= 3.

£ A«nd5
■M 3dF=3 ^ 3K
But. F is the volume of a sphere of radius 3. Therefore
F=|jt(3)^=:36w.

ji A*nd5 = 3F = 3 X 36n: = lOSjff.


Ex. 35. (a). Apply divergence theorem to evaluate

j[fl [(-X+^)tfydz + (y+z)dzdx + (x+y)dx£fy]


where S is tlK surface ofthe sphere + z^ = 4. (Andhra 1989)
Sol. By divergence theorem* the given surface integral is equal
to the volume integral

IS +io-+2)+ +y)]<«'
J£ 2dF=2
(^. dV = 2F, where Fis the
volume of the spherex^ +y^ + z^ = 4
=2
*|^(2)^]=f..
https://www.pdfnotes.co/

254 VECTOR CALCULUS

Ex.35.(b). ^using the Gauss divergence theorem,^aluate

iC {xdydz +ydzdx+zdxdy)where Sis the surface ofthe sphere

;r"+/+/= 4. (Osmania 1991)


Sol. By Gauss divergence theorem,the given surfoce integral is
equal to the volume integral

V ^ ^^ where Kis the volume


enclosed by the spherejc^ +2^ = 4
(l + l + l)dK=3 JJT dK=3K=3 |«(2)»
= 32;r.
Ex.36. IfSis any closed surface enclosing a volume Vand
F =jc 1 + 2yj+ 32 k,prove r/iflir

JOE F.n^=6K
(Rohilkhand 1980,Kanpur 79,Agra 78)
Sol. By divergence theorem,we have

I F*nd5 =
^ divFrfK= div(xl + 2yj+ 32k)JK

=JK
dV^6V.
\ .yj\.
Ex.37. Evaluate

J[][ O^z^l +z^jr^j+z^y^k)●ndS


where Sis thepart ofthe spherej^+^ +z^ = l above the^-plane and
bounded by thisplane, (Agra 1969, Bombay 66)
Sol. By divergence theorem,we have
(/^l+z^x^i +^/k)●ndS

-Jffi div I +2Vj+ 2^y2 k)dV, '


} I ,
's.-

where V is the volume enclosed by S


7- !

2z/dV=2 ,z/dV.
https://www.pdfnotes.co/

GREEN’S,GAUSS’S AND STORE’S THEOREMS 255

We shall use spherical polar coordinates(r,0, to evaluate this


triple integral. In polars dV=(dr)(rdO)(r sin 0d<p)= P‘sin 6 dr dO
d<p. Alsoz = r cos0, y = rsinOsin To cover Kthe limits ofr will be

0to 1,those of0


|will be0to and those of^ wiU be0to 2jc. The triple
integral is
2
U J«=o
Jri ^jr/2 X=0
p2jt 0sin^^) sin0dr d0 dtp
2
jHoX=o J^^^Bcos0sis?^drdBd4,

1 p/2
6j0=o J4>=o sm^0fkii0sm^pd0d(p,
on integrating with respect tor.
[Note that the order of integration is immaterial because the
limits ofr,0and <p are all constants],
=i 2
3*4*2
1 . n/2
ism <pd<pt on integrating with res^t to0

12
4
Xsufipdp-^ 1 i.jT 7t
3 2* 2 12

evalu^ ^^ the surface inte^al into a volume integral

(x^^dz+/dzdii +z^dxify)^
where Sis the surface ofthe spherer? + = 1. (Bombay 1970)
Sol. By divergence theorem, we have

jffi {F-^dydZ’^F2dzdx + F^dxdy)


'dF^ ^ dF
V dx- 0y 0Z- dxdy dz
where V is the volume enclosed by S.
HereFj F^ =/,Fg =z^.
OFj dF2
ar ^
the (ren surface integral
https://www.pdfnotes.co/
256 VECTOR CALCULUS

smedrded<f)y changing to polar


shperical coordinates
llji
= 3x2;rx2xi=-j
Ex.39. Evaluate by divergence theorem the integral
r x?dfdz-\-(]?y-;?)dzdx +{2xy*r^)<>^^<
JJS
where S is the entire surface of the hemispherical region bounded by
z = y/(a^-J^-/)andz = 0. (Meerut 1974)
Sol. HereSis the part ofthe spherejr^+)^+z^ = a^above the
jcy-plane and bounded by this plane.
Let Kbe the volume enclosed by5.By Gauss divergence theorem,
we have

JpT x^dydz + {j?y-2?)dzdx + i2xy+/z)dxify

...(1)

We shall use spherical polar coordinates {r,6,0)to evaluate the


triple integral(1).In polars,z = r cos 0, jr = rsin0cos
y = rsin0sin^,dF= (dr){rdd)(rsin0d^)= r^sinddrdddtp.
Alsoz^ +y^ + z^ = t^. To cover Kthe limits ofr will be0to a,those of
0will be0tojr/2and those of^will be0to 2;r.Hence the triple integral
(1)
^ ra rjt/2 sin0drd0d0
Jr=0 J$=0 */0=O
2jt
r"^ Sin d dr d9dtp
r=oJd—0
Jro rji/2 r
^=0

pjt/2 P2a Lsl®


sin0d0d0
*/0=o »l^=o 5 r=0
5 ‘jr/2
a'
r-/2 p
5 J«=0 J#=0 = yJ^_^jSine.
dO
●jr/2
iTta^ .„ , 2aa’
5 JO s“e<«« =-5—1=-5—
Ex.40. By using Gauss divergence theorem,evaluate
https://www.pdfnotes.co/
GREEN’S,GAUSS’S AND STORE’S THEOREMS 257

J[j[ (xi+,-J+z2k)*ii</S
where S is the closed surface bounded by the cone = and the
planez = 1. (Agra 1973)
Sol. Let V the volume enclosed by the closed surface 5.Then by
Gauss divergence theorem,we have

i
-JK-
-E (2 + 2z)dV^ where V is the region bounded by t|ic
surfacesz 0, z = l,z^ -j? +y^

ifs r
Jz=0 Jy--z Jx=-V(z‘-y^
(1 +z)dxdydz

=8
nz
-1 I dz
r/J_y=0
\z^ 7t
■>£ (1+z) Yi dz = 2jL
dz

= 2jt
3 4 JO = i» 5 + 3
Ex. 41. Evaluate I V»ndS over the entire surface ofthe region

above the xy-plane bounded by the cone z^ = jc^ +y^ and the plane
z^AJf
F = 4jcz i + jcyz^ j + 3z k.
SqK By divergence theorem, we have

ic F*n<f5 =
^ divFdK,
https://www.pdfnotes.co/
258 VECTOR CALCULUS

where Fis the volume enclosed by S.

Here div F = -^ i^xz)+^{xy^P") + -^(3z)= 4z + xz^ + 3.


Also V is the region bounded by the surfaces
z = 0, z = 4, z^=jr^+y^.

Therefore JQII^ divF <iK= M (4z + xz^ + 3)dx(tydz

rr r 'K^-yh
(4z +xz^ + 3)dxdydz
Jz=0 Jy^-zjx=-V-r^'

2Jz=0
r Jy=
r —zjx=0
r (4z + 3)dxtfydz,
rV(z^-y^
xdx = 0
Jx=-Viz^-y^

on integrating with respect to x

='* XIo Jp.0 ~/)dydz


dz
^ 0

“^Jo dz = jr r<‘-- + 3z^)dz


-|4 ^ ^
ss jr
z"*+z^ Q = jT(256 + 64)= 32(hr.
Ex.42. Show that (x^l+/j+z^k)●n dS
vanishes where S denotes the surface of the ellipsoid
—+^
—+?
—= 1
a a c
Solution. We have by divergence theorem

JjjT i^l+/i+^kyndS
div I+ j + z^ k) dV, where V is the volume
enclosed by .S
https://www.pdfnotes.co/

GREEN’S,GAUSS’S AND STORE’S THEOREMS 259

^ {2x+ 2y+ 2z)dxdydz

=^r / byf{l-(z^/c^}

Jz=-c Jy=-6V{l-(zVc^} r (x +y +z)dxdydz

f l,=-W«-(zVcVO'+^)V
on integrating with respect tojc
Note that /(X)<ir =0if/(-x)=-/(x)and

= 2j[V)<fatf/(-it)=/(r)'
“8Jz=-c rbV{i-(z^/c^}
p *fv=0 //1-^-^
/I 4-<fe
rc n,*V{l-(zVc^)^
=*J.=-J,=« 1-^ -/ dydz

-\byf{\-(j?/c^}
^2
y
dz
b ^^{l -(z^/c^)}
Jy=o

=fb Jz=-c
r z[ffl-4)sin-‘l)d.
2

_8 p f, _ ;r
^dz = 0.
6 Jz=ic^ 2 [* c2j 2
Ex.43. C/5C divergence theorem to evaluate

M [xifydz +ydzdx +zdxify]y

where Sis the surfacejp' +y^ +z^ = 1.


Sol. Let Kbe the volume bounded by the surfaceSofthe sphere
a:^ + y^ + 2^ = 1. Then by Gauss divergence theorem,we have

JJ^ [xitydz +ydzdx +zdxdy]


https://www.pdfnotes.co/
\
260 VECTOR CALCULUS

dV = 3K,where V is the volume enclosed by the

sphere = 1 whose radius is 1


= 3 ●|;r-l^ = 4;r.

Ex. 44. Use divergence theorem tofind Y*ndSfor the vector

F =xi -yj + 2z kover the sphere]? +y^ + (z - 1)^ :== 1.


Sol. Here 5 is the surface of the sphere +y^ + (z - 1)^ = 1
whose centre is the point (0,0,1) and radius 1. Let Kbe the volume
' enclosed by S. Then by Gauss divergence theorem, we have

Jjj^ F»ndS = E (divF)dF

=ii(rK(‘-‘+2)</K=2j[[t,dF
= 2K, where V is the volume of the sphere
x^ + y^ + (z - 1)^ = 1 whose radius is 1
8

Ex. 45. ^F = or i + iy j + cz k, where a,b,c are constants, show


that
Ajt
ffs = Y(a+b + c),
S being the surface of the sphere (x - 1)^ + (y — 2)^ + (z - 3? = 1-
(Gauhati 1971)
Sol. By the divergence theorem, we have

/X (n*¥)dS = M (V*F) dV, where V is the volume


enclosed by the sphere S whose radius is 1

\
=JK
jt.
= (a + b + c)dV^ (a + h + c) K= (n + b + e) |
https://www.pdfnotes.co/

GREEN’S,GAUSS'S AND STQKE’S THEOREMS 261

since the volume Venclosed by a sphere of unit,radius is (1)-^


4‘
3
4
= -JT (fl + h + c).
Ex.46. Verify the divergence theoremfor
F = 4jri-2/j+z2k
taken over the region bounded by the surfaces = 4, z = 0,z = 3.
(Allahabad 1978)
Sol. Let S denote the closed surface bounded by the qrlinder
+y2 4 and pjanes z = 0,z = 3. Also let V be the volume
bounded by the surface S. Gauss divergence theorem,we have
[r IlLdiyFdV. ●
We have
M divFdV =

= fj[fy(^-4y + 2z)df'
2r f r
4Jz=0
f Jx= rV(4-x^ (2+z)dzdxdy
f-2Jy~0
[*.* 2y is an odd function ofy J
’2
● V^(4-,:*)
=4 dzdx
h=aJx=-2 2y+zy.y°o

‘£f. [2\^(4 -jc^)+2\^(4 -x^)]dzdx


i3

*£=-2 + dx
2=0
2 21
2 >/(4 -x^)dx = 4-^‘2 dx
https://www.pdfnotes.co/

262 VECTOR CALCULUS

2
Jt
= 84 2- = 84;r.
= 84 |>/(4-j?)+ 2sui"‘|JO 2
Now we shall evaluate the surface integral

E V*ndS.
For evaluating this surface integral give complete solution of
solved example 61 on page 197.
Thus ¥*ndS = 84ar.

We see that
E ¥*ndS divFdK.

This completes the verification of Gauss divergence ^eorem.


Ex. 47. Use Gauss divergence theorem to ^JI ¥*ndSy

where ¥ = 2j^y + 4x2^ k and Sis the closed surface in thefirst


octant bounded by^ + = 9andx = 2. (Kanpur 1976)
Sol. Let Kbe the volume enclosed by the closed surface S,Then
by Gauss divergence theorem,we have

JI 1^ F»nd5 = ISv
“JJIk +

-s (4xy“2y + 8xz)dF,where Kis the volume in the first

octant bounded by the cylinder +z^ = 9and the planes


X = 0, X = 2

^(9-z*)
^ Jx—0●lz=0 (2xy -y + Axz)dxdzdy

= ^J[!o£oP-5>" + 4xzy x-0 dxdz

= 2jCo£o p (9 - - i (9 - + 4XZ V(9 -z'^)]dxdz


https://www.pdfnotes.co/

GREEN’S,GAUSS’S AND STOKE’S THEOREMS 263

t2
dz
= Y(9-z^)-|(9-z^)+ 2x2W(9-z^
j:=0
3r
=2
0 L2(9-2^)-(9-/)+ 821^(9-2^] <iz

=^£[9-2^-4(-22)(9-2^)*'^] d2

= 2 9z-
J-4-|(9-2^H*
^ ^ Jo
= 2[27-9 + f-27]= 2(18 + 72)= 180.
Ex.48. Verify divergence theoremfor thefunction
F=;;i+j:j+z^k over the cylindrical region bounded byj^ -¥y^
z = 0andz = h. (Kanpur 1975; Allahabad 79)
Sol. Let S denote the closed surface bounded by the qrlinder
j^+y^ = a^ and the planes z = 0,2=A. Also let V be the volume
bounded by the surface S. By Gauss divergence theorem,we have

£ F^ndS =
^ divFdK.

We have
^ [div(yi+xj+z2k)]dK

“JOOC

-£oX:-j=« I zdzdxdy

-4 p r
*/z=0*fjr=-a
2 y
L'ly=o
dzdx
https://www.pdfnotes.co/

\
264 VECTOR CALCULUS

d!x
2=0

=8 yV(a^-ji^)dx = 4k^£V
.2
a jr
= 4/1^ = 4/|2
2 2

=jtan. ...(1)
Now we shall evaluate the surface integral

E ¥*ndS.
The surface S consists of three surfaces:(i) the surface iS^ of the
base of the cylinder i.e., the plane facez = 0,(ii) the surface 52 of the
top face of the cylinder U., the plane face z = h and (iii) the surface
53of the convex portion of the cylinder.
For the surface 5j U.y z = 0, F=yi+xj,puttingz = 0in F.
A unitvector n along the outward drawn normal to5^ is obviously
-k.

if.. Y*ndS
=JX, 0'l+^j)*(-k)<iS = 0.
For the surface S2 z = /i, F=yl+jcj+/*^k,puttingz-h
inF.
A unit vector n along the outward drawn normal to$2is given by
n = k.

it. ¥*udS
“iXj 0'« + Jcj+A^k)*k'''8

ffs, dS = h\ area of the plane face S2of


the cylinder
= h\jca^ = Jta^h^.

For the convex portion i.e., jc^ a vector normal


to 53is given by
V(z^+/)= 2zi + 2yj.
https://www.pdfnotes.co/

GREEN’S,GAUSS’S AND STORE’S THEOREMS 265

n = a unit vector along outward drawn normal at any point of53


2xi + 2yj
— »since2^ + y^ = on 53.
V(4jc^ + 4y^)

on53,F*n =(yi +jfj+ z^k)*

a a ^ a xy.
Also dS - elementary area on the surface
= a dOdz,using cylindrical coordinates r,6,2.

}L F*nrfS

x = acos6, y-asind
rh r2jc
2a cos6a Sinddddz
“Jz=oJe=o
'231 r -\h
= 2^2 COSdSind z dO
w=o 2=0

231
cosdsindrfd = sin2Bdd

-\2si
cos 2d a\
^a\ -
●2 —^[cos 4jc - cos 0]= 0.
Jo
Hence the total surface integral

Jfj^ F*nd5 = 0 + naV + 0 ...(2)


From (1) and (2), we see that
F»n dS.
This verifies divergence theorem.
Ex. 49. ^F = + y - 4) i + 3zy j + (2zz + z^) k, evaluate

JOE (V X F)*n dS where S is the surface of the sphere


+ y^ + z^ = 16 above the xy-plane.
https://www.pdfnotes.co/

266 VECTOR CAIXULUS

Sol. The surface +>»^ + 2^ = 16 meets the plane z = 0 in a


circle Cgiven byjc^ +3^ = 16,z = 0.Let be the plane region bounded
by the circle C. If Sprima and 5^,then 5' is a closed surface. Let Vbe
the region bounded by S\
Ifn denotes the outward drawn(drawn outside the region V)unit
normal vector to S\ then on the plane surface ,we have n =- k.
Note that k is a unit vector normal to drawn into the region V.
Now by an application of Gauss divergence theorem,we have

E curlF»nd5 = 0 [See Ex. 1 page233]

or
E curlF*ndS + curlF»ndS =0

[●.* 5' consists of 5 and ]


or
E curlF*ndS
-E 1
curl F*kdS = 0

[v onSi,n=~k]

or
E curlF-ndS =
E curlF*kdS.

Now curl F = i j k
d d d
dx dz
x^+y-4 3jfy 2xz + z^
= Oi -zj + (3y - 1) k =-z j + (3y - 1) k.
.'. curl F*k = {- zj + (3y - 1) k}»k = 3y - 1.

JIs r2jt r4
= JI,
J6=0 Jr=0 (3rsiii0 - l)rdddry changing to polars

[Note that is a circle inj^r-plane with centre


origin and radius 4]
P4 _ r2n (*4
, ^
6=0 Jr=0
I sinedddr- Jd=0 I ^ rdddr
L ^ Jr=0
https://www.pdfnotes.co/

GREEN’S,GAUSS’S AND STORE’S THEOREMS 267

’2jt
=0- sinddd = 0
f0=O '0=0
0
r i2jt
=-8 6 =- 167T.
L Jo
Ex.50; IfF -yi +(x- 2xz)j- k,evaluate

JJ^ (V X F)*ndS where Sis the surface ofthe sphere


jr above thexy-plane, (Kanpur 1980)
Sol. The surfece +/+z^ =t meets the plane z =0 in a
circle C given by +>^ = ^ ^ q Le^
bounded by the circle C. If5' is the surface consisting of the surfaces
S and 5j,then 5'is a closed surface. Let Kbe the volume bounded by
S'.
Ifn denotes the outward drawn(drawn outside the region V)unit
normal veaor to S', then on the plane surface S^, we have n =- k.
Note that k is a unit vector normal to S^ drawn into the region V.
By Gauss divergence theorem,we have

jK (curlF)«nd5 =
= 0.
(div(curlF)ldK

(●.* div (curl F) = 0]

● JJ (curlF).n<iS + jU (curl F)»ndS = 0

[*.● 5'consists of 5 and ]


or
je (curlF)*nd5 (curlF)*kd5 = 0

[V on5i,n=-k]
or
JE (curlF)«nd5
Now curl F = i
1
(curl F)*krfS’.
J k
d d d
dx dz
y X - 2xz -xy
=i J
https://www.pdfnotes.co/

268 VECTOR CALCULUS

+ k l-(x-2xz)--^,(y)
dx dy
-xl+yj-2zk.
(curl F)*k = (jf I +3?j- 2z k)● k
=- 2z = 0 over the surface bounded by the
circle +)T z = 0.
0dS = 0.
Hence |J^ (curlF)*nd.i'= s1

Ex.51. Evaluate (V X A)»n where

A = {xy^ + log(z + 1)-sinx\ k and S is the surface of the sphere


+z^ = above thexy-plane.
Sol. Proceed as in solved example 50.
Here curl A = i j k
d d d
dx dy dz
0 0 xye^ + log(z + 1)- sinx
a r
=i xye^ + log(z + 1)- sinx
dy

~ ^ ic + 1)"*
= xc^i -(ye^ - cosx)j.
(curl A)^k =[xe^i — (ye^ - cosx)j]«k = 0.
.*. (curl A)*k = 0over the surface 5^.

Hence 0d5 = 0.
JJ (curlA).ndS = J][^
Ex.52. Evaluate (V X A)»ndSyWhere

A =(x -z)H-(x^ +yz)^ — 3xy^ k and S is the surface of the cone


z = 2- v^(x^ + y^)above thexy-plane. (Meerut W14)
Solution. Thesurfacez = 2- V(x^ +y^)meets thexy-plane in
a circle C given by x^ +y^ = 4, z = 0. Let Sj be the plane region
bounded by the circle C. If S' is the surface consisting of the surfaces
https://www.pdfnotes.co/

GREEN’S,GAUSS’S AND STORE’S THEOREMS 269

S and Sj, then S' is a closed surface. By application of divergence


theorem,we have

St curl A*nd5 = 0 [See Ex. 1 page233]


or
ft curlA«nd5 +
K curlA*nd5 = 0

or
ft curlA*n^5
■ft curl A»kdS [●.* on , n = - k]

Now curl A = i j k
d d d
dx dy dz
X -
2 x^+yz -3xy^

= i (~ 6xy-y) + j (- 1 + 3^) + k(3x^ - 0).


.% curl A*k = 3r^.

jE curlA*nd5
-JK,
a=o
JrTji r2 Changing to polars

=3 cos^0d0
4 0
= 12

t
ji/2
= 12x4
cos20d0
y = 48x^xf
Z Z =123t.
Ex. 53. Evaluate
I
F = (x^ + y - 4) i + 3xy j + (2xz
(V X F)»ne/5, w/icre

tP’)k and S is the surface of the


paraboloid z = 4 ~ (x^ +>'^) above thexy-plane.
Sol. The surface z = 4 - (x^ +y2) meets the plane z = 0 in a
drcleCgiven byx^ + = 4, z — Q. Let5j be the plane region bounded
by the circle C. If S' is the surface consisting of the surfaces S and S
then S' is a closed surface. Let Kbe the volume bounded by S'.
https://www.pdfnotes.co/

270 VECTOR CALCULUS

Ifn denotes the outward drawn(drawn outside the region K)unit


normal vector to 5', then on the plane surface 5^, we have n =- k.
Note that k is a unit vector normal to Si drawn into the region K
By Gauss divergence theorem,we have

JX- (curlF)*n<iS = [divcurlF]dK= 0,


since div curl F = 0.

(curlF)*nd5 + JJ^ (curl F)»n =0


[●/ 5' consists of S and ]

or (curlF)«nd5- (curlF)*kd5 = 0

[v on 5i,n=-k]

or
JJ^ (curlF)*n</5 = (curlF)*kd5.
Now curl F = I j k
d d d
dx dy dz
jt^+y-4 ^xy 2xz+2^
=i + ^(3xy) -j ^(2xz+z^)-^(J^+y-4)

= 0i-2zj + (3y-l)k.
/. (curl F)«k = [- 2z j + (3y - 1) k]«k
= 3y “ 1 over tlie surface bounded by the circle
jP- z:=^0.

Hence (curlF)*nrf5 = (3y-l)dS

=r r
jx=—2jy=
(4-x^
(3y - l)dxdy
V(4-x^
(-l)dxdy [: 3y is an odd function ofy]
https://www.pdfnotes.co/

GREEN’S,GAUSS’S AND STORE’S THEOREMS

.-,r M*-''dx
Jx==-2 J)»=0

V(4-JT^)dx
t2
71
=~4 |V(4-A:2)+ 2sin^|
=-4p--j =-<te.
0

Ex.54. Evaluate
j[j[ (a:^ + b/+ cz^)dS
over the sphere +y^ +z^ = I using the divergence theorem.
Solution. Let us first put the integral

f[ (ax^ + b/+ cz^)dS in the form


& E*ndS,
where n is unit normal vector to S.
The normal vector to <p {x,y,z)sjr^ +^2 ^2 _ j _ q
= V0 = 2jri + 2)^j + 22k.
n= _ 2xi + 2yjH-2zk
|V^I V[4(j^+y^ +z^)]
^xl+yj+zk [V x^+y^ +z^= l,on5]
Now we are to choose F such that
F*n =^¥^(xi+yj+zk)=ax^ + b/+ c?.
Obviously F = njc i + byj+ cz k.

ff.
Now (ax^ + by^ + C2^)dS

-jE F»ndS,where F = njc i + byj+ czk

=jtt div FdV, by divergence theorem

-fi (a+b + c)dV [V diV F = n + h+c]

<fK=(n + h + c)K
=(a+ b+c)|jT,since the volume ^enclosed by the sphereSof
unit radius is|jt.
https://www.pdfnotes.co/

272 VECTOR CALCULUS

Ex.55. Compute
1/2
(i) JJ (aV + ftV +oV) dS.and

(ii) f£ (<,V +6V + cV)-‘^dS


over the ellipsoid a:? + + cz^ = 1.
Sol.(i). Let us first put the integral
(flV + hV + dS in the form

s.
where n is a unit normal vector to the closed surface S whose equa
tion is a)? + b^ + cz^ = 1.
The normal vector to^(x,y,z) + by^ + cz^ - 1 = Ois
= V(p- lax 1 + Ibyi + 2cz k.
V4> laxi + lbyj+ lczk axi + by^+czk
"-IWT“'/(4aV +46V +4c^2:*)
Now we are to choose Fsuch that F*n = + b^y + c z ).
'vObviously F = flz1 + byi + cz k.
Now (a^:^ + b^/+ c^z^)^^^dS

F*n dS,where F = or i + hyj + czk

divFdK by divergence theorem; Kis the volume


enclosed by the closed surface S

dV=(a + b+c)V
4 (1 1 l\ 4ji(a+ b + c)
= 's'"* (Vfl'Vb'Vcj ~ 3 V(abc)
can
Note that the equation of the ellipsoid oj^ + by^ + cz^ = 1
be written as + Jll. 4. J— - 1 and the volume of the ellipsoid
1/a 1/b 1/c

= 1 is abc.
https://www.pdfnotes.co/

vjKEEN’S,GAUSS’S AND STORE’S THEOREMS 273

(li) Proceed as in part(i)of this question.


Here we are to choose F such that
F*n = + c^z^)on S,
Obviously F=jci+yj+2k,beca.use then
F*n = gy?'+ + cz^ 1
on 5.
V + bV + V(flV + bV +
Note that on 5, + by^ + cz^ = 1.
L
Now
ffs + bV +
dS

F«nd5,whereF=xi+yj+zk

(V ●F)dV, by divergence theorem; Vis the volume


enclosed by the surface S

hi
jt ● J_.JL.J_ =
4jt
d 3dV

= 3F=3-| Va >lb yfc V{abc)


Ex. 56. Evaluate
ffs (^+^) dS, where S is the surface ofthe
cone z^ = 3 (x^ + y^) bounded byz = 0 andz = 3.
Sol. Let S be the surface of the cone z^ = 3 (x^ + y^) bounded
by the planes z = 0 and z = 3. The plane z = 3 cuts the surface
z^ = 3 (x^ + y^) in the circle x^ + y^ = 3, z = 3. Let be the plane
region bounded by this circle. Let S' be the closed surface consisting of
the surfaces S and .
Let us first put the integral

Jfs in the form

£ F»n dSs
where n is a unit vector along the outward drawn normal io the surface
S whose equation is<p (x,y,z) s 3 (x^ +y^) - z^ = 0.
We have n = ^<p 6xi + 6yj-2zk
https://www.pdfnotes.co/

274 VECTOR CALCULUS

3jci + 3yj-zk _ 3jci 4-3yj-zk


V[9(jc^+y2)+z^] \^(3z^ +z^)
since on 5, 3(x^ +y^)=z^
3xi + 3yj ~zk
2

Now take F =^(x i + yj). Then on 5, F*n = + y\


By Gauss divergence theorem,we have

S. , ¥*ndS ■E divFdK,
where Vis the volume enclosed by the closed surface 5'.
...(1)

(2 ' 2 '
We have div F = div -zx i + -zyj
3 y
d (2 \ ^ d I2 ^ 2^2 4
“ at ay “3^ 3^" 3^-

JE divFdK = JE ^z dK, where Vis the volume


bounded by z = 0, z = 3 andx^ +y^ =
V{(z^/3)-y^}
4 p r zdzdydx
3 Jz=0 Jy=-z/V3 Jx =-V{(zV3)V}
V{(z^/3)-y^}
1.2 r T' r
3 ●/z=0 */y=—z/'l3 Jx = 0
zdzdydx

=-f r '2/^3

3 J2=0 Jy = -z/yf3
nv{(zV3)-/)
z X
L J x=0
dzdy

8 p
3 Jz=aJy=

dzdy
-IZ/V3
2^ . -1 f 3- d!z
6®” [2/^3 j Jy=o

\
fr- ●sin ^ 1 d!z = M.i.fL
3 6 2 Jo
https://www.pdfnotes.co/

GREEN’S,GAUSS’S AND STORE’S THEOREMS 275

r
^ 1 £L _ On.
3*6*2 4 3 *6*2* 4 ...(2)

Also
If, F»ndS +
£ 1
F»n dS

[v S' consists of5and 5^]

ffs f (Jcl+yj).kd5,

since on 5i, n = k, z = 3

-ffs dS +0
From (1),(2)and (3),we have
=J0C ...(3)

dS^9jt.

Ex.57. Prove that

f,n^rjy.[ Fxf»dS + X F*cur/fdV.

Sol. We have
X Fxf*i/S
-X (F X f)*n dS,
where n is a unit normal vector to the surface S

= [V*(F X f)]dV,by Gauss divergence theorem

=X ^
X fcurlFrfF-
X F«curl fdV.

A Xf*curlF<fK =x F X f*rfS +
X F*curl TdK

Ex. 58. Gauss’s theorem. Let S be a closed surface and let r


denote the position vector ofany point(jc,>',z) measuredfrom an origin
a Then

E? n^5

is equal to (i)zero ifO lies outside S;(ii) An ifO lies inside S.


Proof, (i) When origin O is outside S. In this case F = -^ is
r
continuously differentiable throughout the region V enclosed by S.
Hence by divergence theorem,we have
https://www.pdfnotes.co/

276 VECTOR CALCULUS

/-\
r
i£ div -r dV=0y since div — = 0.
r
(ii) When origin O is inside S.
In this case divergence theorem
cannot be applied to the region V
enclosed by 5 since F has a / S

point of discontinuity at the ori


gin.To remove this difficulty let us
enclose the origin by a small \
sphere 2ofradius e.
The function F is continu-
- ously differentiable at the points
of the region V enclosed between
S and 2.Therefore applying divergence theorem for this region F,we
have

Ns 7*"'^
ir
div — dW -0,since div = 0.
\ y

ffs te
Now on the sphere 2» the outward arawii normal n is directed
towards the centre. Therefore on 2» we have
r
n =— e

-JL? ●ndS-
-fL 7* r V
-7 d2»smceon2,/*= e

d2~ = 4jt.

Hence
£? ●nd5 = 4jr.

§9. Stoke*s Theorem. Let S be a piecewise smooth open surface


bounded by a piecewise smooth simple closed curve C. Let F (jc»y, z) be
a continuous vectorjunction which fuis continuousfirstpartial derivatives
in a re^n ofspace which contains Sin its. interior. Then
https://www.pdfnotes.co/

GREEN’S,GAUSS’S AND STORE’S THEOREMS 277

(curlF)»dS
where C is traversed in the positive direction. The direction ofC is called
positive ifan observer, walking on the boundary ofSin thisdirection, with
his headpointing in the direction ofoutward drawn normal n to S,has the
surface on the left.
(Meerut 85;Rohilkhand 90; Osmania 89;Kakatiya 90,92;
Tinipati89,93)
Note. T*dr =
'C UF-^]<fa =
t i ds) ^ (F*t) where t is unit
tangent vector to C. Therefore F«t is the component of F in the
direction of the tangent vector ofC.Also(V x F)*n is the component
of curl F in the direction of outward drawn normal vector n of S.
Therefore in words Stoke’s theorem may be stated as follows:
The line mteg^al of the tangential component of vector F taken
around a simple closed curve C is equal to the surface integral of the
normal component ofthe curl o/F taken over any surface S having C as
its boundary.
Cartesian equivalent of Stokers theorem.
Let F = Fji + ^2j+ F3 k. Let outward drawn normal vector n
ofS make angles a, y with positive directions ofjc,y,z axes.
Then n = cos a i + cos/8j+ cos y k.
Also V X F = i j k
d d d‘

ar dy dz
F, ^2 ^3
'aFg df2 dF.\
i+ k.
az dz ar I ** ar
\ ^j
'aFg dF2] cosa +(dF, dF,\
(V X F)»n = ‘
dz dz dx^ cos/S
■>

dF,]
+ -7^- cosy.
dx dy
Also F»rfr = (Fj i + F2 j + F^ky(dxi + (fyj + dz k)
= F^dx + F2(fy + F^dz.
.’. Stoke’s theorem can be written as
https://www.pdfnotes.co/
278 VECTOR CALCULUS

VdFg dp2
cos a
^Fi dr + F2^v + Fgdz = J[j^ Ll^ dz
(dF2 dF,\
cos/3 + - cosy dS.
dz ar ex
\ \
Proof of Stoke*s theorem. Let5 be a surface which is such that
its projections on the;i^,>z andzr planes are regions bounded by simple
closed curves. Suppose S can be represented simultaneously in the
forms
2 =/(Jc>y)»y = g(x,z), x = h (z,y)
where/,g,h are continuous functions and have continuous first partial
derivatives.

Consider the integral


WehaveVx(Fil)=
JK
i
[Vx(Fii)]«ndS.
j k
d d d
dx dy dz
Fi 0 0
dFj dFj
k.
dz j- dy
ZA
I n
Y
S'
Ids

<■
X
R

r
https://www.pdfnotes.co/

GREEN’S,GAUSS’S AND STORE’S THEOREMS 279

{bF. . dF. dF1 dF1


[Vx(Fii)].n = dz j*n—r^k*n dz cos/S- cos y.
/
fdF dF
f£ [V X (Fj i)J*n<i5 -jE dz-cos/5- -cos y dS.

We shall prove that

d:
Let R
aFj
dz cos/S -
dFi
cosy dS =
J,Fidr.
the orthogonal projeiction of5 on thejty-plane and let
r be its boundary which is oriented as shown in the figure. Using the
representationz =/(jc,y)of5,we may write the line integral over C as
a line integral over T.Thus

Fj{x,y,z)dx = £f^ ^,y,f(x,y)]dx

=^ {Fj[x,y,/(x,y)]dx-hO^}
dF
-I R dy
-dx^, by Green’s theorem in plane
for the region R.
Bm
dy dz dy
[V z=f(x,y)\

'dFi ^ dF^ Of' dxdy ...(1)


^ dz dy
Now the equationz =/(jr,y)of the surface S can be written as
<p(x,y,z)sz-f(x,y)= 0.
We have grad 0 =- i- + k.
ax dy
Let Jgrad^l =/I.
Since grad 4> is normal to 5,therefore, we get

a
But the components of both n and grad (f> in positive direction of
z-axis are positive. Therefore
n _ I grad^
a
https://www.pdfnotes.co/

280 VECTOR CALCULUS

or cosai + cos/8j + cosy k=- a dx


a dy a
1
cos a = iif. cos/S =-ii£. cosy = -●
a dx a dy
dxdy
Now dS = -adxdy.
cosy

a \
dz cos/S -
dF
^ cosy dS

_ ^^1 1
— adxdy
dz [ ady^ dy a

=_ Jjr
ff + dxdy. ...(2)
\ dy dz dy /
From (1) and (2), we get
'dFj dF
dz cosjS- dy-cosy dS

= ffs [Vx(fi ●)]●"<« ...(3)


Similarly, by projections on the other coordinate planes, we get
...(4)

£^3^ "ffs lVX(F3k)]*“‘'S ...(5)


Adding (3), (4), (5), we get

£ (Fidx+F2dy+Fsdz)=J£ [V X (Fj i + F2j+F3k)]»nd5


or
JOE (V X F)»ndS.
If the surfoce S does not satisfy the restrictions imposed above,
even then Stoke’s theorem will be true provided 5 can be subdivided
into surfaces Sj, 52, with boundaries Cj, C2,...» which do
satisfy the restrictions. Store’s theorem holds for each such surface.
The sum of surface integrals over 5^,52,... ,5^ will give us surface
integral over 5 while the sum of the integrals over.Cj, C2,..., Q will
give us line integral over C.
https://www.pdfnotes.co/

GREEN’S,GAUSS’S AND STOKE’S THEOREMS 281

Note. Green’s theorem in plane is a special case of Stoke’s


theorem. If is a region in the j»y-plane bounded by a closed curve C,
then in vector form Green’s theorem in plane can be written as

(^x P)*kdR =^ F*dr.


This is nothing but a special case ofStoke’s theorem because here
k = n = outward drawn unit normal to the surface of region R.
Solved Examples
Ex. 1. Froye that
£
Sol. By Stoke’s theorem
r»dr = 0.
(Andhra 1989)

(curl r)*n = 0,since curl r = 0.

Ex. 2. Prove that ●dr.


^ ipWiffdr = — ^
Sol. By Stoke’s theorem, we have

£^(H>)‘dr = f£ [curl grad (0V>)]*n</5


= 0, since curl grad (^) = 0.
But V (^) = + Y’V^.

^ (0Vy; + V'V^)«dr = 0
Ot
^ ^V^*dr = — ^ ‘ipV<j>*dr.

Ex. 3. (a) Prove that ^ (pVtp^dv —


Sql. By Stoke’s theoreni, we have
f£ [V^ X V^]*ndS.

[V X (^Vv»)]»ndS

-£ [V^ X Vy» + ^ curl grad t^]*n dS

= JJ^ [V 0 X Vy;]«n dS, since curl grad = 0.


Ex. 3(b) Show that *>dr = 0, C being a closed curve.
Sol. Applying Stoke’s theorem to the vector function <pV(p, we
have

[curl(^V0)]»ndS
https://www.pdfnotes.co/

282 VECTOR CALCULUS

-If. [0 curl V0 + X V0]*ndS

= rrJJs
= 0.
0**ndS [●.● curl = 0 and x = 0]

Ex. 4. Prove that dS X V0. (Kanpur 1977)


Sol. Let A be any arbitrary constant vector. Let F = ^A. Apply
ing Stoke’s theorem for F, we get

CF-dr=ff (Vx(0A)]*nrf5 -If. [V^ X A + 0 curl A]» dS

If. (V^ X A)»dS, since curl A = 0.

.●.£(M)** = JX A*(dSxV^)
or A* rfSxV0 =0.
Since A is an arbitrary vector, therefore we must have
= dsxv^.
Ex. 5. ^ Stoke’s theorem prove that div curl F = 0.
Sol. Let V be any volume enclosed by a closed surface. Then by
divergence theorem

M V«(curlF)dK

/X (curlF)*n^f5.
Divide the surface S into two portions
5j and S2 by a closed curve C. Then

(curlF)*nd5

-XX, (curlF)*n^/5i

...(1)
^ JX F)*nrf52.
By Stoke’s theorem right hand side of (1) is
F»dr = 0.
^ F*<ir - ^
https://www.pdfnotes.co/

GREEN’S,GAUSS’S AND STORE’S THEOREMS 283

Negative sign has been taken in the second integral because the
positive directions about the boundaries of the two surfaces are oppo¬
site.

M V«(curlF)dK=0.
Now this equation is true for all volume elements K Therefore
we have V ●(curl F)=0
or divcurlF = 0.
Ex.6. Stake’s theorem prove that curlffrad ^ = 0.
Solution. LetS be any surface enclosed by a simple closed curve
C.Then by Stokers theorem,we have

JJ^ (curl grad ^)»ndS =^ grad


Now grad = j+ k ●(dxi + {fyj +dzk)
dr
dz J
dx + d±
dy-¥ dz dz — dtp.
dr dy

<p ^ »where >4 is any point on C


= 0.
Therefore we have (curl grad ^)»ndS = 0.
Now this equation is true for all surface elements S.
Therefore we have, curl grad ^ = 0.
Ex.7. (a). Veri^ Stoke’stheorem/orF=yi+zj+jrkwhereS
is the upper half surface of the sphere +y^ + z^ = 1 and C is its
boundary. (Meerut 1981; Rohilkhand 91; Agra 70; Andhra 92)
i
Sol. The boundary C of 5 is a circle in the;i^-plane of radius unity
and centre origin. The equations of the curve C are +
:^ = i,
z = 0. Suppose X = cos r, y = sin r, z = 0, 0 < r < 2n: are«pararnetric
equation of C. Then
F»dr =
c J>^ (yh+ zj + xk)»(dri + cfyj+ dzk)

-i(ydx + zdy +xdz) = ^ydr, since on C, z = Oanddz = 0


=J[
2jc
dx
smt—dt
dt =j: 2ji
- sin^ tdt
https://www.pdfnotes.co/

284 VECTOR CALCULUS

2st
1 C'^ 1 sin2r
t-
=~2jo (l-“s2r)d(=-2 2 Jo
~-ji. ...(1)
Now let us evaluate curl F»n d5. We have curl F
&
=VxF= i j k =-i-j -k.
AAA
ar dy dz
y z X
If is the plane region bounded by the circle C, then by an
application of divergence theorem,we have

Jtt curlF»nrfS =
JJ^ curlF»kd5 [SeeEx.50 page267]

-JJ (-l-l-B-kdS.JJ (-■).!-JI, dS = -5i.


But 5j = area of a circle of radius 1 = (1)^ = jr.

s. curlF»n^fS =-;r.

Hence from (1) and (2), the theorem is verified.


...(2)

Ex. 7 (b). Ven^ Stake’s theorem for


F = (jc^ +.y - 4) i + 3xy j + (2xz'+ z^) k
where S is the upper half of the sphere +y^ + z^ = 16 ^nd C is its
boundary. (Osmania 1991)
Sol. The boundary C of S is the circle jc^ M- = 16, z = 0 lying
in the j«y-plane. Suppose z = 4 cos r, y = 4 sin r, Z= 0,0 < r < 27t are
parametric equations of C. Then

^ F»rfr = ^ {(z^+y - 4) 1 + 3zyj


+ (2zz + z^ k}» {dxi + j+ k)

=^ (z^ + y - 4) dx+ 3xydy + (2zz +z^)dz


> since on C, z = 0 and dz = 0

dx
^ dt
dt + 3zy dt
https://www.pdfnotes.co/

GREEN’S,GAUSS’S AND STORE’S THEOREMS 285

r
“Jo [(16cos^r+ 4sin/-4)(-4sin/)
+ 3● 16 sin / cost.4 cos t dt
2ji 2ji
' =128 cos^rsinrrfr -16
ji/2
£ six^'tdt + 16
£ sin tdt

= 128.0- 16.4 'sin^/dt + 16.0 =-64


2a
Note that
£ /(x)dr = 0, if/(2a-x)=-/(x)

and = 2
^^f{x)dxAffCla- =m
Now let us evaluate
JJ^ (curl F)»n dS. We have curl F
= V X F= i j k
d d d
dx dy dz
j^+y-4 3xy 2xz + z^
= Oi-2j+(3y - 1)k =-2j+(3y - 1)k.
If 5j is the plane region bounded by the circle C, then by an
application of Gauss divergence theorem,we have

JJ^ curlF»nrf5 =JI 1


curlF»krf5 [See Ex.50 page 267]

-ffs, (-^J^(3y-i)k}-kds= f£^ (3y-l)dS


Jr2ji r4 (3rsin6 — VjrdBdr^ changing to polars
0=0 Jr=i
[Note that is a circle in j^^-plane
with centre origin and radius 4]
=- 167T.
...(2)
[SeeEx.49pag265]
Hence from(1)and (2),Stoke’s theorem is verified.
Ex. 8. Verify Stokers theorem for F =(2x -y)i -yz^j-y^z k,
where S is the upper halfsurface ofthe sphere + y^ +^=landCis
its boundary.
(Agra 1960; Rohilkhand 78; Allahabad 78;Kanpur 70;
Osmania 89,91)
https://www.pdfnotes.co/

286 VECTOR CALCULUS

Sol. The boundary Cof5is a circle in thej»y-plane ofradius unity


and centre origin. Suppose jc = cos r, y = sin r, 2 = 0,0 :S r < 27t are
parametric equations of C.Then

^ F*dr =^[(2x -y)i -y2^j -y^z k]« (dxl +(fyl+dzk)

= ^^[(2x-y)dx-y:^tfy-/zdz]

=^(2x -y)dXf sincez =0anddz =0


●2w
(2cost - Sint)Sintdt
= J^^(2cost sint)§dt=-
n27T
cos 2r 1 1 sin 2r
=-J^^[sin2r-i(l-cos2r)]rfr=~ 2 2 2 2 JO

=-[(“ i + 5)- i ”0)+ i(0- 0)]=;r. ...(1)

And(V X F)= i j k
d d d
dx dy dz
2x-y
=: (- 2y z + 2y z) i - (0 - 0) j + (0 + 1) k = k.
Let Sj be the plane region bounded by the circle C. If S' is the
surface consisting of the surfaces S and , then S' is a closed surface.
.●. by an application of Gauss divergence theorem, we have

jE , curlF*nrf5 = 0 [See Ex. 1 page 233 ]

or
E curlF»ndS +
J[jj^ curlF*nd5 = 0
[-.* S' consists of S and S^ ]
or
JE curlF«ndS
-a, curlF»krf5 = 0

[V onSi, n=-k]
or
£ curlF«nrf5 = JJ^
curl F»kdS.

■If. curlF®iii/5
.Jf »,ir.|.JS
https://www.pdfnotes.co/

GREEN’S,GAUSS’S AND STORE’S THEOREMS 287

-JE.
Note that
1
dS = Sj= ji.

= area of a circle of radius 1


...(2)

Hence from (1)and(2)Stoke’s theorem is verified.


Ex.9. Verify Stokers theoremfor thefunction
F = zi +jcj +yk
where curve is the unit circle in the xy-plane bounding the hemisphere
z — V(1 (Rohllkhand 1981; Kanpur 78;Agra 75)
Sol. Here the surface S is the surface of the sphere +>^^ +
= 1 lying above thejy-plane.The curveCis the boundary ofthe surface
iS and is a circle in thej^-plane of radius 1 and centre origin.
The equations of the curve C are 0. Suppose
z = cos r, y = sin z = 0,0 ^ r < Z>r are parametric equations of C.
Then

(zl +zj +yk)*(dri + + d!zk)

= £(zdx-hxtfy+ydz)=£ xdy,since on Cy z = 0andd!z =0


*2jt

= Jo
●231
cosr dt
dt
■r cost .cost dt
-i2ji
1
2 Jo (l + cos2r)rfr = i t + ^^
Z0 = |-2* = Jl. ...(1)
0

Now let us evaluate

We have curl F = V x F =
E (caT\F)*ndS.

i j k
± ± ±
dx ^ dz
z X y
=i

+k
=:i+j + k.
Let Sj be the plane region bounded by the circle C. If S' is the
surface consisting of the surfaces S and Sj, then S' is a closed surface.
https://www.pdfnotes.co/

288 VECTOR CALCULUS

By Gauss divergence theorem,


where K is the
jjs = JITk (div curl F)volume
dV,
enclosed by S'
~ 0,since div curl F = 0.

■ jj. (curlF)»nd5 + (curl F)*ndS = 0

[●.* S' consists of S and ]

or
ffs (curl F)*krfS' = 0

[v on5j,n=-k]

or
(curl F)*ndS - JJ^ (curl F)*kdS.

J]s (c'»rlF)*nrf5 = (curlF)»kd5

= iC. + dS

=0
= Sj, where S^ is the area of the circle jc^ + = 1, z
= ;r.l^ = JT. ... (2)
From (1) and (2), we see that

^F»dr- Jj^ (curl F) *n dS.


This verifies Stoke’s theorem.
Ex. 10. Stake's theorem /or A = 2y i + 3z j - z^ k, where S
is the upper half surface of the sphere jp" + y^ + z^ = 9 and C is its
boundary. (Meerut 1975)

Sol. Proceed as in solved example 9.


Here the parametric equations of the circle are x = 3 cos r,
y = 3 sin r, z = 0, 0 < r < 2jt.

We have I A .dr = ^ (2y i + 3x j - z^k) ● (^ i + ^ j + ^ k)

(2ydx + 3xdy- z^dz) {2ydx + 3xdy),


since on C, z = 0 and dz = 0
https://www.pdfnotes.co/
GREEN’S,GAUSS'S AND STORE’S THEOREMS 289

&
sin / ●(— 3sinr)
dt)
+ 9cos/ -(3cos/)]<//

= Jo t -IS sin^ t)dt

X
jc/2
fn/2
= 27.4 f cos^tdt-lSA sin^ tdt

= 108-i-f-72-i-| = 9;..
Again here curl A = i j ^
d d d
etc dy dz
2y 3jc -2^

=i

+k
= 0i~0j + k = k.
(curlA)*nrf5 = (curlA)»krf5

£ 1
k*kdS =
If, 1
dS = S i» where 5j is the area

of the circle jc^ + = 9, z = 0


= jt.3^ = 9jt, since radius of the circle is 3.
We see that , A*dr =
JC
This verifies Stoke’s theorem.
XX
(curl A) *11^5.

Ex. 11. Veri^ Stoke’s theorem for the vector


F = zi+jcj+>'k tal^n over the half of the sphere +y^ + P ~a^
lying above the xy-plane. (Gauhati 1973)
Sol. Here let 5 be the surface of the sphere +y^ + z^ =
lying above thej^^-plane and let the curve C be the boundary of this
surface. Obviously the curve C is a circle in thej«y-plane of radius a and
centre origin and its equations are jc^ +y^ = z = 0. Suppose
X
= a cos /, y = « sin /, z = 0,0 < / < 27t are parametric equations of
C.
https://www.pdfnotes.co/
290 VECTOR CALCULUS

By Sioke’s theorem,we have

X "" J[Js ...(1)


Let us verify (1).

We have ^ F»dr = (z i + x}+ yk)*{dx I + dy} + dzk)

=^{z dx +xdy +y dz)= Xdyy since on C,z=0 and diz =0

-X
2ji 2jt
a cost
dt
dt
■t a cost .a cost dt - a
i2ji
2r
= «!f
2 Jo
2t

(1 + cos 2/)dt .= ^
r + sin 2/
2 Jo
o — jia2.
— zji
2
...(2)
Now let us find
JJ^ (curlF)«nrf5.
We have curl F= i j k=i+j + k.
± ± ±
dx dy dz
z X y
If n is a unit vector along outward drawn normal at any point
(x,y,z) on the surface 5 i.e., the surface <f> (;c,y, z) s jc^ = a\
then
2jci + 2yj + 2zk xi +yj +zk » .
n
a
1^01 + 4y^ + 4z^)
since on 5, + z^ = a^,

XX (curl F)*nd5 -& (i+j + k) -


/xi+yj+zk^
a .
dS

= «JI
To evaluate it we shalfUse-polar spherical coordinates (r, 0, <p).
Wehavez =: rcos0, X = rsin0cos0, y =s rsin0sin0.
Here r — a. x = a sin d cos <f>yy — sin 6 sin (pyZ — ct cos 6.
Also dS = an elementary area on the surface of the sphere at the
point («, 0, <p) = a dd. a sin 6 d<p = a^ sin 6 dO d<p.
(cUrlF)-n<iS
https://www.pdfnotes.co/
GREEN’S,GAUSS’S AND STORE’S THEOREMS 291

2ji

a Je=o J4>=o(a sin d cos <p +a sin 0sin0+a cos 6) sin d d9d<f>
2ji
=a ,J0=o
p/2J^=a(sin^6cos <p + sin^9sin 0,+ cos0sin 9)d9d<p
Jl/2 r
= a2
-i2jr

w=o sin^0sin 0- sin^0cos^ ^ cos0sin 0 d9


J^=0
*71/2
'= a2
'0=0 2jt COS0sin 9d9- i = jta^. ...(3)
From (2)and (3), we see that

^F*rfr = (curlF)*nrf5.
This verifies Stoke’s theorem.
Ex. 12. Verify Stake’s theoremfor the vector
A = 3yi-xzj+yz^k, where S is the surface of the paraboloid
2z = + y^ bounded byz = 2and Cis its boundary.(Meerut 1973,77)
Sol. The boundary C of the surface S is the circle in the plane
2 = 2 whose equations arex^ = 4, z = 2. The radius of this circle
is 2 and centre (0,0,2). Suppose x = 2cosr, y = 2sinr, z = 2,
0 < / < 27t are parametric equations of C. By Stoke’s theorem

(curl A)*n dS, where n is a unit vector along


outward drawn normal to the surface S.
We have
^ A»dr = i (3y i -xzj+ yz^ k)*(dx i +dyj+dz k)

=^(3ydx-xzdy +yz^dz)

=^(3ydx-2x dy),since on C,z = 2and dz = 0


dx -
dt
2jt
dt)
[Note that here the surface S lies below the curve C and so direction
of C is positive if C is traversed in clockwise sense]
2jt

-L [3-2sin / ● (- 2sin/)- 2-2cos / ● 2cos/]rf/

Q [-12sin^/- 8cos^/]i//
Jf*2jc
https://www.pdfnotes.co/
292 VECTOR CALCULUS

ji/2 :t/2
=4
X sin^ tdt + 8
f. cos^ tdt

=4 124-?+
22
84-?
22 = 44*20
4 = 20;r. ...(1)

Let be the plane region bounded by the circle C. If S' is the


surface consisting of the surfaces S and ,then S'is a closed surface.
Let Fbe the volume bounded by S'.
By Gauss divergence theorem,we have

E (curlA)«nrfS = M divcurlAdK
0, since div curl A = 0.
(curl A)*nrfS + (curl A)*n =0

[v S' consists of5and ]


or (curl A)*ndS=- (curl A)»n dS

Jfjl^ (curlA)-k</S.
[v on^i, n = k]
Now curl A = I j k
± ± ^
dz
dx ^
3y -xz yz2
=i A(y,2)_A(_„)l_jrA(,,2)_±(3y)
+k
dx
=(z^+jc)i-(z + 3)k.

(curlA)«nrfS=- [(z^+x)i -(z 3)k]»krf5

=il, + 5 dS, since on


= 55i» where 5^ is the area of a circle ofradius 2
z=2

= 5.Jt.2^ = 20jt. ...(2)


https://www.pdfnotes.co/
GREEN’S,GAUSS’S AND STOKE’S THEOREMS 293

From (1)and we see that

<f> A*dr = IL (curl A)*n dS.


This verifies Stoke’s theorem.
Ex. 13. Ven)^ Stoke’s theoremfor

taken round the rectangle bounded by


x = ±a,y^0,y-b. (Meerut 1967)
Sol. We have

E B

^f
x = -ii
S * ^x = a

O
D y=0

curl F = i j k
d a a
dx dy dz
x^+y^ -2xy. 0
-{—2y — 2y)k =- 4yk.
Also n = k.

(“ AyV)*Vidxdy

N\r
●fy-0Jx=—a
ydxtfy = xy
lo
tfy

=- 4ab^.
https://www.pdfnotes.co/
294 VECTOR CALCULUS

Also
£,F*</r = £.[(x^+/)i- 2xyj]»(dx I + dyj)

=^ +/)<&-29’4’1
= [(-"^+)^)*- + ^AB'*'Jbe'^Jed'
Along DA, y = 0 and dy = 0. Along>4B, jc = « and dx = 0.
AlongBE, y = b anddy=^0. Along£D, x =- a anddx = 0.

f
Jc F.dr= Jx=-a
r ^dx+ Jy=0
f\- laydy
*£z laydy

= J-a
2 \A“=-
="X/*~‘^X^‘^=-2ab^-4a ^
/Jo
Thus
^ F»dr = (curl F)-n dS.
- Hence the theorem is verified.
Ex. 14. Verify Stake’s theoremfor the Junction
F=j/i+j«yj
integrated along the rectangle, in the plane z = 0, whose sides are along
the lines X ==0, y-0, x-a andy - b. (Meerut 1976)
Sol. We have
curl F = i j k
a ^
dx ^ dz
j/ JC); 0

D B
(0.6) (a,b)
S 4 i

O A(a,0)
(0,0)
https://www.pdfnotes.co/
GREEN’S,GAUSS’S AND STORE’S THEOREMS 295

= 0i-0j+
= yk.
The closed curve C is the boundary of the rectangle OABD
traversed in anti-clockwise sense. The surface S bounded by C is the
area of rectangle OABD.
Also n = unit normal vector to 5 = k.
By Stoke’s theorem,we have

^F»dr = (curl F)● n dS.


Let us verify it.
We have

= r„rM=r Kf
jy=0jx=0'^ ^ Jjc=0 2 2 Jo
dx

ab^
2 ...(1)
Also + xyj)*(dxi + dyj)

= £<^dic+xyify)
+ xydy)
+
Xo +xydy)+ £^(x^dx +xydy).
Along OAy y = 0and rfy = 0, jc varies from 0 to n;
along^, X = , dr = 0, y varies from 0 to b;
along BDy y = b, tfy = 0, jc varies from to 0;
and along DO,.r = 0, dr = 0, y varies from b to 0.

pr + a r?
aydy +
lO
r-'-r Qdy

3
L-'JO 0
_ ab^ a
"3 2 3 ■" 2 ...(2)
From (1) and (2), we see that
https://www.pdfnotes.co/
296 VECTOR CALCULUS

^ F*dr = (curl F)*n dS.


This verifies Stoke’s theorem.
Ex. 15. Verify Stake's theoremfor thefunction
F=jr^i+j^j,
integrated round the square, in the planez = 0,whose sides are along the ‘
linesx -0, y-0, x = a, y == a. (Bombay 1970)
Sol. Proceed as in solved example 14.
Show that F*rfr = = (cvLi\¥)»ndS.
Ex. 16. Verify Stake's theorem for a vector field defined by
F= i + 2xyjin the rectangular region in thexy-plane bounded
by the linesx = 0, x = n,j/ = Qandy = b. (Kanpur 1975)
Sol. Proceed as in solved example 14.
Ex.17. Verify Stake's theoremfor thepnction
¥{x,y,z)^xyi+xy^}
integrated round the square with vertices(1,0,0), (1,1,0), (0,1,0)and
(0,0,0), where i and\ are unit vectors alongx-axis andy-axis respectively.
(Meerut 1979)
Sol. Proceed as in solved example 14.
We observe that thez-coordinate of each vertex of the square is
zero. Therefore the square lies in the Jty-plane. Its vertices in the
j«y-plane are the points (0,0), (1,0), (1,1) and (0,1). Also here
n = k.

Ex. 18. Verify Stake's theorem /or F =-yh + ,where S is the


circular disc x^ < 1, z = 0.
Sol. The boundary Cof5is a circle inj^-plane of radius one and
centre at origin.
Suppose X = cos r, y = sin r, z = 0, 0 < / < 2;r are parametric
equations of C.Then

c F*^/r =^(-/i+x^j)*(d&ci + dKj+^fek)


(f
dt

= [-sin^r(-sinr)+ cos^/(cosr)]i/r
https://www.pdfnotes.co/
GREEN’S,GAUSS’S AND STORE’S THEOREMS 297

Jt/2
-■ (cos'* t + sin'* t)d/ = 4 (cos^/ + sin'*?)rf/
3.1 Jt 2nji
_4|M£E^. ™ ” I _ —.
4.2 2^4.2 2 2
Also V X F = i
^ j k = (3jc^ + 3^2) k.
w d
etc ^ ~Sz
0

Here n = k because the surface S is the;<y-plane.


(V X F)*n = (3r^ + 3/) k-k = 3 (3? +/y

..f£ (VxF).ndS = 3j[j[ (,^+/)dS

“^X=o Jr^o'^>'‘‘^dr,ch3ng^g to polars

Thus
£^'“^=ffs
Hence the theorem is verified.
(VxF)»nrf5 = ~-

Ex. 19. Evaluate by Stake's theorem

^ (e^dx + 2ydy - dz)


where C is the curve + >’^ = 4, z = 2. (Meerut 1969; Agra 72)
Sol
. ^(e^dx + 2yify-dz)

§c “k)*(dri +dyj +dzk)

=
F*</r, where F = ^ j - k.
Now curl F = i j k = Oi + Oj + Ok = 0.
a a d
dx Dy dz
2y -1
.’. By Stoke’s theorem

(curl F)«n c/5


https://www.pdfnotes.co/
298 VECTOR CALCULUS

= 0, since curl F = 0.
Ex.20. Evaluate by Stoke's theorem

^ iyzdx-^xzdy -^-xydz)
where C is the curve jp" +y^ = U z=^ y^. (Andhra 1989,Kanpur 80)
Sol. Here F = yz I + jczj +jcy k.
CurlF= i j k
A A ±
dx dy dz
yz JCZ xy
=(X -X)i -(y -y)j+(z -z)k = 0.
By Stoke’s theorem

^ F*dr = (curl F)*ndS


= 0,since curl F = 0.
Ex. 21. Evaluate (xydx+ xy^dy) by Stoke’s theorem where
Cis the positively oriented square with vertices(1,0),(- 1,0),(0,1)and
(0,-1).
F*i/r, where
Sol. Wehave^ {xydx +xy^dy)- 1C

F = xy i + xy^jand r = x i + y j.
By Stoke’s theorem,

^ F*i/r = (curl F)»n dS,


where 5 is any surface bounded by the square C and n is unit normal
vector to the surface 5.
https://www.pdfnotes.co/
GREEN’S,GAUSS’S AND STORE’S THEOREMS 299

Let us take the surface S as the area bounded by the square C.


Since the square lies in thejiy-plane,therefore n = k.
Now curl F = i j k -(y^'-x)k.
± ± d
dx dy dz
xy xy^ 0

/.(Curl F)*n =(/-x)k*k ^y^-x.


The given line integral

^{xydx +x^dy)= ffs —x)dS, where 5is the area


of the square ABCD.
Equation of the st. line j4B isx + y = 1 /.e., y = l —x and the
equation of the st. line5C is -jr + y = 1 /.e., y=x+ L

-x)dxdy

.r r Jx=0jy=-(l-x)
1-x
(y^ -x)dxdy
rO
= 2jx=-lX=0 (y
fx+1
-x)dxdy + 2 r r (/-
Jx=0 Jy=0 x)dxdy

[*.'● the integrand y^ —x is an even function ofy ]

T.
ll-JC

= -i
\/
.y=0
or+ 2 f Y-
●/.v=0 3 -xy
J y=0
dx

-x(l-x) dx
t1
=2 +2
-1 3 0

i2 +^ i3 +^ ±
12

12 3^^ ^ 12 3*

Ex. 22. (a) Evaluate dr by Stoll's theorem where

^ ~y i + j “ (x + z) ^ aitd C is the boundary of the triangle with


vertices at (0,0,0), (1,0,0), (1,1,0).
https://www.pdfnotes.co/
300 VECTOR CALCULUS

Sol. We have
CurlF = i J k = 0i+j+ 2(j:-)^)k-
d d d
dx ay dz
y2 / -ix +z)
Also we note that z co-ordi
nate of each vertex of the triangle is ^
zero. Therefore the triangle lies in a(1.1)
thej^-plane. So n = k.
CurlF*n =[j + 2(x->>)k]*k
= 2(z-y). ik
In thefigure,we have only,con
sidered thez-y plane.
The equation ofthe line OB is
y=z.
By Stoke’s theorem

§c ~ JX
dx
2 J y=0
dx = 2

Ex. 22 (b). 7/ F =(2z^ +/)i +(3y - 4j:)j, evaluate ^ F*rfr


where C is the boundary ofthe triangle with vertices at(0,0),(2,0)and
(2,1). (Andhra 1992)
Sol. We have
CurlF= i j k = 0i+0j+(-4-2y)k
d d
dx dz
/?(2,1)
2z^+y^ 3y-4z 0
=-2(2+y)k.
By Stoke’s theorem,

Jc JX (0,0) (2,0) A X
https://www.pdfnotes.co/
GREEN’S,GAUSS’S AND STORE’S THEOREMS 301

where 5 is any surface bounded by the curve C and n is unit normal


vector to the surface S. <
Let us take the surface S as the area of the given triangle. Since
the triangle lies in thej^^-plane, therefore n = k
(curl F)*n =-2(2 + y)k*k =- 2(2 +y).

E -2(2+y)dS,
where S is the area of the triangle OAB.
The equation of the st. line OB isy = ijc.

E -2(2+y)dS
*x/2

r.r=o

[●.' for the region S, x varies from 0 to 2 andy varies


from 0 tojr/2]
rix/2

' J>'=0 dx,


integrating with respect toy regardingjr as constant

=r- 8
2c-
t]^= 14
12
0

12 =-*-h-3
14
■■■£ F«c/r=- 3
Ex. 23. Evaluate by Stake’s theorem

i P {sin zdx — cosxffy + sin ydz)


where C is the boundary of the rectangle
0 <ar :< jr,0 ^y < l,z = 3.
Sol. Here F = sinz i - cosjcj + sinyk
CurlF = i j k
d d d
ax dz = cosyi + coszj + sinxk
sinz — cosz siny
Since the rectangle lies in the plane z = 3, therefore n = k
.*. curlF»n = (cosyi + coszj + sirizk)«k = sinz..
By Stoke’s theorem
https://www.pdfnotes.co/
302 VECTOR CALCULUS

(curlF) ●ndS
JC

■f r
Jy=:0»/x = 0 sinxdxety ■t =0
sinxdx = 2.

Ex. 24. By converting into a line integral evaluate

X£ (V X A)*n dS, where A -(x - z)i + (x^ +yz)y- 3xy^ k

and S is the surface of the cone z = 2 - V(r^ + above the xy-plane.


(Meerut 1974)
Sol. The ;«y-plane cuts the surface 5 of the cone
z=2- +y^) in the circle C whose equations are jp" + y^ - 4 J

z = 0. Thus the boundary of the surface S is the circle C.


The surface S lies above the circle C. Let the parametric equa^
tions of the curve C be jc = 2 cos /,y = 2 sin r, z = 0,0 < r < 27t. '
By Stoke’s theorem, we have '

E (VxA)»ndS' =
i
A»dr

^Jc + ●(dxi + £tyj+dzk)


= (x-z)dx + (jp+yz)tfy - 3xy^dz

[ on C, z = 0 and rfz = 0 ]
= J^(xdx+x^dy)
dt
i = o ^ dt dt]
2jz
= I [2 cosr ● (- 2 sin r) + 8 cos^r -2cos/ ]dt

=-^X
2ji
2 sin t cos tdt-r 16 CJQS^tdt

i
2jt jr/2
smTxdt + 16*4 cos'* t dt

cos2r -\2ji
= -2 + 64- 31 £
2 Jo 4-2 ' 2
= 0 + 12 JT = 12jt.
Ex. 25. By converting into a line integral evaluate
https://www.pdfnotes.co/
GREEN’S. GAUSS’S AND STORE’S THEOREMS 303

xc (V X F)*nc?5

where F = (a:^ +>.- 4)i + 3^j+(2o- +z^)k S the sutfitce of


the paraboloidz = A — (jp" + above the xy-plane.
Sol. The .^-plane cuts the surface 5 of the paraboloid
z = 4- in the circle C whose equations are _ 4^
2 = 0. Thus the boundary of the surface5is the circle Cand the surface
5 lies above the circle C. Let the parametric equations of the curve C
hex = 2cos/,y = 2sinr,2 = 0,0 < r < 27t.
By Stoke’s theorem,we have

J][ (VxF)»ndS==^ F*dr

~ Jc + + ● (dzi+ifyj+ dzk)
~ Jc (^^'*’y ~ ^^y^ +(2xz+z^)dz

~ Jc + ,sinceonC,z = Oanddz = 0

+ 3xy dt dt

p2ji p
^ Jo + 2sinr - 4)(- 2sinr)+ 3-2 cos / ● 2sin t ●

2ji 2jt
2cos/]dt
cos^rsin/<i/- 4 siiPtdt
2ji

Zt
+8

jr/2
X sinrrfr + 24
2n

X cos^ t Sin tdt

=8
COS^f
3 Jo
-4-2-2-
X sin^rrfr + 8 -cost
2jc
0
t2j[
COS^t
-24
3 JO
24
=8-0-16~|+ 8-0 T'0= -4»-
Ex. 26.Evaluate
XX (V X F)*n dS;where F =(y -z + 2)i +
(_yz + 4)j -zzk and S is the surface of the cube x = y =z = 0,
z = y =z = 2above the xy-plane.
/

https://www.pdfnotes.co/
304 VECTOR CALCULUS

Sol. Thejty-plane cuts the surface of the cube in a square. Thus


the curve C bounding the surface 5 is the square, say OABD,in the
Av-piane whose vertices in thej>y-plane are the points
0(0,0),^(2,0),B(2,2),i>(0,2).
[Draw figure as in solved example 14]
By Stoke’s theorem,we Imve

JJ (VxF).ndS = i ¥ ●dr

=^ '(y-z + 2)i + (yz + 4)j-xzk ●(dxl +dyj+dzk)

= J^\y-z + 2)dx + {yz + 4)dy-xzdz


[ V onC,z = 0and^fe = 0]
^ J^[(y + 2)dx + 4dy'
(y + 2)dr + 4dy

*Ld l(y + 2)dx + 4dy + Lt{y4-2)dx4-4dy

[ ●/ on OAyy = 0, = 0 andz varies from 0 to 2;


onylB,jc = 2, ifr = 0 andy varies from.O to 2;
on BD,y = 2, = 0,z varies from 2 to 0;
and on DO, z = 0, dz = 0,y varies from 2 to 0 ]
2 r -i2 ttO rnO
=2 . X JO +4 L y JO^+4 L X JZ +4 L y JZ
= 4 + 8- 8- 8= -4.
Ex. n. Apply Stoke’s theorem to prove that
f {ydx + zdy-\rxdz) --2.y/2jta^
where C is the curve given by
+y^ + 2^ -2ax-2ay-QyX-\‘y = 2a
and begins at the point (2a, 0,0) and goes at first below the z-plane
\ (Meerut 1982) Agra 69),
Solution. The centre of the sphere x^+y^ + z^-2ax-2ay-0
is the point (a, a, 0). Since the plane z + y = 2a passes through the
point (a, a, 0), therefore the circle C is great circle of this sphere.
Radius of the circle C
https://www.pdfnotes.co/

GREEN’S,GAUSS’S AND STORE’S THEOREMS 305

= radius of the sphere = = a \^2.


Now
+2>fy+Mtz)=£(yl +zj+2k)«rfr

=/x [curI(j>l +zj*jck)].n<iS,


where S is any surface ofwhich circle Cis boundary[Stoke’s theorem].
Now curl(yi+2j+jck)= i j k
± ± ±
dx dy dz
y z X
= -l-j-k=-(i+j+ k).
Let us take S as the surface of the plane jr +y = 2a bounded by
the circle C.Then*a vector normal to5is grad (jr + y)= I +j.
n = unit normal to5=

●●● £(y<^+zdy +xdz)


■E- (l+j + k). dS

-kE dS =
2
- ^ (area of the circle of radius a V2)

Ex. 28. Use Stokers theorem to evaluate


E
whereF=yi + (x - 2xz)J - xy k and Slg^ surface ofsphere
(VxF)«nrf5,

^ + — a^, above the xy-plane


Sol. The boundary C of the surface 5 is the circle x^ +y^ =
2 = 0. Supposes = a cos r.y = a sin r,2 = 0,0 s f < 2;r are paramet-
nc equations of C. By Stoke’s theorem, we have

ff
s (VxF).nrf5

^ ^ X^^ 222) j -xy k]»(l<ir + jrfy + kdz)

=^j^lydx + (x-2xz)ify--xydz]
https://www.pdfnotes.co/

306 VECTOR CALCULUS

[ V onC,z- Oanddz = 0]

— [asin ^(— <i sin ^)+ « cos t {a cos t)]dt


sin2ri2jr
=fl2j^^(cos^/-sin^r)dt f ca&2tdt-(?
2 Jo
= 0.

Ex. 29. Evaluate


the surface inteffal JJ^ cur/F^n dS fy trans
forming it into a line integral, S being that part of the surface of the
paraboloidz = 1 -a? ^/forwhichz S 0,fl«d F =)i + 2j +J:k.
(Bombay 1979)
Sol. The boundary C of the surface S is the circle jc^ + = 1,
z = 0. Suppose X = cos f,y = sin r,z = 0,p s r < 2JT are parametric
equations ofC.By Stoke’s theorem,we have
F»dr
JDE CMlF.n<iS = J[
= +;?! +^k)*(l<fc +Jrfl’+ k<b)= +zify+xdz

[ V onC,z = Oanddz = 0]
dx
^dt
dt = J^sinr(-sinr)<ft = - sin^rdr
— Jt.
=-4 sto*(<ir=-4xixY =

evaluate curlF*ndS taken over the portion of the surface


E».3^F= 0>*+z^
x^+y^ +z^-2ar + oz=0 above the plane z = 0, and verify Stoke’s
theorem.
Sol. The surface x^.+y^ +z^-2ax + flz =0 meets the plane
z =0 in the drcie C given by x^ +/- 2ax = 0, z = 0. The polar
equation of the circle C lying in thejq^-plane is r= 2a cos 6,
0se<3t. Also the equationx^+/-2mf =0canbe written as
https://www.pdfnotes.co/

GREEN’S,GAUSS’S AND STORE’S THEOREMS 307


2
(x-a)^+/=a .Therefore the parametricequations ofthecircle Ccan
be taken as
x = a + aoo&t.y = asinr,z = 0,0 ^ r < 27t.
Let S denote the portion of the surface ^2 _ 2^^ az„
= 0l3dng above the planez =0and denotethe plane region bounded
by the ^le C.By an application ofdivergence theorem,we have
JJs ■E.
curlF ●kdS.

Now curl F»k = i j k


d d d
dr dz ●k

>* + z*-** jr*+y“-2*

[ V i●k=J●k = p]
= 2(f-yy_
E curlF *iirf5
■E cuflF«k<iS ●iE,2e-rt«
=2t r*J0 = O «|rts0
cos$
(r COS 0 - r sin 0) rd0 dr.
changing to polars
2aoosd
I

'a
= 2x Sa^ f (cos 0 — sin 0) cos^ 0d0
3 Jo
= 3 Jo
f cosrBdB
'a

£■ cos*esine<» = 0
16
= 2x^ J, ods^fl^
16 a* 3jja 2_ 2na*.
= 2X^ 4x2 2~" (1)
Also <2r

■4
~ + [ V onC,z = 0and<fe = 0]
https://www.pdfnotes.co/
308 VECiTOR CALCULUS

dt

-fl^sin^r](fl cosr + a sint)dt


= J[^[(a + «cos0^

= J[^(l + cos^^ + 2cosf - sin^O (cosf + sint)dt

= 2cos^t dt,the other integrals vanish

= 2fl^ X 4 = 8fl^ X i X Y = 2?ia^ ...(2)

Comparing(1)and(2),we see that

f£aaiF-ndS=£F*dr.
Hence Stoke’s theorem is verified.
Ex.31.Show that

Jfj[ ^cmt/F ● dS = ^F*dr - Jfj(^. (grad4> X F)»dS.


Sol.Here Cis the closed curve bounding the surface S.Applying ,
Stokers theorem to the vector ^F,we.have

i (4>¥)*dr ■a curl(0F)*nd5

= iC (Vx(^F)].<iS
= r [(grad^) xF + 0curlF]»dS
[ *.● curl A) = ( grad x A + 0 curl A ]

=a (grad^ X F)»dS +

Hence by transposition, we have


a ^curlF*dS.

<p curl F*dS = ^ "JX (grad


Ex. 32. Iff^^itandg = Vv> am two vectorpoint Junctions, such
that
vV = o,v^V' = o
show that

JJ (g.V)f.«S = X(» X g)*ar + a (fV)g«(iS.


https://www.pdfnotes.co/

GREEN’S,GAUSS’S AND STORE’S IHEOREMS 309


Sol.
Stoke s theorem to the vector f x g,we have

Vx(fx^)«nrf5

-jK curl(f X g)*</S

“JK [<8*^l'-g'iivf-(f*V)g + f<livg] .</s


■£ fe*V)f-g div -(f.V)g +fdiv Vv>1‘.is
[ ●.* f = and g = Vv» ]
■£ [(g*V) f - 0g - (f.V) g + 0 f ]«dS

[ div = V* = 0, given
and similarly div = VV = 0, ^en]

= Jl (f*V)g.dS.
Hence by transposition, we have

Ik fe*V)f<iS = j[(fxg).* + jjj^ (fV)g.ds.


Ex. 33. Ptove that a necessary and sufficient condition that

£ F»rfr =
= Ofor every c/osedauveC lying in a Simply connected region
' R is that V X F = 0 identicalfy.
Sol. Sufficiency. Suppose R is simpty connected and curl F = 0
everywhere m i?. I^t C be any closed path in R. Since R is simply
, rannected, therefore we cm find a surface 5 in i? having C as its
, boundary. Therefore by Stoke’s theorem

Necessi^. Suppose F ● dr => 0 for eveiy closed path C and


assume that V x F ^ 0 at some pointed.
Th®”‘aWngVxFascontinnous,theremusteidstaregionwIth
^ as an interior point, where V x F0. Let S be a sur&ce
m this region whose normal n at each point is in the same as
https://www.pdfnotes.co/

310 VECTOR CALCULUS

V X F,i£. V X F = An where A is a positive constant. Let C be the


bound^ ofS.Then by Stokers theorem

^ F*dr=J][(VxF)«ndS ■a
Xn*ndS

= A5>a
This contradicts the hypothesis that F *dr = 0 for every

closed path C. Therefore we must have V' x F = 0 everywhere inR.


§ 10. line integrals Independent of path.
Let F (r,y,z) =/(x,y,2) I + g (x,y,z) j + h (jc,y,z) k be a v^or
point funcUon defined and continuous in a regioni? of space. LetP and
Qhc two points ini? and let Cbe a path joining? to Q. Then
...(1)
J^F ● dr = {fdx + fffy ’^hdz)
is called the line integral of F along C. In general the value of this line
integral depends not onty on the end points ? and (2 of the path C but
alsoonC
In other words, if we integrate from P to Q along different paths,
we shall, in gene:;al, get different values of the integral. The line integral
(1) is said to be independent of path in R, tffor every pair of end points
PandQinRthe value of the integral is the same for all paths CinR
startingfrom Pond ending at Q,
In Ibis case the value ofthis line integral will depend on the choice
of P and Q and not on the choice of the path joining P to Q.
B^BsAtion,Thee3perssionfdx + gdy + hdzissaidtobeanexact
dijferential ^ there exists a single valued scalar point Junction (x,y,z), ;
having corUinuous first partial derivatives such that' I
dxf =fdx ■¥ gdy hdz.
It can be easily seen that/de + g<fy + hdz is an exact differential if and
only if the vector function
F=/i + gj + hk
is the gradient of a single valued scalar function 4> (x,y, z).
Because F = grad^

it and only if/1 + g j + h k = ^ i + ^ J + ^ k


it and only if/ dx 8 9z

It and on^r + A <fe = 1^* +


it and only if/dr + gd^ + fr dr =
https://www.pdfnotes.co/

GREEN’S,GAUSS’S AND STORE’S THEOREMS 311

Thus F = grad^ if, and only iffdx + gtfy + hdz is an eixact


differential ^0.
Theorem 1. Letf(x,y,z),g(x,y*z)and h (x,y,z)be continuous in
a region R ofspace. Then the line integral

J{fdx-¥gdy'^hdz)
is independent ofpath in R ifand only ifthe di^erentialform under the
integralsign is exactin R (Meerut1968)
Or
Let F(x,y,z) be continuous in region R ofspace. Then the line
integral

X F*rfr

is independent of the path C in R joining P and Q ft and only


F =sgrad4> where ^(x,y,z) is a sin^-valued scalar junction having
continuousfirstpartialderivativesin R. (Kerala 1975)
Proof. Suppose F = grad^in LetPand ]2be ai^two points
●in P and let C be any path from P to Q in P.
Then
●dr

●(dxl + tfyi+dzk)

= Jcf \dr dx +
= = =4>(Q)-4>(P).
Thus the line integral depends only on points P and 0 not
the path joining them. This is true of course onty if ^ (x,y,z)is single
valued at all points P and Q.
Conversely suppose the line integral F ● r is independent of
the path Cjoining any two points P and )2 in P. Let P be a fixed point
’ I'd ’ ^o) ii^ ^ 12 be any point (x,y, z) in P.
Let

-X (*.y.«)
(Xo«>o*^o) -X,
F*rfr
(*.y.*)
(*0*^0 »*b) ‘

Differentiating both sides with respect to 5, we get


!)*●
https://www.pdfnotes.co/

312 VECTOR CALCULUS

dr
ds ds
But
ds dx ds^ ds^ dz ds

(dx dy* dz ) \ds ds* ds )


dr
= v^.-
dr
●●● or (V^-F)*x = 0.
Now this result is true irrespective of the path joining P to Q
ix.this result is true irrespective ofthe direction of^which is tangent
vector to C.Therefore we must have
- F =0
ie., V^ = F.
This completes the proofof the theorem.
Definition. A vectorfield F defined and continuous in a

Ion ¥ ●dr is independent


R ofspace issaid toofthe path C in R joining
be a conservative P and
vectorfield Q where
ifthe P and
line integral
Q are any two points in R
By theorem 1, vector field F (r,y, z) is conservative if and only if
F = V ^ where <f> (z,y, z) is a single valued scalar function having con
tinuous first partial derivatives ini?. The function <j> (x,y, z) is called the
scalar potential of the vectorfield F.
Theorem 2. Let F (z,y, z) be a vectorfunction defined and contin-
CQ
uous in a>egion R ofspace. Then the line integral I F ● dr is indepen
dent of the path joining any two points P and Q in R if and only if

^ F»dr = 0 for every simple closed path in R


Proof. Let C be any simple closed path in R and let the line
integral be independent of path in R. Take two points P and QonC and
subdivide C into two arcs PBQ and QAP. Then
F»dr
PBQAP
'f
https://www.pdfnotes.co/

GREEN’S,GAUSS’S AND STORE’S THEOREMS 313

= J F»dr + J* F*dr 4$.


PBQ QAP A Q
F*dr
PBQ PAQ
= 0,since the integral from J C
P to j2 along a path through B is
equal to the integral from P to Q
along a path throughyl.
Conversely suppose that the
integral under consideration is zero on eveiy simple closed path in R,
LetP and Q be any two points inP which join PtoQ and do not cross.
Then

F*dr +
f
PBQAP
F.dr=J
PBQ
F*dr.
QAP PBQ PAQ

§
But as given,we have
PBQAP
F*dr = 0.

"Spbq F*dr -L F»dr = 0

or f F»dr = JPAQ
f F*dr.
JPBQ
This completes the proof of the theorem.
Theorem 3.Let F(x,y,z)=fi + gj+ hkbea continuous vector
junction havingcontinuousfirstpartialderivatives in a reg^n R ofspace.

If ^fdx ■¥ gdy + hdz is independent of path in R and consequently


fdx + gdy + hdz is an exact differential in R, then curlF-Q everywhere
in R Conversely, ifR is simply connected and curl F = 0 everywhere in R,

then fdx + gdy ●¥ hdz is an exact differential inRor ^fdx + g(ty + hdzi^
independent ofpath in P. (AUahabad 1979)
Proof.
Suppose J* {fdx + g(fy + hdz) is independent of path in
R. Then fdx + gcfy + hdz is an exact differential in R. Therefore
F =/i + g j + A k = grad
curl F = curl (grad rp) - 0.
https://www.pdfnotes.co/

314 VECTOR CALCULUS

Conversely suppose R is simply connected and curl F =0everywhere


' inR.LetC be any simple closed path inR.Since is simply connected,
therefore we can find a surface S in R having C as its boundary.
Therefore by Stoke*s th^rem

^ F«rfr = /X (curlF)*iid5 = 0.

i
Thus |> F*dr is zero for every simple closed path C in i?.
c
Therefore
J* F*dr is independent of path in R.
Therefore F = V ^ and consequently/dir + gify + fuizis2Ln exact
differential
Note.Theassumption thatR besimply connected is essentialand
cannot be omitted. It is obvious from the following example.
y
Example.Let F = —

Here F is not defined at origin. In every region R of theJiy-plane


not containing the origin,we have

CurlF = i J k

d d d
dr dz
y y
0

= 0l + 0j +|^[-r^]
_ fjc^+y^-2r^ jr^ + y^-^ k =0k»0.
-| ^
Suppose R is simply connected. For example letR be the region
enclosed by a simple closed curve C not enclosing the origin. Then

£ F*dr“f
●'C
—Z-—-rdr+
jc^+y^
2^
jc^+y^
\'
= Jjjt
ff a ( y dxify.
https://www.pdfnotes.co/

GREEN’S,GAUSS’S AND STORE’S THEOREMS 315

by Green’s theorem in plane


= 0.
Suppose R is not simply connected. Let R be the region of the
ji^-plane contained between concentric circles of
| radii
| and and hav
ing centre at origin. Obviously R is not simply connected. We have
2 = 0,everywhere in i?. Let C be a closed curve m R.The parametric
. equations ofC can be taken asX = cos = sin r, 2 = 0,0 /< 27t.

We have
i F-dr
=/-
y ... . y .

star ^t ^ dt
●// = 0 cos^r+ sta^f cos^r + sta^r

= J^ (sta^r+ cos^/)dr = 27r.

Thus we see that F«dr^0.

Definitioii. Irrotational vector field. .4 vectorfield F is said to be


irrotational tf curl F = 0.
We see that an irrotational field F is characterised by any one of
the three conditions:
(i) F = V0,
(ii) VXF = 0,

' (iii) i F ● dr = 0 for every closed path.


Any one of these conditions implies the other two.
Solved Examples
Ex. hAre the followingforms exact ?
(i) xdx^ydy + zdz. (ii) e^dx -\-e^dy + e^dz.
(iii) yzdx ’¥xzdy +xydz, (iv) yVdr + IxyPify + 3x^2^dz.
SoL(i)Wehave
xdx -yify + zdz = (xi -yi + 2k)*(dri + dyj + dzk)
= F»dr, where
F=jd-yj+2k.
https://www.pdfnotes.co/

316 VECTOR CALCULUS

We have Curl F =
I j k = 0i + 0j + 0k = 0.
A A ±
ar ^ dz
X -y z
the given form is exact
(ii) Here F = e'i + + e^k. We have
CurlF = I j k = 0i + 0j+(e*-e^)k.
AAA
ar dv az

Since curl F 0,therefore the given form is not exact


(iii) Here F =>?zi + xzj + We have
CurlF =
I j k =(t-Jc)i-()’-)')j +(2-z)k = 0.
± ± ±
dx dy dz
yz xz xy
Since curl F = 0,therefore the given form is exact.,
(iv) Here F =yV I + + 3jq^ k. We have
Curl F = I j k
a a a
dx dz
y^z^ 2xyP 3xy^2^
=(6xy2^ -6xy^)i -(3y^- 3>^z^)j
+(2yz^-2y2^k
= 0.
the given form is exact
Ex.2.In each offollowingcasesshow thattheg^ven dijferentialform
is exact andfind afunction <p such that theform equals d<f>:
(i) xdx-ytfy-zdz. (ii) dx zdy ydz.
(iii) cosxdx-2yzdy-y^dz,
(iv) (z^ - 2xy)dx - + 2xzdz.
Sol.(i) Here F =jd - -zk. We have
CurlF =
i j k = 0i + 0j + 0k = 0.
A A A
dx dy dz
X -y —z
https://www.pdfnotes.co/
GREEN’S,GAUSS’S AND STORE’S THEOREMS 317

the given form is exact.


Let F = V
or Jd->j-zk = dr k.Then
dy dz
d0
dr = X whence^ = y + 0^,2) ...(1)
d4> _
= -y whence^ = -I
j+f2(x,z) ...(2)

dz = -zwhence4,^-j+f^(x^yy ...(3)
The constants of integration are functions of the variables not
involved in the integration because the derivatives are partial.
(1),(2),(3)each represents These agree if we choose

fi(y*2)= - j^’fi(t.^)= ’/3 0c,y)= ●


x^-3^-z^
2 to which may be added any constant.

Hence^ s 2 + C,where Cis a constant.


(ii) Here F = i +zj + yk. We have
CurlF =
i j k = 0i +0j+ 0k = 0.
± ± ±
dx dy dz
1 z y
the given form is exact.
LetF = V4>
or i +zj+yk = k.Then
dr dyJ ^ dz
dip
dr = 1 whence^ =x +/j(y,z) ...(1)
dtp
dy =z whence^ ^+/j(r,z) ...(2)
dip
dz =y whence^ =yz +/3(x,y) ...(3)
(1)» (2),(3)each represents 0.These agree if we choose
/i(y.^) /z ix,z) =x,/3(x,y) =x.
^ = X +yz to which may be added any constant.
^=x +yz + C.
https://www.pdfnotes.co/

318 VECTOR CALCULUS

(iii) Here F = cosjd - -y^k. We have


Curl F = i j k
d d d
dx dy dz
cosx -2yz .y
=(-^ + ^)l + 0j + 0k = 0.
the given form is exact.
LetF = V^,
or k.Then
cosjri - 2yzj -yhi=^i+ dz
d<p
...(1)
dr = cosjrwhence^ = sinjc +fi(y,z)
^=^2yz whence^ + A (x,z) ...(2)

^ whence^ = +/s{x^y). ...(3)


dz
(1),(2),(3)each represents <f>. These agree if we choose
fi(y*2)= -A /2(X.Z)= sinx,/3(x,y)= sinx.
^ = sinx - to which may be added any constant.
^ = sinx-y^z + C. .
(iv)Here F =(z^ - 2xy)i -x^j + 2xz k. We have
CurlF = i j k =(M + Oj + Ok =0
.
d d d
dr ^ dz
z^-2xy -x^ 2xz
.*. the given form is exact.
Let F = V ^
3^
or (z^-2xy).l + 2zz k = + dz k.Then

.(1)
^=z^- 2xy whence =z^-x^ +/i fyyZ)
..(2)
^= -x^whence^ =s -x^ +/2(x,z)
...(3)
dz = 2xz whence0= x z^ + A(x,y).
(1),(2),(3)each represents 4>. These agree if we choose
fi (y.z)= 0,/z(r,z) =zz*,/a(z,}')=
https://www.pdfnotes.co/

GREEN’S,GAUSS’S AND STORE’S THEOREMS 319

j^y to which may be added any constant.


^ = 2^jc-jc^y+ C.
Ex.3^ Show that
{^:?cosx - Aj?z)dx + Iz^sinxdy +(3y^si smx-x'^)dz
is an exact differential ofsomefunction <p andfind thisfunction.
Sol. Let F =(yV cosx -4A)i +2z^sinxj+
(3yV sinx -x^)k.
We have curl F
i j k
' d d d
ar dz
^2^cosx~4x^z 2z^sinx 3y^z^sinx-x"*
= * |^(33^^sinx-x^-^(2z^ysinx)

“j ^(3y^sinx” ^(3^cosx - 4x^z)


+k sinx)- (yV cosx - 4x^z)

= sinx -6s^ sinx)i - (3yV cosx-

(3zV cosx -4x^]j+(2z^^x-2yz^cosx)k


= 0i-qj+ 0k = 0.
there exists a scalar function ^(x,y,z)such that
F = V^.
F*dr = V4>*dr
or cosx - 4x\)dr + 2z^y sinx +(3yVsinx -x'*)dz = d<p.
Hence (yV cosx— 4A)dr + 2z^sinxd^ +(3yVsinx -x"*)dz
is an exact differential ofsome function
NowF = V^=> F =
ar 3y^^ az
Equating the coefficients ofI,j,k on both sides,we get
ar =yVcosx -4Awhence^ =yVsinx -x^z +/i(y,z) ...(1)
d<f>
9y = 2A sinx whence^ =zV sinx+/2(z,x) ...(2)

^= 3yV sinx -x^ whence^ = yV sinx -xlz +fy(x,y) ...(3)


(1),(2),(3)each represents <p. These agree if we choose
https://www.pdfnotes.co/
320 VECTOR CALCULUS

fi(y.z)= 0./2(J.*)= - fi(f-y)= 0-


^ ss);^ sinjc -x^z to which maybe added any constant.
Hence 4> = y^z^ sinx — x^z + C.
Ex.4.Show thatF =(2xy +z^)i + x^j + 3xAisa conservative
forcefield Find the scalar potential Find also the work done in moving
an object in thisfieldfrom
(1.-2,1)to(3,1,4).
Sol. The field F will be conservative if V x F = 0.
We have
V X F= i = 0.
d ^ d_ ±
5
az dz
2xy+^ 3xz^
Therefore F is a conservative force field.
LetF =
or (2xy +z^)l +x^j+ 3xz^k = + ^k.Then
. ax ay oz
^ ...(1)
ax = 2xy +z^whence^ =x^ + A +/i (y,z)
...(2)
^=x^whence^ =x^ +f2(x,z)
...(3)
^= 3xz^ whence^t—xP+f^(x,y) '
(1),(2),(3)each represents <p. These agree if we choose
h (y.^)= o» h(^yy)
.-. 0=x^ + xz^ to which may be add^ any constant.
<p =^jPy +xz^ + C.
(3,1,4)
Work done F»dr

(3,1.4) ● (3,1,4)
(1.-2.1)
di>= 4>. (1,-2,1)
'(3.1,4)
= x^+xz^. (1,-2,1)= 202.
Ex.5. Show that the vectorfield Fgiven by
F =(y +sinz)i +xj+xcojzk
is conservative. Find its scalarpotential
Sol. We have
https://www.pdfnotes.co/

GREEN'S,GAUSS’S AND STORE’S THEOREMS ' 321

V X F= k = 0.
J
dr
i d
dy a?
j'+ sinz X xcosz
the vector field F is conservative.
LetF =
or
(y+ sinz)i+xj +xcoszk =^1+ +-^k.Then
^ dy dz
d<f>
dr =y + sinz whence^ =xy +xsmz +/i(y,z) ...(1)
d^
= x whence^ =xy+/2(x,z) ...(2)

dz X cosz whence^ = x sinz +f^(x,y) ...(3)


(1)» (2),(3)each represents <p. These agree if we chO(^
fi(y*2)= 0,/2(x,z) =xsinz,/a(x,y) =xy.
—xy + X sinz to which may be added any constant.
^=xy+xsinz + C.
Ex.6,Show that the vectorfield
F =(2V +yz)I +(2jt^ +jtz + 2)Z^J +(2y\ +i)«)k
is conservative
Sd.We have
V X F= I J k
d d d
dr dz
2xy^ +yz 2j?y +xz + 2yz^ 2y^ +xy

=[(4yz +x)-(x + 4yz)]I -0^-y)j+[(4xy+z)-(4xy+z)Jk


= 0l-0j+ 0k = 0.
the vector field F is conservative.
Ex.7.Show that F^xf+yj -f zk ir amservadve andfind^such
rAfl/F = V0. (Kanpur 1980)
Sol. We have Curl F
322
https://www.pdfnotes.co/
VECTOR CALCULUS

= I j k
± ± ±
dx ^ dz
X y z
= f* _ dzi 1 + \dz dx)* lar ^1 ^
= 0i + 0j+ 0k = 0.
the vector field F is conservative.
LetF =

te., jd+)5j+2k =^i


ax + Mj
^ + MtT|jgn
dz

^=xysbencei,= ^j^+f^(y,z) ...(1)

^=y whence^ =1/+/j(z,x) ...(2)

^?zwhence^ =l22+/3(i,y) ...(3)


(1).(2),(3)each represents <p. These agree if we choose

fi(y»2)=f (A-r)= +|x^,

● ● ^ -r 2^+ 2)^ + 2^to which may be added any constant,

r Hence^«l(x2+/+22^ + C.
^S.showthat
F =(^/+f)« + ^«wy-z)j+(r-j-)k« a conservative
vectorfield andfind afimction ^such that
F = V0.
(Bombay 1966)
Sol. We have curl F
I J k
d d d
\ dr dz
siny +z xcosy-z x-y

+j[;|(sto,.+z)-±(t-y)
https://www.pdfnotes.co/

GREEN’S,GAUSS’S AND STORE’S THEOREMS 323

+ k —(xcos)^-2)-~(smy +2)
=(- 1 + 1)i + XI - 1)j+(cosy - cosy)k
= 0i + Qj + 0k = 0.
/. the vector field F is conservative.
Let F =

Le,, (siny +z)i +(xcosy -z)j+(x -y)k = +-^j+ fc


dx ^ az
Then — = sihy +z whence^ =xsiny +xz +/j(y,z) ...(1)
d4>
dy = Xcosy -z whence^ =xsiny -yz +/2(z,x) ...(2)
d4>
^=x-y whence 4>=xz-yz+f^(x,y) ...(3)
(1),(2),(3)each represents <f>. These agree if we choose
/i fyyX)= -yz,/2(z,z) =xz,/3(x,y) =xsiny.
^ =xsiny+xz-yz.
^ = X siny + xz —yzto which may be added any constant
Hence0=X siny +xz-yz + C.
Ex.9.Show that the vectorfield defined by
F =(2^-z^l +(x^ +z)j +(y - 3*z^)k is conservative, and
findthescalarpotentialof¥. (BombayI97O)
Sol. We have curl F
i j k
d d d
dx dz
2xy-z^ x^+z y-3xz^
=i

=(1 -1)* +(-3z*+ J +(2x -2*)k


= 01 + Oj + Ok = 0.
the vector field F is conservative.

Let F = +-^k.Tken

dz 2xy -z^ whence 4> +f^(y,z) (1)


https://www.pdfnotes.co/

324
VECTOR CALCULUS

=jc^ +2whence0 +/j(z,x) ...(2)

^=3^- 3zz2whence^ ^yz -xz^ +/3(x,y) ...(3)


(1)» (2),(3)each represents fp. These agree if we choose
/i fy.2)=zy,f2(z,x)= -A,/3(x,>)= A-
-^x^zyio which may be added any constant.
Hence^ z=j^y-:^x+zy + C.
to.Evaluate

£Ixyz^dz +(tV +ZC<Jiyz)<fy +(2x^yz +ycosyz)dz


when Cisaivpathftom (0,0,1)to(1,Ijr,2). (Meerut 1968)
SW.We have F = i +(zV+zcos yz)j
+(2x^yz+ycosyz)k.
.*. V X F =s i j k
d d d
dx dy dz
2xyz^ +zcosyz 2x^yz+ycosyz
=(2A + cosyz -yzsinyz- 2x^z- cosyz
+yzsinyz)I -(4xyz-4xyz)j+(2x^- 2xz^)k = 0.
the given line integral is independent of path in space.
LetF = V^.Then
Ah ^
= 2zyz^whence^ +/j(y,z) ...(1)
x^+z
9y amyi whence^ =x^^y + sin;>z +/z(z,z) ...(2)

^= 2z^yz +>roos;>zwhenoe^ =x^y^ + sin;>z + (z,y) ...(3)


W»(2)f(3)each represeats These agree ifwe choose
ft <y,z)= sin>z,/z(Z.Z)= 0,/3(jc,y) = 0.
A^ + sinyz to which may be added any constant.
The given line integral is therefore
■ (1,^/4,2)
d(fyi? + sinyz)= + sinyz (0,0,1)
= jr + sini;r =;r +1.
https://www.pdfnotes.co/

GREEN’S,GAUSS’S AND STORE’S THEOREMS 325

Ex. \1» Evaluate

^^yzdx-{‘{xz’{-\)dy+xydz,
where C is anypathfrom (1,0,0)to(2,1,4). (Agra 1972; Meerut64)
Sol. We hi.ve F +(rz + 1)j+ k.
V X F= i j k
d d d
dr dy dz
yz xz+1 xy
=(r-jc)i -(y j+(z-z)k = 0.
the differential form yzdx +(xz + l)ify +jiydz k exact and
the given line integral is independent of path.
LetF = V^

or y^l +(xz+l)J+:yk =^l


or +^J
qv + ^k.Tben
dz
dr yz whence^ = xyz +/j(y,z) ...(1)
d0
=zz + 1 whence^ =xyz+y +/2(z,z) ...(2)
d0
dz = xy whence <p = xyz 4-/3(jr,y) ...(3)
(1),(2),(3)each represents 4>. These agree if we choose
f\ iy>2) f2(r,2)'= 0,/3(r,y) =y.
^ =zyz + y to which may be added any constant
The given line integral is therefore
(2.1,4)
J (1,0.0)^ +y)= [xyz +y. (1.0.0)
=[8 + 1 -0-0]= 9.
Ex. 12. Show that the form under the integral «gn is exact and
evaluate

(0.2,1) [z^dx + 3yz^ +


/(2.0.1). + y2)dz].
Sol. Here F =ze*I + 2yzj+(e* + fc.
We have curl F|
= k 1
j
d d d
ar dy dz
ze* 2yz ^*+y2
=(^-^)i -(d*-e*)j+ Ok = 0.
the form under the integral sign is exact and consequently the
https://www.pdfnotes.co/

326
VECTOR CALCULUS

line integral is independent of path in space


LetF = V^ '
or
+ 2)^j + +3^ k =-^1 + ^k.Then
=ze*whence^ =ze»+/j O',z) ...(1)

= 2j-zwhence^ =)r^+jfj(z,:t) ... (2)


i !

●^ = e*+/whence^ = e»z+y^+/3 \
>..(3)
(1), (2), (3) each represents (p. These agree if we choose
fl O-.Z) = A/z (Z,Z) = «^,/3 (jc,y) = 0.

line inre^T fc att^re


■ (2,0,1)
“ S(^> 2,1) ^ (2^ +7^2) = 2^ +^Z -I (0,2,1)

= [e^ + 0-l-4]*‘c2_5

- Ex. 13.//F = cosyl-xsinyl evaluate £ where C is the


curve y = V(1 - x^) in the x-yplane from (1,0) to (0,1).
Sol. We have
X “X -xsin)Yfy)
“X -y^) sinydy.
It is difficult to evaluate the integrals direcUy. However weob-
serve that
CurlF = i j k
d d d
dx dz
cosy -xsiny 0
= (M + Oj + {— siny + siny)k = 0.
the given line integral is independent of path.
Let F —
or cosyi -xsin;'j = Mj + + k.Then
dz

= cosy whence <p=x cosy +/j (y, z) ...(1)
dip
“Xsmy»whence^=xcosy+/2(r,z) ...(2)
https://www.pdfnotes.co/
GREEN’S,GAUSS’S AND STORE’S THEOREMS 327

a0
dz =0whence^ = fs ...(3)
From (1),(2),(3),we see that^ = x cosy,
i^e given line integral is equal to

t (0.1)
d {x cosy)= X cosy
(1.0) -1(1.0)
=.[0- lcos0]= -l.
Ex.14.Show that the vectorfield Fgiven by

is irrotationaL Find a scalar <p such thatF = vi.


Sol. We have
CurlF = i j k
d d d
dr by dz
J^-yz y^-zr z^-xy
_ =(-x-»-x)i-(-y-t-y)j +(-2+z)k = 0.
.*. The vector field F is irrotationaL
Let F = V^
or
<^-yz)1 + O^-zr)j+ k= Then
or ^ dz

-;>2whence^ =T ...(1)

by =y^-zx whence^ ^-xyz-¥f2ix,z) ...(2)

^=z2
dz -xy whence^ = -j-xyz +f^(x,y). ...(3)
(1)» (2),(3)each represents <p. These agree if we choose

^+ y>/2 (●». z)=


/lO’.z)='| >/3 (z. y) = ●
z*+y*+z»
Therefore 0 = -xyz-t-C.
3
Ex. 15. Show that the following vectorfunctions F are irrotational
andfind the corresponding scalar such that
F = V^.
(i) F = (ywy + ZC05X) i + (xcosy j/hz) j
+ (ycosz + sinx) k. (Calcutta 1975)
(ii)F = (ysinz -5wx) I -f- (xsinz + 2yz)i + (xy cosz
(iii)F=x^l -Hy^j +z^
https://www.pdfnotes.co/
328 VECTOR CALCULUS

Sol.(i) We have curl F


i j k
d d d
dx dz
siny+zcosx xcosy + sinz ycosz + sinx
=i
^(y cosz + sinjr)-^(r cosy + sin z)

+j ^(siny +zcosjc)-^(ycosz + sinx)


+k
—(xcosy+sinz)-^(siny+zcosx)
=(cosz- cosz)i +(cosx - cosx)j+(cosy - cosy)k
= 0i + 0j+ 0k = 0.
.*. the vector F is irrotational.
LetF = V^
Le., (siny +z cosx)i +(x cosy + sinz)j+(y cosz + sinx)k

dx dy^ dz k.Then

~= siny +z^x whence^ =x siny +zsinx +/i(y,z) ...(1)

dy =X cosy + sinz whence 4>=x siny + y sinz +/2(z,x) ...(2)


dtp
dz =ycosz + sinx whence^ =ysinz +zsinx +/3(x,y) ...(3)
(1)» (2),(3)each represents <p. These agree if we choose
h iyy2) =ysinz,/2(z;x)=zsinx,/3(x,y)=xsiny.
4> =xsiny +zsinx +y sinz to which may be added any con-
stant
i Hence <p =x siny +zsinx +y sinz + C.
(ii)Do yourselt Ans.^=xy sinz + cosx + y%:+ C.

(iii) Do yourselfc Ans.^= J(x^+y^ +z"*) + C.


Ex. 16. Find a,b,c^ F =(3x - 3y ^ oz)I +(bx + 2y -4z)j
+(2x + <y +z)kisirrotadomd. (CaUcutl974)
Sol.The vector F is irrotational ifand onfy ifcurl F = 0. We have
CurlF= ■ i J k
d d d
dr dy dz
3r — 3y + oz hx +^ — 4z 2r + <y +z
https://www.pdfnotes.co/

GREEN’S,GAUSS’S AND STORE’S THEOREMS 329

=1
(2* + cy +Z)-^(te + 2»I- <b)

+k
■l^(bx + ty -^)-±(3x-^ + az)
= (e + 4)l + (a-2)J + (* + 3)t
NowcurlF = 0 if(c + 4)i + (a-2)j + (6+ 3)k = 0
!.&, ifc + 4 = 0, fl - 2 = 0, 6 + 3 = 0
tc., ifa = 2, = ~ 3, c = - 4.
Hence the given vector F is irrotational if« = 2, h = - 3, c = - 4.
Ex. 17. Show that
(2xcosy + zsiny)dx + (xzcosy -j?siny)dy +xsinydz = 0
is an exact differential equation and hence solve it
Sol. The given differential equation is exact if there exists a scalar
function ^ {x^y^z) such that
(2r COS); + 2 sin;;) dr + (jcz cosy - jc^siny) dy+xsmydz = d<p.
Let F = (2z cosy + z siny) I + (rz cosy - siny) j + z siny k.
We have curl F
i j k
d d d
dx dz
2z cosy + z siny xz cosy - siny x siny

=i (z siny) - ~ (zz cosy - z2 siny)

+ j ;^(2zcosy+zsiny)-^(zsiny)
+k
^ (zz cosy - z^ siny) - ^ (2z cosy + z siny)
= (z cosy - z cosy) i + (siny - siny) j
+ [(z cosy - 2r siny)-(-2z siny + z cosy)] k
= 0i + 0j + 0k = 0.
the vector F is conservative.
Hence there exists a scalar function d> (z,y, z) such that
F = V^.
F«dr = V^*dr
\
or (2z cosy + z siny) dx + (zz cosy - z^ siny) + z siny dr = d4>.
Hence the given differential equation is exact.
https://www.pdfnotes.co/
330
VECTOR CALCULUS

91 k. Therefore
NowF = V^ = M,+|j+ dz

■^ = 2j: cosy + z sin;^ whence ^ ^ ,..(1)

-^ = zz cosy - siny whence <p-xz siny + z^ cosy + /2 (z,z)... (2)


dz =z siny whence ^ = zz: siny + /g (z,yj ...(3)
(1)> (2), (3) each represents <p. These agree if we choose
h (y»^) = 0»/2 (2yX) = 0,/3 (z,y) = z^-cosy.
^ = z^ cosy + zz siny.
Now the given differential equation reduces to
’ d<f>^0 whose solution is 0 = C
te., z^ cosy + xi siny = C.
18.^F is irrotational in a simply connectedregidn R, show that
ihereexistsascalarfield<f>suchthatF ^gradtp. , (Calicut 1975)
there^cu?*?^ Rotational in a simply connected region R,
C be any siMle closed path in R. Then by Stoke’s theorem
i (curl F)*n dSy where S is any surface in R
whose boundary is the closed curve C.
= 0, since curl F = 0.
CQ
the line integral I F*dr is independent of the path joining
any two points P and Q in R.
. Let P be a fixed point (ro,yo. Zg) «» « and let Q by any point
(»,)',z)inR.Let
(x,y,z)
<f<y.z) =/ F*dr <x,y,z) I
■/(^o»>'o>^o) dyJ
Differentiating both sides with respect to we get
^=F.^-
ds ds

ds 9x ds dy ds dz ds
dr
= V0*^-
●●●
or (V0 -F).^ = O.
NowthisresnltistmeirrespectJveofthepathJoiningFtoeie,
https://www.pdfnotes.co/
GREEN’S,GAUSS’S AND STORE’S THEOREMS 331

this result is true irrespective of the direction of ~ which is tangent


vector to C.Therefore we must have
-F = 0 .\
i.e., V^ = F Le., F = grad^. .
§ 11. Physical interpretation of divergence and curl.
. [Meerut 1968]
Physical interpk^tation ofdivergence.Suppose that there is afluid
motion whose velocity at anypoint isy(x,y,z). Then the loss offluid per
unit volume per unit time iii a smallparallelopipeld having centre atP(x,
edges parallel to the co-ordinate axes and having lengths
. oXy (5y,dz respectively, is given approximately by
divv = V»\.
Letv = vji + V2j + V3k,
x-component of velocity v atP = Vj(x,y,z).
x-component of v at centre offace AF£Z)which is perpendic
ular tox-axis and is nearer to origin
Vj X 2
5x bv
= Vj(x,y,z)- + ... by Taylor’s theorem
Za
5> E
c4
●p

G
0

.
y . 6x dv 1
approximately.
Similarly x-component of v at centre of opposite face
GHCB = v,+~ —I
1 2 dx approximately.
volume of fluid entering the parallelopiped across per
1 . ^
unit time= 'v, - dtdv.'l r y
oy dz.
I* 2 arj
Also volume of fluid going out the paralleiopiped across GHCB per
https://www.pdfnotes.co/

332 VECTOR CALCULUS

dx dvi
unit time= Vi1 +
2 Ox dy bz.
loss in volume per unit time in the direction ofx-axis
^ , bx bx . 3vi
v"* - (*’1 - Tl^J
Similarly, loss in volume per unit time in>^ direction
&
bx by bz.
and loss in volume per unit time in z direction
awg
dz bx by bz.
total loss of the fluid per unit volume per unit time

bixby bz
bxbybz
s= V ● V = div V.
dx dy dz
Physical interpretation of curl. Let5 be a circular disc of small
radius r and centre P bounded by the circle C. Let F(jc,y,z) be a
continuously differentiable vector function in S.Then by Stokers theo-
rem
dS,
jTF«rfr = (curlF)*nrf5 =(curlF)«nJJ*

by mean value theorem of integral calculus where (curl F)*n'is some


/ value intermediate between the maximum and minimum values of
(curl F)● n over 5.

■■■£ F*</r =(curlF) ● n 5. (curl F) *n = £ F-dr


5
Taking limit as r -► 0, we get at P

lim
(curlF)‘»n = r-*0 5
£
F ●.dr

Now (curl F)*n is normal component of curl F at P and

£ F ● </r is circulation of F about C. Therefore the normal component


of the curl can be interpreted physically as the limit of the circulation
per unit area.
https://www.pdfnotes.co/

Most Popular Books in MATHEMATICS

A. R. Vasishtha
Trigonometry
Matrices A. R. Vasishtha

A. R. Vasishtha
Algebra
Differential Calculus A. R. Vasishtha

A. R. Vasishtha
Integral Calculus
A. R. Vasishtha
Differential Equations
A. R. Vasishtha
Geometry of2 Dimensions
A. R. Vasishtha
Geometry of 3 Dimensions
A. R. Vasishtha
Modem Algebra
Vector Calculus A. R. Vasishtha

A. R. VasisMaa
Ordinary & Partial Diflfcrrential Equations

Partial Differential Equations A. R. Vasishtha

Statics A. R. Vasishtha

A. R. Vasishtha
Dynamics

Real Analysis A. R. Vasishtha

Numerical Analysis A. R. Vasishtha

Hydrostatics A. R. Vasishtha

1SW 81-8283-94
THE KRISHNA Prakashan
KRISHNA
GROUP Media (P) Ltd..Meerut

‘:>>7K8 I 82*^R3 94<

Buy Books Online at WWW. krishnaprakashan write to us at InfoOkrishnaprakashan^

You might also like